Download as pdf or txt
Download as pdf or txt
You are on page 1of 274

SEQUENCES &

SERIES
30 January 2024
Revision: 1553

AZIZ MANVA
AZIZMANVA@GMAIL.COM

ALL RIGHTS RESERVED


Get all the files at: https://bit.ly/azizhandouts
Aziz Manva (azizmanva@gmail.com)

TABLE OF CONTENTS
3.4 More Topics 116
TABLE OF CONTENTS ................................. 2 3.5 Infinite Geometric Series 120
3.6 Finite Geometric Series 132
1. BASICS ...................................................... 3
1.1 Figurate Numbers (Sequences) 3 4. FURTHER TOPICS .............................. 150
1.2 Common Sequences and Series 14 4.1 Summation Notation 150
1.3 Pairing, Factoring and Telescoping 25 4.2 Product and Other Notations 164
2. ARITHMETIC SEQUENCES & SERIES 32 4.3 Log Sequences; AP-GP General Revision 174
4.4 Harmonic Sequences and Series 191
2.1 Basics 32 4.5 𝑹𝑴𝑺 ≥ 𝑨𝑴 ≥ 𝑮𝑴 ≥ 𝑯𝑴 (In)equality 201
2.2 Algebra 49 4.6 Recursive and Cyclic Sequences 216
2.3 Symmetry,Structure,Proof 59 4.7 Arithmetico-Geometric Sequences/Series 232
2.4 Arithmetic Series 76 4.8 Telescoping 241
4.9 Miscellaneous Sequences 253
3. GEOMETRIC SEQUENCES & SERIES . 90 4.10 Polynomial Sequences 262
3.1 Basics 90 4.11 Series Expansions 270
3.2 Exponential Growth and Decay 102 4.12 Further Topics 271
3.3 More Definitions 109

P a g e 2 | 274
Get all the files at: https://bit.ly/azizhandouts
Aziz Manva (azizmanva@gmail.com)

1. BASICS
1.1 Figurate Numbers (Sequences)
A. Triangular Numbers
The figurate numbers form geometrical figures – triangles, squares, pentagons and others1. We begin our study
of figurate numbers by looking at triangles. The colored green squares in the diagram below form the shape of a
triangle. They also form the shape of a staircase.

The numbers in the first 𝑛 rows of the triangular arrangement above are
called triangular numbers. Hence, the first few triangular numbers are:
𝐹𝑖𝑟𝑠𝑡 𝑇𝑟𝑖𝑎𝑛𝑔𝑢𝑙𝑎𝑟 𝑁𝑢𝑚𝑏𝑒𝑟 = 𝑛1 = 1
𝑆𝑒𝑐𝑜𝑛𝑑 𝑇𝑟𝑖𝑎𝑛𝑔𝑢𝑙𝑎𝑟 𝑁𝑢𝑚𝑏𝑒𝑟 = 𝑛2 = 1 + 2 = 3
𝑇ℎ𝑖𝑟𝑑 𝑇𝑟𝑖𝑎𝑛𝑔𝑢𝑙𝑎𝑟 𝑁𝑢𝑚𝑏𝑒𝑟 = 𝑛3 = 1 + 2 + 3 = 6

Example 1.1: Finding Numbers


A. Find the sixth triangular number.
B. Find the eighth triangular number.
C. Find the tenth triangular number using a pattern.

Part A and B
𝑆𝑖𝑥𝑡ℎ 𝑇𝑟𝑖𝑎𝑛𝑔𝑢𝑙𝑎𝑟 𝑁𝑢𝑚𝑏𝑒𝑟 = 1 + 2 + 3 + 4 + 5 + 6 = 21
𝐸𝑖𝑔ℎ𝑡ℎ 𝑇𝑟𝑖𝑎𝑛𝑔𝑢𝑙𝑎𝑟 𝑁𝑢𝑚𝑏𝑒𝑟 = 1 + 2 + 3 + 4 + 5 + 6 + 7 + 8 = 36
Part C
1 + 2 + 3 + 4 + 5 + 6 + 7 + 8 + 9 + 10
Add the first number and the last number:
= 1 + 10 = 11
Add the second number and the second last number
= 2 + 9 = 11
Add the third number and the third last number
= 3 + 8 = 11
Add the fourth number and the fourth last number
= 4 + 7 = 11
Add the fifth number and the fifth last number
= 5 + 6 = 11
The number of 11′𝑠 is the same as the number of pairs that can be made.
And the number of pairs is exactly half of the total number of numbers.
Hence, the total is
10
= 11 × = 11 × 5 = 55
2

1.2: Sum of Natural Numbers


We can view the triangular numbers as representing the sum of the first 𝑛 natural numbers, and find a formula
for them.
𝑛+1
1 + 2 + 3 + ⋯ + (𝑛 − 2) + (𝑛 − 1) + 𝑛 = 𝑛 ( )
2

1 See here for a small discussion on why they are called figurate.

P a g e 3 | 274
Get all the files at: https://bit.ly/azizhandouts
Aziz Manva (azizmanva@gmail.com)

Pair the Numbers as below:


𝟏 + 𝟐 + 𝟑 + ⋯ + (𝒏 − 𝟐) + (𝒏 − 𝟏) + 𝒏
Add the first number and the last number:
=1+𝑛 =𝑛+1
Add the second number and the second last number
=2+𝑛−1=𝑛+1
Add the third number and the third last number
=3+𝑛−2=𝑛+1
Add the fourth number and the fourth last number
=4+𝑛−3=𝑛+1
Each pair adds up to:
𝑛+1
And the number of pairs is
𝑛
2
Final Sum
𝑛 𝑛(𝑛 + 1)
= × (𝑛
⏟ + 1) =

2 2
𝑇𝑜𝑡𝑎𝑙 𝑜𝑓
𝑁𝑢𝑚𝑏𝑒𝑟
𝑜𝑓 𝑃𝑎𝑖𝑟𝑠 𝑒𝑎𝑐ℎ 𝑝𝑎𝑖𝑟

Example 1.3: Using the Formula


A. What is the sum of the first 100 natural numbers?
B. Anand counted the natural numbers from 1 to 25. What is the sum of all the numbers he counted?
C. What is the sum of the numbers from 20 to 40?
1 2 10
D. Compute + + ⋯ +
5 5 5
E. What is the sum of the numbers from 51 to 100?
F. What is the greatest prime factor of the sum of the arithmetic sequence 1 + 2 + 3+. . . +80?
(MathCounts 2002 School Sprint)
G. Find the sum of the first ten even numbers.
H. Find the sum of the first twenty multiples of three.

Part A 40 × 41 19 × 20
= − = 820 − 190 = 630
Substitute 𝑛 = 100 in the formula: 2 2
𝑛(𝑛 + 1) 100 × 101 Part D
= = 50 × 101 = 5050 1 2 10 55
2 2 + +⋯+ = = 11
Part B 5 5 5 5
Substitute 𝑛 = 25 in the formula: Part E
𝑛(𝑛 + 1) 25 × 26
= = 25 × 13 = 250 + 75 = 325 We want the sum:
2 2
Part C 51 + 52 + ⋯ + 100
We want the sum We add and subtract 1 + 2 + ⋯ + 50:
20 + 21 + ⋯ + 40 1 + 2 + ⋯ + 100 − (1 + 2 + ⋯ + 50)
We add and subtract 1 + 2 + ⋯ + 19: Apply the formula for the sum of the first 𝑛
1 + 2 + ⋯ 19 + 20 + ⋯ + 40 − ⏟
⏟ (1 + 2 + ⋯ + 19) numbers:
𝐴𝑑𝑑 𝑖𝑡 ℎ𝑒𝑟𝑒 𝑆𝑢𝑏𝑡𝑟𝑎𝑐𝑡 𝑖𝑡 ℎ𝑒𝑟𝑒 100 × 101 50 × 51
Apply the formula for the sum of the first 𝑛 = − = 5050 − 1275 = 3775
2 2
numbers: Part F

P a g e 4 | 274
Get all the files at: https://bit.ly/azizhandouts
Aziz Manva (azizmanva@gmail.com)

80 × 81 Part H
1 + 2 + ⋯ + 80 = = 40 × 81 = 23 × 34 × 5
2 3 + 6 + 9 + 12 + ⋯ + 60
⇒ 𝐺𝑟𝑒𝑎𝑡𝑒𝑠𝑡 𝑃𝑟𝑖𝑚𝑒 𝐹𝑎𝑐𝑡𝑜𝑟 = 5 = 3(1 + 2 + ⋯ 20)
Part G 20 × 21
2 + 4 + 6 + 8 + ⋯ + 20 = 3[ ]
2
Factor out 2 from each term: = 630
10 × 11
2(1 + 2 + 3 + ⋯ + 10) = 2 [ ] = 110
2

Example 1.4
A. The natural numbers are written following some rule as 1,3,6,10, …The tenth number is: (NMTC
Primary/Screening 2007/1)
B. Look at the rows of numbers shown in the diagram. The first
number in the 50𝑡ℎ row is: (NMTC Primary/Screening
2004/2)
C. It takes one block to build a staircase with one step, three
blocks to build a staircase with two steps, and six blocks to
build a staircase with three steps. The 𝑛𝑡ℎ step of the
staircase needs 𝑛 blocks. What is the total number of blocks
needed to build a staircase with 7 steps?
D. A 12-hour clock strikes once at 1:00, twice at 2:00, etc. How many times does the clock strike in a 24-
hour period? (MathCounts 1990 School Sprint)
E. Sean adds up all the even integers from 2 to 500, inclusive. Julie adds up all the integers from 1 to 250,
inclusive. What is Sean's sum divided by Julie's sum? (MathCounts 2007 Warm-Up 9)

Part A Rather than finding the 50th Row, we find the last
We have the triangular numbers: number in the 49𝑡ℎ Row
𝐹𝑖𝑟𝑠𝑡 𝑁𝑢𝑚𝑏𝑒𝑟: 𝑛1 = 1 49 × 50
1 + 2 + ⋯ + 49 = = 49 × 25 = 1225
𝑆𝑒𝑐𝑜𝑛𝑑 𝑁𝑢𝑚𝑏𝑒𝑟: 𝑛2 = 1 + 2 = 3 2
3×4 And hence, the first number in the 50𝑡ℎ Row is
𝑇ℎ𝑖𝑟𝑑 𝑁𝑢𝑚𝑏𝑒𝑟: 𝑛3 = 1 + 2 + 3 = =6
2 1225 + 1 = 1226
4×5 Part C
𝐹𝑜𝑢𝑟𝑡ℎ 𝑁𝑢𝑚𝑏𝑒𝑟: 𝑛4 = 1 + 2 + 3 + 4 = = 10
2 Total Blocks = 1 + 2 + ⋯ + 7
𝑛(𝑛+1)
Substitute 𝑛 = 7 in :
10 × 11 2
𝑇𝑒𝑛𝑡ℎ 𝑁𝑢𝑚𝑏𝑒𝑟: = 5 × 11 = 55 7×8
2 = 7 × 4 = 28
Part B 2
We can get the blue numbers as the 𝑛𝑡ℎ triangular Part D
number: 1+2+...+12=78
𝐹𝑖𝑟𝑠𝑡 𝑅𝑜𝑤: 𝑛1 = 1 Ans = 78*2=156
𝑆𝑒𝑐𝑜𝑛𝑑 𝑅𝑜𝑤: 𝑛2 = 1 + 2 = 3 Part E
3×4 J=1+2+...+250
𝑇ℎ𝑖𝑟𝑑 𝑅𝑜𝑤: 𝑛3 = 1 + 2 + 3 = =6 S=2+4+...+500=2(1+2+...+250)=2J
2
4×5 S/J=2J/J=2
𝐹𝑜𝑢𝑟𝑡ℎ 𝑅𝑜𝑤: 𝑛4 = 1 + 2 + 3 + 4 = = 10
2

Example 1.5: Back Calculations


If the sum 1 + 2 + 3 + ⋯ + 𝐾 is a perfect square 𝑁 2 and if 𝑁 is less than 100, then the possible values for 𝐾 are:
(AHSME 1956/36)
A. Only 1

P a g e 5 | 274
Get all the files at: https://bit.ly/azizhandouts
Aziz Manva (azizmanva@gmail.com)

B. 1 and 8
C. Only 8
D. 8 and 49
E. 1, 8 and 49

𝐾 = 1 ⇒ 1 + 2 + 3+. . . +𝐾 = 1 = 12 ⇒ 𝑉𝑎𝑙𝑖𝑑
𝐾 = 8 ⇒ 1 + 2 + 3 + ⋯ + 𝐾 = 36 = 62 ⇒ 𝑉𝑎𝑙𝑖𝑑
𝐾 = 49 ⇒ 1 + 2 + 3 + ⋯ + 𝐾 = 1225 = 352 ⇒ 𝑉𝑎𝑙𝑖𝑑

Option E is correct

Example 1.6: Back Calculations


A. If 75 is written as the sum of 10 consecutive natural numbers, the maximum of the numbers is: (NMTC
Primary-III/Screening/36)
B. The number 21 is the sum of two consecutive integers 10 and 11. What is the largest number of positive,
consecutive integers whose sum is 21? (MathCounts 2008 State Countdown)
C. When reading a book, Charlie made a list by writing down the page number of the last page he finished
reading at the end of each day. (He always finished reading a page that he started.) His mom thought his
list indicated the amount of pages he had read on each day. At the end of the 8th day of reading, she
added the numbers on his list and thought Charlie had read 432 pages. If Charlie started reading the
book on page one, and he read the same amount of pages each day of this eight-day period, how many
pages did he actually read by the end of the day? (MathCounts 2006 Chapter Sprint)
D. An airplane climbs 100 feet during the first second after takeoff. In each
succeeding second it climbs 100 feet more than it climbed during the previous
second. How many seconds does it take for the plane to reach an altitude of
12,000 feet above its takeoff height? (MathCounts 2007 School Sprint)
E. The pages of a book are numbered 1 through 𝑛. When the page numbers of the
book were added, one of the page numbers was mistakenly added twice,
resulting in an incorrect sum of 1986. What was the number of the page that was
added twice? (AIME 1986/6)
F. Starting from her tenth birthday, Dawn reads 1 page on the first day, 2 pages the
second day, 3 pages the third day, etc? What is the least number of days that she
can read so that her total pages read are a multiple of 56?
G. A number of linked rings, each 1 cm thick, are hanging on a peg. The top ring has
an outside diameter of 20 cm. The outside diameter of each of the other rings is 1
cm less than that of the ring above it. The bottom ring has an outside diameter of
3 cm. What is the distance, in cm, from the top of the top ring to the bottom of the
bottom ring? (AMC 10A 2006/14)

Part A Hence, the maximum of the numbers is:


The sum of the numbers from 1 to 10 is: 12
10 × 11 Part B
1 + 2 + ⋯ + 10 = = 5 × 11 = 55
2 1+2+...+6=21
If we increase every number by 1, then we get, and 1+2+...+7=28
the overall total increases by 10: Ans=6
2 + 3 + ⋯ + 11 = 65
If we add 1 to every number one more time, than Part C
the total again increases by 10: Let the number of pages read each day be 𝑎. Then,
3 + 4 + ⋯ + 12 = 75 Charlie will write

P a g e 6 | 274
Get all the files at: https://bit.ly/azizhandouts
Aziz Manva (azizmanva@gmail.com)

𝑎, 2𝑎, 3𝑎, … ,8𝑎 112 = 24 × 7


His mom will add: The factors of 2 can be split across 𝑛 and 𝑛 + 1, but
𝑎 + 2𝑎 + ⋯ + 8𝑎 = 432 one of the two must be a multiple of 7.
Take a common: 7 × 8 = 56 ⇒ 𝑇𝑜𝑜 𝑆𝑚𝑎𝑙𝑙
8×9 Check with 14:
𝑎(1 + 2 + ⋯ + 8) = 432 ⇒ 𝑎 ( ) = 432
2 14 × 15, 14 × 13 ⇒ 𝐵𝑜𝑡ℎ 15 𝑎𝑛𝑑 13 𝑎𝑟𝑒 𝑜𝑑𝑑
36𝑎 = 432 ⇒ 𝐷𝑜𝑒𝑠 𝑛𝑜𝑡 𝑤𝑜𝑟𝑘
The actual number of pages will be: (In fact, if the multiple of 7 is even, the number
432 before and after will be odd. So, we do not need to
8𝑎 = ( ) × 8 = 96
36 check unless we reach 7 × 16 = 112)
Part D
Suppose the flight takes 𝑛 seconds: Check with 21:
100 + 200 + ⋯ + 100𝑛 = 12000 21 × 22 ⇒ 𝑂𝑛𝑙𝑦 1 𝑓𝑎𝑐𝑡𝑜𝑟 𝑜𝑓 2
100(1 + 2 + 3 + ⋯ + 𝑛) = 12000 21 × 20 ⇒ 𝑂𝑛𝑙𝑦 2 𝑓𝑎𝑐𝑡𝑜𝑟𝑠 𝑜𝑓 2
𝑛(𝑛 + 1) Check with 35:
= 120
2 35 × 36 ⇒ 𝑂𝑛𝑙𝑦 2 𝑓𝑎𝑐𝑡𝑜𝑟𝑠 𝑜𝑓 2
𝑛(𝑛 + 1) = 240 35 × 34 ⇒ 𝑂𝑛𝑙𝑦 1 𝑓𝑎𝑐𝑡𝑜𝑟 𝑜𝑓 2
Factor pairs of 240 are: Check with 49:
(1,240), (2,120), (3,80), (4,60), (5,48), 49 × 50 ⇒ 𝑂𝑛𝑙𝑦 1 𝑓𝑎𝑐𝑡𝑜𝑟𝑠 𝑜𝑓 2
(6,40), (8,30), (10,24), (12,20), (15,16) 49 × 48 ⇒ 4 𝐹𝑎𝑐𝑡𝑜𝑟𝑠 𝑜𝑓 2 ⇒ 𝑊𝑜𝑟𝑘𝑠
𝑛 = 15 Hence,
You can also do this using quadratics: 𝑛 = 48
𝑛(𝑛 + 1) = 240 ⇒ 𝑛2 + 𝑛 − 240 = 0 Part G
Part E If the rings were 𝑛𝑜𝑡 linked, then you could put next
𝑛(𝑛 + 1) 10 × 11 to each other, and the total distance would be:
𝑛 = 10 ⇒ = = 55
2 2 3 + 4 + ⋯ + 20
40 × 41
𝑛 = 40 ⇒ 𝑛(𝑛 + 1) = = 820 = 1 + 2 + ⋯ + 20 − 3
2 20 × 21
60 × 61 = −3
𝑛 = 60 ⇒ 𝑛(𝑛 + 1) = = 1830 2
2 = 210 − 3 = 207
61 × 62
𝑛 = 61 ⇒ 𝑛(𝑛 + 1) = = 1892
2
𝟔𝟐 × 𝟔𝟑 Since the rings are linked, we must subtract the
𝒏 = 𝟔𝟐 ⇒ 𝒏(𝒏 + 𝟏) = = 𝟏𝟗𝟓𝟑 overlap. There are:
𝟐
63 × 64 20 − 2 = 18 𝑅𝑖𝑛𝑔𝑠
𝑛 = 63 ⇒ 𝑛(𝑛 + 1) = = 2016 18 − 1 = 17 𝑂𝑣𝑒𝑟𝑙𝑎𝑝𝑠
2
𝑂𝑣𝑒𝑟𝑙𝑎𝑝 𝐷𝑖𝑠𝑡𝑎𝑛𝑐𝑒 = 2 × 17 = 34
Hence, the number of pages in the book are: And hence the final answer is:
62 207 − 34 = 173
Part G: Alternate Solution
The wrong total is 1986. The actual total is 1953. The distance from the top of the largest, to its
Hence, it is more by: bottom, is simply:
1986 − 1953 = 33 20
66 The bottom of the top ring is 2 cm below the top of
= 33
2 the 2nd ring. Hence, its distance from the bottom of
Part F the 2nd ring is
Let the number of days she reads be 𝑛. Then for 19 − 2 = 17
some natural number 𝑘: Similarly, the bottom of the 2nd ring is 2 cm below
𝑛(𝑛 + 1) the top of the 3rd ring, and hence its distance from
= 56𝑘 ⇒ 𝑛(𝑛 + 1) = 112𝑘
2 the bottom of the 3rd ring is:
Prime Factorize 112: 18 − 2 = 16

P a g e 7 | 274
Get all the files at: https://bit.ly/azizhandouts
Aziz Manva (azizmanva@gmail.com)

We continue this pattern till we reach the last ring, 20 + 17 + 16 + ⋯ + 1


getting a total of: 17 × 18
= 20 + = 20 + 153 = 173
2

Example 1.7: Positions


The positive integers are arranged in rows and columns as shown. More
rows continue to list the positive integers in order, with each new row
containing one more integer than the previous row. How many integers
less than 2000 are in the column that contains the number 2000?
(Gauss Grade 7 2012/25)

The number at the rightmost end are:


1,3,6,10 ⇒ 𝑇𝑟𝑖𝑎𝑛𝑔𝑢𝑙𝑎𝑟 𝑁𝑢𝑚𝑏𝑒𝑟𝑠
The rightmost number in the row in which 2000 occur (using trial and error) is:
63 × 64
= 2016
2
We start counting the numbers in the column above 2016, and going left:
(2016, 0), (2015,1), (2014,2), … , (2016 − 𝑛, 𝑛)
Since
2016 = 2000 − 16 ⇒ 𝑛 = 16
There are
16 𝑁𝑢𝑚𝑏𝑒𝑟𝑠 𝑎𝑏𝑜𝑣𝑒 2000

Example 1.8: Average


Find the average of the positive numbers less than 100 which are multiples of 3.

The numbers are:


3 + 6 + 9 + ⋯ + 99

Factor 3 from each term


= 3(1 + 2 + 3 + ⋯ + 33)

The sum of the above expression is:


33 ∙ 34
= 3( )
2

The list {1,2, … ,33} has 33 numbers and hence to find the average, we divide by 33:
33 ∙ 34 1 34
= 3( )× = 3 ( ) = 3(17) = 51
2 33 2

Example 1.9: Average


Find the average of the first 𝑛 multiples of the natural number 𝑚.

The numbers are:


𝑚 + 2𝑚 + 3𝑚 + ⋯ + 𝑛𝑚

Factor 𝑚 from each term

P a g e 8 | 274
Get all the files at: https://bit.ly/azizhandouts
Aziz Manva (azizmanva@gmail.com)

𝑚(1 + 2 + 3 + ⋯ + 𝑛)
= 𝑚(1 + 2 + 3 + ⋯ + 𝑛)

The sum of the above expression is:


𝑛(𝑛 + 1)
= 𝑚( )
2

The list {1,2, … , 𝑛} has 𝑛 numbers and hence to find the average, we divide by 33:
𝑛(𝑛 + 1) 1 𝑛+1
= 𝑚( )× = 𝑚( )
2 𝑛 2

B. Square Numbers

1.10: Sum of Odd Numbers


The sum of the first 𝑛 odd numbers:
1 + 3 + 5 + ⋯ = 𝑛2

𝒏 𝒕𝒆𝒓𝒎𝒔

1 1 3 1 3 5 1 3 5 7 1 3 5 7 9 1 3 5 7 9 11

1 + 3 = 4 = 22
1 + 3 + 5 = 9 = 32
1 + 3 + 5 + 7 = 16 = 42
1 + 3 + 5 + 7 + 9 = 25 = 52
1 + 3 + 5 + 7 + 9 + 11 = 36 = 62

Example 1.11
Find the sum of the numbers in each part below. Write each number as the square of a natural number.
A. 1 + 3 + 5
B. 1 + 3 + 5 + 7
C. 1 + 3 + 5 + 7 + 9
D. 1 + 3 + 5 + 7 + 9 + 11
E. 1 + 3 + 5 + ⋯ + 101
F. (−39) + (−37)+. . . + (−1) (MathCounts 1990 School Sprint)
G. −1 − 49 − 3 − 47 − ⋯ − 23 − 25

1 + 3 + 5 ⇒ 𝑛 = 3 ⇒ 𝑛 2 = 32 = 9

3 𝑁𝑢𝑚𝑏𝑒𝑟𝑠
1 + 3 + 5 + 7 ⇒ 𝑛 = 4 ⇒ 𝑛2 = 42 = 16

4 𝑁𝑢𝑚𝑏𝑒𝑟𝑠
1 + 3 + 5 + 7 + 9 ⇒ 𝑛 = 5 ⇒ 𝑛2 = 52 = 25

5 𝑁𝑢𝑚𝑏𝑒𝑟𝑠
11 + 1
1 + 3 + 5 + 7 + 9 + 11 ⇒ 𝑛 = = 6 ⇒ 𝑛2 = 62 = 36
2
P a g e 9 | 274
Get all the files at: https://bit.ly/azizhandouts
Aziz Manva (azizmanva@gmail.com)

Using sum of odd numbers:


39 + 1 2
−(1 + 3 + 5 + 7 + ⋯ + 39) = − ( ) = −202 = −400
2
Rearrange to get:
49 + 1
−1 − 3 − 5 − ⋯ − 47 − 49 = − ( ) = −252 = −625
2

Example 1.12
A. Find the sum to 10 terms of 1 + 3 + 5 + ⋯. Also find the last term added in the above sequence?
B. Andy added all the odd numbers from 1 to 100. Find the total he got.
C. Sheela added all the single digit odd numbers. Find the total she got.
D. Cho added all the two-digit odd numbers. Find Cho’s total
E. Find the sum of all odd three-digit integers.
F. What is the sum of the odd integers between 46 and 64? (MathCounts 2001 National Countdown)

Part A Part E
1 + 3 + 5 + ⋯ = 100 = 102
⏟ 101 + 103 + ⋯ + 999
10 𝑇𝑒𝑟𝑚𝑠 =⏟
1 + 3 + ⋯ 99 + 101 + 103 + ⋯ + 999
Since there are 10 terms, the last number will be: 𝐴𝑑𝑑
10𝑡ℎ 𝑂𝑑𝑑 𝑁𝑜. = 10𝑡ℎ 𝐸𝑣𝑒𝑛 𝑁𝑜. −1
− (1
⏟+ 3 + ⋯ 99)
= 2(10) − 1 = 20 − 1 = 19
𝑆𝑢𝑏𝑡𝑟𝑎𝑐𝑡
Part B = 5002 − 502 = 250000 − 2500 = 247500
We want: Part F
99 + 1 100 63+1 64
1 + 3 + ⋯ 99 ⇒ 𝑛 = = = 50 There are 2 = 2 = 32 odd integers upto 63. And
2 2
𝑛2 = 502 = 2500 we do not want the odd integers from 1 to 45,
Part C which are
1 + 3 + 5 + 7 + 9 ⇒ 𝑛 = 5 ⇒ 𝑛2 = 52 = 25
⏟ 45 + 1 46
= = 23
5 𝑁𝑢𝑚𝑏𝑒𝑟𝑠 2 2
Part D Use Complementary Counting:
11 + 13 + ⋯ + 99 322 − 232 = 1024 − 529 = 495
Add and Subtract 1 + 3 + 5 + 7 + 9: We can also do this using arithmetic series:
= 𝟏 + 𝟑 + ⋯ + 𝟗 + 11 + 13 + ⋯ + 99 𝐹𝑖𝑟𝑠𝑡 + 𝐿𝑎𝑠𝑡 47 + 63
𝑛×( ) = 9( ) = 9 × 55 = 495
− (1 + 3 + ⋯ + 9) 2 2
= 2500 − 25 = 2475

Example 1.13
Look at the following dot diagram. This pattern continues.
The value of 1 + 3 + 5 + ⋯upto 100 terms is the number of
dots shown in which pattern? What is the sum 1 + 3 + 5 + ⋯
upto 100 terms (NMTC Primary/Screening 2004/01,
Adapted)

𝑃𝑎𝑡𝑡𝑒𝑟𝑛 1 = 1 𝐷𝑜𝑡 = 12
𝑃𝑎𝑡𝑡𝑒𝑟𝑛 2 = 4 𝐷𝑜𝑡𝑠 = 22
𝑃𝑎𝑡𝑡𝑒𝑟𝑛 3 = 9 𝐷𝑜𝑡𝑠 = 32
𝑛𝑡ℎ 𝑝𝑎𝑡𝑡𝑒𝑟𝑛 = 𝑛2 𝑑𝑜𝑡𝑠
Part A

P a g e 10 | 274
Get all the files at: https://bit.ly/azizhandouts
Aziz Manva (azizmanva@gmail.com)

100𝑡ℎ 𝑃𝑎𝑡𝑡𝑒𝑟𝑛
Part B
1002 = 10,000

Example 1.14
The total number of dots in the first 100 rows:
(NMTC Primary/Screening 2005/09)

The number of dots is:


1 + 3 = 4 = 22
1 + 3 + 5 = 9 = 32
1 + 3 + 5 + 7 = 16 = 42
1 + 3 + 5 + 7 + 9 = 25 = 52

Hence, the number of dots in 100 rows is


1 + 3 + 5 + ⋯ = 1002 = 10,000

100 𝑇𝑒𝑟𝑚𝑠

Example 1.15
The natural numbers are written in the following form:
1
234
98765
10 11 12 13 14 15 16
The number 2010 is which position in which row? (NMTC Primary/Screening 2010/10)

To get the row number, we need to first check which two perfect squares does 2010 lie between? After some
trial and error, we find that:
1936 = 442 < 2010 < 452 = 2025

1
2 3 𝟒 = 𝟐𝟐
𝟐
𝟑 =𝟗8765
10 11 12 13 14 15 𝟏𝟔 = 𝟒𝟐

Note that odd perfect squares are on the left, and even perfect squares are on the right.
The number that we want is between 442 and 452 . Hence, our row will look like this:

… 1936
𝟐𝟎𝟐𝟓 2024 … 1939 1938 𝟏𝟗𝟑𝟕

P a g e 11 | 274
Get all the files at: https://bit.ly/azizhandouts
Aziz Manva (azizmanva@gmail.com)

2025 𝑖𝑠 2025 − 2024 = 1𝑠𝑡 𝑁𝑢𝑚𝑏𝑒𝑟 𝑓𝑟𝑜𝑚 𝑡ℎ𝑒 𝐿𝑒𝑓𝑡


2024 𝑖𝑠 2025 − 2023 = 2𝑛𝑑 𝑁𝑢𝑚𝑏𝑒𝑟 𝑓𝑟𝑜𝑚 𝑡ℎ𝑒 𝐿𝑒𝑓𝑡
2010 𝑖𝑠 2025 − 2009 = 16𝑡ℎ 𝑁𝑢𝑚𝑏𝑒𝑟 𝑓𝑟𝑜𝑚 𝑡ℎ𝑒 𝐿𝑒𝑓𝑡

Example 1.16
Candice is drawing squares with the same intersection point for their
diagonals. The first square has a side length of 1 cm, the second a side length
of 2 units, the third 3 units and so on. She colors the first(smallest) square
𝑑𝑎𝑟𝑘 𝑏𝑙𝑢𝑒. Then, she colors the area outside the first square, and inside the
second square 𝑔𝑟𝑒𝑒𝑛. She colors the area outside the second square, but
inside the third square 𝑙𝑖𝑔ℎ𝑡 𝑏𝑙𝑢𝑒. The diagram shows the first three
squares. She continues this process, using a different color each time.
A. Let 𝑡𝑛 be the area occupied by the 𝑛𝑡ℎ color that she uses. Find the
sequence 𝑡1 , 𝑡2 , 𝑡3 , . ..
B. Find the sum to 100 terms of 𝑡1 + 𝑡3 + 𝑡5 + ⋯

𝐴𝑟𝑒𝑎 𝑜𝑓 1𝑠𝑡 𝑆𝑞𝑢𝑎𝑟𝑒 = 1


𝐴𝑟𝑒𝑎 𝑜𝑓 2𝑛𝑑 𝑆𝑞𝑢𝑎𝑟𝑒 = 4
𝐴𝑟𝑒𝑎 𝑜𝑓 3𝑟𝑑 𝑆𝑞𝑢𝑎𝑟𝑒 = 9
𝐴𝑟𝑒𝑎 𝑜𝑓 4𝑡ℎ 𝑆𝑞𝑢𝑎𝑟𝑒 = 16

𝑡1 = 4 − 1 = 3
𝑡2 = 9 − 4 = 5
𝑡3 = 16 − 9 = 7

Example 1.17
Ali has an elephant who loves peanuts. He gives his elephant one peanut on the 1st of the month, 3 peanuts on
the second day of the month, 5 peanuts on the third day of the month, and so on. When the month changes, he
gives his elephant 1 peanut on the first day of the new month. He must feed his elephant the full quota of
peanuts every day.
A. How many peanuts will his elephant eat in February in a non-leap year?
B. Ali has 400 peanuts in stock on 1st Feb before feeding any peanuts to his elephant. He has a just-in-time
peanut order policy. They are delivered only when needed. On which day in February will he get his first
delivery.
C. How many more peanuts will Ali's elephant eat in March as compared to Feb.

Part A
1 + 3+. . += 282 = 784
Part B
400 = 202 ⇒ 20 𝐷𝑎𝑦𝑠 ⇒ 𝐷𝑒𝑙𝑖𝑣𝑒𝑟𝑦 𝑤𝑖𝑙𝑙 ℎ𝑎𝑝𝑝𝑒𝑛 𝑜𝑛 21𝑠𝑡 𝐷𝑎𝑦
Part C
312 − 282 = 961 − 784 = 177 𝑃𝑒𝑎𝑛𝑢𝑡𝑠

Example 1.18
A book has 100 pages. Strangely, the author insists on giving only odd numbers to the pages

1 , ⏟
3 , … , 199

1𝑠𝑡 𝑃𝑎𝑔𝑒 2𝑛𝑑 𝑃𝑎𝑔𝑒 100𝑡ℎ 𝑃𝑎𝑔𝑒
If the sum of the numbers on the pages can be written as 𝑝𝑎 𝑞𝑏 , where 𝑝 and 𝑞 are prime numbers, and 𝑎 and 𝑏

P a g e 12 | 274
Get all the files at: https://bit.ly/azizhandouts
Aziz Manva (azizmanva@gmail.com)

are positive integers, find


𝑝+𝑞+𝑎+𝑏

There are 100 pages, which will be numbered


1 + 3 + 5 + ⋯ = 1002 = 104 = 24 × 54 ⇒ 2 + 5 + 4 + 4 = 15

Example 1.19
A grocer makes a display of cans in which the top row has one can and each lower row has two more cans than
the row above it.
A. If the display contains 100 cans, how many rows does it contain? (AMC12B 2004/8)
B. What is the number of cans in the bottom row?

1 + 3 + 5 + ⋯ = 100 = 102 ⇒ 10 𝑁𝑢𝑚𝑏𝑒𝑟𝑠 ⇒ 10 𝑅𝑜𝑤𝑠


𝑁𝑜. 𝑜𝑓 𝐶𝑎𝑛𝑠 = 10𝑡ℎ 𝑂𝑑𝑑 𝑁𝑜. = 2(10) − 1 = 20 − 1 = 19

Example 1.20
The sum of 5 consecutive even integers is 4 less than the sum of the first 8 consecutive odd counting numbers.
What is the smallest of the even integers? (AMC 10B 2003/3)

(1 + 3 + 5+. . . +15) = 64
64 − 4 = 60
60
= 12
5
𝑁𝑢𝑚𝑏𝑒𝑟𝑠 𝑎𝑟𝑒 8,10,12,14,16
𝑆𝑚𝑎𝑙𝑙𝑒𝑠𝑡 𝑖𝑠 8

Example 1.21
Anisha arranged her Lego blocks in groups of size 𝑥, starting from the smallest possible group, and increasing
sequentially, except that no group was divisible by 2. She had none left over. The blocks were then counted to be
1370. If a single group was counted twice, which group was it?

1 + 3 + 5 + ⋯ = 𝑥2
Hence, we need the largest square which is smaller than 1370, which is:
372 = 1369

1370 − 1369 = 1
Hence, the group of blocks that had a single block in it was counted twice.

Example 1.22
The sequence
1,2,1,2,2,1,2,2,2,1,2,2,2,2,1,2,2,2,2,2,1, 2, . ..
consists of 1’s separated by blocks of 2’s with 𝑛 2’s in the 𝑛𝑡ℎ block. The sum of the first 1234 terms of this
sequence is (AHSME 1996/24)

1st 2nd 3rd 4th 5th


Block Block Block Block Block
1 2,1 2,2,1 2,2,2,1 2,2,2,2,1
No. of 1 2 3 4 5 . . . Triangular Numbers
Numbers

P a g e 13 | 274
Get all the files at: https://bit.ly/azizhandouts
Aziz Manva (azizmanva@gmail.com)

Total of 1 3 5 7 9 . . . Square Numbers


those
Numbers

The numbers of numbers in each block is given by the triangular numbers. Hence, we will be able to have:
𝑛(𝑛 + 1) 49 × 50
𝑛 = 49 ⇒ = = 1225 ⇒ 49 𝐶𝑜𝑚𝑝𝑙𝑒𝑡𝑒 𝐵𝑙𝑜𝑐𝑘𝑠
2 2
The total of those 49 complete blocks will be given by:
492 = 2401
But we need the total of
1225 + 9 = 1234 𝑇𝑒𝑟𝑚𝑠
After the 49th block, there will be 49 consecutive 2’s. But we only need nine of those to get the total to 1234
terms.
So we need to add:
2401 + (9 × 2) = 2401 + 18 = 2419
C. Others

Example 1.23
Pentagonal Numbers
Hexagonal Numbers
Centered Hexagonal Numbers

Example 1.24
In the triangle shown, the first diagonal line, 1, 2, 3, 4, … , begins at 1 and each number after the first is one
larger than the previous number. The second diagonal line, 2, 4, 6, 8, … begins at 2 and each number after the
first is two larger than the previous number. The nth diagonal line begins at n and each number after the first is
n larger than the previous number. In which horizontal row does the number 2016 first appear? (Gauss Grade 7
2016/25)

A number from the 𝑛𝑡ℎ diagonal line appears in the 𝑚𝑡ℎ horizontal row when:
𝑛 × 𝑚 = 2016 ⇔ ⏟
⏟ 𝑛+𝑚−1
𝑁𝑢𝑚𝑏𝑒𝑟 𝑅𝑜𝑤 𝑁𝑢𝑚𝑏𝑒𝑟

To minimize 𝑛 + 𝑚 − 1, we minimize 𝑛 + 𝑚, which is itself minimized by having n and m as close to each other
as possible.

We can list the factor pairs of 2016, and the factor pair with minimum difference between the factors is:
42 × 48 = 2016
𝑛 + 𝑚 − 1 = 42 + 48 − 1 = 89

1.2 Common Sequences and Series


A. Defined Sequences

Example 1.25
Two sequences 𝑆1 and 𝑆2 are are as under:
2 4 6
𝑆1 : , , , …
1 3 5

P a g e 14 | 274
Get all the files at: https://bit.ly/azizhandouts
Aziz Manva (azizmanva@gmail.com)

1 3 5
𝑆2 : , , , …
2 4 6
2𝑛 2𝑛−1
The 𝑛𝑡ℎ term of 𝑆1 is 𝑆1 : 𝑡ℎ
and the 𝑛 term of 𝑆2 is . The value of the difference between the 2012𝑡ℎ
2𝑛−1 2𝑛
terms of 𝑆1 and 𝑆2 is: (NMTC Primary/Screening 2012/A/5)

Method I: Substitute first and simplify later:


2𝑛 2 × 2012 4024
2012𝑡ℎ 𝑡𝑒𝑟𝑚 𝑜𝑓 𝑆1 = = =
2𝑛 − 1 2 × 2012 − 1 4023
2𝑛 − 1 2 × 2012 − 1 4023
2012𝑡ℎ 𝑡𝑒𝑟𝑚 𝑜𝑓 𝑆2 = = =
2𝑛 2 × 2012 4024

4024 4023 40242 − 40232


− =
4023 4024 4024 × 4023
Factor the numerator using the property 𝑎2 − 𝑏 2 = (𝑎 + 𝑏)(𝑎 − 𝑏):
(4024 + 4023)(4024 − 4023) (4024 + 4023)(1) 8047
= = =
4024 × 4023 4024 × 4023 4024 × 4023

Method II: Simplify first and substitute later


2𝑛 2𝑛 − 1 (2𝑛)2 − (2𝑛 − 1)2 4𝑛2 − (4𝑛2 − 4𝑛 + 1) 4𝑛 − 1
− = = =
2𝑛 − 1 2𝑛 2𝑛(2𝑛 − 1) 2𝑛(2𝑛 − 1) 2𝑛(2𝑛 − 1)
Substitute 𝑛 = 2012:
2012(4) − 1 8048 − 1 8047
= =
(2012 × 2)(2012 × 2 − 1) (4024)(4024 − 1) (4024)(4023)

B. Prime Numbers

Example 1.26
Find the 27𝑡ℎ term in the sequence:
2,3,5,7,11,13,17, …

Let’s apply common differences:


2,3,5,7,11,13,17, …
1,2,2,4,2,4
The common differences don’t have a clear pattern from which to make a rule.

So, we look for another rule. The above sequence consists of prime numbers, starting from 2.
There are 25 prime numbers less than 100.
∴ 25𝑡ℎ 𝑃𝑟𝑖𝑚𝑒 𝑁𝑢𝑚𝑏𝑒𝑟 = 97
∴ 26𝑡ℎ 𝑃𝑟𝑖𝑚𝑒 𝑁𝑢𝑚𝑏𝑒𝑟 = 101
∴ 27𝑡ℎ 𝑃𝑟𝑖𝑚𝑒 𝑁𝑢𝑚𝑏𝑒𝑟 = 103

Example 1.27
The 25𝑡ℎ term in the sequence:
(1,2), (2,3), (3,5), (4,7), (5,11), (6,13), …
(NMTC Primary/Screening 2004/03)

We can think of this as sequences put together:


𝐹𝑖𝑟𝑠𝑡 𝑆𝑒𝑞𝑢𝑒𝑛𝑐𝑒: 1,2,34,5,6,7 ….

P a g e 15 | 274
Get all the files at: https://bit.ly/azizhandouts
Aziz Manva (azizmanva@gmail.com)

𝑆𝑒𝑐𝑜𝑛𝑑 𝑆𝑒𝑞𝑢𝑒𝑛𝑐𝑒: 2,3,5,7,11,13,17, …

The first number in each bracket is just the term number. Hence, for the 25th term, it will be just
25

The second term in each bracket is a prime number, starting from 2, and then going to the next largest prime,
etc. Hence, for the 25th, we want the 25th prime number, which is:
97
Our final answer is:
(25,97)

C. Composite Numbers

Example 1.28
Here is a sequence of composite numbers having only one prime factor, written in ascending order
4,8,9,16,25,27,32, … . The 15𝑡ℎ number of this sequence is:
(NMTC Primary/Screening 2012/B/2)

You cannot have prime numbers:


2,3,5,7, … ⇒ 𝑅𝑢𝑙𝑒𝑑 𝑂𝑢𝑡 Powers of 2 4 8 16 32 64 128
You cannot have 1: Powers of 3 9 27 81 243
⇒ 𝑁𝑒𝑖𝑡ℎ𝑒𝑟 𝑝𝑟𝑖𝑚𝑒 𝑛𝑜𝑟 𝑐𝑜𝑚𝑝𝑜𝑠𝑖𝑡𝑒. Powers of 5 25 125 625
Powers of 7 49 343
Powers of 11 121
The only remaining numbers are those which are a power
Powers of 13 169
of a single prime.
4,8,9,16,25,27,32,49,64,81,121,125,128,169,243
D. Double Sequences

Example 1.29
Find the next four numbers in the sequence: 11,88,16,80,21,72, … (NMTC Primary-III/11, Adapted)

This is a double sequence. The odd terms have the rule 𝐴𝑑𝑑 5.
11,16, 21, 26, 31
The even terms have the rule 𝑆𝑢𝑏𝑡𝑟𝑎𝑐𝑡 8.
88,80,72,64,56
The final sequence is:
11,88,16,80,21,72, 26
⏟ , 64 ⏟ , 31
⏟ , 56

21+5 72−8 26+5 64−4

Example 1.30
In the sequence 1,4,3,6,5,8,7,10 … we have 𝑡2𝑛−1 = 2𝑛 − 1 and 𝑡2𝑛 = 𝑡2𝑛−1 + 3 i.e. every odd term is that odd
number and the next even term is 3 more than the previous odd term. If 𝑡𝑚 = 2010, then 𝑚 = (NMTC
Primary/Screening 2010/14)

We can think of this as a double sequence:


𝑂𝑑𝑑 𝑇𝑒𝑟𝑚𝑠: 1,3,5,7, …
𝐸𝑣𝑒𝑛 𝑇𝑒𝑟𝑚𝑠: ⏟
4 , ⏟
6 , ⏟ 8 , 10 ⏟ , … , 2010

2𝑛𝑑 4𝑡ℎ 6𝑡ℎ 8𝑡ℎ 𝟐𝟎𝟎𝟖𝒕𝒉
𝑇𝑒𝑟𝑚 𝑇𝑒𝑟𝑚 𝑇𝑒𝑟𝑚 𝑇𝑒𝑟𝑚 𝑻𝒆𝒓𝒎

P a g e 16 | 274
Get all the files at: https://bit.ly/azizhandouts
Aziz Manva (azizmanva@gmail.com)

𝑡2𝑛 = 𝑡2𝑛−1 + 3 = 2𝑛 − 1 + 3 = 2𝑛 + 2
Hence, the value of the term is 2 more than the term number.
E. Remainders

Example 1.31
𝑀𝑜𝑛, 𝑇𝑢𝑒, 𝑊𝑒𝑑, 𝑇ℎ𝑢, 𝐹𝑟𝑖, 𝑆𝑎𝑡, 𝑆𝑢𝑛, 𝑀𝑜𝑛, 𝑇𝑢𝑒, …
Consider the sequence above. Find, in the above sequence, the:
A. 9𝑡ℎ term
B. 49𝑡ℎ term
C. 100𝑡ℎ term
D. 729𝑡ℎ term
E. 1002𝑛𝑑 term

There are seven days in the week. This means the sequence will repeat every seven days.

𝑀𝑜𝑛
⏟ , 𝑇𝑢𝑒
⏟ ,⏟𝑊𝑒𝑑 , 𝑇ℎ𝑢
⏟ , 𝐹𝑟𝑖
⏟ ,⏟𝑆𝑎𝑡 , 𝑆𝑢𝑛
⏟ , 𝑀𝑜𝑛
⏟ , 𝑇𝑢𝑒
⏟ ,…
1 2 3 4 5 6 7≡0 (𝑚𝑜𝑑 7) 8≡1(𝑚𝑜𝑑 7) 9≡2(𝑚𝑜𝑑 7)

9 = 2 + 7 ≡ 2 (𝑚𝑜𝑑 7) ⇒ 𝑇𝑢𝑒𝑠𝑑𝑎𝑦
49 ≡ 0 (𝑚𝑜𝑑 7) ⇒ 𝑆𝑢𝑛𝑑𝑎𝑦
100 ≡ 2 (𝑚𝑜𝑑 7) ⇒ 𝑇𝑢𝑒𝑠𝑑𝑎𝑦
729 ≡ 29 ≡ 1 (𝑚𝑜𝑑 7) ⇒ 𝑀𝑜𝑛𝑑𝑎𝑦
1002 ≡ 1 + 1001 ≡ 1 + (7 × 11 × 13) ≡ 1 (𝑚𝑜𝑑 7) ⇒ 𝑀𝑜𝑛𝑑𝑎𝑦

Example 1.32
If the first day of each sequence is a Sunday, and it is a non-leap year, what day is the last day of the sequence?
A. 𝐽𝑎𝑛 1, 𝐽𝑎𝑛 2, … , 𝐽𝑎𝑛 31
B. 𝐽𝑎𝑛 1, 𝐽𝑎𝑛 2, … , 𝐷𝑒𝑐 31

𝐽𝑎𝑛 1 → 𝑆𝑢𝑛𝑑𝑎𝑦 → 1 (𝑚𝑜𝑑 7)


Part A
𝐽𝑎𝑛 31 → 31 ≡ 3 (𝑚𝑜𝑑 7) → 𝑇𝑢𝑒𝑠𝑑𝑎𝑦
Part B
𝐷𝑒𝑐 31 → 364 ≡ 0 (𝑚𝑜𝑑 7) → 𝑆𝑢𝑛𝑑𝑎𝑦

Example 1.33
𝐽𝑎𝑛, 𝐹𝑒𝑏, 𝑀𝑎𝑟, … , 𝐷𝑒𝑐, 𝐽𝑎𝑛, 𝐹𝑒𝑏 …
In the sequence above, find the:
A. 12𝑡ℎ Term
B. 50𝑡ℎ Term
C. 100th Term
D. 1730𝑛𝑑 term

12 𝑇𝑒𝑟𝑚 = 12 (𝑚𝑜𝑑 12) → 𝐷𝑒𝑐


50 ≡ 2 (𝑚𝑜𝑑 12) → 𝐹𝑒𝑏
100 ≡ 4(𝑚𝑜𝑑 12) → 𝐴𝑝𝑟𝑖𝑙
1730 = 1728 + 2 = 123 + 2 ≡ 2 (𝑚𝑜𝑑 12) → 𝐹𝑒𝑏

P a g e 17 | 274
Get all the files at: https://bit.ly/azizhandouts
Aziz Manva (azizmanva@gmail.com)

Example 1.34
The 2009th letter of the sequence 𝑀𝐴𝑇𝐻𝑇𝐴𝐿𝐸𝑁𝑇 𝑀𝐴𝑇𝐻𝑇𝐴𝐿𝐸𝑁𝑇 𝑀𝐴𝑇𝐻𝑇𝐴𝐿𝐸𝑁𝑇 … (NMTC Primary/Screening
2009/06)

The sequence consists of MATHTALENT repeated.


MATHTALENT consists of 10 letters.

We find the remainder when 2009 is divided by 10:


2009 ≡ 9 (𝑚𝑜𝑑 10) → 𝑁

Example 1.35
A fan switch is in the off position. A boy turns it continuously to 𝑠𝑙𝑜𝑤, 𝑚𝑒𝑑𝑖𝑢𝑚, ℎ𝑖𝑔ℎ, 𝑜𝑓𝑓, 𝑠𝑙𝑜𝑤, 𝑚𝑒𝑑𝑖𝑢𝑚, … for
each operation counting 1,2,3,4,5,6, … . After 2009 operations in which position will the switch be? (NMTC
Primary/Screening 2009/09)

Off Slow Medium High


Remainder 0 1 2 3
4 5 6 7
8 9 10 11
2009 ≡ 1(𝑚𝑜𝑑 4) → 𝑆𝑙𝑜𝑤 𝑃𝑜𝑠𝑖𝑡𝑖𝑜𝑛

Example 1.36
Laxman starts counting backwards from 100 by 7′𝑠. He begins 100,93,86, … .Which number will not come in his
countdown? (NMTC Primary-III/Screening/20)
A. 65
B. 30
C. 23
D. 15

Note that each number in the sequence has remainder 2 when divided by 7.
Hence, the correct answer option will have a remainder other than 2 when divided 7.

This is
15
= 2𝑅1 ⇒ 𝑂𝑝𝑡𝑖𝑜𝑛 𝐷 𝑖𝑠 𝑐𝑜𝑟𝑟𝑒𝑐𝑡
7
F. Squares of Natural Numbers

Example 1.37
A. What percent of the first ten thousand natural numbers are perfect squares?
B. In the sequence 1,2,34,5, … ,100000000 the percentage of square numbers is: (NMTC
Primary/Screening 2009/11)

Part A
The squares upto 10,000 = 1002 are
100
× 100 = 1%
10000

P a g e 18 | 274
Get all the files at: https://bit.ly/azizhandouts
Aziz Manva (azizmanva@gmail.com)

Part B
The squares upto 100,000,000 = 10,0002 are:
12 , 22 , 32 , … , 100002 ⇒ 10,000 𝑁𝑢𝑚𝑏𝑒𝑟𝑠
And the percentage is:
10,000 1
× 100 = % = 0.01%
100,000,000 100

Example 1.38
𝑛(𝑛 + 1)(2𝑛 + 1)
12 + 22 + ⋯ + 𝑛 2 =
6
Using the formula above, find the sum of the squares:
A. of the first seven natural numbers.
B. of the numbers {−10, −9, −8, … . ,9,10}
C. of the natural numbers from 4 to 20 (Calculator Allowed)

Part A
𝑛(𝑛+1)(2𝑛+1)
Substitute 𝑛 = 7 in 6
giving:
𝑛(𝑛 + 1)(2𝑛 + 1) (7)(8)(15)
= = 7 × 4 × 5 = 140
6 6
Part B
Break this into two parts
(−10)2 + (−9)2 + ⋯ + (−1)
(10)2 + (9)2 + ⋯ + (1)
Note that each part will have the same sum since
(−𝑥)2 = 𝑥 2
So, we find
(10)(11)(21)
2( ) = 770
6
Part C
42 + 52 + ⋯ + 202
Add and subtract 12 + 22 + 32 = 1 + 4 + 9 = 14
20 × 21 × 41
12 + 22 + ⋯ + 202 − 14 = − 14 = 2870 − 14 = 2856
6

Example 1.39
𝑛(𝑛 + 𝑐)(2𝑛 + 𝑘)
The sum of the squares of the first n positive integers is given by the expression 6
, if 𝑐 and 𝑘 are,
respectively: (AHSME 1957/30)

𝑐=𝑘=1

P a g e 19 | 274
Get all the files at: https://bit.ly/azizhandouts
Aziz Manva (azizmanva@gmail.com)

Example 1.40
Cannonballs are stacked in a pyramid format in three dimensions
as shown. Each cannonball is stacked in the dip or gap between
four cannonballs in the layer below it.2
A. Find the number of cannonballs in each of the first four
rows starting from the topmost row, which has a single
cannonball, as can be seen in the photo.
B. Find the number of cannonballs in rows 1 to 𝑛. Write it as
a series. Also, write it as a single formula.
C. If the number of cannonballs in 𝑛 rows is 55, find 𝑛.

Part A
The topmost layer has to be, as seen in the photo,
1 𝐶𝑎𝑛𝑛𝑜𝑛𝑏𝑎𝑙𝑙
Each cannonball is stacked on four cannonballs. Hence, the second layer must be:
4 𝐶𝑎𝑛𝑛𝑜𝑛𝑏𝑎𝑙𝑙𝑠
If you draw out the number of cannonballs needed to place 4 cannonballs in a 2 by 2
square, you need a three by three square, giving us
9 𝐶𝑎𝑛𝑛𝑜𝑛𝑏𝑎𝑙𝑙𝑠
If you draw out the number of cannonballs needed to place 9 cannonballs in a 4 by 4
square, you need a three by three square, giving us
16 𝐶𝑎𝑛𝑛𝑜𝑛𝑏𝑎𝑙𝑙𝑠

Part B
𝑛(𝑛 + 1)(2𝑛 + 1)
⏟2 + 22 + ⋯ + 𝑛2 =
1
⏟ 6
𝑺𝒆𝒓𝒊𝒆𝒔 𝑽𝒆𝒓𝒔𝒊𝒐𝒏
𝑭𝒐𝒓𝒎𝒖𝒍𝒂

Part C

Example 1.41
Oranges are stacked in a three-dimensional pyramid with a single orange stacked in the gap between four
oranges in the layer below it. The topmost layer has a single orange. The oranges are then rearranged into a
single layer in the shape of a triangle, with one orange in the gap between two oranges in the row below it, and
one orange to start the arrangement. The number of rows in the triangle is double the number of layers in the
pyramid. Find all possible values that the number of oranges can take.

Let the number of layers in the pyramid be 𝑛. The total number of oranges will be:

2The above question only asks for the number of spheres. Packing maximum spheres in a given space is a much more
difficult problem, and is a current research topic in Maths.

P a g e 20 | 274
Get all the files at: https://bit.ly/azizhandouts
Aziz Manva (azizmanva@gmail.com)

𝑛(𝑛 + 1)(2𝑛 + 1)
12 + 22 + ⋯ + 𝑛 2 =
6
Since the number of layers in the pyramid is 𝑛, the number of rows in the triangle will be double of that, which
is 2𝑛.
And the total number of oranges must be:
2𝑛(2𝑛 + 1)
1 + 2 + ⋯ + 2𝑛 =
2
But the total number of oranges in the pyramid is the same as the number of oranges in the triangle:
2𝑛(2𝑛 + 1) 𝑛(𝑛 + 1)(2𝑛 + 1)
=
2 6
Multiply both sides by 6:
6𝑛(2𝑛 + 1) = 𝑛(𝑛 + 1)(2𝑛 + 1)
Divide both sides by 𝑛(2𝑛 + 1):
𝑛+1=6⇒𝑛 =5
The total number of oranges can only be:
55
G. Cubes of natural numbers

1.42: Cube of the first 𝒏 natural numbers


2
𝑛(𝑛 + 1)
13 + 23 + ⋯ + 𝑛 3 = [ ]
2

Note the interesting property that the sum of the cubes of the first 𝑛 natural numbers is the square of the sum of
the first 𝑛 natural numbers.

Example 1.43
Find the sum
A. 13 + 23 + 33 + 43
B. 13 + 23 + ⋯ + 103
C. 53 + 63 + ⋯ + 103

2
3 3
4(5)
3 3
1 +2 +3 +4 =[ ] = 102 = 100
2
2
10(11)
13 + 23 + ⋯ + 103 = [ ] = 552 = 3025
2

53 + 63 + ⋯ + 103 = 3025 − 100 = 2925

Example 1.44
If 𝑥, 𝑦 ∈ 𝑁, then for what values of 𝑥 and 𝑦 does the inequality below hold true?
2
𝑥(𝑥 + 1)
[ ] < 13 + 23 + 33 + ⋯ + 𝑦 3
2

2 2
𝑥(𝑥 + 1) 𝑦(𝑦 + 1)
[ ] <[ ] ⇒𝑥<𝑦
2 2

P a g e 21 | 274
Get all the files at: https://bit.ly/azizhandouts
Aziz Manva (azizmanva@gmail.com)

H. Removing Squares

Example 1.45
A set of tiles numbered 1 through 100 is modified repeatedly by the following operation: remove all tiles
numbered with a perfect square, and renumber the remaining tiles consecutively starting with 1. How many
times must the operation be performed to reduce the number of tiles in the set to one? (AMC 10A 2002/22)

Brute Force Method


Step I
We start with the tiles:
1,2, … ,100
We remove the tiles
12 , 22 , … , 102 ⇒ 10 𝑇𝑖𝑙𝑒𝑠
The number of tiles left is 100 − 10 = 90. We renumber the remaining tiles, giving us:
1,2, … ,90
Step II
We remove the tiles
12 , 22 , … , 92 ⇒ 9 𝑇𝑖𝑙𝑒𝑠
The number of tiles left is 90 − 9 = 81. We renumber the remaining tiles, giving us:
1,2, … ,81
Simply calculate the number of steps required.

100 90 81 72 64 56 49 42 36 30 25 20 16 12 9 6 4 2 1
10 9 9 8 8 7 7 6 6 5 5 4 4 3 3 2 2 1
1 2 3 4 5 6 7 8 9 10 11 12 13 14 15 16 17 18

Algebraic Method
We originally with a perfect square number of tiles. Let that perfect square be 𝑛2 . After the first operation, we
are left with
𝑛2 − 𝑛 𝑡𝑖𝑙𝑒𝑠
𝑛2 − 𝑛 will have 𝑛 − 1 perfect squares smaller than it, so subtract that:
𝑛2 − 𝑛 − (𝑛 − 1) = 𝑛2 − 2𝑛 + 1 = (𝑛 − 1)2

Example 1.46: Continuation


Answer the question above if:
A. there are 2500 tiles instead of 100?
B. there are 2500 tiles instead of 100, and you stop when you reach 100 tiles instead of reaching 1 tile?
C. if there are 104 tiles instead of 100

Part A
You want to go from
2500 = 502 → 12 = 1 ⇒ 50 − 1 = 49
Number of steps needed
= 49 × 2 = 98
Part B
You want to go from
2500 = 502 → 100 = 102 ⇒ 50 − 10 = 40
Number of steps needed

P a g e 22 | 274
Get all the files at: https://bit.ly/azizhandouts
Aziz Manva (azizmanva@gmail.com)

= 40 × 2 = 80
Part C

100 90 81 72 64 56 49 42 36 30 25 20 16 12 9 6 4 2 1
10 9 9 8 8 7 7 6 6 5 5 4 4 3 3 2 2 1
1 2 3 4 5 6 7 8 9 10 11 12 13 14 15 16 17 18

104 94 85 76 68 60 53 46 40 34 29 24 20 16 12 9 6 4 2 1
10 9 9 8 8 7 7 6 6 5 5 4 4 4 3 3 2 2 1
1 2 3 4 5 6 7 8 9 10 11 12 13 14 15 16 17 18

19 Steps

I. Number Theory

Example 1.47
In the sequence, 1, 22, 333,...,10101010101010101010, 1111111111111111111111,the sum of the digits in the
200𝑡ℎ term is: (NMTC Primary/Screening 2004/03)

The pattern that we observe is:


1 repeats once
2 repeats twice
10 repeats ten times
11 repeats 11 times.

The 𝑛𝑡ℎ term is 𝑛 repeated 𝑛 times. Hence, the 200𝑡ℎ term will consist of the number 200 repeated 200 times.
200200200. . .200

We need the sum of the digits of the above number.


The zeroes do not matter, because they will not increase the sum.

The 2 is repeated 200 times. Hence, the sum of digits is:


2 × 200 = 400

Example 1.48
In the sequence of numbers 1,2,11,22,111,222,... the sum of digits in the 999𝑡ℎ term is (NMTC
Primary/Screening 2004/06)

The even numbered terms:


2𝑛𝑑 𝑇𝑒𝑟𝑚: 1 𝑇𝑤𝑜
4𝑡ℎ 𝑇𝑒𝑟𝑚: 2 𝑇𝑤𝑜′𝑠
6𝑡ℎ 𝑇𝑒𝑟𝑚: 3 𝑇𝑤𝑜′𝑠
𝑛
𝑛𝑡ℎ 𝑇𝑒𝑟𝑚: 𝑇𝑤𝑜′𝑠
2

The odd numbered terms which are one less than the even numbered terms have the same number of digits:
1000
1000𝑡ℎ 𝑇𝑒𝑟𝑚 ℎ𝑎𝑠 = 500 𝐷𝑖𝑔𝑖𝑡𝑠
2

P a g e 23 | 274
Get all the files at: https://bit.ly/azizhandouts
Aziz Manva (azizmanva@gmail.com)

999𝑡ℎ 𝑇𝑒𝑟𝑚 𝑎𝑙𝑠𝑜 ℎ𝑎𝑠 500 𝐷𝑖𝑔𝑖𝑡𝑠

𝑆𝑢𝑚 𝑜𝑓 𝐷𝑖𝑔𝑖𝑡𝑠 = ⏟
1 + 1 + ⋯ + 1 = 1 × 500 = 500
500 𝑇𝑖𝑚𝑒𝑠

Example 1.49
A group of 12 pirates agree to divide a treasure chest of gold coins among themselves as follows. The 𝑘 th pirate
𝑘
to take a share takes 12 of the coins that remain in the chest. The number of coins initially in the chest is the
smallest number for which this arrangement will allow each pirate to receive a positive whole number of coins.
How many coins does the 12th pirate receive? (AMC 10A 2013/21)

1
Let the chest initially have 𝐶 coins. The first pirate takes 12 of the coins, leaving
11
𝐶∙ 𝑐𝑜𝑖𝑛𝑠
12
2
The second pirate takes of the remaining coins, leaving
12
11 10
𝐶∙ ∙ 𝑐𝑜𝑖𝑛𝑠
12 12
Continue this process, and the last pirate gets:
11 10 9 8 7 6 5 4 3 2 1
𝐶∙ ∙ ∙ ∙ ∙ ∙ ∙ ∙ ∙ ∙ ∙
12 12 12 12 12 12 12 12 12 12 12
Prime factor, write in exponent form and simplify:
11 × 7 × 52 × 34 × 28 11 × 7 × 52 × 34 × 28 11 × 7 × 52
𝐶∙ = 𝐶 ∙ = 𝐶 ∙
1211 222 × 311 214 × 37

The smallest value that 𝐶 can take is 214 × 37 .

The last pirate will get:


11 × 7 × 52 = 1925
J. Formula for Nth Term

Example 1.50: Exponents


A. Find 𝑆100 − 𝑆99 if 𝑆1 = 1, 𝑆2 = 7, 𝑆3 = 72 , …
B. If 𝑃𝑛 = 𝑆𝑛+1 − 𝑆𝑛 , then find a formula for 𝑃𝑛 , and also find its first four terms.
C. If 𝑆1 = 1, 𝑆2 = 𝑥, 𝑆3 = 𝑥 2 , …then repeat parts A and B for the new sequence
D. Write the sequence 5,30,180,1080 in terms of the sequences given above.

Part A
The formula for the 𝑛𝑡ℎ term is 7𝑛−1 :
𝑆100 − 𝑆99 = 799 − 798 = 798 (7 − 1) = 6 × 798
Part B
𝑃𝑛 = 𝑆𝑛+1 − 𝑆𝑛 = 7𝑛 − 7𝑛−1 = 7𝑛−1 (7 − 1) = 6 × 7𝑛−1
𝑃1 = 7 − 1 = 6
𝑃2 = 49 − 7 = 42
𝑃3 = 343 − 49 = 294
𝑃4 = 2058
Part C
𝑆100 − 𝑆99 = 𝑥 99 − 𝑥 98 = 𝑥 98 (𝑥 − 1)

P a g e 24 | 274
Get all the files at: https://bit.ly/azizhandouts
Aziz Manva (azizmanva@gmail.com)

𝑃𝑛 = 𝑆𝑛+1 − 𝑆𝑛 = 𝑥 𝑛 − 𝑥 𝑛−1 = 𝑥 𝑛−1 (𝑥 − 1)


Part D
5 = 5 × 1 = 61 − 60
30 = 5 × 6 = 62 − 61
180 = 5 × 62 = 63 − 62
1080 = 5 × 63 = 64 − 63

1.3 Pairing, Factoring and Telescoping


A. Pairing
Sometimes terms do not cancel. Rather, combining terms makes the calculations simple.
Being able to see which terms to combine requires practice.

Example 1.51
A. Find the value of: 1 − 2 + 3 − 4 + 5 − 6 + 7 − 8
B. Find the value of: 1 − 2 + 3 − 4 + 5 − 6 + 7 − 8 + 9
C. Find the value of:1 − 2 + 3 − 4 + ⋯ + (𝑛 − 1) − 𝑛, 𝑛 𝑖𝑠 𝑒𝑣𝑒𝑛
D. Find the value of:1 − 2 + 3 − 4 + ⋯ − (𝑛 − 1) + 𝑛, 𝑛 𝑖𝑠 𝑜𝑑𝑑

Part A
Make a pair of two consecutive numbers:
1−2 + ⏟
⏟ 3−4 + ⏟
5−6 + ⏟
7 − 8 = −1 − 1 − 1 − 1 = −4
1−2=−1 3−4=−1 5−6=−1 7−8=−1

Part B
1−2 + ⏟
⏟ 3−4 + ⏟
5−6 + ⏟
7 − 8 + 9 = −1 − 1 − 1 − 1 + 9 = 5
1−2=−1 3−4=−1 5−6=−1 7−8=−1

You can also do this by pairing the first term with the last term, and so on, leaving the middle term
𝟏−𝟐+𝟑−4+5−𝟔+𝟕−𝟖+𝟗= 5
Part C
The answer to this question is the same as the answer to Part A, except that there we had four pairs, and hence
we multiplied −1 by 4.
𝑛
Here, we have 𝑛 numbers, and hence we will have pairs, giving us:
2
𝑛
1−2 + ⏟
⏟ 3 − 4 + ⋯+ ⏟(𝑛 − 1) − 𝑛 = ⏟ −1 − 1 − ⋯ − 1 = −
1−2=−1 3−4=−1 (𝑛−1)−1=−1 𝑛 2
𝑇𝑖𝑚𝑒𝑠 −1
2

Part D
Method I: Make pairs of all except the last number
1−2 + ⏟
⏟ (𝑛 − 2) − (𝑛 − 1) + 𝒏
3 − 4 + ⋯+ ⏟
1−2=−1 3−4=−1 (𝑛−2)−(𝑛−1)=−1
Each pair adds up −1 giving us:
(𝑛 − 1) 𝑛+1
−1 − 1 − ⋯ − 1 + 𝑛 = −
⏟ +𝑛 =
𝑛−1 2 2
𝑇𝑖𝑚𝑒𝑠 −1
2
Method II: Make pairs of all the numbers
𝟏 − 𝟐 + 3 − 4 + ⋯ − (𝒏 − 𝟏) + 𝒏
Add the pairs:
1+𝑛 =𝑛+1
−2 − (𝑛 − 1) = −2 − 𝑛 = −(𝑛 + 1)

P a g e 25 | 274
Get all the files at: https://bit.ly/azizhandouts
Aziz Manva (azizmanva@gmail.com)

All the pairs will cancel, expect for the middle number, which will be
𝑛+1
2

Example 1.52
A. Saket added up all the even numbers from 1 to 101. Then, from the total he obtained, he subtracted all
odd numbers between 0 and 100. The answer he would have obtained is: (NMTC Primary-
III/Screening/49)
B. What is the positive difference between the sum of the first 20 positive even integers and the sum of the
first 15 positive odd integers? (MathCounts 2001 National Sprint)
C. Find the sum of the largest prime factor and the smallest prime factor of (−498) + (−497) + ⋯ + 500.
D. What is the difference between the sum of the first 2003 even counting numbers and the sum of the first
2003 odd counting numbers? (AMC 10A 2003/1, AMC 12A 2003/1)
E. Asha adds all the odd integers from 1 through 101, inclusive, and then subtracts all the even integers in
that same range from her sum. What result does she obtain? (MathCounts 2007 State Countdown)

Part A
⏟+ 4 + ⋯ + 100) − (1
(2 ⏟+ 3 + ⋯ 99)
𝐸𝑣𝑒𝑛 𝑁𝑢𝑚𝑏𝑒𝑟𝑠 𝑂𝑑𝑑 𝑁𝑢𝑚𝑏𝑒𝑟𝑠
Rearrange:
(2 − 1) + (4 − 3) + ⋯ + (100 − 99)

50 𝑃𝑎𝑖𝑟𝑠
Note that every pair simplifies to 1:
1 + 1 + ⋯ + 1 = 50

50 𝑇𝑖𝑚𝑒𝑠

Part B
⏟+ 4 + ⋯ + 40) − (1
(2 ⏟+ 3 + ⋯ + 29)
𝐸𝑣𝑒𝑛 𝑁𝑢𝑚𝑏𝑒𝑟𝑠 𝑂𝑑𝑑 𝑁𝑢𝑚𝑏𝑒𝑟𝑠
Rearrange:
(2 − 1) + (4 − 3) + ⋯ + (30 − 29) + 32 + 34 + 36 + 38 + 40

15 𝑃𝑎𝑖𝑟𝑠
Simplify:
1 + 1 + ⋯ + 1 + 36 + 36 + 36 + 36 + 36 = 15 + 36 × 5 = 195

15 𝑇𝑖𝑚𝑒𝑠

Part C
(−𝟒𝟗𝟖) + (−𝟒𝟗𝟕) + ⋯ + 𝟒𝟗𝟕 + 𝟒𝟗𝟖 + 499 + 500
All the pairs will cancel out:
499 + 500 = 999
Prime Factorize:
999 = 33 × 37 ⇒ 3 + 37 = 40
Part D
(1 + 3 + ⋯ )
2+4+⋯ −⏟

2003 𝑁𝑢𝑚𝑏𝑒𝑟𝑠 2003 𝑁𝑢𝑚𝑏𝑒𝑟𝑠
Rearrange:
= (2
⏟− 1) + (4 − 3)+. . .
2003 𝑃𝑎𝑖𝑟𝑠
Each pair simplifies to 1. Since there are 2003 such pairs, the total will be
1*2003=2003

P a g e 26 | 274
Get all the files at: https://bit.ly/azizhandouts
Aziz Manva (azizmanva@gmail.com)

Part E
(1+3+...+101)-(2+4+...+100)
=(2-1)+(4-3)+...+(100-99)+101
=-50+101
=51

Example 1.53
The terms 𝑥, 𝑥 + 2, 𝑥 + 4, … , 𝑥 + 2𝑛 form an arithmetic sequence, with 𝑥 an integer. If each term of the
sequence is cubed, the sum of the cubes is −1197. What is the value of 𝑛 if 𝑛 > 3? (MathCounts 2004 State
Sprint)

𝑥 is odd since the final answer is odd.


(−1)3 + (−3)3 + ⋯ + (−9)3 = −1225
−1125 − (−1197) = −28 = (−1)3 + (−3)3
−9, −7, −5 ⇒ 𝑛 = 2
−9, −7, −5, −3, −1,1,3 ⇒ 𝑛 = 6

Example 1.54
What is the greatest number of consecutive integers whose sum is 45? (AMC 10A 2019/5, AMC 12A 2019/4)

−44, −43, … ,43,44,45 ⇒ 90 𝑁𝑢𝑚𝑏𝑒𝑟𝑠

Example 1.55
If 𝑆𝑛 = ⏟
1 − 2 + 3 − 4 + ⋯, then find:
𝑛 𝑡𝑒𝑟𝑚𝑠
A. 𝑆2021 + 𝑆2022
B. 𝑆𝑛 + 𝑆𝑛+1

Part A
2022
𝑆2022 = − = −1011
2
2021 + 1 2022
𝑆2021 = = = 1011
2 2
𝑆2021 + 𝑆2022 = 0
Part B
𝑛 (𝑛 + 1) + 1 −𝑛 + 𝑛 + 2 2
𝑛 is even: 𝑆𝑛 + 𝑆𝑛+1 = − + = = =1
2 2 2 2
𝑛 + 1 −(𝑛 + 1) 0
𝑛 is odd: 𝑆𝑛 + 𝑆𝑛+1 = + = =0
2 2 2

Example 1.56
Consider the sequence
1, −2,3, −4,5, −6, . ..
𝑡ℎ (−1)𝑛+1
whose 𝑛 term is ∙ 𝑛. What is the average of the first 200 terms of the sequence? (AHSME 1997/6)

Example 1.57
Daryl first writes the perfect squares as a sequence
1, 4, 9, 16, 25, 36, 49, 64, 81, 100, . ..
After the number 1, he then alternates by making two terms negative followed by leaving two terms positive.

P a g e 27 | 274
Get all the files at: https://bit.ly/azizhandouts
Aziz Manva (azizmanva@gmail.com)

Daryl’s new sequence is


1, −4, −9, 16, 25, −36, −49, 64, 81, −100, . ..
What is the sum of the first 2011 terms in this new sequence? (CEMC Grade 8 2011/25)

Consider groups of four consecutive terms in the sequence, starting from the first. They can be written:
𝑛2 − (𝑛 + 1)2 − (𝑛 + 2)2 + (𝑛 + 3)2
Re-arrange:
(𝑛 + 3)2 + 𝑛2 − (𝑛 + 1)2 − (𝑛 + 2)2
Expand and simplify:
= 𝑛2 + 6𝑛 + 9 + 𝑛2 − [𝑛2 + 2𝑛 + 1 + 𝑛2 + 4𝑛 + 4]
= 2𝑛2 + 6𝑛 + 9 − [2𝑛2 + 6𝑛 + 5]
=4

Hence, every set of 4 terms adds up to 4.


We have 2011 terms. We use complementary counting and find the total for 2012 terms, and then adjust it for
the missing term.
= 2012 𝑡𝑒𝑟𝑚𝑠 = 503 𝑔𝑟𝑜𝑢𝑝𝑠 ⇒ 𝑇𝑜𝑡𝑎𝑙 = 503 × 4 = 2012

But, the 2012𝑡ℎ term is missing, and that is:


20122 = 4,048,144

The final answer is:


2012 − 4048144 = 4046132

B. Factoring

Example 1.58
You are given a sequence of 58 terms; each term has the form 𝑃 + 𝑛 where 𝑃 stands for the product
2 × 3 × 5 ×. . .× 61 of all prime numbers less than or equal to 61, and 𝑛 takes, successively, the values
2, 3, 4, . . . ,59. Let 𝑁 be the number of primes appearing in this sequence. Then 𝑁 is: (AHSME 1960/)

For each value of the sequence, we can take some prime p common:
𝑃
𝑃 + 𝑛 = 𝑝 ( + 1)
𝑛
Hence, the number of primes is zero.

C. Telescoping

1.59: Telescope
We can sometimes write out a series in such a way that adjacent terms start cancelling.
Telescoping products use cancellation in the numerator and the denominator.

Example 1.60: Products


2 3 4
A. × ×
3 4 5
2 3 99
B. 3
× × … × 100
4
𝑎 𝑎+1 𝑎+𝑛−1
C. 𝑎+1
× 𝑎+2 × … × 𝑎+𝑛

P a g e 28 | 274
Get all the files at: https://bit.ly/azizhandouts
Aziz Manva (azizmanva@gmail.com)

4 5 𝑎 1
D. If 5 × 6 × … × 𝑎+1 = 𝑛 , 𝑛 ∈ ℕ, then find the possible values of 𝑎.

Part A
Note that the denominator and the numerator of successive terms cancels, leaving us with only the numerator
of the first term, and the denominator of the last term:
2 𝟑 𝟒 2
× × =
𝟑 𝟒 5 5
Part B
The idea here is the same as in Part A. We are again left with the numerator of the first term, and the
denominator of the last term:
2 𝟑 𝟗𝟗 2 1
× × …× = =
𝟑 𝟒 100 100 50
Part C
𝑎 𝒂+𝟏 𝒂+𝒏−𝟏 𝑎
× ×…× =
𝒂+𝟏 𝒂+𝟐 𝑎+𝑛 𝑎+𝑛
Part D
4 5 𝑎 4 1
× × …× = =
5 6 𝑎+1 𝑎+1 𝑛

4𝑛 = 𝑎 + 1 ⇒ 𝑎 = 4𝑛 − 1, 𝑛 ≥ 2
For example:
4 1
𝑛 = 2 ⇒ 𝑎 = 4(2) − 1 = 8 − 1 = 7 ⇒ 𝑎 + 1 = 8 ⇒ =
8 2
However, 𝑛 = 1 is not possible since
𝑎 = 4(1) − 1 = 4 − 1 = 3 ⇒ 𝑇ℎ𝑖𝑠 𝑖𝑠 𝑙𝑒𝑠𝑠

Example 1.61: Setting up a Telescope


1 1
A. (1 + 3) (1 + 4) (CEMC Fermat 2012/4)
1 1 1 1 1 1
B. Evaluate (1 − 2) (1 − 3) (1 − 4) (1 − 5) (1 − 6) (1 − 7)
1 1 1 1
C. The product of the 9 factors (1 − ) (1 − ) (1 − ) ⋯ (1 − ) = (AMC 8 1985/9)
2 3 4 10
1 1 1 1 1
D. Evaluate (1 − 4) (1 − 5) (1 − 6) … (1 − 23) (1 − 24)
1 1 1 1 1
E. Evaluate (1 − ) (1 − ) (1 − ) … (1 − ) (1 − )
2 3 4 𝑛−1 𝑛
1 1 1 1 1
F. If (1 − 2) (1 − 3) (1 − 4) … (1 − 𝑛) = 32 , then 𝑛 = ?
3 5 7 2𝑛+1
G. √1 ∙ 3 ∙ 5 … 2𝑛−1 = 9 (CEMC Fermat 2002/21)

1 1 4 5 5
(1 + ) (1 + ) = ( ) ( ) =
3 4 3 4 3

Part A
1 2 1 1
1−
= − =
2 2 2 2
Evaluate each term. Then, don’t multiply! See that the terms cancel.
1 2 3 4 5 6 1
( )( )( )( )( )( ) =
2 3 4 5 6 7 7
Part B

P a g e 29 | 274
Get all the files at: https://bit.ly/azizhandouts
Aziz Manva (azizmanva@gmail.com)

1 𝟐 𝟑 𝟗 1
( )( )( )…( ) =
𝟐 𝟑 𝟒 10 10
Part C
Use three dots to indicate that the pattern continues (rather than writing out the whole sum). All terms except
the first numerator and the last denominator will cancel.
3 4 5 23 3 1
( )( )( )…( ) = =
4 5 6 24 24 8
Part D
This question has 𝑛 instead of the last denominator, but the same pattern:
1 2 3 𝑛−1 1
( )( )( )…( )=
2 3 4 𝑛 𝑛
Part E
From the previous example, we see that the denominator of the last fraction is the denominator of the answer.
Hence, 𝑛 = 32

Part G
Square both sides:
3 5 7 2𝑛 + 1
∙ ∙ … = 81
1 3 5 2𝑛 − 1
Telescope:
2𝑛 + 1 = 81 ⇒ 2𝑛 = 80 ⇒ 𝑛 = 40

Example 1.62
One half of the water is poured out of a full container. Then one third of the remainder is poured out. Continue
the process: one fourth of the remainder for the third pouring, one fifth of the remainder for the fourth pouring,
etc. After how many pourings does exactly one tenth of the original water remain? (AMC 8 1992/25)

1 2 9 1
( )( )…( ) = ⇒ 𝑁𝑜. 𝑜𝑓 𝑇𝑒𝑟𝑚𝑠 = 9 ⇒ 9 𝑃𝑜𝑢𝑟𝑖𝑛𝑔𝑠
2 3 10 10

𝟏
1.63: Breaking
𝒏(𝒏+𝟏)
1 1 𝑛+1 𝑛 1
− = − =
𝑛 𝑛+1 ⏟ 𝑛(𝑛 + 1) 𝑛(𝑛 + 1) 𝑛(𝑛 + 1)
𝑇𝑎𝑘𝑒 𝑡ℎ𝑒 𝐿𝐶𝑀

1 1 3 2 1 1
− = − = =
2 3 6 6 6 2×3
1 1 5 4 1 1
− = − = =
4 5 20 20 20 4 × 5

Example 1.64: Breaking


1 1 1 1 1 1 1
A. The value of 1×2 + 2×3 + 3×4 + 4×5 + 5×6 + 6×7 + 7×8 (NMTC Primary/Screening 2007/13)
1 1 1 1 1
B. Evaluate 2 + 6 + 12 + 20 + 30
1 1 1 1
C. Evaluate + + + ⋯+
2 6 12 𝑛(𝑛−1)

Part A
Apply the formula to break each term into two:

P a g e 30 | 274
Get all the files at: https://bit.ly/azizhandouts
Aziz Manva (azizmanva@gmail.com)

𝟏 𝟏 𝟏 𝟏 𝟏 𝟏 𝟕
(𝟏 − ) + ( − ) + ⋯ + ( − ) = 𝟏 − =
𝟐 𝟐 𝟑 𝟕 𝟖 𝟖 𝟖
Part B
Rewrite the denominators:
1 1 1 1 1
+ + + +
1×2 2×3 3×4 4×5 5×6
Apply the formula to break each term into two:
𝟏 𝟏 𝟏 𝟏 𝟏 𝟏 𝟏 𝟏 1
(1 − ) + ( − ) + ( − ) + ( − ) + ( − )
𝟐 𝟐 𝟑 𝟑 𝟒 𝟒 𝟓 𝟓 6
Everything cancels, except the first and the last term:
1 5
1− =
6 6
Part C
Rewrite the denominators:
1 1 1 1
+ + ⋯+ +
1×2 2×3 (𝑛 − 1)(𝑛 − 2) 𝑛(𝑛 − 1)
Apply the formula to break each term into two:
𝟏 𝟏 𝟏 𝟏 𝟏 𝟏 1
(1 − ) + ( − ) + ⋯ + ( − )+( − )
𝟐 𝟐 𝟑 𝒏−𝟐 𝒏−𝟏 𝒏−𝟏 𝑛
Everything cancels, except the first and the last term:
1 𝑛−1
=1− =
𝑛 𝑛

P a g e 31 | 274
Get all the files at: https://bit.ly/azizhandouts
Aziz Manva (azizmanva@gmail.com)

2. ARITHMETIC SEQUENCES & SERIES


2.1 Basics
A. Basics

Example 2.1: Identifying Rules


An arithmetic sequence is a sequence where each term is obtained by adding or subtracting a fixed number to
the previous term. The fixed number is called the 𝑐𝑜𝑚𝑚𝑜𝑛 𝑑𝑖𝑓𝑓𝑒𝑟𝑒𝑛𝑐𝑒.
Identify the rule in the following sequences. Then, write the next three terms in the sequence.
A. 12,17,22,27, …
B. 31,28,25,22 …
1 3 1
C. , , ,…
2 8 4
1 3
D. − 2 , − 10 , −0.1, …

𝑅𝑢𝑙𝑒 𝑖𝑠: 𝐴𝑑𝑑 5 ⇒ 12, 17,22,27,32,37,42, …


𝑅𝑢𝑙𝑒 𝑖𝑠: 𝑆𝑢𝑏𝑡𝑟𝑎𝑐𝑡 ⇒ 31,28,25,22,19,16,13, …
3 1 3 4 1 1 3 2 3 1 1 3 1 1 1
𝑅𝑢𝑙𝑒 𝑖𝑠: − = − = − , − = − = − ⇒ , , , , 0, − …
8 2 8 8 8 4 8 8 8 8 2 8 4 8 8
3 5 2 1 3 2 1 3
𝑅𝑢𝑙𝑒 𝑖𝑠: − − (− ) = , − − (− ) = ⇒ − , − , −0.1,0.3,0.5, …
10 10 10 10 10 10 2 10

Example 2.2: Converting Rules to Sequences


If the common difference is positive, it is 𝑎𝑑𝑑𝑒𝑑. If the common difference is negative, it is 𝑠𝑢𝑏𝑡𝑟𝑎𝑐𝑡𝑒𝑑.
Write the first five terms of the given arithmetic sequences.
A. First term nine, and common difference four
B. First term five, and common difference 3
2 1
C. First term , and common difference −
3 3
2 1
D. First term , and common difference
5 2

9,13,17,21,25, …
5,8,11,14,17, …
2 1 1 2
, , 0, − , − , …
3 3 3 3
2 9 14 19 24
, , , ,
5 10 10 10 10

Example 2.3: Identifying the 𝒏𝒕𝒉 Term


Given that the sequences below are arithmetic, identify the
A. Fifth term in the sequence with first term seven, and common difference 2.
B. Seventh term in the sequence with first term −2, and common difference −3.
3 1
C. Tenth term in the sequence with first term 7, and common difference 3.
2 1
D. Fifteenth term in the sequence with first term , and common difference − .
11 7
E. 50th term of the sequence with first term 2, and common difference 8.

Part A
7,9,11,13,15

P a g e 32 | 274
Get all the files at: https://bit.ly/azizhandouts
Aziz Manva (azizmanva@gmail.com)


7 ,⏟
7 + 1 × 2,⏟
7 + 2 × 2,⏟
7 + 3 × 2,⏟
7+4×2
1𝑠𝑡 2𝑛𝑑 3𝑟𝑑 4𝑡ℎ 5𝑡ℎ
𝑇𝑒𝑟𝑚 𝑇𝑒𝑟𝑚 𝑇𝑒𝑟𝑚 𝑇𝑒𝑟𝑚 𝑇𝑒𝑟𝑚
7 + 4 × 2 = 7 + 8 = 15
Part B
−2 + (6)(−3) = −2 − 18 = −20
Part C
3 1 3 3
+ (9) ( ) = + 3 = 3
7 3 7 7
Part D
2 1 2 9
+ (14) (− ) = −2=1
11 7 11 11
Part E
2 + 49 × 8 = 2 + 392 = 394

Example 2.4: Term


Identify the term number of:
A. The largest negative term of the sequence 45,41,37, …
B. The largest two-digit term of the sequence 15,28,41, …

Part A
45
⏟ , 41
⏟ , 37
⏟ …
1𝑠𝑡 2𝑛𝑑 3𝑟𝑑
𝑇𝑒𝑟𝑚 𝑇𝑒𝑟𝑚 𝑇𝑒𝑟𝑚
To make 45 negative, we need to subtract 46.
𝐶𝑜𝑚𝑚𝑜𝑛 𝐷𝑖𝑓𝑓𝑒𝑟𝑒𝑛𝑐𝑒 = 4 ⇒ 𝑊𝑒 𝑛𝑒𝑒𝑑 𝑡𝑜 𝑠𝑢𝑏𝑡𝑟𝑎𝑐𝑡 48 = 4 × 12

After I subtract four 12 times, we will have reached


12 + 1 = 13𝑡ℎ 𝑇𝑒𝑟𝑚
Part B
Rewrite the sequence:
13 × 1 + 2, 13 × 2 + 2, 13 × 3 + 2
It is much easier to work with since we know how it corresponds to multiples 13.
13 × 7 + 2 = 91 + 2 = 93 ⇒ 𝑇𝑒𝑟𝑚 𝑁𝑢𝑚𝑏𝑒𝑟 7
13 × 8 + 2 = 104 + 2 = 106 > 100 ⇒ 𝑵𝒐𝒕 𝑽𝒂𝒍𝒊𝒅

Example 2.5
A. The first three terms of an arithmetic sequence are 1, 10 and 19, respectively. What is the value of the
21st term? (MathCounts 2008 National Countdown)
2 4
B. What is the eighth term in the arithmetic sequence , 1, , . ..? Express your answer in simplest form.
3 3
(MathCounts 2003 State Countdown)

Part A
𝐶𝑜𝑚𝑚𝑜𝑛 𝐷𝑖𝑓𝑓𝑒𝑟𝑒𝑛𝑐𝑒 = 9
We add the common difference twenty times to the first term to get the 21st term:
= 1 + 20(9) = 1 + 180 = 181
Part B
d=1/3
a_8=2/3 + (7)(1/3)=3

P a g e 33 | 274
Get all the files at: https://bit.ly/azizhandouts
Aziz Manva (azizmanva@gmail.com)

Example 2.6
Connie is starting an exercise program. On June 1, she will do 25 sit-ups. Each day after that, she will increase
her number of sit-ups by four. On which date during the month of June will Connie first do more than 100 sit-
ups in one day? (MathCounts 2005 Chapter Countdown)

𝐵𝑎𝑠𝑒 = 25
𝐼𝑛𝑐𝑟𝑒𝑎𝑠𝑒 = 75
75
𝐷𝑎𝑦𝑠 = = 18.75 ⇒ 𝑅𝑜𝑢𝑛𝑑 𝑢𝑝 𝑡𝑜 19
4
𝐷𝑎𝑡𝑒 = 1 + 19 = 20

Example 2.7: Number of Terms


A. How many terms of the arithmetic sequence 88, 85, 82,... appear before the number -17 appears?
(MathCounts 2004 Chapter Sprint)
B. How many integers belong to the arithmetic sequence 13,20,27,34,...,2008?

Part A
35

Part B
Subtract 6:
7,14,21,,...,2002

These are all multiples of 7.


2002=1001*2=7*11*13*2=7*286

Algebra:
13 + 7(𝑛 − 1) = 2008 ⇒ 𝑛 = 286

Example 2.8: Finding the common difference


Find the common difference in each part below, if the given sequences are arithmetic.
A. The 14𝑡ℎ term is 100, and the first term is 9.
B. The 9𝑡ℎ term is 247, and the first term is 4.

Part A
In the 14𝑡ℎ term, the common difference has been added 13 times.
𝑇ℎ𝑖𝑟𝑡𝑒𝑒𝑛 𝑡𝑖𝑚𝑒𝑠 𝑡ℎ𝑒 𝑐𝑜𝑚𝑚𝑜𝑛 𝑑𝑖𝑓𝑓𝑒𝑟𝑒𝑛𝑐𝑒 = 100 − 9 = 91
𝐶𝑜𝑚𝑚𝑜𝑛 𝐷𝑖𝑓𝑓𝑒𝑟𝑒𝑛𝑐𝑒 = 7
Part B
In the 9𝑡ℎ term, the common difference has been added 8 times.
𝐸𝑖𝑔ℎ𝑡 𝑡𝑖𝑚𝑒𝑠 𝑡ℎ𝑒 𝑐𝑜𝑚𝑚𝑜𝑛 𝑑𝑖𝑓𝑓𝑒𝑟𝑒𝑛𝑐𝑒 = 247 − 4 = 243
243 240 3 3
𝐶𝑜𝑚𝑚𝑜𝑛 𝐷𝑖𝑓𝑓𝑒𝑟𝑒𝑛𝑐𝑒 = = + = 30
8 8 8 8

Example 2.9
What is the positive difference between the 2000𝑡ℎ term and the 2005𝑡ℎ term of the arithmetic sequence

P a g e 34 | 274
Get all the files at: https://bit.ly/azizhandouts
Aziz Manva (azizmanva@gmail.com)

−8, −2,4,10, . ..? (MathCounts 2005 State Sprint)

We want five times the common difference


= 5𝑑 = 5(10 − 4) = 5(6) = 30

Example 2.10: Multi-Step


A. The fifth term of an arithmetic sequence is two, and the second term of a sequence is five. Find the
seventh term of the sequence.
B. The seventh term of an arithmetic sequence is 3, and the fourth term is 7. Find the hundredth term.
C. In an arithmetic sequence, 𝑡𝑝 = 2𝑞, 𝑡𝑞 = 3𝑝 where 𝑝 is the largest single digit prime number and 𝑞 is the
smallest two-digit prime number. Find 𝑡𝑟 , where 𝑟 is the largest two-digit prime number.

Part A
𝑡1 , 𝑡2 = 5, 𝑡3 , 𝑡4 , 𝑡5 = 2, 𝑡6 , 𝑡7 =?
Going from the second term to the fifth term means adding the common difference thrice.
𝑇ℎ𝑟𝑒𝑒 𝑡𝑖𝑚𝑒𝑠 𝑡ℎ𝑒 𝑐𝑜𝑚𝑚𝑜𝑛 𝑑𝑖𝑓𝑓𝑒𝑟𝑒𝑛𝑐𝑒 = 2 − 5 = −3
3
𝐶𝑜𝑚𝑚𝑜𝑛 𝐷𝑖𝑓𝑓𝑒𝑟𝑒𝑛𝑐𝑒 = − = −1
3
To find the seventh term, add the common difference to the fifth term twice:
𝑎7 = 2 + 2(−1) = 0
Part B
𝑡4 = 7, 𝑡5 , 𝑡6 , 𝑡7 = 3
𝑇ℎ𝑟𝑒𝑒 𝑡𝑖𝑚𝑒𝑠 𝑡ℎ𝑒 𝑐𝑜𝑚𝑚𝑜𝑛 𝑑𝑖𝑓𝑓𝑒𝑟𝑒𝑛𝑐𝑒 = 3 − 7 = −4
4
𝐶𝑜𝑚𝑚𝑜𝑛 𝑑𝑖𝑓𝑓𝑒𝑟𝑒𝑛𝑐𝑒 = −
3
𝑡4 → 𝑡100 ⇒ 𝐴𝑑𝑑 𝑡ℎ𝑒 𝑐𝑜𝑚𝑚𝑜𝑛 𝑑𝑖𝑓𝑓𝑒𝑟𝑒𝑛𝑐𝑒 96 𝑡𝑖𝑚𝑒𝑠
4
𝑡100 = 7 + (96) (− ) = 7 − 128 = −121
3
Part C
𝑝 = 7, 𝑞 = 11 ⇒ 𝑡7 = 22, 𝑡11 = 21
11 − 7 = 4
𝐹𝑜𝑢𝑟 𝑡𝑖𝑚𝑒𝑠 𝑡ℎ𝑒 𝑐𝑜𝑚𝑚𝑜𝑛 𝑑𝑖𝑓𝑓𝑒𝑟𝑒𝑛𝑐𝑒 = 21 − 22 = −1
1
𝐶𝑜𝑚𝑚𝑜𝑛 𝑑𝑖𝑓𝑓𝑒𝑟𝑒𝑛𝑐𝑒 = −
4
𝑟 = 97 ⇒ 𝑡𝑟 = 𝑡97
97 − 11 = 86
1 43
𝑡97 = 21 + (86) (− ) = 21 − = 21 − 21.5 = −0.5
4 2

Example 2.11
What is the 5th term of an arithmetic sequence of 20 terms with first and last terms of 2 and 59, respectively?
(MathCounts 2009 Chapter Countdown)

d=(59-2)/19=3
5th Term=14

Example 2.12
Four primes 𝑎, b, c and d form an increasing arithmetic sequence with a>5 and common difference 6. What is

P a g e 35 | 274
Get all the files at: https://bit.ly/azizhandouts
Aziz Manva (azizmanva@gmail.com)

the ones digit of a? (MathCounts 2009 State Sprint)

Consider possible units digit of a prime.


1,7,3,9
3,9,5,1
7,3,9,5
9,5,1,7

Only the first is possible.


a=11

Example 2.13
The sequence of integers in the row of squares and in each of the two columns of
squares form three distinct arithmetic sequences. What is the value of N?
(MathCounts 2005 Chapter Sprint)

-7

B. Remainders

Example 2.14
A. 12,17,22,27, …
B. 31,28,25,22 …

12,17,22,27, … ⇒ 10 + 2(𝑚𝑜𝑑 5)
31,28,25,22 … ⇒ 33 − 2(𝑚𝑜𝑑 3)

Example 2.15
Laila writes a list of numbers. Her first number is 4. Each number after the first is 7 more than the previous
number. Which of the following numbers appears in Laila's list? (CEMC Gauss 7 2020/11)
A. 45
B. 46
C. 47
D. 48
E. 49

The numbers that Laila writes form an arithmetic sequence with first term 4, and common difference 7:
4,11,18,25, …
Note that each number is:
4(𝑚𝑜𝑑 7)
Of the options, the only one which is 4(𝑚𝑜𝑑 7) is Option B.
Hence, option B.

Example 2.16
For the arithmetic sequence 1000, 987, 974, 961,... what is the least positive integer in the sequence?
(MathCounts 2007 Warm-Up 16)

P a g e 36 | 274
Get all the files at: https://bit.ly/azizhandouts
Aziz Manva (azizmanva@gmail.com)

Difference is 13. One less than 1001, which is a multiple of 13.


Hence, 12.

Example 2.17
An arithmetic sequence has first term 7, and common difference 4. Identify the:
A. largest two-digit number in the sequence (not the term number)
B. smallest four-digit number in the sequence (not the term number)

Part A
The terms of the sequence are
{7,11,15,19,23, … } = {4 × 1 + 3, 4 × 2 + 3,4 × 3 + 3}
Each of the terms has remainder 3 when divided when by 4. That is each is:
3(𝑚𝑜𝑑 4) ≡ −1(𝑚𝑜𝑑 4)
Hence, identify the largest two-digit number which has remainder 3 when divided by 4:
100 → 𝑅𝑒𝑚𝑎𝑖𝑛𝑑𝑒𝑟 𝑍𝑒𝑟𝑜 ⇒ 99 → 𝑅𝑒𝑚𝑎𝑖𝑛𝑑𝑒𝑟 3
Part B
As seen above, the number has to be one less than a multiple of 4, which gives
1003

Example 2.18
A. What is the smallest term that is at least six digits of the sequence 12, 17,22,27?
B. What is the smallest term greater than 520 of the sequence 17, 20, 23, 26, …?
C. What is the largest three-digit term of the sequence 123, 134,145, …? What is the largest four-digit
term?

Part A
This sequence increases by 5 every time.
The remainder when I divide any term of this sequence by 5 is always:
2
The smallest number that is at least six digits is
100,000
And the smallest number that is at least six digits, and has remainder 2, when divided by 5 is
100,002
Part B
Every term of the sequence has remainder 2 when divided by 3.
520 → 𝑆𝑢𝑚 𝑜𝑓 𝐷𝑖𝑔𝑖𝑡𝑠 = 7
Hence, 520 is 1 more than a multiple of 3.
We want our number to be 2 more than a multiple, which will be:
521 → 𝑆𝑢𝑚 𝑜𝑓 𝐷𝑖𝑔𝑖𝑡𝑠 = 8
Part C
123, 134,145, …
Every term of the sequence has remainder 2, when divided by 11.
Recall that
1001 = 7 × 11 × 13 ⇒ 1003 𝑖𝑠 𝑎 𝑡𝑒𝑟𝑚 ⇒ 1003 − 11 = 992 𝑖𝑠 𝑙𝑎𝑟𝑔𝑒𝑠𝑡 𝑡ℎ𝑟𝑒𝑒 𝑑𝑖𝑔𝑖𝑡 𝑡𝑒𝑟𝑚
Consider 9999:
(9 + 9) − (9 + 9) = 18 − 18 = 9999 𝑖𝑠 𝑑𝑖𝑣𝑖𝑠𝑖𝑏𝑙𝑒 𝑏𝑦 11
9999 − 11 = 9988 𝑖𝑠 𝑑𝑖𝑣𝑖𝑠𝑖𝑏𝑙𝑒 𝑏𝑦 11
9988 + 2 ℎ𝑎𝑠 𝑟𝑒𝑚𝑎𝑖𝑛𝑑𝑒𝑟 2 𝑤ℎ𝑒𝑛 𝑑𝑖𝑣𝑖𝑑𝑒𝑑 𝑏𝑦 11

P a g e 37 | 274
Get all the files at: https://bit.ly/azizhandouts
Aziz Manva (azizmanva@gmail.com)

Example 2.19
Find the sum of the smallest positive term, and the largest negative term of the sequence:
1004,991,978, …

1001 = 7 × 11 × 13 ⇒ 1004 = 1001 + 3 = 13𝑛 + 3


Every term of this sequence has remainder 3, when divided by 13.

The smallest positive term


=3
The largest negative term
= 3 − 13 = −10
Sum of the smallest positive term and largest negative term
= 3 − 10 = −7

Example 2.20
Find the product of the smallest positive term, and the largest negative term of the sequence
1721 1718 1715
, , …
4 4 4

1721 → 𝑆𝑢𝑚 𝑜𝑓 𝐷𝑖𝑔𝑖𝑡𝑠 = 1 + 7 + 2 + 1 = 11 = 9 + 2


Whenever the numerator of a term is divided by 3, the remainder is always 2.

The smallest positive term


2
=
4
The largest negative term
2 3 1
= − =−
4 4 4
Product of the smallest positive term and largest negative term
2 1 1
= ( ) (− ) = −
4 4 8

Example 2.21
Shashank has 500 apples. He eats 7 apples every day. Identify for how many days he has the full quota of apples,
and how many apples he will eat on the day his apples actually get over.

500
= 71 𝑅3 ⇒ 72 𝐷𝑎𝑦𝑠, 3 𝐴𝑝𝑝𝑙𝑒𝑠 𝑜𝑛 𝑡ℎ𝑒 𝑙𝑎𝑠𝑡 𝑑𝑎𝑦
7

Example 2.22
Jane runs a lemonade stand. She sold 20 glasses of lemonade every day last week, which was the first week she
started. This week, she sold 27 glasses of lemonade each day. Next week, she plans to sell 34 glasses of
lemonade each day. If this pattern continues, what is the first week that she will sell in excess of 1000 glasses of
lemonade in a day. (Jane is hard-working, and works seven days a week).

The number of glasses of lemonade that Jane sold on a daily basis was:
20
⏟ , 27 ⏟ , 34 ⏟ ,…
𝐹𝑖𝑟𝑠𝑡 𝑆𝑒𝑐𝑜𝑛𝑑 𝑇ℎ𝑖𝑟𝑑
𝑊𝑒𝑒𝑘 𝑊𝑒𝑒𝑘 𝑊𝑒𝑒𝑘

P a g e 38 | 274
Get all the files at: https://bit.ly/azizhandouts
Aziz Manva (azizmanva@gmail.com)

Note that the above is an arithmetic sequence with first term 20, and common difference 7.
Also, note that every term has remainder 6, when divided by 7.
20 = 2 × 7 + 6, 27 = 3 × 7 + 6, 34 = 4 × 7 + 6

1001 = 7 × 11 × 13 = 7 × 143
1001 + 6 = 1007
143 × 7 + 6 = 1007
Hence, the sequence becomes:
𝟐 × 7 +6,
⏟ 𝟑 × 7 + 6,
⏟ 𝟒 × 7 + 6,…,⏟
⏟ 𝟏𝟒𝟑 × 7 + 6
20 27 34 1007
{2,3,4, … ,143} ⇒ 143 − 2 + 1 = 142 𝑁𝑢𝑚𝑏𝑒𝑟𝑠

Example 2.23: Calendar


Amanda plays chess every 3𝑟𝑑 day. That is, if she has played chess on Monday, she plays it again on Thursday.
Today is Monday, the 1st of Jan on a non-leap year and she just played chess.
A. When will she play chess again on a Monday?
B. When will she play chess again on the first day of the month?

Part A
Monday’s repeat every seven days:
𝑀𝑜𝑛
⏟,⏟ 𝑇𝑢𝑒 , 𝑊𝑒𝑑
⏟,⏟ 𝑇ℎ𝑢 , 𝐹𝑟𝑖
⏟ , 𝑆𝑎𝑡
⏟ , 𝑆𝑢𝑛
⏟ , 𝑀𝑜𝑛
⏟ ,…
𝑥 𝑥+1 𝑥+2 𝑥+3 𝑥+4 𝑥+5 𝑥+6 𝑥+7
Chess is played every third day:

She played chess on a Monday. She will again play chess on a Monday after:
𝐿𝐶𝑀(3,7) = 21 𝑑𝑎𝑦𝑠
She will play chess again on a Monday on
1 + 21 = 22𝑛𝑑 𝐽𝑎𝑛
Part B
1,4,7, …
Note:
➢ The above is an arithmetic sequence with first term 1, and common difference 3.
➢ When any term of the above sequence is divided by 3, the remainder is 1.

February:
1𝑠𝑡 𝐹𝑒𝑏 = 32𝑛𝑑 𝑑𝑎𝑦 = 𝑅𝑒𝑚𝑎𝑖𝑛𝑑𝑒𝑟 2 𝑤ℎ𝑒𝑛 𝑑𝑖𝑣𝑖𝑑𝑒𝑑 𝑏𝑦 3
2𝑛𝑑 𝐹𝑒𝑏 = 33𝑟𝑑 𝑑𝑎𝑦 = 𝑅𝑒𝑚𝑎𝑖𝑛𝑑𝑒𝑟 0 𝑤ℎ𝑒𝑛 𝑑𝑖𝑣𝑖𝑑𝑒𝑑 𝑏𝑦 3
3𝑟𝑑 𝐹𝑒𝑏 = 34𝑡ℎ 𝑑𝑎𝑦 = 𝑅𝑒𝑚𝑎𝑖𝑛𝑑𝑒𝑟 1 𝑤ℎ𝑒𝑛 𝑑𝑖𝑣𝑖𝑑𝑒𝑑 𝑏𝑦 3
Mar:
1𝑠𝑡 𝑀𝑎𝑟 = 1𝑠𝑡 𝐹𝑒𝑏 + 28 = 32 + 28 = 60 ⇒ 𝑅𝑒𝑚𝑎𝑖𝑛𝑑𝑒𝑟 0 𝑤ℎ𝑒𝑛 𝑑𝑖𝑣𝑖𝑑𝑒𝑑 𝑏𝑦 3
2𝑛𝑑 𝑀𝑎𝑟 = 61 ⇒ 𝑅𝑒𝑚𝑎𝑖𝑛𝑑𝑒𝑟 1 𝑤ℎ𝑒𝑛 𝑑𝑖𝑣𝑖𝑑𝑒𝑑 𝑏𝑦 3
Apr:
1𝑠𝑡 𝐴𝑝𝑟 = 1𝑠𝑡 𝑀𝑎𝑟 = 60 + 31 = 91 ⇒ 𝑅𝑒𝑚𝑎𝑖𝑛𝑑𝑒𝑟 1 𝑤ℎ𝑒𝑛 𝑑𝑖𝑣𝑖𝑑𝑒𝑑 𝑏𝑦 3
The dates on which she plays chess are:
1,4,7, … ,31 , ⏟
⏟ 𝟑, 6, … ,27 , ⏟
𝟐, 5, … ,29 , ⏟
𝟏
𝐽𝑎𝑛𝑢𝑎𝑟𝑦 𝐹𝑒𝑏𝑟𝑢𝑎𝑟𝑦 𝑀𝑎𝑟𝑐ℎ 𝐴𝑝𝑟𝑖𝑙

P a g e 39 | 274
Get all the files at: https://bit.ly/azizhandouts
Aziz Manva (azizmanva@gmail.com)

C. Double Sequences

Example 2.24
Consider the sequences
𝑋 = 6,11,16, … , 𝑌 = 6,10,14, …
A. Find the sequence of numbers which are present in both 𝑋 and 𝑌
B. Find the smallest three-digit term of the sequence that you found in Part A.

𝑋 = 𝟔, 11,16,21, 𝟐𝟔, …
𝑌 = 𝟔, 10,14,18,22, 𝟐𝟔, …
The first common term is 6.
Every time the value increases by 𝐿𝐶𝑀(4,5) = 20. Hence, 20 is the common difference.

Hence, the sequence of numbers which is present in both X and Y is:


6,26,46,66,86,106 ⇒ 106 𝑖𝑠 𝑡ℎ𝑒 𝑠𝑚𝑎𝑙𝑙𝑒𝑠𝑡 𝑡ℎ𝑟𝑒𝑒 𝑑𝑖𝑔𝑖𝑡 𝑡𝑒𝑟𝑚 𝑜𝑓 𝑡ℎ𝑒 𝑠𝑒𝑞𝑢𝑒𝑛𝑐𝑒

Example 2.25
Magical mushrooms come in packs that give always you three more mushrooms than a multiple of four. Magical
toadstools come in packs that always give you 2 more mushrooms than a multiple of five. I buy a pack of each,
and get the same number of mushrooms, and toadstools. If the total number of mushrooms and toadstools I
have is less than 1000, and I do not have a negative number of mushrooms and toadstools, what is the possible
number of values for the number of mushrooms I have?

𝑀𝑢𝑠ℎ𝑟𝑜𝑜𝑚𝑠 = 3,7,11,15, …
𝑇𝑜𝑎𝑑𝑠𝑡𝑜𝑜𝑙𝑠 = 2,7,12,17, …
𝐹𝑖𝑟𝑠𝑡 𝑐𝑜𝑚𝑚𝑜𝑛 𝑇𝑒𝑟𝑚 = 7
Each time, the sequence increases by 𝐿𝐶𝑀(4,5) = 20.
1000
𝑀𝑎𝑥 𝑛𝑢𝑚𝑏𝑒𝑟 𝑜𝑓 𝑚𝑢𝑠ℎ𝑟𝑜𝑜𝑚𝑠/𝑡𝑜𝑎𝑑𝑠𝑡𝑜𝑜𝑙𝑠 = = 500
2
The possible values for both mushrooms and toadstools to be the same is:
7,27,47,67, … ,487 ⇒ 20 × 𝟎 + 7,20 × 𝟏 + 7, … ,20 × 𝟐𝟒 + 7
{0,1,2, … ,24} ⇒ 25 𝑉𝑎𝑙𝑢𝑒𝑠

Example 2.26
Ronald is a creature of habit. Every three days, he plays golf in the morning. Every five days, he goes horse-
riding in the evening. Today is Wednesday, and he played golf today morning and will go horse-riding today
evening. After 500 days, what will be the day when he plays golf and goes horse-riding on the same day.

Suppose today is day 0.


The days on which he plays golf will be:
3,6,9,12, …

The days on which he goes horse-riding will be:


5,10,15,20, …

The days on which he will do both activities will be 𝐿𝐶𝑀(3,5) = 15


15,30,45,60, …

Hence, we want the smallest multiple of 15 that is greater than 500.

P a g e 40 | 274
Get all the files at: https://bit.ly/azizhandouts
Aziz Manva (azizmanva@gmail.com)

450 𝑖𝑠 𝑎 𝑚𝑢𝑙𝑡𝑖𝑝𝑙𝑒 𝑜𝑓 15
450 + 60 = 510 𝑖𝑠 𝑎 𝑚𝑢𝑙𝑡𝑖𝑝𝑙𝑒 𝑜𝑓 15

We want to know the day for


𝑊𝑒𝑑𝑛𝑒𝑠𝑑𝑎𝑦 + 510 = 𝑊𝑒𝑑𝑛𝑒𝑠𝑑𝑎𝑦 + 504
⏟ + 6 = 𝑇𝑢𝑒𝑠𝑑𝑎𝑦
72 𝑊𝑒𝑒𝑘𝑠

D. Patterns

Example 2.27
1 3 1
Let 𝑎𝑛 be the first negative term in the sequence {100 4 , 99 4 , 99 4 … }. Find 𝑎𝑛 + 𝑛.

1 3 1 1
𝑎 = 100 , 𝑑 = 99 − 100 = −
4 4 4 2
First Negative Term
List out a few terms:
1 3 1 3 1
100 , 99 , 99 , 98 , 98 , …
4 4 4 4 4
When the series nears zero, we will get the numbers:
3 1 1 3
, ,− ,−
4 4 4 4
And hence the first negative term is
1
𝑎𝑛 = −
4
Number of Terms
Every number has two terms associated with it
1 1
− , ⏟ 1,2, … 100 , 100 ⇒ 𝑇𝑜𝑡𝑎𝑙 𝑁𝑢𝑚𝑏𝑒𝑟 𝑜𝑓 𝑇𝑒𝑟𝑚𝑠 = 1 + 200 + 1 = 202 ⇒ 𝑛 = 202
⏟ 4 100×2=200 𝑇𝑒𝑟𝑚𝑠 ⏟ 4
1 𝑇𝑒𝑟𝑚 1 𝑇𝑒𝑟𝑚

Finding the sum


1 3
𝑎𝑛 + 𝑛 = − + 202 = 201
4 4
E. Common Difference

Example 2.28
Let 𝑎1 , 𝑎2 , 𝑎3 , . .. be an increasing arithmetic sequence of integers. If 𝑎4 𝑎5 = 13, what is 𝑎3 𝑎6 ? (AOPS Alcumus,
Algebra, Arithmetic Sequences)

We have been given:


𝑎4 𝑎5 = 13
(This is an example of a Diophantine equation, where the number of variables is more than the number of
equations, and we solve only in integers).
The LHS is a product of two integers. Hence, the RHS must also be a product of two integers. There are exactly
two possible cases:
13 = (1)(13) = (−1)(−13)
𝐶𝑎𝑠𝑒 𝐼: 𝐶𝑜𝑚𝑚𝑜𝑛 𝑑𝑖𝑓𝑓𝑒𝑟𝑒𝑛𝑐𝑒: 13 − 1 = 12
−11
⏟,⏟ 1 , 13 ⏟ ⇒ 𝑎3 𝑎6 = (−11)(25) = −275
⏟ , 25
𝑎3 𝑎4 𝑎5 𝑎6

P a g e 41 | 274
Get all the files at: https://bit.ly/azizhandouts
Aziz Manva (azizmanva@gmail.com)

𝐶𝑎𝑠𝑒 𝐼𝐼: 𝐶𝑜𝑚𝑚𝑜𝑛 𝑑𝑖𝑓𝑓𝑒𝑟𝑒𝑛𝑐𝑒: − 1 − (−13) = −1 + 13 = 12


−25
⏟ , −13
⏟ , −1 ⏟ ⇒ 𝑎3 𝑎6 = (−25)(11) = −275
⏟ , 11
𝑎3 𝑎4 𝑎5 𝑎6

Example 2.29
Two arithmetic sequences 𝐴 and 𝐵 both begin with 30 and have common differences of absolute value 10, with
sequence A increasing and sequence 𝐵 decreasing. What is the absolute value of the difference between the
51st term of sequence 𝐴 and the 51st term of sequence 𝐵? (MathCounts 2009 State Target)

30
⏟ ,⏟
30 + 10 × 1 , ⏟
30 + 10 × 2 , … , ⏟
30 + 10 × 50
1𝑠𝑡 2𝑛𝑑 𝑇𝑒𝑟𝑚 3𝑟𝑑 𝑇𝑒𝑟𝑚 51𝑠𝑡 𝑇𝑒𝑟𝑚
𝑇𝑒𝑟𝑚 𝑜𝑓↑ 𝑆𝑒𝑞 𝑜𝑓↑ 𝑆𝑒𝑞 𝑜𝑓↑ 𝑆𝑒𝑞
30
⏟ ,⏟
30 − 10 × 1 , ⏟
30 − 10 × 2 , … , ⏟
30 − 10 × 50
1𝑠𝑡 2𝑛𝑑 𝑇𝑒𝑟𝑚 3𝑟𝑑 𝑇𝑒𝑟𝑚 51𝑠𝑡 𝑇𝑒𝑟𝑚
𝑇𝑒𝑟𝑚 𝑜𝑓↓ 𝑆𝑒𝑞 𝑜𝑓↓ 𝑆𝑒𝑞 𝑜𝑓↓ 𝑆𝑒𝑞

30 − 500, 30 + 500

30 + 500 − (30 − 500) = 30 + 500 − 30 + 500 = 1000


Shortcut
𝐴 𝑖𝑛𝑐𝑟𝑒𝑎𝑠𝑒𝑠 𝑏𝑦 500
𝐵 𝑑𝑒𝑐𝑟𝑒𝑎𝑠𝑒𝑠 𝑏𝑦 500
𝑇𝑜𝑡𝑎𝑙 𝐷𝑖𝑓𝑓𝑒𝑟𝑒𝑛𝑐𝑒 = 1000

Example 2.30
In 1960, there were 450,000 cases of measles reported in the U.S. In 1996, there were 500 cases reported. How
many cases of measles would have been reported in 1987 if the number of cases reported from 1960 to 1996
decreased linearly? (MathCounts 2005 State Target)

𝑇𝑜𝑡𝑎𝑙 𝑌𝑒𝑎𝑟𝑠 = 1996 − 1960 = 36


𝑌𝑒𝑎𝑟𝑠 𝐶𝑜𝑣𝑒𝑟𝑒𝑑 = 1987 − 1960 = 27
27 3
𝑅𝑎𝑡𝑖𝑜 = =
36 4
3 3
(450,000 − 500) = × 495,500 = 337,125
4 4

Example 2.31
In each blank below a single digit is inserted such that the following six three-digit numbers, in this order, form
an arithmetic sequence:
1 __ __, __ __ 9, 2 __ 2, __ 6 __ , 2 __ __ , __ 3 __
What is the value of the next number in the sequence? (MathCounts 2004 State Team)

The second number must start with 1, and the fourth number must start with 2:
1 __ __, 1 __ 9, 2 __ 2, 26 __ , 2 __ __ , __ 3 __

From the second to the third number, the units digit changes from 9 to 2.
9 + 𝑢 = 12 ⇒ 𝑢 = 3

Hence, the units digit of the common difference must be 3.


Hence, we can write the units digit of each number. And we can also write the units digit of the next number in

P a g e 42 | 274
Get all the files at: https://bit.ly/azizhandouts
Aziz Manva (azizmanva@gmail.com)

the sequence:
1 __ 6, 1 __ 9, 2 __ 2, 265 , 2 __ 8, __ 31,

We now have the middle digit of 265, and the middle digit __ 31.
➢ There is one number between them, so twice the common difference must be added to get from one
number to the other.
➢ There is a carryforward in the 𝑢𝑛𝑖𝑡𝑠 𝑑𝑖𝑔𝑖𝑡 from 2__8 to __31, so 1 ten comes from there.
➢ The middle digit changes from 6 to 3, so there is a carryforward in the 𝑡𝑒𝑛𝑠 𝑑𝑖𝑔𝑖𝑡:
6 + 1 + 2𝑡 = 13 ⇒ 𝑡 = 3
6 + 1 + 2𝑡 = 23 ⇒ 𝑡 = 8

Try 𝑡 = 3, 𝑢 = 3:
166, 199, 232, 265, 298, 331, 𝟑𝟔𝟒

Example 2.32
The first term of an arithmetic sequence is 1, another term of the sequence is 91 and all of the terms of the
sequence are integers. How many distinct arithmetic sequences meet these three conditions? (MathCounts
2008 State Sprint)

First term is one and another term (which must be second term or higher) is 91. Hence, it is an increasing
sequence.
Difference between two terms
= 91 − 1 = 90

Common difference must be a factor of 90:


𝜏(90) = 𝜏(2 × 32 × 5) = (1 + 1)(2 + 1)(1 + 1) = 12

2.33: Common Difference Property


➢ If the difference between two terms of a sequence is constant, then the sequence is an arithmetic
sequence.
➢ 𝐶𝑜𝑛𝑣𝑒𝑟𝑠𝑒: If a sequence is arithmetic, then the difference between two terms of a sequence is constant.

Example 2.34
Show that the sequence is arithmetic. Find the common difference as well.
A. 7,19,31,43, …
1 7 5
B. , ,
3 12 6
C. 𝑥 + 3,2𝑥 + 1,3𝑥 − 1

Part A
43 − 31 = 31 − 19 = 19 − 7 = 12
Part B

7 1 7 4 3 1
− = − = =
12 3 12 12 12 4
5 7 10 7 3 1
− = − = =
6 12 12 12 12 4
Part C
2𝑥 + 1 − (𝑥 + 3) = 𝑥 − 2

P a g e 43 | 274
Get all the files at: https://bit.ly/azizhandouts
Aziz Manva (azizmanva@gmail.com)

3𝑥 − 1 − (2𝑥 + 1) = 𝑥 − 2

Example 2.35
A. 3𝑥 + 5,2𝑥 + 1,4𝑥 + 7 is an arithmetic sequence. Find the common difference and the value of each term

Since the sequence is arithmetic, the difference between two terms must be the same:
10
2𝑥 + 1 − (3𝑥 + 5) = 4𝑥 + 7 − (2𝑥 + 1) ⇒ −𝑥 − 4 = 2𝑥 + 6 ⇒ 3𝑥 = −10 ⇒ 𝑥 = −
3

Substitute into the expression for the common difference to find the value:
10 10 12 2
−𝑥 − 4 = − (− ) − 4 = − =−
3 3 3 3
10 20 18 2
2𝑥 + 6 = 2 (− ) + 6 = − + =−
3 3 3 3
Find each term:
10
1𝑠𝑡 𝑇𝑒𝑟𝑚 = 3𝑥 + 5 = 3 (− ) + 5 = −10 + 5 = −5
3
10 20 3 17
2𝑛𝑑 𝑇𝑒𝑟𝑚 = 2𝑥 + 1 = 2 (− ) + 1 = − + = −
3 3 3 3
10 40 21 19
3𝑟𝑑 𝑇𝑒𝑟𝑚 = 4𝑥 + 7 = 4 (− ) + 7 = − + =−
3 3 3 3

2.36: Extension of Common Difference Property

Example 2.37
A. The second and fifth terms of an arithmetic sequence are 17 and 19, respectively. What is the eighth
term? (MathCounts 2009 State Countdown)
B. The tenth term of an arithmetic sequence is 50. The fourteen term is 70. What is the twelfth term?
C. Show that the 2𝑛𝑑 term, the 5𝑡ℎ term and the 8𝑡ℎ term of an arithmetic sequence themselves form an
arithmetic sequence.
D. The eleventh and twenty-first terms of an arithmetic sequence are 21 and 11, respectively. What is the
thirty-first term?
E. Find the 19𝑡ℎ term of the arithmetic sequence with 𝑝𝑡ℎ term 5, and 𝑞 𝑡ℎ term 20, given that 𝑝 is the
largest single digit prime number, and the 𝑞 is the smallest two-digit prime number.

Part A
17
⏟ , 17
⏟ + 1(2) , 17
⏟ + 2(2)
2𝑛𝑑 5𝑡ℎ 8𝑡ℎ
𝑇𝑒𝑟𝑚 𝑇𝑒𝑟𝑚 𝑇𝑒𝑟𝑚
Part B
Twelfth term is exactly between the tenth term and the fourteenth term.
So, it is the average of the terms
50 + 70 120
= = = 60
2 2
Part C

𝑎 ,⏟𝑎 + 3𝑑 , ⏟
𝑎 + 6𝑑
2𝑛𝑑 5𝑡ℎ 8𝑡ℎ
𝑇𝑒𝑟𝑚 𝑇𝑒𝑟𝑚 𝑇𝑒𝑟𝑚
The above is an arithmetic sequence with first term 𝑎 and common difference 3𝑑.

P a g e 44 | 274
Get all the files at: https://bit.ly/azizhandouts
Aziz Manva (azizmanva@gmail.com)

Part D
21
⏟ , 11
⏟ , ⏟
1
11𝑡ℎ 21𝑠𝑡 31𝑠𝑡
𝑇𝑒𝑟𝑚 𝑇𝑒𝑟𝑚 𝑇𝑒𝑟𝑚
Part E

5 , 20
⏟ , 50

7𝑡ℎ 11𝑡ℎ 19𝑡ℎ
𝑇𝑒𝑟𝑚 𝑇𝑒𝑟𝑚 𝑇𝑒𝑟𝑚

2.38: Arithmetic Mean Property and its converse


Any term of an arithmetic sequence is the arithmetic mean of the terms that precede and follow it.
𝑎𝑛−1 + 𝑎𝑛+1
𝑎𝑛 =
2

𝐶𝑜𝑛𝑣𝑒𝑟𝑠𝑒: If the general term of a sequence is the arithmetic mean of the terms that precede and follow it, then
that sequence is an arithmetic sequence.

The arithmetic mean property has both theoretical and practical importance. We can apply it:
➢ To find missing values when we know that a sequence is arithmetic.
➢ To prove that a sequence is arithmetic

Example 2.39: Showing that the property holds


Show that the arithmetic mean property holds in the following arithmetic sequence:
1 2 3
3 ,4 ,5
5 5 5

Find the average of the first and the last term, and show that it is equal to the middle term:
1 3
1 3 3 +5
𝐴𝑣𝑔 (3 , 5 ) = 5 5 = 8 4 × 1 = 44 × 1 = 22 = 4 2 = 𝑀𝑖𝑑𝑑𝑙𝑒 𝑇𝑒𝑟𝑚
5 5 2 5 2 5 2 5 5

Example 2.40
The terms in each part below form an arithmetic sequence. Find the values of the variables.
A. 10, 17, 𝑎
B. 𝑏, 22, 15
C. 20, 𝑦, 70
D. 10, 𝑧, 15
E. 2, 𝑝, 23
F. 22 , 𝑞, 24

17 − 10 = 7 ⇒ 𝑎 = 17 + 7 = 24
15 − 22 = −7 ⇒ 𝑏 = 22 − (−7) = 22 + 7 = 29

20 + 70 90
𝑦= = = 45
2 2
10 + 15 25
𝑧= = = 12.5
2 2
2 + 8 10
𝑝= = =5
2 2

Example 2.41

P a g e 45 | 274
Get all the files at: https://bit.ly/azizhandouts
Aziz Manva (azizmanva@gmail.com)

A. What is the integer value of 𝑦 in the arithmetic sequence 22 , 𝑦, 24? (MathCounts 2007 State Countdown)
B. What is the value of 𝑦 in the arithmetic sequence 𝑦+6, 12, 𝑦? (MathCounts 2005 Warm-Up 9)
C. The first term of an arithmetic sequence is 5. The third term is 12. Find the second term.

Part A
4 + 16 20
𝑦= = = 10
2 2
Part B
𝑦+6+𝑦
12 = ⇒ 24 = 2𝑦 + 6 ⇒ 18 = 2𝑦 ⇒ 𝑦 = 9
2
Part C
5 + 12 17
𝑡2 = = = 8.5
2 2

F. Finding Numbers

Example 2.42
Find the numbers, 𝑖𝑓 𝑝𝑜𝑠𝑠𝑖𝑏𝑙𝑒, in each situation below:
A. Three consecutive integers that add up to zero
B. Three consecutive integers that add up to 45
C. Three consecutive odd integers that add up to 81
D. Three consecutive even integers that add up to 72
E. Four consecutive odd integers that add up to 104
F. Four consecutive integers that add up to 70
G. Four consecutive even integers that add up to 72

We use the property stated above. The average of 72


= 24 ⇒ 22,24,26
the consecutive integers will be the middle term of 3
the arithmetic sequence. Part E
Part A 104
= 26 ⇒ 23,25, 𝟐𝟔, 27,29
0 4
= 0 ⇒ 𝑁𝑢𝑚𝑏𝑒𝑟𝑠 𝑎𝑟𝑒 − 1,0,1
3 Part F
Part B 70
= 17.5 ⇒ 16,17, 𝟏𝟕. 𝟓, 18,19
45 4
= 15 ⇒ 14,15,16
3 Part G
Part C 72
⇒ 14,16, 𝟏𝟖, 20,22 = 14 + 16 + 20 + 22 = 72
81 4
= 27 ⇒ 25,27,29 ⇒ 𝑵𝒐𝒕 𝑪𝒐𝒏𝒔𝒆𝒄𝒖𝒕𝒊𝒗𝒆
3
14 + 16 + 18 + 20 = 68
Part D 16 + 18 + 20 + 22 = 76
There are no valid solutions for this Part.

Example 2.43
A. The sum of three consecutive integers is 27. What is the product of the integers? (MathCounts 2006
State Countdown)
B. The sum of four positive integers that form an arithmetic sequence is 46. Of all such possible sequences,
what is the greatest possible third term? (Mathcounts 2005 State Sprint)

P a g e 46 | 274
Get all the files at: https://bit.ly/azizhandouts
Aziz Manva (azizmanva@gmail.com)

Part A
8 × 9 × 10 = 720
Part B
46 23
= = 11.5
4 2
Take different values of 𝑑. As the value of 𝑑 increases, so will the value of the third term:
𝑑 = 1 ⇒ 10,11, 𝟏𝟏. 𝟓, 12,13
𝑑 = 3 ⇒ 7,10, 𝟏𝟏. 𝟓, 13,16
𝑑 = 5 ⇒ 4,9, 𝟏𝟏. 𝟓, 14,19
𝑑 = 7 ⇒ 1,8, 𝟏𝟏. 𝟓, 15,22
When 𝑑 increases to 9, the smallest term becomes negative, which we cannot allow:
𝑑 = 9 ⇒ −𝟐, 7,16,25 ⇒ 𝑪𝒐𝒏𝒕𝒓𝒂𝒅𝒊𝒄𝒕𝒊𝒐𝒏 ⇒ 𝒅 ≠ 𝟗
Hence, the greatest possible third term occurs when
𝑑 = 5 ⇒ 𝑎3 = 15

Example 2.44: Geometry


Recall that the sum of the interior angles of a polygon with 𝑛 sides is given by
(𝑛 − 2) × 180
A. If the measures of the angles in a triangle form an arithmetic sequence, then find the value of the angle
that forms the middle term.
B. If the measures of the angles in a pentagon form an arithmetic sequence, then find the value of the angle
that forms the middle term.

Part A
The sum of the angles in a triangle is 180°. If the angles form an arithmetic sequence, then the middle term
must be:
180
= 60°
3
Part A
The sum of the angles in a triangle is 540°. If the angles form an arithmetic sequence, then the middle term
must be:
540
= 108°
5

Example 2.45
A. The measures of the angles of a triangle form an arithmetic sequence with a non-zero common
difference. If all of the measures are integer values (when measured in degrees), find the number of
such triangles if the angles cannot all have the same measure. (Two triangles are considered the same if
they have angles with the same measure).
B. Answer the above question if the triangle is acute.

Part A
60,60,60 ⇒ 𝑁𝑜𝑡 𝑉𝑎𝑙𝑖𝑑
𝑑 = 1 ⇒ 59,60,61
𝑑 = 2 ⇒ 58,60,62
.
.
.
𝑑 = 59 ⇒ 1,60,119

P a g e 47 | 274
Get all the files at: https://bit.ly/azizhandouts
Aziz Manva (azizmanva@gmail.com)

Hence, the total number of triangles is the number of possible values of the common difference:
𝑛(1,2,3, … ,59) = 59

Part B
𝑑 = 1 ⇒ 59,60,61
𝑑 = 2 ⇒ 58,60,62
.
.
.
𝑑 = 29 ⇒ 31,60,89

Hence, the total number of triangles is the number of possible values of the common difference:
𝑛(1,2,3, … ,29) = 29

Example 2.46
One angle of a triangle is 60°. The other two angles have integer measures. Find the number of such triangles if
the other two angles are both even.

First Angle 2° 4° . . . 60°


Second Angle 118° 116° . . . 60°

And hence we have to count the number of numbers in the list:


2,4,6, … ,60
Divide each number by 2 to get:
1,2,3, . . ,30 ⇒ 30 𝑁𝑢𝑚𝑏𝑒𝑟𝑠 ⇒ 𝟑𝟎 𝑻𝒓𝒊𝒂𝒏𝒈𝒍𝒆𝒔

Example 2.47
One angle of a triangle is 60°. The other two angles have integer measures. Find the number of such triangles if
the other angles are both odd.

Use complementary counting

60
⏟ − 30
⏟ = 𝟑𝟎 𝑻𝒓𝒊𝒂𝒏𝒈𝒍𝒆𝒔 𝒘𝒊𝒕𝒉 𝑻𝒘𝒐 𝑶𝒅𝒅 𝑨𝒏𝒈𝒍𝒆𝒔
𝑇𝑜𝑡𝑎𝑙 𝐸𝑣𝑒𝑛
𝑇𝑟𝑖𝑎𝑛𝑔𝑙𝑒𝑠 𝐴𝑛𝑔𝑙𝑒𝑠

First Angle 1° 3° . . . 59°


Second Angle 119° 117° . . . 61°

And hence we have to count the number of numbers in the list:


1,3,5, … ,59
Add 1 to each number to get:
2,4,6, … ,60
Divide each number by 2 to get:
1,2,3, . . ,30 ⇒ 30 𝑁𝑢𝑚𝑏𝑒𝑟𝑠 ⇒ 𝟑𝟎 𝑻𝒓𝒊𝒂𝒏𝒈𝒍𝒆𝒔

Example 2.48
One angle of a triangle is 60°. The other two angles have integer measures. Find the number of such triangles if
the other two angles are both even, and all angles are acute.

P a g e 48 | 274
Get all the files at: https://bit.ly/azizhandouts
Aziz Manva (azizmanva@gmail.com)

First Angle 32° 34° . . . 60°


Second Angle 88° 86° . . . 60°

And hence we have to count the number of numbers in the list:


32,34, … ,60
Divide each number by 2 to get:
16,17, . . ,30 ⇒ 30 − 16 + 1 = 15 𝑁𝑢𝑚𝑏𝑒𝑟𝑠 ⇒ 𝟏𝟓 𝑻𝒓𝒊𝒂𝒏𝒈𝒍𝒆𝒔
G. Other Formulas

Example 2.49
A. The sum of the first 20 positive even integers is also the sum of four consecutive even integers. What is
the largest of these four integers? (MathCounts 2010 State Sprint)
B. If the lengths of the sides of a right-angled triangle with hypotenuse 15 form an arithmetic sequence,
then find the area of the triangle.
C. (𝐶ℎ𝑎𝑙𝑙𝑒𝑛𝑔𝑒) Find the smallest prime that is the fifth term of an increasing arithmetic sequence, all four
preceding terms also being prime. (AIME 1999/1)

Part A
20 × 21
2 + 4 + ⋯ + 40 = 2(1 + 2+. . . +20) = 2 ( ) = 420
2

420
= 105 ⇒ 𝑁𝑢𝑚𝑏𝑒𝑟𝑠 = 102, 104, 106, 108
4
Part B
{3,4,5} × 3 = {9,12,15} 𝑖s a Pythagorean Triplet where the terms form an arithmetic sequence. Since the
question asks for area, there must be a unique answer for the area, and hence the numbers that we have found
will give the correct answer.
1
Area = × 9 × 12 = 54
2
Part C
(Ans = 29)
List the first few primes
2,3,5,7,11,13,17,23,29

2.2 Algebra
A. Basics

2.50: Arithmetic Sequence


An arithmetic sequence with first term 𝑎 has a common difference 𝑑 between successive elements.
𝑎, 𝑎 + 𝑑, 𝑎 + 2𝑑, … , 𝑎 + (𝑛 − 1)𝑑

First term is called 𝑎


Common difference is 𝑑

Some of examples of arithmetic sequences are:


12, 17,22,27, … ⇒ 𝑅𝑢𝑙𝑒 𝑖𝑠: 𝐴𝑑𝑑 5 ⇒ 𝑎 = 12, 𝑑 = 5
31,28,25, … ⇒ 𝑅𝑢𝑙𝑒 𝑖𝑠: 𝑆𝑢𝑏𝑡𝑟𝑎𝑐𝑡 3 ⇒ 𝑎 = 31, 𝑑 = −3

P a g e 49 | 274
Get all the files at: https://bit.ly/azizhandouts
Aziz Manva (azizmanva@gmail.com)

Example 2.51
Below are some arithmetic sequences. Identify the first term and common difference for each.
A. 3,4,5,6,7 …
B. 3,3,3,3,3 …
C. 1,3,5,7,9 …
D. 5,3,1, −1, …
3 5 3
E. 4
, 1, 4 , 2

First Term Common


Difference
3,4,5,6,7 … 𝑎=3 𝑑=1

3,3,3,3,3 … 𝑎=3 𝑑=0


1,3,5,7,9 … 𝑎=1 𝑑=2
5,3,1, −1, … 𝑎=5 𝑑 = −2
3 5 3 3 1
, 1, , 𝑎= 𝑑=
4 4 2 4 4

3 4 3 1 5 5 4 1
1− = − = , −1= − =
4 4 4 4 4 4 4 4

2.52: Term Number


The terms of an arithmetic sequence are identified using t to indicate a term, and a subscript for the term
number:
𝑡1 , 𝑡2 , 𝑡3 , … , 𝑡𝑛

𝑡1 = 𝑎
𝑡2 = 𝑎 + 𝑑
𝑡3 = 𝑎 + 2𝑑
.
.
.
𝑡𝑛 = 𝑎 + (𝑛 − 1)𝑑

2.53: General Term


For an arithmetic sequence, the general term (also the called the 𝑛𝑡ℎ term) is given by:
𝑡𝑛 = 𝑎 + (𝑛 − 1)𝑑

𝐹𝑖𝑟𝑠𝑡 𝑇𝑒𝑟𝑚 = 𝑎 = 𝑎 + 0𝑑
𝑆𝑒𝑐𝑜𝑛𝑑 𝑇𝑒𝑟𝑚 = 𝑎 + 𝑑 = 𝑎 + 1𝑑
𝑇ℎ𝑖𝑟𝑑 𝑇𝑒𝑟𝑚 = 𝑎 + 2𝑑

And in general, 𝑑 is added one less than the term number:


𝑎 + (𝑛 − 1)𝑑

Example 2.54
Write the general term for each sequence below. All sequences are arithmetic.
A. 3,4,5,6,7 …

P a g e 50 | 274
Get all the files at: https://bit.ly/azizhandouts
Aziz Manva (azizmanva@gmail.com)

B. 3,3,3,3,3 …
C. 5,3,1, −1, …
3 5 3
D. 4
, 1, 4 , 2
E. 4, 7, 10, 13, ….
F. 7, 12, 17, ….
G. 10,9.5, 9,8.5, ….

Part A
Substitute 𝑎 = 3, 𝑑 = 1 in 𝑡𝑛 = 𝑎 + (𝑛 − 1)𝑑:
3 + (𝑛 − 1)1 = 3 + 𝑛 − 1 = 2 + 𝑛

Check
𝑛 = 1 ⇒ 𝑡1 = 2 + 𝑛 = 2 + 1 = 3
𝑛 = 2 ⇒ 𝑡2 = 2 + 𝑛 = 2 + 2 = 4
Parts B-G
𝑃𝑎𝑟𝑡 𝐵: 𝑎 = 3, 𝑑 = 0 ⇒ 𝑡𝑛 = 3 + (𝑛 − 1)(0) = 3 + (𝑛 − 1) × 0 = 3
𝑃𝑎𝑟𝑡 𝐶: 𝑎 = 5, 𝑑 = −2 ⇒ 𝑡𝑛 = 5 + (𝑛 − 1)(−2) = 5 − 2𝑛 + 2 = 7 − 2𝑛
3 1 3 1 3 1 1 1 1
𝑃𝑎𝑟𝑡 𝐷: 𝑎 = , 𝑑 = ⇒ + (𝑛 − 1) ( ) = + 𝑛 − = + 𝑛
4 4 4 4 4 4 4 2 4
3 1
𝑃𝑎𝑟𝑡 𝐸: 𝑎 = , 𝑑 = ⇒ 𝑎 + (𝑛 − 1)𝑑 = 4 + (𝑛 − 1)3 = 4 + 3𝑛 − 3 = 1 + 3𝑛
4 4
𝑃𝑎𝑟𝑡 𝐹: 𝑎 = 7, 𝑑 = 5 ⇒ 7 + (𝑛 − 1)5 = 7 + 5𝑛 − 5 = 2 + 5𝑛
𝑃𝑎𝑟𝑡 𝐺: 𝑎 = 10, 𝑑 = −0.5 ⇒ 10 + (𝑛 − 1)(−0.5) = 10.5 − 0.5𝑛

Example 2.55
Find a general formula for the
A. 𝑛𝑡ℎ odd natural number
B. 𝑛𝑡ℎ even natural number

1,3,5,7,9 … ⇒ 𝑎 = 1, 𝑑 = 2 ⇒ 1 + (𝑛 − 1)(2) = 1 + 2𝑛 − 2 = 2𝑛 − 1
2,4,6,8,10, … ⇒ 𝑎 = 2, 𝑑 = 2 ⇒ 2 + (𝑛 − 1)(2) = 2 + 2𝑛 − 2 = 2𝑛

Example 2.56
The Rock is pumping iron. He starts by putting 10 pounds on the bar in the first set. Then, he adds 15 pounds
for the second set. In the third set, he adds another 15 pounds. Show that:
A. The pattern that is being followed is an arithmetic sequence
B. Find the first term, common difference, and general term of the arithmetic sequence.

10,25,40,55
𝐶𝑜𝑚𝑚𝑜𝑛 𝑑𝑖𝑓𝑓𝑒𝑟𝑒𝑛𝑐𝑒 = 𝑑 = 55 − 40 = 40 − 25 = 25 − 10 = 15
𝐹𝑖𝑟𝑠𝑡 𝑇𝑒𝑟𝑚 = 𝑎 = 10
(𝑛
10 + − 1)(15) = 10 + 15𝑛 − 15 = 15𝑛 − 5
B. Finding Terms

Example 2.57
The general term of a sequence is 3𝑛 + 4.
A. Find the first five terms of the sequence.
B. Confirm that the sequence is arithmetic.

P a g e 51 | 274
Get all the files at: https://bit.ly/azizhandouts
Aziz Manva (azizmanva@gmail.com)

C. Find the first term and the common difference of the sequence.
Part A
𝑡1 : 3𝑛 + 4 = 3(1) + 4 = 3 + 4 = 7
𝑡2 : 3𝑛 + 4 = 3(2) + 4 = 6 + 4 = 10
𝑡3 : 3𝑛 + 4 = 3(3) + 4 = 9 + 4 = 13
𝑡4 : 3𝑛 + 4 = 3(4) + 4 = 12 + 4 = 16
𝑡5 : 3𝑛 + 4 = 3(5) + 4 = 15 + 4 = 19
Parts B and C
19 − 16 = 16 − 13 = 13 − 10 = 10 − 7 = 3
Hence, the sequence is arithmetic with a common difference of 3.
𝑎 = 7, 𝑑 = 3

Example 2.58
Find the 50𝑡ℎ term of an arithmetic sequence with first term 2, and common difference 8.

Substitute 𝑎 = 2, 𝑑 = 8, 𝑛 = 50 in 𝑎 + (𝑛 − 1)𝑑:
2 + (50 − 1)(8) = 2 + 400 − 8 = 2 + 392 = 394

Example 2.59
An arithmetic sequence has first term 7, and common difference 4.
A. Identify the tenth term in the sequence.
B. Identify the value of the largest two-digit number in the sequence (not the term number)
C. Identify the smallest four-digit number in the sequence (not the term number)

Part A
𝑎 + (𝑛 − 1)𝑑 = 7 + (10 − 1)4 = 7 + 36 = 43

Part B
7,11,15
Each of the terms has remainder 3 when divided by 4.
Largest two-digit that has remainder 3 when divided by 4
= 99

97 1
𝑎 + (𝑛 − 1)𝑑 < 100 ⇒ 7 + 4𝑛 − 4 < 100 ⇒ 4𝑛 < 97 ⇒ 𝑛 < = 24
4 4
The largest integer 𝑛 which satisfies this is 𝑛 = 24.
Substitute 𝑛 = 24 in the formula:
𝑎 + (𝑛 − 1)𝑑 = 7 + (24 − 1)4 = 7 + 23 × 4 = 99
Part C
997
𝑎 + (𝑛 − 1)𝑑 > 100 ⇒ 7 + 4𝑛 − 4 > 1000 ⇒ 4𝑛 > 997 ⇒ 𝑛 >
7
The smallest integer which satisfies this inequality is 𝑛 = 250, which on substituting gives:
𝑎 + (𝑛 − 1)𝑑 = 7 + (250 − 1)4 = 7 + 249 × 4 = 1003

Example 2.60
Jane runs a lemonade stand. She sold 20 glasses of lemonade every day last week, which was the first week she
started. This week, she sold 27 glasses of lemonade each day. Next week, she plans to sell 34 glasses of

P a g e 52 | 274
Get all the files at: https://bit.ly/azizhandouts
Aziz Manva (azizmanva@gmail.com)

lemonade each day. If this pattern continues, what is the first week that she will sell in excess of 1000 glasses of
lemonade in a day. (Jane is hard-working, and works seven days a week).

The number of glasses of lemonade sold every day is:


20
⏟ , 27 ⏟ , 34
⏟ ,…
1𝑠𝑡 2𝑛𝑑 3𝑟𝑑
𝑊𝑒𝑒𝑘 𝑊𝑒𝑒𝑘 𝑊𝑒𝑒𝑘
This is an arithmetic sequence with
𝑆𝑎𝑙𝑒 𝑝𝑒𝑟 𝑑𝑎𝑦 𝑖𝑛 1𝑠𝑡 𝑊𝑒𝑒𝑘 = 𝐹𝑖𝑟𝑠𝑡 𝑇𝑒𝑟𝑚 = 𝑎 = 20
𝐶𝑜𝑚𝑚𝑜𝑛 𝐷𝑖𝑓𝑓𝑒𝑟𝑒𝑛𝑐𝑒 = 𝑑 = 7

The general term:


20 + (𝑛 − 1)(7) = 20 + 7𝑛 − 7 = 13 + 7𝑛
Hence,
13 + 7𝑛 > 1000 ⇒ 7𝑛 > 987 ⇒ 𝑛 > 141
Smallest 𝑛 that will work is
𝑛 = 142 ⇒ 142𝑛𝑑 𝑊𝑒𝑒𝑘

Example 2.61
𝑎 𝑎
A. Let 𝑎1 , 𝑎2 , 𝑎3 , . .. be an arithmetic sequence. If 𝑎4 = 3, what is 𝑎5 ? (AOPS Alcumus, Algebra, Arithmetic
2 3
Sequences)
B. Find the ratio of the fifth and the third term in an arithmetic sequence, if the fourth term is three times
the second term.

Part A
𝑎 + 3𝑑
= 3 ⇒ 𝑎 + 3𝑑 = 3𝑎 + 3𝑑 ⇒ 𝑎 = 3𝑎 ⇒ 𝑎 = 0
𝑎+𝑑
𝑎5 𝑎 + 4𝑑 4𝑑
= = =2
𝑎3 𝑎 + 2𝑑 2𝑑
Part B
This is the same question, framed in different language.

C. Common Difference

Example 2.62: Finding the common difference


A. The 14𝑡ℎ term of an arithmetic sequence is 100. If the first term is
9, find the common difference.
B. The sequence of integers in each of the two rows of squares and in
each of the two columns of squares form four separate arithmetic
sequences. What is the least positive integer value for 𝑁?
(MathCounts 2005 National Team)

Part A
𝑡14 = 100 ⇒ 𝑎 + (𝑛 − 1)𝑑 = 100 ⇒ 9 + 13𝑑 = 100 ⇒ 𝑑 = 7
Part B
The first column is an arithmetic sequence with common difference
= +75 − 87 = −12

P a g e 53 | 274
Get all the files at: https://bit.ly/azizhandouts
Aziz Manva (azizmanva@gmail.com)

The first row is an arithmetic sequence with common difference:


26 − 75 49
= =− = −7
7 7
The common difference for second row and second column must be
integers:
𝐹𝑟𝑜𝑚 𝑡ℎ𝑒 𝑆𝑒𝑐𝑜𝑛𝑑 𝐶𝑜𝑙𝑢𝑚𝑛: 𝑁 = 40 − 3𝑎

(40,37,34, … , ⏟
4 , 1)
𝑎=12
𝐹𝑟𝑜𝑚 𝑡ℎ𝑒 𝑆𝑒𝑐𝑜𝑛𝑑 𝑅𝑜𝑤: 𝑁 = 39 − 5𝑏

(39,34,29, … , ⏟
4 )
𝑏=7
Smallest number in both lists is 4.

2.63: Common Difference Property


If the difference between two terms of a sequence is constant, then the sequence is an arithmetic sequence.

𝑎1 , 𝑎2 , 𝑎3 … . 𝑎𝑛
𝑎2 − 𝑎1 = 𝑎3 − 𝑎2 = ⋯ = 𝑎𝑛 − 𝑎𝑛−1 = 𝑑

𝑎1 , 𝑎2 , 𝑎3 … . 𝑎𝑛 ⇔ 𝑎1 , 𝑎1 + 𝑑, 𝑎1 + 2𝑑, … , 𝑎1 + (𝑛 − 1)𝑑
Which is precisely the definition of an arithmetic sequence.

Example 2.64: Using the common difference property


A. Find 𝑛 if ⏟
𝑛 + 5,⏟
2𝑛 + 1 , ⏟
4𝑛 − 3 are consecutive terms of an arithmetic sequence.
𝒂𝟏 𝒂𝟐 𝒂𝟑
B. The first three terms of an arithmetic progression are 𝑥 − 1, 𝑥 + 1,2𝑥 + 3, in the order shown. The value
of x is: (AHSME 1962/3)
C. The largest and smallest of three consecutive terms in an arithmetic sequence differ by 14. Half of the
smallest term is added to each term and the sum of the resulting three numbers is 120. What is the
value of the original smallest term? (MathCounts 2008 State Countdown)
D. The sum of four positive integers that form an arithmetic sequence is 46. Of all such possible sequences,
what is the greatest possible third term? (MathCounts 2005 State Sprint)

Part A 𝑎, 𝑎 + 𝑑, 𝑎 + 2𝑑 ⇒ 2𝑑 = 14 ⇒ 𝑑 = 7
Consecutive terms of an arithmetic sequence differ 𝑎
𝑎 + (𝑎 + 7) + (𝑎 + 14) + × 3 = 120
by the common difference. Hence, 2
3𝑎 9𝑎
𝑎2 − 𝑎1 = 𝑎3 − 𝑎2 3𝑎 + 21 + = 120 ⇒ = 99 ⇒ 𝑎 = 22
Substituting the terms as given in the question: 2 2
Part C
2𝑛 + 1 − ⏟
⏟ (𝑛 + 5) = ⏟
(4𝑛 − 3) − ⏟ (2𝑛 + 1)
𝒂𝟐 𝒂𝟏 𝒂𝟑 𝒂𝟐
𝑎, 𝑎 + 𝑑, 𝑎 + 2𝑑, 𝑎 + 3𝑑
2𝑛 + 1 − 𝑛 − 5 = 4𝑛 − 3 − 2𝑛 − 1 4𝑎 + 6𝑑 = 46 ⇒ 2𝑎 + 3𝑑 = 23 ⇒ 3𝑑 = 23 − 2𝑎
23 − 2𝑎
𝑛 − 4 = 2𝑛 − 4 𝑑=
𝑛 = 2𝑛 3
23 − 2𝑎
𝑛=0 𝑎 + 2𝑑 = 𝑎 + 2 ×
3
Part B 3𝑎 + 46 − 2𝑎 46 − 𝑎
𝑥 + 1 − (𝑥 − 1) = 2𝑥 + 3 − (𝑥 + 1) = =
3 3
2=𝑥+2 𝑆𝑚𝑎𝑙𝑙𝑒𝑠𝑡 𝑣𝑎𝑙𝑢𝑒 𝑜𝑓 𝑎 𝑖𝑠 1, 𝑤ℎ𝑖𝑐ℎ 𝑔𝑖𝑣𝑒𝑠 𝑑 = 7
𝑥=0 1,8,15,22
Part C

P a g e 54 | 274
Challenge 2.65: Using the common difference property
𝑥
The first four terms in an arithmetic sequence are 𝑥 + 𝑦, 𝑥 − 𝑦, 𝑥𝑦, and , in that order. What is the fifth term?
𝑦
(AMC 10B 2003/24)

The common difference is:


𝑑 = 𝑥 − 𝑦 − (𝑥 + 𝑦) = −2𝑦
Hence, we must have:
3𝑟𝑑 𝑇𝑒𝑟𝑚 = 𝑥 − 3𝑦, 4𝑡ℎ 𝑇𝑒𝑟𝑚 = 𝑥 − 5𝑦
But the two values for the third term must be same:
3𝑦 𝑥 3
𝑥 − 3𝑦 = 𝑥𝑦 ⇒ 𝑥 − 𝑥𝑦 = 3𝑦 ⇒ 𝑥(1 − 𝑦) = 3𝑦 ⇒ 𝑥 = ⇒ =
1−𝑦 𝑦 1−𝑦
Similarly, the two values for the fourth term must be the same:
𝑥
𝑥 − 5𝑦 =
𝑦
3𝑦 𝑥 3
Substitute 𝑥 = and = in the above:
1−𝑦 𝑦 1−𝑦
3𝑦 3
− 5𝑦 =
1−𝑦 1−𝑦
Multiply both sides by 1 − 𝑦:
3𝑦 − 5𝑦(1 − 𝑦) = 3 ⇒ 3𝑦 − 5𝑦 + 5𝑦 2 = 3 ⇒ 5𝑦 2 − 2𝑦 − 3 = 0
Factor and solve:
3
(5𝑦 + 3)(𝑦 − 1) = 0 ⇒ 𝑦 ∈ {− , 1}
5
Substitute 𝑦 = 1 in the expressions for the third and fourth term:
𝑥
𝑥𝑦 = 𝑥, = 𝑥 ⇒ 𝑑 = 0 ⇒ 𝐶𝑜𝑛𝑡𝑟𝑎𝑑𝑖𝑐𝑡𝑖𝑜𝑛 ⇒ 𝑦 ≠ 1
𝑦
3
Substitute 𝑦 = − 5 in 𝑥 − 3𝑦 = 𝑥𝑦:
3 3 8 9 9
𝑥 − 3 (− ) = 𝑥 (− ) ⇒ 𝑥 = − ⇒ 𝑥 = −
5 5 5 5 8
And, finally, the fifth term is:
9 3 9 21 −45 + 168 123
𝑥 − 7𝑦 = − − 7 (− ) = − + = =
8 5 8 5 40 40

Example 2.66
A. (𝐽𝑢𝑚𝑝𝑖𝑛𝑔 𝑇𝑒𝑟𝑚𝑠) The fifth term of a sequence is two, and the second term of a sequence is five. Find
the seventh term of the sequence.
B. (𝑅𝑎𝑑𝑖𝑐𝑎𝑙𝑠) If the fourth term of an arithmetic sequence is 5√7 − 3, and the first term is 8 − √7, then
find the second term.
C. (𝑆𝑖𝑚𝑢𝑙𝑡𝑎𝑛𝑒𝑜𝑢𝑠 𝐸𝑞𝑢𝑎𝑡𝑖𝑜𝑛𝑠) The seventh term of an arithmetic sequence is 25. Find the 𝑛𝑡ℎ term if the
fifth term added to twice the third term equals the twelfth term.

Part A difference to the fifth term


Going from the second term to the fifth term means 𝑎7 = 𝑎5 + 2𝑑 = 2 − 2 = 0
adding the common difference thrice. Therefore, Part B
the common difference must be: To go from the first term to the fourth term, you
2−5 3 need to add the common difference three times:
3𝑑 = 2 − 5 ⇒ 𝑑 = = − = −1
3 3
To find the seventh term, add twice the common
Get all the files at: https://bit.ly/azizhandouts
Aziz Manva (azizmanva@gmail.com)

(5√7 − 3) − (8 − √7) −11 + 6√7 3𝑑 15𝑑


𝑑= = 𝑎 + 6𝑑 =
⏟ + 6𝑑 = = 25
3 3 𝑺𝒆𝒗𝒆𝒏𝒕𝒉
2 2
The second term is equal to the common difference 𝑻𝒆𝒓𝒎
2 10
added to the first term: 𝑑 = 25 ×
=
−11 + 6√7 13 + 3√7 15 3
𝑡2 = 8 − √7 + = 3𝑑 3 10
3 3 𝑎 = = × =5
2 2 3
Part C 10
(𝑎 + 4𝑑) + 2 ⏟
⏟ (𝑎 + 2𝑑) = ⏟𝑎 + 11𝑑 𝑛𝑡ℎ 𝑡𝑒𝑟𝑚 = 𝑎 + (𝑛 − 1)𝑑 = 5 + (𝑛 − 1) ( )
3
𝑭𝒊𝒇𝒕𝒉 𝑻𝒆𝒓𝒎 𝑻𝒉𝒊𝒓𝒅 𝑻𝒆𝒓𝒎 𝑻𝒘𝒆𝒍𝒇𝒕𝒉 𝑻𝒆𝒓𝒎
3𝑑
𝑎 =
2

D. Arithmetic Mean Property

2.67: Arithmetic Mean Property and its converse


Any term of an arithmetic sequence is the arithmetic mean of the terms that precede and follow it.
𝒂𝒏−𝟏 + 𝒂𝒏+𝟏
𝒂𝒏 =
𝟐

𝑎𝑛−1 + 𝑎𝑛+1 [𝑎 + (𝑛 − 2)𝑑] + [𝑎 + 𝑛𝑑] 2𝑎 + 2𝑛𝑑 − 2𝑑


= = 𝑎 + (𝑛 − 1)𝑑
= 𝑎 + 𝑛𝑑 − 𝑑 = ⏟
⏟ 2 ⏟ 2 2 𝑳𝑯𝑺
𝑹𝑯𝑺 𝑺𝒖𝒃𝒔𝒕𝒊𝒕𝒖𝒕𝒆 𝒕𝒉𝒆 𝒅𝒆𝒇𝒊𝒏𝒊𝒕𝒊𝒐𝒏

Converse: If the general term of a sequence is the arithmetic mean of the terms that precede and follow it, then
that sequence is an arithmetic sequence.

The arithmetic mean property has both theoretical and practical importance. We can apply it:
➢ To find missing values when we know that a sequence is arithmetic.
➢ To prove that a sequence is arithmetic

Example 2.68
A. Show that the sequence 3,7,11 is arithmetic.
B. Check whether the sequence 2𝑥 + 3, 𝑥 + 6, 9 is arithmetic using the arithmetic mean property. If it is,
then find the common difference.
C. The first term of an arithmetic sequence is 5. The third term is 12. Find the second term using the
arithmetic mean property, and then by finding the common difference. Which is better?
1
D. (𝐿𝑖𝑛𝑒𝑎𝑟 𝐸𝑞𝑢𝑎𝑡𝑖𝑜𝑛) Given that the numbers 2 , 𝑥 − 1, 3𝑥 form an arithmetic sequence, find the value of 𝑥.
(AOPS Alcumus, Algebra, Arithmetic Sequences)
E. (𝑄𝑢𝑎𝑑𝑟𝑎𝑡𝑖𝑐 𝐸𝑞𝑢𝑎𝑡𝑖𝑜𝑛) If −4𝑥 − 12, 𝑥 2 + 2, −6𝑥 + 4 form an arithmetic sequence, find the largest
possible value of 𝑥.

Part A 𝑎1 + 𝑎3 11 + 3 14
𝐴𝑣𝑔 (𝑎1 , 𝑎2 ) = = = = 7 = 𝑎2
Method I: Common Difference Property 2 2 2
𝑑1 = 𝑎2 − 𝑎1 = 7 − 3 = 4 Part B
𝑑2 = 11 − 7 = 4 ⇒ 𝑑1 = 𝑑2 Arithmetic Mean Property
𝑆𝑒𝑞𝑢𝑒𝑛𝑐𝑒 𝑖𝑠 𝑎𝑟𝑖𝑡ℎ𝑚𝑒𝑡𝑖𝑐 𝑎1 + 𝑎3 (2𝑥 + 3) + 9 2𝑥 + 12
= = = 𝑥 + 6 = 𝑎2
Method II: Arithmetic Mean Property 2 2 2
𝑆𝑒𝑞𝑢𝑒𝑛𝑐𝑒 𝑖𝑠 𝑎𝑟𝑖𝑡ℎ𝑚𝑒𝑡𝑖𝑐

P a g e 56 | 274
Get all the files at: https://bit.ly/azizhandouts
Aziz Manva (azizmanva@gmail.com)

We can find the common difference difference.


𝑑1 = 𝑎2 − 𝑎1 = (𝑥 + 6) − (2𝑥 + 3) = −𝑥 + 3 Part D
𝑑2 = 𝑎3 − 𝑎2 = 9 − (𝑥 + 6) = −𝑥 + 3 Use the arithmetic mean property:
Part C 1
+ 3𝑥 1 5
Method I: Arithmetic Mean Property 2 = 𝑥 − 1 ⇒ + 3𝑥 = 2𝑥 − 2 ⇒ 𝑥 = −
The second term is the average of the first term and 2 2 2
Part E
the third term:
Method I: Arithmetic Mean Property
𝑎1 + 𝑎3 5 + 12 17
𝑎2 = = = (−4𝑥 − 12) + (−6𝑥 + 4)
2 2 2 = 𝑥2 + 2
Method II: Finding the common difference 2
−5𝑥 − 4 = 𝑥 2 + 2
Find the common difference:
7 Collect like terms to get a quadratic, solve it, and
2𝑑 = 12 − 5 = 7 ⇒ 𝑑 = take the larger solution:
2
Use the common difference to find the required 𝑥 2 + 5𝑥 + 6 = 0 ⇒ (𝑥 + 2)(𝑥 + 3) = 0
term: 𝑥 ∈ {−2, −3} ⇒ 𝑥 = −2
7 17 Method II: Common Difference Property
𝑎+𝑑 =5+ = 𝑥 2 + 2 − (−4𝑥 − 12) = −6𝑥 + 4 − (𝑥 2 + 2)
2 2
In this case, the arithmetic mean property method 𝑥 2 + 4𝑥 + 10 = −6𝑥 + 2 − 𝑥 2
is shorter since we do not need to find the common 2𝑥 2 + 10𝑥 + 12 = 0
𝑥 2 + 5𝑥 + 6 = 0

Challenge 2.69
𝑆1 , 𝑆2 , 𝑆3 are arithmetic sequences. 𝑝 and 𝑞 are positive integers. The middle terms of 𝑆1 , 𝑆2 and 𝑆3 themselves
form an arithmetic sequence 𝑆4 . Find the common difference of 𝑆4 for the second smallest value of 𝑝 + 𝑞.
𝑆1 = {𝑝, 𝑥, 𝑝3 }, 𝑆2 = {𝑝2 , 𝑦, 𝑝4 }, 𝑆3 = {𝑞, 𝑧, 𝑞 3 }, 𝑆4 = {𝑥, 𝑦, 𝑧}

The middle term of the first three arithmetic sequences themselves form an arithmetic sequence, as given in 𝑆4 .
Rather than solve this algebraically, we try small values of 𝑝 and 𝑞, as given in the table below:

𝑆3 𝑆1 𝑝 𝑜𝑟 𝑞 𝑝 1 2 3
𝑞 𝑝 1 2 3 𝑝2 1 4 9
𝑞3 𝑝3 1 8 27 𝑝4 1 16 81
𝑞 + 𝑞3 𝑝 + 𝑝3 10 30 𝑝2 + 𝑝4 1 20 90
𝑞 + 𝑞3 𝑝 + 𝑝3 1 5 15 𝑦 1 10 45
𝑧= 𝑥= 𝑝2 + 𝑝4
2 2 =
2

Smallest value
𝑝 = 𝑞 = 1 gives us 𝑥 = 𝑦 = 𝑧 = 1

Second Smallest Value


𝑥 = 5, 𝑦 = 10, 𝑧 = 15 ⇒ 𝑝 = 2, 𝑞 = 3 ⇒ 𝐶𝑜𝑚𝑚𝑜𝑛 𝑑𝑖𝑓𝑓𝑒𝑟𝑒𝑛𝑐𝑒 = 𝑑 = 5
E. Recursive Definition
A recursive definition is a definition where a term is defined in terms of what comes before it.

2.70: Recursive Definition


An arithmetic sequence can be defined as:
𝑎𝑛 = 𝑎𝑛−1 + 𝑑, 𝑎1 = 𝑐

P a g e 57 | 274
Get all the files at: https://bit.ly/azizhandouts
Aziz Manva (azizmanva@gmail.com)

An arithmetic sequence can be defined either recursively, or explicitly, and both definitions are equivalent.
It is also possible to convert from one form to another, and this is useful.

Example 2.71
Find the first five terms of the sequence defined as:
A. 𝑎𝑛 = 𝑎𝑛−1 + 2, 𝑎1 = 7
3
B. 𝑎𝑛 = 𝑎𝑛−1 − ( ) , 𝑎1 = 5
4

Part A 3 3 20 3 17
𝑎2 = 𝑎1 − =5− = − =
𝑎1 = 7 4 4 4 4 4
𝑎2 = 𝑎1 + 2 = 7 + 2 = 9 14
𝑎3 =
𝑎3 = 𝑎2 + 2 = 9 + 2 = 11 4
11
𝑎4 = 𝑎3 + 2 = 11 + 2 = 13 𝑎4 =
𝑎5 = 𝑎4 + 2 = 13 + 2 = 15 4
8
Part B 𝑎5 = = 2
4
𝑎1 = 5

Example 2.72
Write the arithmetic sequence below using a recursive definition:
A. 21, 23.5, 26
13 1
B. 4 , − 3
C. 𝑥, 𝑥 − 2
3 1 1 3
D. 4 𝑥 + 3 𝑦, 3 𝑥 − 4 𝑦
E. 𝑥, 𝑦

Part A 1 3 3 1
𝑑 = 𝑥 − 𝑦 − ( 𝑥 + 𝑦)
𝑎𝑛 = 𝑎𝑛−1 + 2.5, 𝑎1 = 21 3 4 4 3
Part B 4 9 9 4
= ( − ) 𝑥 + (− − ) 𝑦
1 13 4 39 43 12 12 12 12
𝑑=− − =− − =− 5 13
3 4 12 12 12 =− 𝑥− 𝑦
43 13 12 12
𝑎𝑛 = 𝑎𝑛−1 − , 𝑎1 = 5 13 3 1
12 4 𝑎𝑛 = 𝑎𝑛−1 − 𝑥 − 𝑦, 𝑎1 = 𝑥 + 𝑦
Part C 12 12 4 3
𝑑 = −2 Part E
𝑎𝑛 = 𝑎𝑛−1 − 2, 𝑎1 = 𝑥 𝑑 = 𝑦−𝑥
Part D 𝑎𝑛 = 𝑎𝑛−1 + 𝑦 − 𝑥

2.73: Converting from Recursive Definition to Explicit Definition


An arithmetic sequence given in recursive form can be converted into explicit form
𝑎
⏟𝑛 = 𝑎𝑛−1 + 𝑑, ⏟𝑛 = 𝑐 + (𝑛 − 1)𝑑
𝑎1 = 𝑐 ⇔ 𝑎
𝑅𝑒𝑐𝑢𝑟𝑠𝑖𝑣𝑒 𝐷𝑒𝑓𝑖𝑛𝑖𝑡𝑖𝑜𝑛 𝐸𝑥𝑝𝑙𝑖𝑐𝑡 𝐷𝑒𝑓𝑖𝑛𝑖𝑡𝑖𝑜𝑛

𝑎1 = 𝑐
𝑎2 = 𝑐 + 𝑑
𝑎3 = 𝑐 + 𝑑 + 𝑑 = 𝑐 + 2𝑑
𝑎𝑛 = 𝑐 + (𝑛 − 1)𝑑

P a g e 58 | 274
Get all the files at: https://bit.ly/azizhandouts
Aziz Manva (azizmanva@gmail.com)

Example 2.74: Converting from Recursive to Explicit


Write the arithmetic sequences below given in recursive form into explicit form, and vice versa.

Numerical Sequences Sequences with Variables


5 13 3 1
E. 𝑎𝑛 = 𝑎𝑛−1 − 12 𝑥 − 12 𝑦, 𝑎1 = 4 𝑥 + 3 𝑦
A. 𝑎𝑛 = 𝑎𝑛−1 + 2.5, 𝑎1 = 21
43 13
F. 𝑎𝑛 = 𝑎𝑛−1 + 𝑦 − 𝑥, 𝑎1 = 𝑥
B. 𝑎𝑛 = 𝑎𝑛−1 − 12 , 𝑎1 = 4 G. 𝑎𝑛 = 𝑥 + (𝑛 − 1)𝑦
C. 𝑎𝑛 = 2 + (𝑛 − 1)(5) H. 𝑎𝑛 = 𝑥 + 𝑦𝑛
3 1
D. 𝑎𝑛 = + (𝑛 − 1) ( )
4 5

Part A 3 1 5 13
𝑎𝑛 = 𝑥 + 𝑦 + (𝑛 − 1) (− 𝑥 − 𝑦)
𝑎𝑛 = 21 + (𝑛 − 1)2.5 4 3 12 12
= 21 + 2.5𝑛 − 2.5 = 18.5 + 2.5𝑛 9 5 4 13 5 13
= 𝑥 + 𝑥 + 𝑦 + 𝑦 − 𝑥𝑛 − 𝑦𝑛
Part B 12 12 12 12 12 12
13 43 14𝑥 + 17𝑦 − 5𝑥𝑛 − 13𝑦𝑛
𝑎𝑛 = + (𝑛 − 1) (− ) =
4 12 12
39 43 43 41 43 Part F
= − 𝑛+ = − 𝑛 Part G
12 12 12 6 12
Part C 𝑎𝑛 = 𝑎𝑛−1 + 𝑦, 𝑎1 = 𝑥
𝑎𝑛 = 𝑎𝑛−1 + 5, 𝑎1 = 2 Part H
Part D Substitute 𝑛 = 1 to find the first term:
1 3 𝑥 + 𝑦(1) = 𝑥 + 𝑦
𝑎𝑛 = 𝑎𝑛−1 + , 𝑎1 =
5 4 𝑎𝑛 = 𝑎𝑛−1 + 𝑦, 𝑎1 = 𝑥 + 𝑦

Part E

2.3 Symmetry,Structure,Proof
A. Difference of Terms

Example 2.75: Odd and Even Terms


The number of terms in an A.P. (Arithmetic Progression) is even. The sum of the odd and even-numbered terms
are 24 and 30, respectively. If the last term exceeds the first by 10.5, the number of terms in the A.P. is (AHSME
1973/26)

Let
𝐹𝑖𝑟𝑠𝑡 𝑇𝑒𝑟𝑚 = 𝑎1 , 𝐶𝑜𝑚𝑚𝑜𝑛 𝑑𝑖𝑓𝑓𝑒𝑟𝑒𝑛𝑐𝑒 = 𝑑, 𝑁𝑜. 𝑜𝑓 𝑇𝑒𝑟𝑚𝑠 = 𝑛,
Then:
𝐿𝑎𝑠𝑡 𝑇𝑒𝑟𝑚 = 𝑎1 + (𝑛 − 1)𝑑
Since the last term exceeds the first by 10.5:
𝑎 + (𝑛 − 1)𝑑 − 𝑎 = ⏟
𝑛𝑑 − 𝑑 = 10.5
𝑬𝒒𝒖𝒂𝒕𝒊𝒐𝒏 𝑰
The sum of odd and even numbered terms is 24 and 30, respectively. Set up sums for only odd and only terms:
𝑂𝑑𝑑 𝑇𝑒𝑟𝑚𝑠 = ⏟ 𝑎1 + 𝑎3 + ⋯ + 𝑎𝑛−1 = 24
𝑬𝒒𝒖𝒂𝒕𝒊𝒐𝒏 𝑰𝑰
(𝑎1 + 𝑑) + (𝑎3 + 𝑑) + ⋯ + (𝑎𝑛−1 + 𝑑) = 30
𝐸𝑣𝑒𝑛 𝑇𝑒𝑟𝑚𝑠 = 𝑎2 + 𝑎4 + ⋯ + 𝑎𝑛 = ⏟
𝑬𝒒𝒖𝒂𝒕𝒊𝒐𝒏 𝑰𝑰𝑰
Subtract Equation II from Equation III:

P a g e 59 | 274
Get all the files at: https://bit.ly/azizhandouts
Aziz Manva (azizmanva@gmail.com)

𝑛𝑑
𝑑 + 𝑑 + ⋯+ 𝑑 = 6 ⇒
⏟ =6⇒ ⏟
𝑛𝑑 = 12
𝑛 2 𝑬𝒒𝒖𝒂𝒕𝒊𝒐𝒏 𝑰𝑽
𝑡𝑖𝑚𝑒𝑠
2
Substitute 𝑛𝑑 = 12 in Equation I:
12 − 𝑑 = 10.5 ⇒ 𝑑 = 1.5
3
Substitute 𝑑 = 1.5 = in Equation IV:
2
3
𝑛 = 12 ⇒ 𝑛 = 8
2
B. Odd Number of Terms in an Arithmetic Sequence

5, ⏟
7 , ⏟9 … 15⏟

𝑎 𝑎+𝑑 𝑎+2𝑑 𝑎+(𝑛−1)𝑑
𝑎=5,𝑑=2,𝑛=5
When given three terms of an arithmetic sequence, we assume them as
𝑎−𝑑 ,
⏟ ⏟
𝑎 , 𝑎+𝑑

𝟏𝒔𝒕 𝑻𝒆𝒓𝒎 𝟐𝒏𝒅 𝑻𝒆𝒓𝒎 𝟑𝒓𝒅 𝑻𝒆𝒓𝒎
This help reduce calculations because when we add them, we get:
(𝑎 − 𝒅) + 𝑎 + (𝑎 + 𝒅) = 𝑎 + 𝑎 + 𝑎 = 3𝑎

Example 2.76: Three Terms in an Arithmetic Sequence


A. The three terms in an arithmetic sequence add up to 17. Find the middle term.
B. The five terms in an arithmetic sequence add up to 17. Find the middle term.

Part A
Method I
Suppose that the first term is 𝑎, and the common difference is 𝑑. Then, the terms are:
𝑎, 𝑎 + 𝑑, 𝑎 + 2𝑑
17
𝑎 + 𝑎 + 𝑑 + 𝑎 + 2𝑑 = 17 ⇒ 3𝑎 + 3𝑑 = 17 ⇒ 3(𝑎 + 𝑑) = 17 ⇒ 𝑎 + 𝑑 =
3
Method II
Suppose that the middle term is 𝑎, and the common difference is still 𝑑:
𝑎 − 𝑑, 𝑎, 𝑎 + 𝑑
17
(𝑎 − 𝒅) + 𝑎 + (𝑎 + 𝒅) = 17 ⇒ 3𝑎 = 17 ⇒ 𝑎 =
3

Part B
Method I
Suppose that the first term is 𝑎, and the common difference is 𝑑. Then, the terms are:
𝑎, 𝑎 + 𝑑, 𝑎 + 2𝑑, 𝑎 + 3𝑑, 𝑎 + 4𝑑

𝑎 + (𝑎 + 𝑑) + (𝑎 + 2𝑑) + (𝑎 + 3𝑑) + (𝑎 + 4𝑑) = 17


5𝑎 + 10𝑑 = 17
5(𝑎 + 2𝑑) = 17
17
𝑎 + 2𝑑 =
5
Method II
Suppose that the middle term is 𝑎, and the common difference is still 𝑑:
𝑎 − 2𝑑, 𝑎 − 𝑑, 𝑎, 𝑎 + 𝑑, 𝑎 + 2𝑑

17
(𝑎 − 𝟐𝒅) + (𝑎 − 𝒅) + 𝑎 + (𝑎 + 𝒅) + (𝑎 + 𝟐𝒅) = 17 ⇒ 5𝑎 = 17 ⇒ 𝑎 =
5

P a g e 60 | 274
Get all the files at: https://bit.ly/azizhandouts
Aziz Manva (azizmanva@gmail.com)

2.77: Finding the middle term


If three terms of an arithmetic sequence add up to 𝒙, then the middle term of the sequence is given by
𝒙
𝟑

Assume the three terms in the sequence and equating them to the sum:
𝑥
(𝑎 − 𝒅) + (𝑎) + (𝑎 + 𝒅) = 𝑥 ⇒ 3𝑎 = 𝑥 ⇒ 𝑎 =
3

2.78: Middle term is average of all the terms (for odd number of Terms)
The average of an arithmetic sequence with an odd number of terms is equal to its middle term.
𝑎1 + 𝑎2 + ⋯ + 𝑎𝑛
= 𝑀𝑖𝑑𝑑𝑙𝑒 𝑇𝑒𝑟𝑚
𝑛

Three Term Case


𝑎1 = 𝑎 − 𝑑, 𝑎2 = 𝑎, 𝑎3 = 𝑎 + 𝑑
Suppose we consider three terms:
𝑎1 + 𝑎2 + 𝑎3 (𝑎 − 𝑑) + 𝑎 + (𝑎 + 𝑑) 3𝑎
= = = 𝑎 = 𝑀𝑖𝑑𝑑𝑙𝑒 𝑇𝑒𝑟𝑚
3 3 3
Five Term Case
𝑎1 = 𝑎 − 2𝑑, 𝑎2 = 𝑎 − 𝑑, 𝑎3 = 𝑎, 𝑎4 = 𝑎 + 𝑑, 𝑎5 = 𝑎 + 2𝑑
Suppose we consider three terms:
𝑎1 + 𝑎2 + 𝑎3 + 𝑎4 + 𝑎5 (𝑎 − 2𝑑) + (𝑎 − 𝑑) + 𝑎 + (𝑎 + 𝑑) + (𝑎 + 2𝑑) 5𝑎
= = = 𝑎 = 𝑀𝑖𝑑𝑑𝑙𝑒 𝑇𝑒𝑟𝑚
3 3 5

Example 2.79
A. Seven consecutive terms of an arithmetic sequence add up to -729. What is the middle term of that
sequence?

729

7

Example 2.80
If angles in a polygon are in arithmetic sequence, then their sum must be equal to the sum of the angles of a
polygon of that many sides. Hence, it is a disguised way of giving the sum of the sequence.
A. If the measures of the angles in a triangle form an arithmetic sequence, then find the value of the angle
that forms the middle term.
B. The angles of a pentagon are in arithmetic progression. One of the angles in degrees, must be: (AHSME
1962/20)
C. How many non-similar triangles have angles whose degree measures are distinct positive integers in
arithmetic progression? (AMC 10A 2006/19)

By symmetry, the middle term must be the average of all the terms, which then add up to the angles of a
triangle.
Part A
𝑎1 + 𝑎2 + 𝑎3 180
∴ 𝑎2 = = = 60
3 3
Part B

P a g e 61 | 274
Get all the files at: https://bit.ly/azizhandouts
Aziz Manva (azizmanva@gmail.com)

𝑎1 + 𝑎2 + 𝑎3 + 𝑎4 + 𝑎5 540
∴ 𝑀𝑖𝑑𝑑𝑙𝑒 𝑇𝑒𝑟𝑚 = 𝑎3 = = = 108°
3 5
Part C
𝑎1 + 𝑎2 + 𝑎3 180
∴ 𝑎2 = = = 60
3 3
This means the remaining two angles must add up to 120:
𝑎1 + 𝑎2 = 120
This is a Diophantine Equation, and we can tabulate the number of solutions by deciding the value of 𝑎1 :

𝑎1 59 58 57 . . . 1
𝑎2 60 60 60 . . . 60
𝑎3 61 62 63 . . . 119

1 ≤ 𝑎1 ≤ 59 ⇒ 𝑎1 𝑐𝑎𝑛 𝑡𝑎𝑘𝑒 59 𝑣𝑎𝑙𝑢𝑒𝑠

Example 2.81
Does there exist a triangle whose angles have measures which are:
A. Consecutive integers
B. Consecutive odd integers
C. Consecutive even integers

Part A
𝐶𝑜𝑛𝑠𝑒𝑐𝑢𝑡𝑖𝑣𝑒 𝑖𝑛𝑡𝑒𝑔𝑒𝑟𝑠: 59, 60, 61
Part B
𝑂𝑑𝑑
⏟ + ⏟
𝑂𝑑𝑑 + ⏟
𝑂𝑑𝑑 = 𝑂𝑑𝑑 ⇒ 𝑁𝑜𝑡 𝑝𝑜𝑠𝑠𝑖𝑏𝑙𝑒
1𝑠𝑡 𝑇𝑒𝑟𝑚 2𝑛𝑑 𝑇𝑒𝑟𝑚 3𝑟𝑑 𝑇𝑒𝑟𝑚
Part C
𝑎 = 60° ⇒ 𝑎 − 𝑑 = 58, 𝑎 + 𝑑 = 62

Example 2.82
Thirty-one books are arranged from left to right in order of increasing prices. The price of each book differs by
2$ from that of each adjacent book. For the price of the book at the extreme right a customer can buy the middle
book and the adjacent one. Then:
A. The adjacent book referred to is at the left of the middle book
B. The middle book sells for 36
C. The cheapest book sells for 4
D. The most expensive book sells for 64
E. None of these is correct (AHSME 1961/24)

There are 31 books, so the middle book has 15 books to its right, and 15 books to its left. Let the middle book
have value 𝑥. Then, the prices of the books are:
𝑥 − 30, 𝑥 − 28, … , 𝑥 − 2, ⏟
𝑥 , 𝑥 + 2, … , 𝑥 + 30
𝑴𝒊𝒅𝒅𝒍𝒆
𝑩𝒐𝒐𝒌

The adjacent book referred to could be to the right or to the left. Try the right first:
𝑥 + 𝑥 + 2 = 𝑥 + 30 ⇒ ⏟ 𝑥 = 28 ⇒ ⏟ 𝑥 − 30 = −2
𝑀𝑖𝑑𝑑𝑙𝑒 𝐵𝑜𝑜𝑘 𝐶ℎ𝑒𝑎𝑝𝑒𝑠𝑡 𝐵𝑜𝑜𝑘
Which we reject since the price of a book cannot be negative.

Hence, the book referred to must be to the left, making option A correct.

P a g e 62 | 274
Get all the files at: https://bit.ly/azizhandouts
Aziz Manva (azizmanva@gmail.com)

We can confirm as well:


𝑥 − 2 + 𝑥 = 𝑥 + 30 ⇒ 𝑥 = 32
⏟ ⇒ ⏟
𝑥 − 30 = 2
𝑴𝒊𝒅𝒅𝒍𝒆 𝑩𝒐𝒐𝒌 𝑪𝒉𝒆𝒂𝒑𝒆𝒔𝒕 𝑩𝒐𝒐𝒌

2.83: Sides in Arithmetic Sequence in a Right-Angled Triangle


If the sides of a right-angled triangle form an arithmetic sequence, then they are in the ratio
𝟑: 𝟒: 𝟓
And hence a multiple of the Primitive Pythagorean Triplet (𝟑, 𝟒, 𝟓)

Let the terms in the sequence be


𝑎−𝑑
⏟ , ⏟
𝑎 , 𝑎+𝑑

𝑺𝒉𝒐𝒓𝒕𝒆𝒓 𝑳𝒆𝒈 𝑳𝒐𝒏𝒈𝒆𝒓 𝑳𝒆𝒈 𝑯𝒚𝒑𝒐𝒕𝒆𝒏𝒖𝒔𝒆

And, by the Pythagoras Theorem, we must have:


(𝑎 − 𝑑)2 + 𝑎2 = (𝑎 + 𝑑)2
Expand
𝑎2 − 2𝑎𝑑 + 𝑑2 + 𝑎2 = 𝑎2 + 2𝑎𝑑 + 𝑑2
Simplify:
𝑎2 − 4𝑎𝑑 = 0
Divide by 𝑎 both sides (which we can since 𝑎 > 0 because it is the side of a triangle):
𝑎 − 4𝑑 = 0
We can now find the values of the sides:
𝑎 = 4𝑑 ⇒ 𝑎 − 𝑑 = 3𝑑 ⇒ 𝑎 + 𝑑 = 5𝑑
Sides are:
3𝑑: 4𝑑: 5𝑑 = 3: 4: 5

Example 2.84: Sides of a Triangle


If the lengths of the sides of a right-angled triangle with hypotenuse 15 form an arithmetic sequence, then find
the area of the triangle.

See the previous property:


{3,4,5} × 3 = {9,12,15}
9 × 12
Area = = 54
2

2.85: Average for Odd Number of Terms (Alternate Notation)


𝑎𝑛−𝑘 + ⋯ + 𝑎𝑛−1 + 𝑎𝑛 + 𝑎𝑛+1 + ⋯ + 𝑎𝑛+𝑘
= 𝑎𝑛
2𝑘 + 1

C. Subsequences

2.86: Subsequences
The first, third, and fifth terms of an arithmetic sequence themselves form an arithmetic sequence

Consider a five-term arithmetic sequence with middle term (= 3𝑟𝑑 𝑡𝑒𝑟𝑚) 𝑎 and common difference 𝑑:
𝑎 − 2𝑑 , 𝑎 − 𝑑, ⏟
⏟ 𝑎 , 𝑎 + 𝑑, ⏟
𝑎 + 2𝑑
1𝑠𝑡 3𝑟𝑑 5𝑡ℎ
𝑇𝑒𝑟𝑚 𝑇𝑒𝑟𝑚 𝑇𝑒𝑟𝑚
Isolate and write out only the first, third and fifth terms:

P a g e 63 | 274
Get all the files at: https://bit.ly/azizhandouts
Aziz Manva (azizmanva@gmail.com)

𝑎 − 2𝑑 , ⏟
⏟ 𝑎 ,⏟
𝑎 + 2𝑑
1𝑠𝑡 2𝑛𝑑 3𝑟𝑑
𝑇𝑒𝑟𝑚 𝑇𝑒𝑟𝑚 𝑇𝑒𝑟𝑚

Example 2.87: Subsequences


A. Do the first, fifth and ninth term of an arithmetic sequence themselves form an arithmetic sequence?
B. Do the second, fourth and seventh terms of an arithmetic themselves form an arithmetic sequence?

Part A The common difference of the original sequence is


Write out the terms with first term 𝑎 and common 𝑑. Common difference of the subsequence
difference 𝑑: = (5 − 1)𝑑 = (9 − 5)𝑑 = 4𝑑
𝑎, 𝑎 + 4𝑑, 𝑎 + 8𝑑 Part B
They form an arithmetic sequence with 𝑡2 = 𝑎 + 𝑑
𝐹𝑖𝑟𝑠𝑡 𝑇𝑒𝑟𝑚 = 𝑎, 𝐶𝑜𝑚𝑚𝑜𝑛 𝑑𝑖𝑓𝑓𝑒𝑟𝑒𝑛𝑐𝑒 = 4𝑑 𝑡4 = 𝑎 + 3𝑑
𝑡7 = 𝑎 + 6𝑑
We can get the common difference between the
terms in a different way 𝑡4 − 𝑡2 = (𝑎 + 3𝑑) − (𝑎 + 𝑑) = 2𝑑
𝑡1 = 𝑎 𝑡7 − 𝑡4 = (𝑎 + 6𝑑) − (𝑎 + 3𝑑) = 3𝑑
𝑡5 = 𝑎 + 4𝑑
𝑡9 = 𝑎 + 8𝑑 Does not form an arithmetic sequence

2.88: Sub Sequences


Given an arithmetic sequence
𝑎𝑛−𝑘 , … , 𝑎𝑛−1 , 𝑎𝑛 , 𝑎𝑛+1 , … , 𝑎𝑛+𝑘
the terms
𝑎𝑛−𝑘 , 𝑎𝑛 , 𝑎𝑛+𝑘
also form an arithmetic sequence

Example 2.89
Show how the sub sequences of the arithmetic sequence
5,9,13,17,21,25,29
Are also arithmetic sequences

13 + 21 9 + 25 5 + 29 34
= = = = 17
2 2 2 2
In other words, each of the sequences below is an arithmetic sequence:
13,17,21
9,17,25
5,17,29

D. Even Number of Terms in an Arithmetic Sequence

P a g e 64 | 274
Get all the files at: https://bit.ly/azizhandouts
Aziz Manva (azizmanva@gmail.com)

Example 2.90: Four Terms in an Arithmetic Sequnce


A. Find the first four terms of an arithmetic sequence with first term 𝑎, and common difference 𝑑. Then,
find their sum.
B. Find the first four terms of an arithmetic sequence with first term 𝑎 − 3𝑑, and common difference 2𝑑.
Then, find their sum.
C.

Part A
𝑎, 𝑎 + 𝑑, 𝑎 + 2𝑑, 𝑎 + 3𝑑
𝑆𝑢𝑚 = 𝑎 + 𝑎 + 𝑑 + 𝑎 + 2𝑑 + 𝑎 + 3𝑑 = 4𝑎 + 6𝑑
Part B
𝑎 − 3𝑑, 𝑎 − 𝑑, 𝑎 + 𝑑, 𝑎 + 3𝑑
𝑆𝑢𝑚 = (𝑎 − 3𝑑) + (𝑎 − 𝑑) + (𝑎 + 𝑑) + (𝑎 + 3𝑑) = 4𝑎
Suppose the terms in an arithmetic sequence are
3,5,7,9, 𝑎 = 3, 𝑑 = 2
We can take assume the terms in variables as below:

3, ⏟
5 , ⏟
7 , ⏟
9
𝑎 𝑎+𝑑 𝑎+2𝑑 𝑎+3𝑑
But, the above does not cancel out when we add. So, there is another way to do it, where we assume that
𝑎 = 6 = 𝐴𝑣𝑒𝑟𝑎𝑔𝑒 𝑜𝑓 𝑡ℎ𝑒 𝑓𝑜𝑢𝑟 𝑡𝑒𝑟𝑚𝑠
𝑑 = 1 = 𝐻𝑎𝑙𝑓 𝑡ℎ𝑒 𝑐𝑜𝑚𝑚𝑜𝑛 𝑑𝑖𝑓𝑓𝑒𝑟𝑒𝑛𝑐𝑒
Then, the terms that we have become:

3 , ⏟
5 , ⏟
7 , ⏟
9
𝑎−3𝑑 𝑎−𝑑 𝑎+𝑑 𝑎+3𝑑
Here the common difference in the above sequence is 𝟐𝒅, and not 𝒅.

Example 2.91
The fourth power of the common difference of an arithmetic progression with integer entries is added to the
product of any four consecutive terms of it. Prove that the resulting sum is the square of an integer. (IIT JEE
Advanced, 2000)

(𝑎 − 3𝑑)(𝑎 − 𝑑)(𝑎 + 𝑑)(𝑎 + 3𝑑) + (2𝑑)4 = (𝑎4 − 10𝑑2 𝑎2 + 25𝑑4 ) = (𝑎2 − 5𝑑2 )2

2.92: Finding the average of the two middle terms


If four terms of a sequence add up to 𝒙, then the average of the two middle terms of the sequence is given by
𝒙
b𝟒

Assuming four terms in the sequence and equating them to the sum:
𝑥
(𝑎 − 𝟑𝒅) + (𝑎 − 𝑑) + (𝑎 + 𝒅) + (𝑎 + 3𝑑) = 𝑥 ⇒ 4𝑎 = 𝑥 ⇒ 𝑎 =
4

The average of the two middle terms is:


(𝑎 − 𝑑) + (𝑎 + 𝑑) 2𝑎
= =𝑎
2 2

2.93: Average for Even Number of Terms


The average of an arithmetic sequence with an even number of terms is equal to the average of its two
middlemost terms:

P a g e 65 | 274
Get all the files at: https://bit.ly/azizhandouts
Aziz Manva (azizmanva@gmail.com)

𝒂𝟏 + ⋯ + 𝒂𝒌 + 𝒂𝒌 + ⋯ + 𝒂𝟐𝒌 𝒂𝒌 + 𝒂𝒌
+𝟏 +𝟏
𝟐 𝟐 𝟐 𝟐
=
𝒌 𝟐

Example 2.94: Four Terms in an Arithmetic Sequence


A. The arithmetic mean (ordinary average) of the fifty-two successive positive integers beginning at 2 is:
(AHSME 1969/9)

We want the average of the two middlemost terms. (These are the same terms that we would use to find the
median).
26𝑡ℎ 𝑇𝑒𝑟𝑚 = 26 + 1 = 27
27𝑡ℎ 𝑇𝑒𝑟𝑚 = 27 + 1 = 28

27 + 28
= 27.5
2

Example 2.95: Four Terms in an Arithmetic Sequence


The total of four consecutive terms in an arithmetic sequence is 102.
A. Find the value of the largest term, such that the common difference has the smallest positive integral
value possible.
B. If the value of the second term is 21, find the value of the fourth term.

Part A 24 + 25 + 26 + 27 = 102 (𝑎𝑠 𝑟𝑒𝑞𝑢𝑖𝑟𝑒𝑑) ⇒ 𝑙 = 27


Let the first term be 𝑎 − 3𝑑, and common difference Part B
be 2𝑑. Let the four terms be: Let the terms be:
𝑎 − 3𝑑, 𝑎 − 𝑑, 𝑎 + 𝑑, 𝑎 + 3𝑑 𝑎1 , 𝑎2 , 𝑎3 , 𝑎4
Then the sum of terms is:
4𝑎 = 102 ⇒ 𝑎 = 25.5 𝑎2 + 𝑎3 102 51
= =
If we take the common difference to be one: 2 4 2
1 Substitute 𝑎2 = 21:
2𝑑 = 1 ⇒ 𝑑 = 21 + 𝑎3 51
2 = ⇒ 𝑎3 = 51 − 21 = 30
1 2 2
𝑎 − 𝑑 = 25.5 − = 25
2
1 𝐶𝑜𝑚𝑚𝑜𝑛 𝑑𝑖𝑓𝑓𝑒𝑟𝑒𝑛𝑐𝑒 = 30 − 21 = 9
𝑎 + 𝑑 = 25.5 + = 26
2 The terms are:
1 12,21,30,39
𝑎 − 3𝑑 = 25.5 − 3 ( ) = 24
2 12 + 21 + 30 + 39 = 102
1
𝑎 + 3𝑑 = 25.5 + 3 ( ) = 27
2

Example 2.96: Geometrical Applications


A. The angles in a quadrilateral form an arithmetic sequence. Find the average of the two middle terms.
B. The measures of the interior angles of a convex hexagon form an increasing arithmetic sequence. How
many such sequences are possible if the hexagon is not equiangular and all of the angle degree
measures are positive integers less than 150 degrees? (MathCounts 2010 State Team)

Part A 𝑎2 + 𝑎3 𝑎1 + 𝑎2 + 𝑎3 + 𝑎4 360
∴ = = = 90°
Again, by symmetry, the average of the two middle 2 4 4
terms must be the average of all the terms, which 𝑎2 + 𝑎3 = 180°
then add up to the angles of a quadrilateral. Part B
The sum of angles of a hexagon is:

P a g e 66 | 274
Get all the files at: https://bit.ly/azizhandouts
Aziz Manva (azizmanva@gmail.com)

(𝑛 − 2)180 = (6 − 2)180 = (4)(180) = 720 2𝑑 = 8: 100,108,116,124,132,140


Method I 2𝑑 = 10: 95,105,115,125,135,145
The question says that the hexagon is not Method II
equiangular. Start, nevertheless with an Let the first term be 𝑎 − 6𝑑 and the common
equiangular hexagon: difference be 2𝑑:
120,120,120,120,120,120 (𝑎 − 5𝑑), (𝑎 − 3𝑑), (𝑎 − 𝑑), (𝑎 + 𝑑), (𝑎 + 3𝑑), (𝑎
This is an arithmetic sequence with common + 5𝑑)
difference zero. Make this into an arithmetic
sequence with (𝑎 − 5𝑑) + (𝑎 − 3𝑑) + ⋯ + (𝑎 + 5𝑑) = 720 ⇒ 6𝑎
120 − 5,120 − 3,120 − 1,120 + 1,120 + 3,120 + 5 = 720 ⇒ 𝑎 = 120
2𝑑 = 2: 115,117,119,121,123,125 𝑎 + 5𝑑 < 150
120 + 5𝑑 < 150
120 − 10,120 − 6,120 − 2,120 + 2,120 + 6,120 5𝑑 < 30
+ 10 𝑑<6
2𝑑 = 4: 110,114,118,122,126,130 𝑑∈ {1,2,3,4,5}
2𝑑 = 6: 105,111,117,123,129,135

Example 2.97: Circles


𝐷𝑒𝑓𝑖𝑛𝑖𝑡𝑖𝑜𝑛: The central angle of an arc refers to the angle between two radii from the
center of the circle to the start and endpoints of the arc. For example, in the diagram the
angle subtended by 𝐴𝑟𝑐 𝐴𝐵𝐶 = ∠𝐴𝑂𝐶.
𝐼𝑛𝑓𝑜𝑟𝑚𝑎𝑡𝑖𝑜𝑛: A circle of radius 1 is divided into 𝑛 arcs. The central angle subtended by
these arcs is in arithmetic progression.
A. If 𝑛 = 3, then the measure of the middle arc is 𝑥. If 𝑛 = 4, then the average
measure of the two middle arcs is 𝑦. Find 𝑥 − 𝑦.
B. If 𝑛 = 3, and the measures of the angles subtended by the arcs are distinct positive integers, when
measured in degrees, find how many such circles can be made. (Two circles with different radii, but
same angle measures for their arcs are considered same).
C. If 𝑛 = 5, find the area of the sector formed by the middle term in the arithmetic progression.

1 ≤ 𝑎1 ≤ 119 ⇒ 𝑎1 𝑐𝑎𝑛 𝑡𝑎𝑘𝑒 119 𝑣𝑎𝑙𝑢𝑒𝑠


Part A Part C
360 𝐴𝑟𝑒𝑎 𝑜𝑓 𝐶𝑖𝑟𝑐𝑙𝑒 = 𝜋𝑟 2 = 𝜋(1)2 = 𝜋
𝑥= = 120
3 𝜃
360 𝐴𝑟𝑒𝑎 𝑜𝑓 𝑆𝑒𝑐𝑡𝑜𝑟 = 𝜋𝑟 2 × ,
𝑦= = 90 360
4 𝜃 = 𝐴𝑛𝑔𝑙𝑒 𝑠𝑢𝑏𝑡𝑒𝑛𝑑𝑒𝑑 𝑏𝑦 𝑎𝑟𝑐
𝑥 − 𝑦 = 120 − 90 = 30
Part B Since the angles subtended by the arcs are in
𝑎1 + 120 + 𝑎3 = 360 arithmetic progression, so will the areas of the
𝑎1 + 𝑎3 = 240 corresponding sectors.
can tabulate the number of solutions by deciding
the value of 𝑎1 : Hence, area of the sector formed by middle term in
the arithmetic progression
𝑎1 119 118 117 . . . 1 𝑆𝑢𝑚 𝑜𝑓 𝐴𝑟𝑒𝑎𝑠 𝑜𝑓 𝐴𝑙𝑙 𝑆𝑒𝑐𝑡𝑜𝑟𝑠 𝐴𝑟𝑒𝑎 𝑜𝑓 𝐶𝑖𝑟𝑐𝑙𝑒
𝑎2 120 120 120 . . . 120 = =
5 5
𝑎3 121 122 123 . . . 239 𝜋
=
5

Example 2.98: Triangle Inequality


How many distinct, non-equilateral triangles with a perimeter of 60 units have integer side lengths 𝑎, 𝑏, and 𝑐

P a g e 67 | 274
Get all the files at: https://bit.ly/azizhandouts
Aziz Manva (azizmanva@gmail.com)

such that 𝑎, 𝑏, 𝑐 is an arithmetic sequence? (Mathcounts 2004 National Sprint)

20,20,20 - Equilateral

19,20,21 ⇒ 19 + 20 > 21 ⇒ 39 > 21 ⇒ 𝑽𝒂𝒍𝒊𝒅 𝑻𝒓𝒊𝒂𝒏𝒈𝒍𝒆


10,20,30 ⇒ 10 + 20 > 30 ⇒ 𝑵𝒐𝒕 𝑻𝒓𝒖𝒆 ⇒ 𝑵𝒐𝒕 𝑽𝒂𝒍𝒊𝒅
11,20,29 ⇒ 11 + 20 > 3031 > 30 ⇒ 𝑽𝒂𝒍𝒊𝒅 𝑻𝒓𝒊𝒂𝒏𝒈𝒍𝒆

Hence, the smallest side can take values:


11,12, … ,18,19
Which gives us the triangles:

{(11,20,29),
⏟ (12,20,28) … (18,20,22)(19,20,21)}
𝟗 𝑷𝒐𝒔𝒔𝒊𝒃𝒊𝒍𝒊𝒕𝒊𝒆𝒔

E. Symmetry
Symmetry refers to balance. For example, geometrical figures can have symmetry if they have two equal halves.

Let’s consider symmetry in an arithmetic sequence. A arithmetic sequence can be represented as below:
𝑎 − 𝑛𝑑, …, ⏟𝑎 − 2𝑑 , 𝑎 − 𝑑, ⏟
𝑎 , 𝑎 + 𝑑, ⏟𝒂 + 𝟐𝒅 ,… , 𝑎 + 𝑛𝑑
𝑻𝒘𝒐 𝒕𝒆𝒓𝒎𝒔 𝑴𝒊𝒅𝒅𝒍𝒆 𝑻𝒘𝒐 𝒕𝒆𝒓𝒎𝒔
𝒃𝒆𝒇𝒐𝒓𝒆 𝒎𝒊𝒅𝒅𝒍𝒆 𝒕𝒆𝒓𝒎 𝑻𝒆𝒓𝒎 𝒂𝒇𝒕𝒆𝒓 𝒎𝒊𝒅𝒅𝒍𝒆 𝒕𝒆𝒓𝒎

If we take the average of two terms such that


➢ the first term is 𝑥 terms ahead of the middle term,
➢ the second is 𝑥 terms behind the middle term
We will obtain the middle term itself.

(𝑎 + 2𝑑)
⏟ + (𝑎 − 2𝑑)

𝑇𝑤𝑜 𝑇𝑒𝑟𝑚𝑠 𝑎ℎ𝑒𝑎𝑑 𝑇𝑤𝑜 𝑇𝑒𝑟𝑚𝑠 𝐵𝑒ℎ𝑖𝑛𝑑 2𝑎
= =𝑎
2 2

(𝑎 + 𝑛𝑑)
⏟ + (𝑎 − 2𝑑)

𝑇𝑤𝑜 𝑇𝑒𝑟𝑚𝑠 𝑎ℎ𝑒𝑎𝑑 𝑇𝑤𝑜 𝑇𝑒𝑟𝑚𝑠 𝐵𝑒ℎ𝑖𝑛𝑑 2𝑎
= =𝑎
2 2

2.99: Symmetry in Average and Median


➢ When a sequence is arranged in ascending (or descending) order, the median is the middle term if there
are an odd number of terms.
➢ An arithmetic sequence has a constant difference. Hence, its arithmetic mean is also its median.

Odd Term Case


𝑎 − 𝑑, 𝑎, 𝑎 + 𝑑 ⇒ 𝑀𝑒𝑑𝑖𝑎𝑛 = 𝑎
𝑎 − 𝑑, 𝑎, 𝑎 + 𝑑 ⇒ 𝑀𝑒𝑎𝑛 = 𝑎
Even Term Case
𝑎 − 3𝑑, 𝑎 − 𝑑, 𝑎 + 𝑑, 𝑎 + 3𝑑 ⇒ 𝑀𝑒𝑑𝑖𝑎𝑛 = 𝐴𝑣𝑔(𝑎 − 𝑑, 𝑎 + 𝑑) = 𝑎
4𝑎
𝑎 − 3𝑑, 𝑎 − 𝑑, 𝑎 + 𝑑, 𝑎 + 3𝑑 ⇒ 𝑀𝑒𝑎𝑛 = =𝑎
4

Example 2.100

P a g e 68 | 274
Get all the files at: https://bit.ly/azizhandouts
Aziz Manva (azizmanva@gmail.com)

A. Consider the numbers 1,9,2. Find their mean and median. Is the mean equal to the median?
B. Show that the mean of the arithmetic sequence 3,7,11,15,19 is also its median.
C. The sum of 49 consecutive integers is 75 . What is their median? (AMC 12A 2004/10)
D. The sum of 32 numbers in an arithmetic sequence is 212 . What is their median?

Part A Median is the same as arithmetic mean:


Arrange the numbers is ascending order 𝑆𝑢𝑚
𝑀𝑒𝑑𝑖𝑎𝑛 = 𝐴𝑟𝑖𝑡ℎ𝑚𝑒𝑡𝑖𝑐 𝑀𝑒𝑎𝑛 =
1,2,9 ⇒ 𝑀𝑒𝑑𝑖𝑎𝑛 = 2 𝑛
1 + 2 + 9 12 75
𝐴𝑣𝑒𝑟𝑎𝑔𝑒 = = =4 = = 73 = 343
3 3 49
𝑀 Part D
Part B 𝑆𝑢𝑚
𝑀𝑒𝑑𝑖𝑎𝑛 = 𝐴𝑟𝑖𝑡ℎ𝑚𝑒𝑡𝑖𝑐 𝑀𝑒𝑎𝑛 =
3,7,11,15,19 ⇒ 𝑀𝑒𝑑𝑖𝑎𝑛 = 11 𝑛
𝑀𝑒𝑎𝑛 = 11 212
= 27 = 128
32
Part C

Example 2.101
Let 𝑎1 , 𝑎2 , … 𝑎𝑘 be a finite arithmetic sequence with 𝑎4 + 𝑎7 + 𝑎10 = 17 and 𝑎4 + 𝑎5 + ⋯ + 𝑎13 + 𝑎14 = 77. If
𝑎𝑘 = 13, then 𝑘 = (AHSME 1993/21)

𝑎4 + 𝑎7 + 𝑎10 = 17
Find 𝒌
𝑎7 is the middle term of an arithmetic sequence Now that we know the common difference, we can
consisting of 𝑎4 , 𝑎7 , 𝑎10 . Hence, it is the average of find 𝑘. Start by checking how many times of the
the three terms: common difference 𝑎𝑘 is ahead of 𝑎9 :
𝑆𝑢𝑚 𝑜𝑓 𝑇𝑒𝑟𝑚𝑠 17
𝑎7 = = 𝑎𝑘 = 13 = 7 + 𝑛𝑑
𝑁𝑜. 𝑜𝑓 𝑇𝑒𝑟𝑚𝑠 3 Solve the above equation for 𝑛:
𝑎9 is the middle term of an arithmetic sequence 6
consisting of 𝑎4 , 𝑎5 , … , 𝑎14. Hence, it is the average 𝑛𝑑 = 13 − 7 = 6 ⇒ 𝑛 =
𝑑
of all the terms: Substitute 𝑑 = 3:
2
𝑆𝑢𝑚 𝑜𝑓 𝑇𝑒𝑟𝑚𝑠 77
𝑎9 = = =7 6 6 3
𝑁𝑜. 𝑜𝑓 𝑇𝑒𝑟𝑚𝑠 11 𝑛 = = =6× =9
𝑑 2 2
Since 𝑎9 is two terms ahead of 𝑎7 , the difference 3
between them must be twice of the common So now we know that 𝑎𝑘 is nine terms ahead of 𝑎7 ,
difference. Hence: which tells us:
17 21 − 17 4 2 𝑎𝑘 = 𝑎9 + 9𝑑 = 𝑎18
2𝑑 = 𝑎9 − 𝑎7 = 7 − = = ⇒𝑑=
3 3 3 3

Challenge 2.102
What is the smallest positive integer that can be expressed as the sum of nine consecutive integers, the sum of
ten consecutive integers, and the sum of eleven consecutive integers? (AIME 1993/6)

Defining Variables
Let the integer that we want be 𝑥. We have three different arithmetic sequences:
(𝑎1 , 𝑎2 , … , 𝑎9 ), (𝑏1 , 𝑏2 , … , 𝑏10 ), (𝑐1 , 𝑐2 , … , 𝑐11 )
Using Properties
Focus only on the middle term of each sequence.
We can use the property for the average of an arithmetic sequence with an odd number of terms:

P a g e 69 | 274
Get all the files at: https://bit.ly/azizhandouts
Aziz Manva (azizmanva@gmail.com)

𝑥 𝑥
𝑎5 = , 𝑐6 = ⇒ 𝑥 𝑖𝑠 𝑑𝑖𝑣𝑖𝑠𝑖𝑏𝑙𝑒 𝑏𝑦 9 𝑎𝑛𝑑 11
9 11
And we can also use the property for the average of an arithmetic sequence with an even number of terms:
𝑏5 + 𝑏6 𝑥 𝑥
= ⇒ 𝑏5 + 𝑏6 = ⇒ 𝑥 𝑖𝑠 𝑑𝑖𝑣𝑖𝑠𝑖𝑏𝑙𝑒 𝑏𝑦 5
2 10 5
Hence, the smallest number we want is:
𝐿𝐶𝑀(5,9,11) = 495

Example 2.103
In the five-sided star shown, the letters A, B, C, D, and E are replaced by the numbers 3, 5, 6, 7, and 9, although
not necessarily in this order. The sums of the numbers at the ends of the line segments AB, BC, CD, DE, and EA
form an arithmetic sequence, although not necessarily in that order. What is the middle term of the arithmetic
sequence? (AMC 10A 2005/17) (Ans=12)

F. Structure

Example 2.104
Find the value of 𝑎2 + 𝑎4 + 𝑎6 + 𝑎8 +. . . +𝑎98 if 𝑎1 , 𝑎2 , 𝑎3 . .. is an arithmetic progression with common difference
1, and 𝑎1 + 𝑎2 + 𝑎3 +. . . +𝑎98 = 137. (AIME 1984/1)

𝑎1 + 𝑎2 + 𝑎3 +. . . +𝑎98 = 137
Since we want to find the value of the even terms, rewrite each odd term in terms of the even term that comes
after it:
(𝑎2 − 1) + 𝑎2 + (𝑎4 − 1) + 𝑎4 + ⋯ + (𝑎98 − 1) + 𝑎98 = 137
Add all the terms that have −1 in them, and note that each even term occurs twice:
2(𝑎2 + 𝑎4 + 𝑎6 + 𝑎8 +. . . +𝑎98 ) − 49 = 137
Isolate and solve for the expression we want:
137 + 49
𝑎2 + 𝑎4 + 𝑎6 + 𝑎8 + ⋯ + 𝑎98 = = 93
2

Challenge 2.105
The terms of an arithmetic sequence add to 715. The first term of the sequence is increased by 1, the second
term is increased by 3, the third term is increased by 5, and in general, the 𝑘 𝑡ℎ term is increased by the 𝑘 𝑡ℎ odd
positive integer. The terms of the new sequence add to 836. Find the sum of the first, last, and middle terms of
the original sequence. (AIME 2012/I/2)

This mixes arithmetic sequences with sums of odd integers. The key idea here is to recognize that:
𝑆𝑢𝑚 𝑜𝑓 𝐴𝑟𝑖𝑡ℎ𝑚𝑒𝑡𝑖𝑐 𝑆𝑒𝑞𝑢𝑒𝑛𝑐𝑒 = 715
𝑆𝑢𝑚 𝑜𝑓 𝐴𝑟𝑖𝑡ℎ𝑚𝑒𝑡𝑖𝑐 𝑆𝑒𝑞𝑢𝑒𝑛𝑐𝑒 + ⏟ 𝐾 𝑜𝑑𝑑 𝑝𝑜𝑠𝑖𝑡𝑖𝑣𝑒 𝑖𝑛𝑡𝑒𝑔𝑒𝑟𝑠 = 836
𝑺𝒕𝒂𝒓𝒕𝒊𝒏𝒈 𝒇𝒓𝒐𝒎 𝟏
Hence, if we subtract the first equation from the second equation, we get just the sum of the k odd positive
integers:
𝐾 𝑜𝑑𝑑 𝑝𝑜𝑠𝑖𝑡𝑖𝑣𝑒 𝑖𝑛𝑡𝑒𝑔𝑒𝑟𝑠 = 836 − 715 = 121 = 112 ⇒ 𝐾 = 11

𝑺𝒕𝒂𝒓𝒕𝒊𝒏𝒈 𝒇𝒓𝒐𝒎 𝟏
Since 𝐾 = 11, there must be 11 terms in the original sequence. We need to find the sum of the first, last and
middle terms, by symmetry, is three times the middle term:
𝑎1 + 𝑎𝑚𝑖𝑑𝑑𝑙𝑒 + 𝑎11 = 3(𝑎𝑚𝑖𝑑𝑑𝑙𝑒 )
Also, the middle term must be the average of the 11 terms of the sequence:

P a g e 70 | 274
Get all the files at: https://bit.ly/azizhandouts
Aziz Manva (azizmanva@gmail.com)

715
3(𝑎𝑚𝑖𝑑𝑑𝑙𝑒 ) = 3(𝐴𝑣𝑔) = 3 ( ) = 3 × 65 = 195
11

G. Common AP of Two Sequences

Example 2.106
Let 𝑆 be the set of the 2005 smallest positive multiples of 4, and let 𝑇 be the set of the 2005 smallest positive
multiples of 6. How many elements are common to 𝑆 and 𝑇? (AMC 10A 2005/22)

4,8, 𝟏𝟐, 16,20, 𝟐𝟒, 28,32, 𝟑𝟔, 40, …


6, 𝟏𝟐, 18, 𝟐𝟒, …
The common AP of both sequences is:
12,2436, …
Since every 3rd multiple of 4 is a multiple of 12, we calculate
2005 1
= 668
3 3
Drop the fractional part to get:
668

Example 2.107
Let 1; 4 … and 9; 16; … be two arithmetic progressions. The set 𝑆 is the union of the first 2004 terms of each
sequence. How many distinct numbers are in 𝑆? (AMC10B 2004/21)
Number Theory
The first arithmetic sequence is 𝐴 = 1,4, 7, … with
𝐹𝑖𝑟𝑠𝑡 𝑇𝑒𝑟𝑚 = 𝑎 = 1, 𝐶𝑜𝑚𝑚𝑜𝑛 𝑑𝑖𝑓𝑓𝑒𝑟𝑒𝑛𝑐𝑒 = 𝑑 = 3, 𝐺𝑒𝑛𝑒𝑟𝑎𝑙 𝑇𝑒𝑟𝑚 = 3𝑚 + 1, 𝑚 ∈ (0,2003)

The second arithmetic sequence is 𝐵 = 9,16, … with


𝑎 = 9, 𝑑 = 7, 𝐺𝑒𝑛𝑒𝑟𝑎𝑙 𝑇𝑒𝑟𝑚 = 7𝑛 + 2, 𝑛 ∈ (1, 2003)

We find the numbers that occur in both sequences:


7𝑛 + 2
3𝑚 + 1 = 7𝑛 + 2 ⇒ 𝑚 = ⇒ (𝑚, 𝑛) = (5,2), (12,5), (19,8), …
3

𝑚 = 5,12,19, … ,2000
Add 2 to the above list:
7,14,21 … 2002 = 286 values

Use the formula for the cardinality of the union of a set:


𝑛(𝑆) = 𝑛(𝐴 ∪ 𝐵) = 𝑛(𝐴) + 𝑛(𝐵) − 𝑛(𝐴 ∩ 𝐵) = 2004 + 2004 − 286 = 3722

Method II
Find the common AP of both sets

H. Recursive Creation of Sequences

Example 2.108
Seven students count from 1 to 1000 as follows:
•Alice says all the numbers, except she skips the middle number in each consecutive group of three numbers.

P a g e 71 | 274
Get all the files at: https://bit.ly/azizhandouts
Aziz Manva (azizmanva@gmail.com)

That is, Alice says 1, 3, 4, 6, 7, 9, ..., 997, 999, 1000.


•Barbara says all of the numbers that Alice doesn't say, except she also skips the middle number in each
consecutive group of three numbers.
•Candice says all of the numbers that neither Alice nor Barbara says, except she also skips the middle number in
each consecutive group of three numbers.
•Debbie, Eliza, and Fatima say all of the numbers that none of the students with the first names beginning
before theirs in the alphabet say, except each also skips the middle number in each of her consecutive groups of
three numbers.
•Finally, George says the only number that no one else says.
What number does George say? (AMC 10A 2011/23)

The number from 1 to 1000 are


1,2,3,4,5, … ,1000
Alice says the numbers:
1,3,4,6,7,9, … ,997,999,1000
Alice does not say the numbers
2,5,8, … ⇒ 𝑎 = 2, 𝑑 = 3
Barbara says the numbers:
2,8,11,17, …
Barbara does not say the numbers:
5,14, … ⇒ 𝑎 = 5, 𝑑 = 9
Candice does not say the numbers:
14,41, … ⇒ 𝑎 = 14, 𝑑 = 27
Debbi does not say the numbers:
41,122 … ⇒ 𝑎 = 41, 𝑑 = 81
Eliza does not say the numbers:
122,365 … ⇒ 𝑎 = 122, 𝑑 = 243
Fatima does not say the numbers:
365,1094, … ⇒ 𝑎 = 365, 𝑑 = 729

We now have only one number less than 1000, which is 365.
This is the answer.

I. Statistics

Example 2.109
When the mean, median, and mode of the list 10,2,5,2,4,2, 𝑥 are arranged in increasing order, they form a non-
constant arithmetic progression. What is the sum of all possible real values of 𝑥? (AMC 10 2000/23, AMC12
2000/14)

Find the Mode, the Mean and the Median


𝑀𝑜𝑑𝑒 = 𝐻𝑖𝑔ℎ𝑒𝑠𝑡 𝐹𝑟𝑒𝑞𝑢𝑒𝑛𝑐𝑦 = 2
10 + 2 + 5 + 2 + 4 + 2 + 𝑥 25 + 𝑥
𝑀𝑒𝑎𝑛 = =
7 7

For the median, arrange the numbers in ascending order:


2,2,2,4,5,10
The value of the median will depend on the value of 𝑥:

P a g e 72 | 274
Get all the files at: https://bit.ly/azizhandouts
Aziz Manva (azizmanva@gmail.com)

𝑥, 2,2,2,4,5,10 ,
⏟ 2,2,2, 𝑥, 4,5,10 ,
⏟ 2,2,2,4,5,10, 𝑥 ⇒ 𝑀𝑒𝑑𝑖𝑎𝑛 ∈ {2, 𝑥, 4}

𝑀𝑒𝑑𝑖𝑎𝑛=2 𝑀𝑒𝑑𝑖𝑎𝑛=𝑥 𝑀𝑒𝑑𝑖𝑎𝑛=4

Do some Casework
Case I: 𝑴𝒆𝒅𝒊𝒂𝒏 = 𝟐
𝑀𝑜𝑑𝑒 = 𝑀𝑒𝑑𝑖𝑎𝑛 = 2 ⇒ 𝐴𝑃 𝑖𝑠 𝑐𝑜𝑛𝑠𝑡𝑎𝑛𝑡 ⇒ 𝑪𝒐𝒏𝒕𝒓𝒂𝒅𝒊𝒄𝒕𝒊𝒐𝒏

Case II: 𝑴𝒆𝒅𝒊𝒂𝒏 = 𝟒, 𝒙 > 𝟒


𝑀𝑜𝑑𝑒 = 2, 𝑀𝑒𝑑𝑖𝑎𝑛 = 4 ⇒ 𝐴𝑃 = {0,2,4}{2,3,4}{2,4,6}
25 + 𝑥 25 + 𝑥 25 + 𝑥
= 0 ⇒ 𝑥 = −25 , = 3 ⇒ 𝑥 = −4 , = 6 ⇒ 𝑥 = 17
⏟7 ⏟7 ⏟7
𝒙>𝟒⇒𝑪𝒐𝒏𝒕𝒓𝒂𝒅𝒊𝒄𝒕𝒊𝒐𝒏 𝒙>𝟒⇒𝑪𝒐𝒏𝒕𝒓𝒂𝒅𝒊𝒄𝒕𝒊𝒐𝒏 𝑵𝒐 𝑪𝒐𝒏𝒕𝒓𝒂𝒅𝒊𝒄𝒕𝒊𝒐𝒏

Case III: 𝑴𝒆𝒅𝒊𝒂𝒏 = 𝒙, 𝟐 < 𝒙 < 𝟒


25 + 𝑥
𝑀𝑜𝑑𝑒 = 2, 𝑀𝑒𝑑𝑖𝑎𝑛 = 𝑥, 𝑀𝑒𝑎𝑛 =
7
Using the property that the middle term in an arithmetic progression is the mean of the terms that come before
and after it:
25 + 𝑥
2+ 7 25 + 𝑥
=𝑥 ⇒2+ = 2𝑥 ⇒ 𝑥 = 3
2 7
Find the Final Answer
Sum of all values
= 3 + 17 = 20

J. Quadratics and Simultaneous Quadratics

Challenge 2.110
If the integer 𝑘 is added to each of the numbers 36, 300, and 596, one obtains the squares of three consecutive
terms of an arithmetic sequence. Find 𝑘. (AIME 1989/7)

Let the three consecutive terms of the arithmetic sequence be:


𝑎 − 𝑑, 𝑎, 𝑎+𝑑

By the given condition:


𝑎2 = 300 + 𝑘 ⇒ ⏟
𝑘 = 𝑎2 − 300
𝐸𝑞𝑢𝑎𝑡𝑖𝑜𝑛 𝐼
(𝑎 − 𝑑)2 = ⏟
𝑎2 − 𝟐𝒂𝒅 + 𝑑2 = 36 + 𝑘
𝐸𝑞𝑢𝑎𝑡𝑖𝑜𝑛 𝐼𝐼
(𝑎 + 𝑑)2 = ⏟
𝑎2 + 𝟐𝒂𝒅 + 𝑑2 = 596 + 𝑘
𝐸𝑞𝑢𝑎𝑡𝑖𝑜𝑛 𝐼𝐼𝐼

Solve for the common difference


Add Equations II and III, and isolate 𝑘:
2𝑎2 + 2𝑑2 = 632 + 2𝑘 ⇒ ⏟
𝑘 = 𝑎2 + 𝑑2 − 316
𝑬𝒒𝒖𝒂𝒕𝒊𝒐𝒏 𝑰𝑽
Equate the two values of 𝑘 obtained from Equation I and Equation IV:
𝑎2 + 𝑑2 − 316 = 𝑎2 − 300 ⇒ 𝑑2 = 16 ⇒ 𝑑 = ±4 ⇒ 𝑑
⏟= 4
𝑰𝒏𝒄𝒓𝒆𝒂𝒔𝒊𝒏𝒈 𝑺𝒆𝒒𝒖𝒆𝒏𝒄𝒆

P a g e 73 | 274
Get all the files at: https://bit.ly/azizhandouts
Aziz Manva (azizmanva@gmail.com)

Solve for the middle term


Substitute 𝑑 = 4 in Equations II and III:
𝑎2 − 𝟖𝒂 + 16 = 36 + 𝑘 ,
⏟ ⏟2 + 𝟖𝒂 + 16 = 596 + 𝑘
𝑎
𝑬𝒒𝒖𝒂𝒕𝒊𝒐𝒏 𝑽 𝑬𝒒𝒖𝒂𝒕𝒊𝒐𝒏 𝑽𝑰
Subtract Equation V from Equation VI to get the value of 𝑎 :
16𝑎 = 560 ⇒ 𝑎 = 35 ⇒ 𝑎 − 𝑑 = 35 − 4 = 31 ⇒ (𝑎 − 𝑑)2 = 312 = 961
Find the value of k:
Use Equation II to get:
961 = 36 + 𝑘 ⇒ 𝑘 = 925

K. Proof-Type Questions

Example 2.111
If the 𝑝𝑡ℎ term of an arithmetic progression is 𝑞, and the 𝑞 𝑡ℎ term of an arithmetic progression is 𝑝, then prove
that its 𝑛𝑡ℎ term is (𝑝 + 𝑞 − 𝑛). (NCERT)

𝑝𝑡ℎ term = 𝑞 = 𝑎 + (𝑝 − 1)𝑑


𝑞 𝑡ℎ term = 𝑝 = 𝑎 + (𝑞 − 1)𝑑
Subtract the second equation from the first:
(𝑝 − 𝑞)𝑑 = 𝑞 − 𝑝 ⇒ 𝑑 = −1
Substitute 𝑑 = −1 in the equation for the 𝑝𝑡ℎ term:
𝑎 + (𝑝 − 1)𝑑 = 𝑞 ⇒ 𝑎 + 1 − 𝑝 = 𝑞 ⇒ 𝑎 = 𝑞 + 𝑝 − 1
𝑡ℎ
Find the 𝑛 term:
𝑎 + (𝑛 − 1)𝑑 = 𝑞 + 𝑝 − 1 + (𝑛 − 1)(−1) = 𝑝 + 𝑞 − 𝑛

Example 2.112
1 1
If the 𝑚𝑡ℎ term of an arithmetic progression be , and the 𝑛𝑡ℎ term be , then show that its (𝑚𝑛)𝑡ℎ term is 1.
𝑛 𝑚

1
𝑚𝑡ℎ term = = 𝑎 + (𝑚 − 1)𝑑
𝑛
1
𝑛𝑡ℎ term = = 𝑎 + (𝑛 − 1)𝑑
𝑚
Subtract the second equation from the first:
1 1 𝑚−𝑛 1
(𝑚 − 𝑛)𝑑 = − ⇒ (𝑚 − 𝑛)𝑑 = ⇒𝑑=
𝑛 𝑚 𝑚𝑛 𝑚𝑛
1
Substitute 𝑑 = 𝑚𝑛 in the equation for the 𝑚𝑡ℎ term:
1 1 1 1 1 1 1
𝑎 + (𝑚 − 1)𝑑 = ⇒ 𝑎 + (𝑚 − 1) = ⇒𝑎+ − = ⇒𝑎=
𝑛 𝑚𝑛 𝑛 𝑛 𝑚𝑛 𝑛 𝑚𝑛
Find the 𝑚𝑛𝑡ℎ term:
1 1 1 1
𝑎 + (𝑚𝑛 − 1)𝑑 = + (𝑚𝑛 − 1) = +1− =1
𝑚𝑛 𝑚𝑛 𝑚𝑛 𝑚𝑛

Example 2.113
1 1
If the 𝑚𝑡ℎ term of an arithmetic progression be 𝑛 , and the 𝑛𝑡ℎ term be 𝑚 , then show that the sum of 𝑚𝑛 terms
1
is 2 (𝑚𝑛 + 1).

(The above question should remind you of something…..)

P a g e 74 | 274
Get all the files at: https://bit.ly/azizhandouts
Aziz Manva (azizmanva@gmail.com)

Example 2.114
Show that there is no infinite arithmetic progression which consists of only distinct prime numbers.

We will prove this by contradiction.


Let, if possible be an arithmetic progression as required:
𝑎1 , 𝑎2 , … , 𝑎𝑛

Note that only one pair of primes (2,3) has difference 1. Hence, the common difference must be:
𝑑>1

Then:
𝑎1 + (𝑎1 + 1 − 1)𝑑 = 𝑎1 + 𝑎1 𝑑 = ⏟
⏟ 𝑎1 (1 + 𝑑)
(𝒂𝟏 +𝟏)𝒕𝒉 𝐭𝐞𝐫𝐦 𝑻𝒘𝒐 𝑭𝒂𝒄𝒕𝒐𝒓𝒔
𝑵𝒐𝒕 𝒂 𝒑𝒓𝒊𝒎𝒆

Example 2.115
1 1 1 1 1 1
Show that, if 𝑎, 𝑏, 𝑐 are in arithmetic progression, then 𝑎 (𝑏 + 𝑐 ) , 𝑏 (𝑐 + 𝑎) , 𝑐 (𝑎 + 𝑏) are also in arithmetic
progression.

1 1 1 𝑎𝑏 + 𝑐𝑎 𝑎𝑏 + 𝑏𝑐 𝑏𝑐 + 𝑐𝑎 1 1 1 1 1 1
𝑎, 𝑏, 𝑐 ⇒
⏟ , , ⇒ , , ⇒ 𝑎( + ),𝑏( + ),𝑐( + )
⏟ 𝑎𝑐 𝑎𝑏
𝑏𝑐 ⏟ 𝑏𝑐 𝑎𝑐 𝑎𝑏 ⏟ 𝑏 𝑐 𝑐 𝑎 𝑎 𝑏
𝑨𝑷
𝟏 𝑀𝑢𝑙𝑡𝑖𝑝𝑙𝑦 𝑏𝑦 (𝑎𝑏+𝑏𝑐+𝑐𝑎) 𝑻𝒂𝒌𝒆 𝒂 𝒄𝒐𝒎𝒎𝒐𝒏
𝑴𝒖𝒍𝒕𝒊𝒑𝒍𝒚 𝒃𝒚 𝑺𝒆𝒑𝒂𝒓𝒂𝒕𝒆 𝒐𝒖𝒕 𝒕𝒉𝒆 𝒇𝒓𝒂𝒄𝒕𝒊𝒐𝒏𝒔
𝒂𝒃𝒄 𝑇ℎ𝑒𝑛,𝑠𝑢𝑏𝑡𝑟𝑎𝑐𝑡 1
𝑨𝑷

Example 2.116
If 𝑎, 𝑏, 𝑐 are in arithmetic progression, prove that the following are also are in arithmetic progression:
A. {(𝑏 + 𝑐)2 − 𝑎2 }, {(𝑐 + 𝑎)2 − 𝑏 2 }, {(𝑎 + 𝑏)2 − 𝑐 2 }
1 1 1
B. , ,
√𝑏+√𝑐 √𝑐+√𝑎 √𝑎+√𝑏

Example 2.117
If 𝑎2 , 𝑏 2 , 𝑐 2 are in arithmetic progression, prove that the following are also are in arithmetic progression:
1 1 1
A. , ,
𝑏+𝑐 𝑐+𝑎 𝑎+𝑏
𝑎 𝑏 𝑐
B. , ,
𝑏+𝑐 𝑐+𝑎 𝑎+𝑏

Example 2.118
Let 𝑝𝑡ℎ term of an arithmetic progression be 𝑎, the 𝑞 𝑡ℎ term be 𝑏. Show that the sum of its 𝑝 + 𝑞 terms is
𝑝+𝑞 𝑎−𝑏
{𝑎 + 𝑏 + }
2 𝑝−𝑞

𝐹𝑖𝑟𝑠𝑡 𝑇𝑒𝑟𝑚: 𝑎 − (𝑝 − 1)𝑑


𝐿𝑎𝑠𝑡 𝑇𝑒𝑟𝑚: 𝑎 + (𝑝 + 𝑞 − 1)𝑑 = 𝑎 + 𝑝𝑑 + (𝑞 − 1)𝑑 = 𝑏 + 𝑝𝑑
𝑏 = 𝑎 + (𝑞 − 1)𝑑
𝑎 − (𝑝 − 1)𝑑 + 𝑏 + 𝑝𝑑 = 𝑎 + 𝑏 + 𝑑

P a g e 75 | 274
Get all the files at: https://bit.ly/azizhandouts
Aziz Manva (azizmanva@gmail.com)

2.4 Arithmetic Series


A. Basics
If 𝑛 consecutive terms of an arithmetic sequence are added, you get the 𝑛𝑡ℎ term of the corresponding
arithmetic series.

Example 2.119: Finding the terms of a series


Consider the arithmetic sequence 5,8,11, ... Find the first four terms of the corresponding arithmetic series.

𝑆𝑒𝑞𝑢𝑒𝑛𝑐𝑒 = 5,8,11,14, …

𝑎1 = 5, 𝑎2 = 8, 𝑎3 = 11, 𝑎4 = 14

𝐹𝑖𝑟𝑠𝑡 𝑇𝑒𝑟𝑚=𝑎=5
𝐶𝑜𝑚𝑚𝑜𝑛 𝐷𝑖𝑓𝑓𝑒𝑟𝑒𝑛𝑐𝑒=𝑑=3

𝑆1 = 5
𝑆2 = 5 + 8 = 13
𝑆3 = 5 + 8 + 11 = 24
𝑆4 = 5 + 8 + 11 + 14 = 38

Example 2.120
Jo adds up all the positive integers from 1 to 50. Kate does a similar thing with the first 50 positive integers;
however, she first rounds every integer to its nearest multiple of 10 (rounding 5s up) and then adds the 50
values. What is the positive difference between Jo's sum and Kate's sum? (MathCounts 2007 Workout 2)

Consider the numbers from 1 to 10:


𝐾𝑎𝑡𝑒: 0 + 0 + 0 + 0 + 10 + 10 + 10 + 10 + 10
𝐽𝑜: 1 + 2 + 3 + 4 + 5 + 6 + 7 + 8 + 9 + 10

𝐾𝑎𝑡𝑒 − 𝐽𝑜 = −1 − 2 − 3 − 4 + 5 + 4 + 3 + 2 + 1 + 0
Everything cancels, except
5

Consider the numbers from 11 to 20. The calculations are similar, and the answer will also be
5

Hence, for each set of 10 numbers, Kate's sum is 5 more than Jo's sum. Hence, overall Kate's sum is more than
Jo's sum by:
5 × 5 = 25

Example 2.121
The integers 2 through 9 are each placed in the figure with one integer in each of
the eight smallest triangles. The integers are placed so that the pairs of integers in
each of the four smallest squares have the same sum.
A. What is the sum of one pair? (MathCounts 2006 School Countdown)
B. Determine one possible arrangement of numbers.

2 + 3 + 4 + 5 + 6 + 7 + 8 + 9 = 45 − 1 = 44
44
𝑆𝑢𝑚 𝑜𝑓 1 𝑃𝑎𝑖𝑟 = = 11
4

P a g e 76 | 274
Get all the files at: https://bit.ly/azizhandouts
Aziz Manva (azizmanva@gmail.com)

(2,9), (3,8), (4,7, ), (5,6)


B. Sums of Subsets

Example 2.122
The set {1,2,3,4} has 𝑛 subsets. Let 𝑠𝑚 be the sum of the elements of the 𝑚𝑡ℎ subset. Find 𝑛 + 𝑠1 + 𝑠2 + ⋯ + 𝑠𝑛 .

We do this using cases.


0 Element Sets
{𝜙} ⇒ 𝑆𝑢𝑚 = 0
1 Element Sets
{1}, {2}, {3}, {4} ⇒ 𝑆𝑢𝑚 = 1 + 2 + 3 + 4 = 10
2 Element Sets
{1,2}, {1,3}, {1,4}, {2,3}, {2,4}, {3,4}
Note that each element of the original set occurs thrice. Hence, the sum is:
3(1 + 2 + 3 + 4) = 3(10) = 30
3 Element Sets
{1,2,3}, {1,2,4}, {1,3,4}, {2,3,4}
Compare this with:
{1,2,3, 𝟒}, {1,2, 𝟑, 4}, {1, 𝟐, 3,4}, {𝟏, 2,3,4}
And hence the sum we want is equal to:
4(1 + 2 + 3 + 4) − 1(1 + 2 + 3 + 4) = 30
4 Element Sets
{1,2,3,4} ⇒ 𝑆𝑢𝑚 = 10

𝑠1 + 𝑠2 + ⋯ + 𝑠𝑛 = 0 + 10 + 30 + 30 + 10 = 80
𝑛 = 16
Final Answer
= 80 + 16 = 96

Example 2.123: Change of Scale & Change of Origin


A. A set of four consecutive integers has a sum of 22. If each integer of the set is increased by 2 and then
multiplied by 20, what is the sum of the new set of integers? (MathCounts 2006 State Target)
B. Three consecutive integers add up to a number 𝑥. Each of the integers is increased by 3, and the
numbers are then added. The result of adding the numbers is multiplied by 11. If the final answer is
231, find the numbers.

Part A
𝑎 + 𝑏 + 𝑐 + 𝑑 = 22
𝑎 + 2 + 𝑏 + 2 + 𝑐 + 2 + 𝑑 + 2 = 22 + 8 = 30
20(𝑎 + 2 + 𝑏 + 2 + 𝑐 + 2 + 𝑑 + 2) = 20(30) = 600
Part B
231
= 21 = 6 + 7 + 8 → 3,4,5
10
C. Sum Formula

Example 2.124
A. An auditorium with 20 rows of seats has 10 seats in the first row. Each successive row has one more
seat than the previous row. If students taking an exam are permitted to sit in any row, but not next to

P a g e 77 | 274
Get all the files at: https://bit.ly/azizhandouts
Aziz Manva (azizmanva@gmail.com)

another student in that row, then the maximum number of students that can be seated for an exam is
(AMC 8 1991/13)
B. An auditorium with 30 rows of seats has 10 seats in the first row. Each successive row has one more
seat than the previous row. If students taking an exam are permitted to sit in any row, but not next to
another student in that row, what is the maximum number of students that can be seated for an exam?
(MathCounts 2001 National Sprint)

Seat 1 2 3 4 5 6 7 8 9 10 11 No. of
No. Students
S S S S S 10
=5
2
S S S S S S 11
=6
2
11
Note: is not 6, but in this case, we are rounding up the number.
2

Part A

Seats 10 11 12 13 14 . . . 28 29
Students 5 6 6 7 7 . . . 14 15

We need to add the following twenty numbers:


𝟓 + 𝟔 + 𝟔 + ⋯ + 𝟏𝟒 + 𝟏𝟒 + 𝟏𝟓

We can make pairs, each adding up to twenty. Hence, we can make ten pairs, giving us a total of:
10 × 20 = 200
Part B
Seats 10 11 12 13 14 . . . 38 39
Students 5 6 6 7 7 . . . 19 20

We need to add the following thirty numbers:


𝟓 + 𝟔 + 𝟔 + ⋯ + 𝟏𝟗 + 𝟏𝟗 + 𝟐𝟎
We can make pairs, each adding up to twenty-five. Hence, we can make 15 pairs, giving us a total of:
15 × 25 = 375

2.125: Sum of an Arithmetic Series


The sum of an arithmetic series with 𝑓 = 𝐹𝑖𝑟𝑠𝑡 𝑇𝑒𝑟𝑚, 𝑙 = 𝐿𝑎𝑠𝑡 𝑇𝑒𝑟𝑚, 𝑛 = 𝑁𝑜. 𝑜𝑓 𝑡𝑒𝑟𝑚𝑠 is:

𝑓+𝑙
𝑆𝑛 = 𝑛 × ( ) = No. of Terms(𝐴𝑣𝑔. 𝑜𝑓 𝐹𝑖𝑟𝑠𝑡 𝑎𝑛𝑑 𝐿𝑎𝑠𝑡 𝑇𝑒𝑟𝑚)
2

𝑆 = 𝑎 + (𝑎 + 𝑑) + (𝑎 + 2𝑑) + ⋯ (𝑙 − 2𝑑) + (𝑙 − 𝑑) + 𝑙
Add the first and the last term:
𝑎+𝑙
Add the second term and the second-last term:
𝑎 + 𝑑 + (𝑙 − 𝑑) = 𝑎 + 𝑙
Add the third term and the third-last term:
𝑎 + 2𝑑 + (𝑙 − 2𝑑) = 𝑎 + 𝑙
Hence, we can make pairs, and the total of each pair is:
𝑎+𝑙

P a g e 78 | 274
Get all the files at: https://bit.ly/azizhandouts
Aziz Manva (azizmanva@gmail.com)

The total number of pairs:


𝑛
2
Hence, the sum is
𝑛
(𝑓 + 𝑙)
× ⏟

2 𝑆𝑢𝑚 𝑜𝑓
𝑁𝑢𝑚𝑏𝑒𝑟
𝑜𝑓 𝑃𝑎𝑖𝑟𝑠 𝑒𝑎𝑐ℎ 𝑝𝑎𝑖𝑟

Example 2.126
A. What is the sum of all integers from 80 through 90, inclusive? (MathCounts 2004 State Sprint)
B. Find the sum of the first twelve terms of the arithmetic sequence 3,7,11, …
C. Find the sum of the first eight terms of the decreasing arithmetic sequence with first few terms
35, 29, 23 ….
D. What is the value of the arithmetic series 28 + 30 + 32 +...+86? (MathCounts 2001 National
Countdown)
E. The sum of 18 consecutive positive integers is a perfect square. What is the smallest possible value of
this sum? (AMC 12B 2002/13)

Part A 𝑓+𝑙 35 − 7
𝑆𝑛 = 𝑛 ( ) = 8( ) = 8 × 14 = 112
80 + 81 + ⋯ + 90 2 2
𝑁𝑜. 𝑜𝑓 𝑇𝑒𝑟𝑚𝑠 = 90 − 80 + 1 = 11 Part D
𝑓+𝑙 80 + 90 86 − 28 58
𝑆𝑢𝑚 = 𝑛 ( ) = 11 ( ) = 11 × 85 = 935 +1 = + 1 = 29 + 1 = 30
2 2 2 2
Part B 𝑓+𝑙 (28 + 86)
𝑆 = 𝑛( ) = 30 = 1710
𝐶𝑜𝑚𝑚𝑜𝑛 𝐷𝑖𝑓𝑓𝑒𝑟𝑒𝑛𝑐𝑒 = 𝑑 = 7 − 3 = 4 2 2
The last term that we need is the 12th term: Part E
𝑙 = 𝑎12 = 𝑎 + (𝑛 − 1)𝑑 = 3 + (12 − 1)4 = 47 𝑥, 𝑥 + 1, . . . , 𝑥 + 17
Substitute 𝑓 = 3, 𝑙 = 47, 𝑛 = 12: 𝑓+𝑙
𝑆𝑢𝑚 = 𝑛 ( ) = 9(2𝑥 + 17)
𝑓+𝑙 47 + 3 2
𝑆𝑛 = 𝑛 ( ) = 12 ( ) = 12 × 25 = 300 9 is a perfect square. 2x+17 must also be.
2 2
Part C Smallest 𝑥 which works is 4.
Sum is 225.
𝑙 = 35 ⏟ − 1) (−6)
⏟ + (8 ⏟ = 35 − 42 = −7
𝒂 𝒏 𝒅=𝟑𝟓−𝟐𝟗

Challenge 2.127
𝐶ℎ𝑜𝑜𝑠𝑒 𝑡ℎ𝑒 𝑐𝑜𝑟𝑟𝑒𝑐𝑡 𝑜𝑝𝑡𝑖𝑜𝑛
The first term of an arithmetic series of consecutive integers is 𝑘 2 + 1. The sum of 2𝑘 + 1 terms of this series
may be expressed as: (AHSME 1958/37)
A. 𝑘 3 + (𝑘 + 1)3
B. (𝑘 − 1)3 + 𝑘 3
C. (𝑘 + 1)3
D. (𝑘 + 1)2
E. (2𝑘 + 1)(𝑘 + 1)2

𝑘 2 + 1, 𝑘 2 + 2, 𝑘 2 + 3, … , 𝑘 2 + 2𝑘 + 1
𝐹𝑖𝑟𝑠𝑡 𝑇𝑒𝑟𝑚 = 𝑓 = 𝑘 2 + 1
𝐿𝑎𝑠𝑡 𝑇𝑒𝑟𝑚 = 𝑙 = 𝑘 2 + 2𝑘 + 1

𝑓 + 𝑙 = 𝑘 2 + 1 + 𝑘 2 + 2𝑘 + 1 = 2𝑘 2 + 2𝑘 + 2

P a g e 79 | 274
Get all the files at: https://bit.ly/azizhandouts
Aziz Manva (azizmanva@gmail.com)

𝑓+𝑙 2𝑘 2 + 2𝑘 + 2
𝑆 = 𝑛( ) = (2𝑘 + 1) ( ) = (2𝑘 + 1)(𝑘 2 + 𝑘 + 1)
2 2

= 2𝑘 3 + 2𝑘 2 + 2𝑘 + 𝑘 2 + 𝑘 + 1
= 𝑘 3 + 𝑘 3 + 3𝑘 2 + 3𝑘 + 1
= +(𝑘 + 1)3
Option A.

Example 2.128: Finding the Sum


A. An employee joins a company at a salary of $1000 per month. After a year, he gets a raise, increasing his
salary to 1100 dollars per month. Every year his salary increases by $100 per month. Find the total
money that he makes during a five-year period starting from when he joins.
B. In a potato race, the first potato is 10 m to the right of the starting line. The second potato is 12 m to the
right of the starting line, the third potato is 14 m to the right of the starting line and so on. Participants
start from the starting line, and pick up potatoes. A participant can have only one potato at a time. There
are 20 potatoes, all of which are to be picked up. Participant A follows the instructions. Participant B
misunderstands the instructions and picks up all the potatoes at once and drops them back to the
starting line together. Find the difference between the distance run by A and B.
C. Mary has three daughters, the sum of whose ages is 33. The product of their ages is 1155. The eldest
daughter is the same number of years older than the middle daughter as the middle daughter is older
than the youngest daughter. Find their ages.

Part A
1000 + 1100 + 1200 + ⋯ + 1400

1000, 𝑙 = 1400, 𝑛 = 10
𝑓+𝑙 1000 + 1400 2400
12𝑛 ( ) = 12 × 5 ( ) = 12 × 5 ( ) = 72,000
2 2 2
Part B
Participant A
10 + 12 + 14 + ⋯ + 48
𝑓+𝑙 10 + 48 10 + 48
2𝑛 ( ) = 2 × 20 ( ) = 2 × 10 ( ) = 1160
2 2 1
Participant A
48 × 2 = 96
Difference
= 1160 − 96 = 1064
Part C
Let the ages of daughters be
𝑎 − 𝑑, 𝑎, 𝑎 + 𝑑
The ages of the daughters form an arithmetic sequence. Hence, the
33
𝑎 = 𝐴𝑔𝑒 𝑜𝑓 𝑀𝑖𝑑𝑑𝑙𝑒 𝐷𝑎𝑢𝑔ℎ𝑡𝑒𝑟 = 𝐴𝑣𝑔. 𝑜𝑓 𝐴𝑙𝑙 3 𝑇𝑒𝑟𝑚𝑠 = = 11
3
Hence, the product of the ages
= (11 − 𝑑)(11)(11 + 𝑑) = 1155
121 − 𝑑 2 = 105
𝑑2 = 121 − 105 = 16 ⇒ 𝑑 = ±4
The ages are

P a g e 80 | 274
Get all the files at: https://bit.ly/azizhandouts
Aziz Manva (azizmanva@gmail.com)

11 − 4,11,11 + 4 → 7,11,15

1155 = 3 × 5 × 7 × 11 = 7 × 11 × 15

(Calculator) Example 2.129: Multiples


Consecutive multiples form an arithmetic series. This means that if wants to find the sum of multiples of a
number, we can apply the concepts of arithmetic series to find the sum.

A. What is the sum of all of the multiples of 3 between 100 and 200? (MathCounts 2002 Workout 9)
B. Find the sum of the multiples of 6 from 200 to 500
C. Find the sum of multiples of 14 (but not 21) from 1000 to 2000
D. Find the sum of the numbers from 1 to 100 that are divisible by 2 or 5.

Part A
102
⏟ + 105 + ⋯ + 198

𝒇 𝒍
102 = 3 × 34
198 = 3 × 66
𝑛 = 66 − 34 + 1 = 33

102 + 198
𝑆 = 33 ( ) = 4950
2
Part B
We find the first term, the last term and the number of terms. Be careful with the calculations here, since it is
easy to make an “off by 1” mistake here:
𝑓 = 204 = 34 × 6, 𝑙 = 498 = 83 × 6, 𝑛 = 83 − 34 + 1 = 50
Substitute the values in the formula:
𝑓+𝑙 204 + 498 702
𝑆 =𝑛×( ) = 50 ( ) = 50 ( ) = 17,550
2 2 2
Part C
Sum of Multiples of 14
1008 + 1022 + ⋯ + 1988 = 72 × 14 + 73 × 14 + ⋯ + 142 × 14
From the above, we get:
𝑓 = 1008 = 72 × 14, 𝑙 = 1988 = 142 × 14, 𝑛 = 142 − 72 + 1 = 71
Substitute the values in the formula for the sum:
𝑓+𝑙 1008 + 1988 2996
𝑆14 = 𝑛 × ( ) = 71 ( ) = 71 ( ) = 71 × 1498 = 106,358
2 2 2
Sum of Multiples of 𝑳𝑪𝑴(𝟏𝟒, 𝟐𝟏) = 𝟒𝟐
𝑓 = 1008 = 24 × 42, 𝑙 = 1974 = 47 × 42, 𝑛 = 47 − 24 + 1 = 24
𝑎+𝑙 1008 + 1974 2982
𝑆 =𝑛×( ) = 24 ( ) = 24 ( ) = 24 × 1491 = 35,784
2 2 2
Sum of Multiples of 14, but not 21
𝑆14 − 𝑆42 = 106,358 − 35,784 = 70,574
Part D
50 × 51
2 + 4 + ⋯ + 100 = 2(1 + 2 + ⋯ + 50) = 2 ( ) = 2550
2
20 × 21
5 + 10 + ⋯ + 100 = 5(1 + 2 + ⋯ + 20) = 5 ( ) = 1050
2

P a g e 81 | 274
Get all the files at: https://bit.ly/azizhandouts
Aziz Manva (azizmanva@gmail.com)

10 × 11
10 + 20 + ⋯ + 100 = 10(1 + 2 + ⋯ 10) = 10 ( ) = 550
2
𝑆2 + 𝑆5 − 𝑆10 = 2550 + 1050 − 550 = 3050

D. Sum Formula: Alternate Version

2.130: Sum of an arithmetic Series


𝑛
𝑆𝑛 = [2𝑎 + (𝑛 − 1)𝑑]
2
𝑎 = 𝐹𝑖𝑟𝑠𝑡 𝑇𝑒𝑟𝑚, 𝑛 = 𝑁𝑜. 𝑜𝑓 𝑡𝑒𝑟𝑚𝑠, 𝑑 = 𝐶𝑜𝑚𝑚𝑜𝑛 𝐷𝑖𝑓𝑓𝑒𝑟𝑒𝑛𝑐𝑒

We write the sum twice, first in regular order, and then back to front.

First Term Second Term 𝒏𝒕𝒉 Term


S 𝑎 𝑎+𝑑 . . . 𝑎 + (𝑛 − 1)𝑑

S 𝑎 + (𝑛 − 1)𝑑 𝑎 + (𝑛 − 2)𝑑 𝑎

2S 2𝑎 2𝑎 + (𝑛 − 1)𝑑 . . . 2𝑎 + (𝑛 − 1)𝑑
+ (𝑛 − 1)𝑑

𝑛
2𝑆 = 𝑛[2𝑎 + (𝑛 − 1)𝑑] ⇒ 𝑆 = [2𝑎 + (𝑛 − 1)𝑑]
2

Example 2.131: Finding the Sum


Find the sum of the multiples of 6 from 200 to 500.

𝑎 = 204 = 34 × 6, 𝑑 = 6, 𝑙 = 498 = 83 × 6, 𝑛 = 83 − 34 + 1 = 50
𝑛 50 50
𝑆𝑛 = [2𝑎 + (𝑛 − 1)𝑑] = (204 × 2 + (49 × 6)) = (702) = 17,550
2 2 2

Example 2.132
If the sum to 7 terms of a sequence is 49, and the sum to 6 terms is 35, then find the common difference and the
first term.

We solve the same question using three different 𝑎 + 3𝑑 = 7



methods. 𝑬𝒒𝒖𝒂𝒕𝒊𝒐𝒏 𝑰
Seventh Term
Method I: Symmetry = 𝑆7 − 𝑆6 = 49 − 35 = 14 = 𝑎 + 6𝑑
49
𝑀𝑖𝑑𝑑𝑙𝑒 𝑇𝑒𝑟𝑚 = 𝑎4 = =7 𝑎 + 6𝑑 = 14

7 𝑬𝒒𝒖𝒂𝒕𝒊𝒐𝒏 𝑰𝑰
35 5
𝑀𝑖𝑑𝑑𝑙𝑒 𝑇𝑒𝑟𝑚 = 𝑎3.5 = =5 Subtract Equation I from Equation II
6 6 7
1 5 7 7 3𝑑 = 7 ⇒ 𝑑 =
𝑑 = 𝑎4 − 𝑎3.5 = 7 − 5 = ⇒ 𝑑 = 3
2 6 6 3
3(7) Method III: Formula for Sum of a Series
𝑎1 = 𝑎4 − 3𝑑 = 7 − =7−7=0 Use the formula:
3
𝑛
Method II: Simultaneous Equations (2𝑎 + (𝑛 − 1)𝑑)
2
Sum of first seven terms Sum to seven terms:
= 𝑎 + (𝑎 + 𝑑) + ⋯ + (𝑎 + 6𝑑) = 7𝑎 + 21𝑑 = 49

P a g e 82 | 274
Get all the files at: https://bit.ly/azizhandouts
Aziz Manva (azizmanva@gmail.com)

7 6 70
(2𝑎 + 6𝑑) = 49 ⇒ 𝑎 + 3𝑑 = 7 (2𝑎 + 5𝑑) = 35 ⇒ 2𝑎 + 5𝑑 =
2 2 6
Sum to six terms: And then these can be solved simultaneously.

Example 2.133
The fourth term of an arithmetic sequence is zero, and the sum of the first ten terms is 300. Find the sum of the
first sixteen terms

𝑎 + 3𝑑 = 0 ⇒ 𝑎 = −3𝑑

𝑛
Substitute 𝑛 = 10, 𝑎 = −3𝑑, 𝑆 = 300 in 2 [2𝑎 + (𝑛 − 1)𝑑] = 𝑆10 to find the value of 𝑑 and 𝑎:
10
( ) (−6𝑑 + 9𝑑) = 300 ⇒ 𝑑 = 20 ⇒ 𝑎 = −3𝑑 = −60
2
Substitute 𝑎 = −60, 𝑑 = 20, 𝑛 = 16 to find the sum of the first sixteen terms:
𝑛 16
𝑆16 = [2𝑎 + (𝑛 − 1)𝑑] = (−120 + 15 ∗ 20) = 1440
2 2

Example 2.134: First Term


A. The third term of a finite series in Arithmetic progression is 28. The sum of the first three terms is 54.
The first term of the series is:(JMET 2009/77)
B. When the sum of the first ten terms of an arithmetic progression is four times the sum of the first five
terms, the ratio of the first term to the common difference is: (AHSME 1952/30)
C. For a given arithmetic series the sum of the first 50 terms is 200, and the sum of the next 50 terms is
2700. The first term in the series is: (AHSME 1961/26)

Part A
54
𝑡2 = 𝑀𝑖𝑑𝑑𝑙𝑒 𝑇𝑒𝑟𝑚 = = 18
3
𝑑 = 𝑡3 − 𝑡2 = 28 − 18 = 10
𝑎 = 𝑡2 − 𝑑 = 18 − 10 = 8
Part B
Sum of first five terms
= 𝑎 + (𝑎 + 𝑑) + (𝑎 + 2𝑑) + (𝑎 + 3𝑑) + (𝑎 + 4𝑑) = 5𝑎 + 10𝑑
Sum of first ten terms
𝑛 10
= [2𝑎 + (𝑛 − 1)𝑑] = [2𝑎 + (10 − 1)𝑑] = 10𝑎 + 45𝑑
2 2
By the given condition:
4(5𝑎 + 10𝑑) = 10𝑎 + 45𝑑
20𝑎 + 40𝑑 = 1
10𝑎 = 5𝑑
𝑎 5 1
= =
𝑑 10 2
Part C
Consider the arithmetic series
𝑡1 = 𝑎, 𝑡50 = 𝑎 + 49𝑑
𝑓+𝑙 𝑎 + 𝑎 + 49𝑑
𝑆 = 𝑛( ) = 50 ( )
2 2
25(2𝑎 + 49𝑑) = 200

P a g e 83 | 274
Get all the files at: https://bit.ly/azizhandouts
Aziz Manva (azizmanva@gmail.com)

2𝑎 + 49𝑑 = 8

𝑬𝒒𝒖𝒂𝒕𝒊𝒐𝒏 𝑰

𝑡51 = 𝑎 + 50𝑑, 𝑡100 = 𝑎 + 99𝑑


𝑓+𝑙 𝑎 + 50𝑑 + 𝑎 + 99𝑑
𝑆 = 𝑛( ) = 50 ( )
2 2
25(2𝑎 + 149𝑑) = 2700
2𝑎 + 149𝑑 = 108

𝑬𝒒𝒖𝒂𝒕𝒊𝒐𝒏 𝑰𝑰
Subtract Equation I from Equation II:
100𝑑 = 100
𝑑=1

2𝑎 + 49𝑑 = 8
2𝑎 + 49 = 8
2𝑎 = −41
41
𝑎=−
2

Example 2.135: Common Difference


A. In a given arithmetic sequence the first term is 2, the last term is 29, and the sum of all the terms is 155.
The common difference is: (AHSME 1966/18)
B. The measures of the interior angles of a convex polygon of 𝑛 sides are in arithmetic progression. If the
common difference is 5° and the largest angle is 160°, then 𝑛 equals: (AHSME 1968/20)

Part A
𝑓+𝑙
𝑛( )=𝑆
2
2 + 29
𝑛( ) = 155
2
31
𝑛 = 155
2
𝑛 = 10
The last term is:
𝑎 + (𝑛 − 1)𝑑 = 29
2 + (10 − 1)𝑑 = 29
9𝑑 = 27
𝑑=3
Part B

Example 2.136: Number of Terms


A. An employee joins a company at a salary of $1000 per month. After a year, he gets a raise, increasing his
salary to 1100 dollars per month. Every year his salary increases by $100 per month. In which month
will the total money that he earns first cross 174,000 dollars?
B. A company offers two compensation schemes. Scheme A has a starting salary of $1000 per month, and a
raise of $100 per month every year. Scheme B has a starting salary of $500 and raise of $200 per month
per year. The first raise will be given one year after joining. What is the minimum number of years that
an employee should so that he gets more money under Scheme B as compared to Scheme A.

P a g e 84 | 274
Get all the files at: https://bit.ly/azizhandouts
Aziz Manva (azizmanva@gmail.com)

Part A
1000 + [1000 + (𝑛 − 1)100]
12𝑛 ( ) = 174000
2
2000 + 100𝑛 − 100
𝑛( ) = 14500
2
𝑛(950 + 50𝑛) = 14500
950𝑛 + 50𝑛2 = 14500
𝑛2 + 19𝑛 − 290 = 0
(𝑛 + 29)(𝑛 − 10) = 0
𝑛 = −29 𝑂𝑅 𝑛 = 10
Part B
1000 + [1000 + (𝑛 − 1)100] 500 + [500 + (𝑛 − 1)200]
𝟏𝟐𝒏 ( ) = 𝟏𝟐𝒏 ( )
⏟ 𝟐 ⏟ 𝟐
𝑻𝒐𝒕𝒂𝒍 𝑴𝒐𝒏𝒆𝒚 𝒇𝒓𝒐𝒎 𝑺𝒄𝒉𝒆𝒎𝒆 𝑨 𝑻𝒐𝒕𝒂𝒍 𝑴𝒐𝒏𝒆𝒚 𝒇𝒓𝒐𝒎 𝑺𝒄𝒉𝒆𝒎𝒆 𝑩
1900 + 100𝑛 = 800 + 200𝑛
1100 = 100𝑛
𝑛 = 11

Example 2.137: Number of Terms


A. How many terms in an arithmetic sequence with first term two, and common difference seven must be
added to obtain a total of 80?
B. Let 𝑠1 be the sum of the first 𝑛 terms of the arithmetic sequence 8,12, ... and let 𝑠2 be the sum of the first
𝑛 terms of the arithmetic sequence 17,19, . . .. Assume 𝑛 ≠ 0. Then 𝑠1 = 𝑠2 for which value(s) of n?
(AHSME 1966/19, Adapted)
C. (𝑁𝑢𝑚𝑏𝑒𝑟 𝑇ℎ𝑒𝑜𝑟𝑦) The sum of 𝑛 terms of an arithmetic progression is 153, and the common difference
is 2. If the first term is an integer, and 𝑛 > 1, then the number of possible values for 𝑛 is: (AHSME
1964/28)

Part A
𝑛
Substitute 𝑎 = 2, 𝑑 = 7, 𝑆𝑛 = 80 in 𝑆𝑛 = (2𝑎 + (𝑛 − 1)𝑑):
2
𝑛
80 = (4 + (𝑛 − 1)7) ⇒ 160 = 𝑛(4 + 7𝑛 − 7)
2
160 = 𝑛(7𝑛 − 3)
160 = 5 × 32 ⇒ 𝑛 𝑚𝑢𝑠𝑡 𝑏𝑒 𝑒𝑖𝑡ℎ𝑒𝑟 𝑎 𝑚𝑢𝑙𝑡𝑖𝑝𝑙𝑒 𝑜𝑓 5, 𝑜𝑟 𝑎 𝑚𝑢𝑙𝑡𝑖𝑝𝑙𝑒 𝑜𝑓 2
𝑛 = 5 𝑤𝑜𝑟𝑘𝑠
Part B
𝑛 𝑛
[2(8) + (𝑛 − 1)4] = [2(17) + (𝑛 − 1)2]
2 2
16 + (𝑛 − 1)4 = 34 + (𝑛 − 1)2
(𝑛 − 1)2 = 18
𝑛 = 10
Part C
The sum of the terms is
𝑛
[2𝑎 + (𝑛 − 1)2] = 153
2
𝑛[𝑎 + (𝑛 − 1)] = 153
𝑛 is a positive integer. It must be a factor of 153 greater than 1. This means the only possibilities are
𝑛 = 3, 9, 17, 51, 153
We now must check if a is an integer:
𝑛[𝑎 + (𝑛 − 1)] = 153

P a g e 85 | 274
Get all the files at: https://bit.ly/azizhandouts
Aziz Manva (azizmanva@gmail.com)

153
𝑎= −1+𝑛
𝑛
153
If 𝑛 is a factor of 153, then 𝑛 will be an integer. Adding 1 − 𝑛 will keep it an integer.
5 𝐶ℎ𝑜𝑖𝑐𝑒𝑠

Example 2.138
If the sum of the first 3𝑛 positive integers is 150 more than the sum of the first 𝑛 positive integers, then the sum
of the first 4𝑛 positive integers is (AHSME 1970/22)

The sum of the first 𝑛 positive integers is:


1+𝑛 𝑛(𝑛 + 1)
1 + 2 + ⋯+ 𝑛 = 𝑛( )=
2 2
The sum of the first 3𝑛 positive integers is:
1 + 3𝑛 3𝑛(3𝑛 + 1)
1 + 2 + ⋯ + 3𝑛 = 3𝑛 ( )=
2 2
(3𝑛)(3𝑛 + 1) 𝑛(𝑛 + 1)
= + 150
2 2
9𝑛2 + 3𝑛 = 𝑛2 + 𝑛 + 300
8𝑛2 + 2𝑛 − 300 = 0
4𝑛2 + 𝑛 − 150 = 0
Use the quadratic formula with 𝑎 = 4, 𝑏 = 1, 𝑐 = −150
−𝑏 ± √𝑏 2 − 4𝑎𝑐 −1 ± √1 − 4(4)(−150) −1 ± √2401 −1 ± 49
𝑛= = = =
2𝑎 8 8 8
25
𝑛 ∈ {6, − }
4
Take
24 × 25
𝑛 = 6 ⇒ 4𝑛 = 24 ⇒ 𝑆𝑢𝑚 = = 300
2

Example 2.139
Let 𝑆𝑛 and 𝑇𝑛 be the respective sums of the first 𝑛 terms of two arithmetic series. If 𝑆𝑛 : 𝑇𝑛 = (7𝑛 + 1): (4𝑛 + 27)
for all 𝑛, the ratio of the eleventh term of the first series to the eleventh term of the second series is: (AHSME
1969/33)

Let
𝑆𝑛 ℎ𝑎𝑣𝑒 𝑓𝑖𝑟𝑠𝑡 𝑡𝑒𝑟𝑚 𝑎1 𝑎𝑛𝑑 𝑐𝑜𝑚𝑚𝑜𝑛 𝑑𝑖𝑓𝑓𝑒𝑟𝑒𝑛𝑐𝑒 𝑑1
𝑇𝑛 ℎ𝑎𝑣𝑒 𝑓𝑖𝑟𝑠𝑡 𝑡𝑒𝑟𝑚 𝑎2 𝑎𝑛𝑑 𝑐𝑜𝑚𝑚𝑜𝑛 𝑑𝑖𝑓𝑓𝑒𝑟𝑒𝑛𝑐𝑒 𝑑2
The ratio of 𝑆𝑛 : 𝑇𝑛 is
𝑛 𝑛
[2𝑎1 + (𝑛 − 1)𝑑1 ]: [2𝑎2 + (𝑛 − 1)𝑑2 ]
2 2
𝑛
Divide both sides by :
2
2𝑎1 + (𝑛 − 1)𝑑1 : 2𝑎2 + (𝑛 − 1)𝑑2

Consider 𝑆𝑛 .
Substitute 𝑛 = 1 in 2𝑎1 + (𝑛 − 1)𝑑1 = 7𝑛 + 1:
2𝑎1 = 7(1) + 1 = 8 ⇒ 𝑎1 = 4
Substitute 𝑛 = 2, 𝑎1 = 4:
8 + 𝑑1 (2 − 1) = 7(2) + 1 ⇒ 𝑑1 = 7
Then, the 11𝑡ℎ term

P a g e 86 | 274
Get all the files at: https://bit.ly/azizhandouts
Aziz Manva (azizmanva@gmail.com)

= 𝑎1 + (𝑛 − 1)𝑑1 = 4 + (10)(7) = 74

Consider 𝑇𝑛 :
Substitute 𝑛 = 1 in 2𝑎2 + (𝑛 − 1)𝑑2 = 4𝑛 + 27:
31
2𝑎2 = 31 ⇒ 𝑎2 =
2
31
Substitute 𝑎2 = 2
,𝑛 = 2:
31
2( ) + (2 − 1)𝑑2 = 4(2) + 27 ⇒ 𝑑2 = 4
2
31 111
11𝑡ℎ 𝑇𝑒𝑟𝑚 = 𝑎2 + (𝑛 − 1)𝑑2 = + (10)(4) =
2 2
The ratio is:
111
= 74: = 148: 111 = 4: 3
2

Example 2.140
If the sum of the first 2𝑛 terms of arithmetic progression 2, 5, 8, … is equal to the sum of first 𝑛 terms of
arithmetic progression 57, 59, 61, … , then 𝑛 = (IIT JEE, 2001 Screening)

2𝑛 𝑛
[4 + (2𝑛 − 1)3] = [114 + (𝑛 − 1)2]
2 2
8 + 12𝑛 − 6 = 114 + 2𝑛 − 2
𝑛 = 11

Example 2.141
𝑎1 +𝑎2 +⋯+𝑎𝑝 𝑝2 𝑎
Let 𝑎1 , 𝑎2 , 𝑎3 , … be terms in an arithmetic progression. If 𝑎1 +𝑎2 +⋯+𝑎𝑞
= 𝑞2 , then find 𝑎 6 . (JEE Main 2006)
21

Use the formula for the sum to 𝑛 terms in the relation given in the question:
𝒑 𝑝−1
[2𝑎1 + (𝑝 − 1)𝑑] 𝑝𝟐 𝑎1 + ( )𝑑 𝑝
2 = ⇒ 2 =
𝒒 𝟐 𝑞−1
[2𝑎1 + (𝑞 − 1)𝑑] 𝑞 𝑎 + ( ) 𝑑 𝑞
2 1 2
Equate the relation obtained with the ratio required in the question:
𝒑−𝟏
𝑎6 𝑎1 + 𝟓𝑑 𝑎1 + ( 𝟐 ) 𝑑 𝑝
= = =
𝑎21 𝑎1 + 𝟐𝟎𝑑 𝑎 + (𝒒 − 𝟏) 𝑑 𝑞
1 𝟐
Use the method of undetermined coefficients. The coefficients in red must be equal, and hence can be equated.
Same with the coefficients in green.
𝑝−1 𝑞−1 𝑝 11
= 5 ⇒ 𝑝 = 11, = 20 ⇒ 𝑞 = 41 ⇒ =
2 2 𝑞 41
E. Exponents

Example 2.142
Recall from the laws of exponents that 𝑥 𝑚 × 𝑥 𝑛 = 𝑥 𝑚+𝑛 .

Expressions B. 𝑥 1 × 𝑥 3 × 𝑥 5 × … × 𝑥 2𝑛+1
Simplify Equations
A. 𝑥 1 × 𝑥 2 × 𝑥 3 × … × 𝑥 𝑛 Find the value of 𝑛 in each equation below

P a g e 87 | 274
Get all the files at: https://bit.ly/azizhandouts
Aziz Manva (azizmanva@gmail.com)

C. 𝑥 1 × 𝑥 2 × 𝑥 3 × … × 𝑥 𝑛 = 𝑥 15 Inequalities
D. 𝑥 1 × 𝑥 2 × 𝑥 3 × … × 𝑥 𝑛 = 𝑥 55 𝑛 is a positive integer. Find all solutions for 𝑛 in
E. 𝑥 1 × 𝑥 3 × 𝑥 5 × … × 𝑥 2𝑛+1 = 𝑥 36 each equation below:
F. 𝑥 1 × 𝑥 3 × 𝑥 5 × … × 𝑥 2𝑛+1 = 𝑥 100 G. 𝑥 1 × 𝑥 2 × 𝑥 3 × … × 𝑥 𝑛 < 𝑥 55

𝑛(𝑛 + 1) = 110
Expressions 𝑛 = 10
Part A Part E
𝑛(𝑛+1) (𝑛 + 1)2 = 36
𝑥 1 × 𝑥 2 × 𝑥 3 × … × 𝑥 𝑛 = 𝑥 1+2+⋯+𝑛 = 𝑥 2
𝑛 + 1 = ±6
Part B
𝑛 ∈ {−7,5}
𝑥 1 × 𝑥 3 × 𝑥 5 × … × 𝑥 2𝑛+1 = 𝑥 1+3+⋯+(2𝑛+1)
2 𝑛>0⇒𝑛=5
= 𝑥 (𝑛+1) Part F
Equations (𝑛 + 1)2 = 100
Part C 𝑛 + 1 = ±10
𝑛(𝑛+1)
𝑥 2 = 𝑥 15 𝑛 ∈ {−11,9}
𝑛(𝑛 + 1) 𝑛>0⇒𝑛=9
= 15 Part G
2
𝑛(𝑛 + 1) = 30 𝑛(𝑛 + 1)
< 55
𝑛=5 2
Part D 𝑛 ∈ {1,2,3, … ,9}
𝑛(𝑛 + 1)
= 55
2

Example 2.143
1 2 3 𝑛
Given the sequence 1011 , 1011 , 1011 , … , 1011 , the smallest value of 𝑛 such that the product of the first 𝑛 members
of this sequence exceeds 100,000 is: (AHSME 1965/24, AHSME 1971/29)

1 2 3 𝑛
1011 × 1011 × 1011 × … × 1011 > 100,000
Replace the inequality with an equation, and then take the value of 𝑛 one larger than the solution.
1 2 3 𝑛
1011+11+11+⋯+11 = 105
1 2 3 𝑛
+ + + ⋯+ =5
11 11 11 11
1 + 2 + ⋯+ 𝑛
=5
11
1 + 2 + ⋯ + 𝑛 = 55
𝑛(𝑛 + 1)
= 55
2
𝑛(𝑛 + 1) = 110
𝑛 = 10

𝐸𝑥𝑐𝑒𝑒𝑑𝑠 = 11
F. Further Questions

Example 2.144
For 𝑝 = 1, 2, . . . ,10 let 𝑆𝑝 be the sum of the first 40 terms of the arithmetic progression whose first term is 𝑝 and
whose common difference is 2𝑝 − 1; then 𝑆1 + 𝑆2 +. . . +𝑆10 is: (AHSME 1974/29)

P a g e 88 | 274
Get all the files at: https://bit.ly/azizhandouts
Aziz Manva (azizmanva@gmail.com)

For an arithmetic series with first term a, common difference d, and n terms, we have
𝐹𝑖𝑟𝑠𝑡 𝑇𝑒𝑟𝑚 = 𝑎, 𝐿𝑎𝑠𝑡 𝑇𝑒𝑟𝑚 = 𝑎 + (𝑛 − 1)𝑑
For the arithmetic series 𝑆𝑝 , the first term is p, the common difference is 2p-1, and hence:
40𝑡ℎ 𝑇𝑒𝑟𝑚 = 𝑝 + (40 − 1)(2𝑝 − 1) = 𝑝 + 39(2𝑝 − 1) = 79𝑝 − 39
The sum to 40 terms is:
𝑓+𝑙 𝑝 + 79𝑝 − 39
𝑆𝑝 = 𝑛 ( ) = 40 ( ) = 1600𝑝 − 780
2 2
We want to find:
𝑆1 + 𝑆2 +. . . +𝑆10
= [1600(1) − 780] + [1600(2) − 780] + ⋯ + [1600(10) − 780]
The 780 occurs in each term. Separate it out, and factor out 1600 from the other terms:
1 + 2 + ⋯ + 10
= [1600 ( )] − 780(10)
2
Apply the formula for the sum of the first n natural numbers:
10 × 11
= 1600 ( ) − 7800
2
= 88000 − 7800
= 80200 − 7800

Example 2.145
A checkerboard of 13 rows and 17 columns has a number written in each square, beginning in the upper left
corner, so that the first row is numbered 1,2, . . . ,17, the second row 18,19, . . . ,34, and so on down the board. If
the board is renumbered so that the left column, top to bottom, is 1,2, . . . ,13, the second column 14,15, . . . ,26
and so on across the board, some squares have the same numbers in both numbering systems. Find the sum of
the numbers in these squares (under either system). (AMC 12 2000/16)

P a g e 89 | 274
Get all the files at: https://bit.ly/azizhandouts
Aziz Manva (azizmanva@gmail.com)

3. GEOMETRIC SEQUENCES & SERIES


3.1 Basics
A. Basics

3.1: Definition
A geometric sequence with first term 𝑎 has a common ratio 𝑟 between successive elements:
𝑎, 𝑎𝑟, 𝑎𝑟 2 , … , 𝑎𝑟 𝑛−1
Where
𝐹𝑖𝑟𝑠𝑡 𝑡𝑒𝑟𝑚 = 𝑎
𝐶𝑜𝑚𝑚𝑜𝑛 𝑟𝑎𝑡𝑖𝑜 = 𝑟
𝑁𝑢𝑚𝑏𝑒𝑟 𝑜𝑓 𝑇𝑒𝑟𝑚𝑠 = 𝑛

Example 3.2
A. Anand has invested $100 in a bank, and the bank pays 10% per year, compounded yearly. Find the
amount at the beginning of each year for the first four years. Show that the sequence is a geometric
sequence.
B. Is the sequence 1,1,1,1, … a geometric sequence? If it is, find the common ratio. Is it an arithmetic
sequence? If it is, what is the common difference?

Part A
Amount at the beginning of:
1𝑠𝑡 𝑌𝑒𝑎𝑟 = 100
2𝑛𝑑 𝑌𝑒𝑎𝑟 = 100 × 1.1 = 110
3𝑟𝑑 𝑌𝑒𝑎𝑟 = 110 × 1.1 = 121
4𝑡ℎ 𝑌𝑒𝑎𝑟 = 121 × 1.1 = 133.1
5𝑡ℎ 𝑌𝑒𝑎𝑟 = 133.1 × 1.1 = 146.41

Basically, the amounts can be written as


100, 100 × 1.1,100 × 1.12 , 100 × 1.13 , 100 × 1.14
Part B
𝐺𝑒𝑜𝑚𝑒𝑡𝑟𝑖𝑐: 𝑟 = 1
𝐴𝑟𝑖𝑡ℎ𝑚𝑒𝑡𝑖𝑐: 𝑑 = 1

3.3: Common Ratio: Consecutive Terms


Any two consecutive terms of a geometric sequence will be
𝑎𝑟 𝑥 , 𝑎𝑟 𝑥+1
If we divide any two consecutive terms of a geometric sequence, we get
𝑎𝑟 𝑥+1 𝑟 𝑥+1−𝑥
= =𝑟
𝑎𝑟 𝑥 1

Example 3.4: Finding the Common Ratio


Find the common ratio for the following geometric sequences and state it. Hence, extend the sequences and find
the next two terms.

Numbers B. 2,8,32,128, … 8 2 1
C. , ,
5 25 250
,…
A. 5,10,20,40, … Fractions

P a g e 90 | 274
Get all the files at: https://bit.ly/azizhandouts
Aziz Manva (azizmanva@gmail.com)
36 12 4 1 1 1 27𝑝2 9𝑝 3
D. , , ,…
27 9 3
G. , ,
10 100 1000
,… I. , − 32𝑞2 , 16𝑞
64𝑞3
E. 1250,250,50, … Variables
125 25 5 𝑥4
F. , , ,…
81 27 9 H. 𝑥 2 𝑦, 𝑥 3 , 𝑦

𝑁𝑒𝑥𝑡 𝑇𝑤𝑜 𝑇𝑒𝑟𝑚𝑠 = 10, 2


Part A
10 20 40 Part F
= 2, = 2, =2
5 10 20 5 25 5 27 3
𝑁𝑒𝑥𝑡 𝑇𝑤𝑜 𝑇𝑒𝑟𝑚𝑠 = 80,160 𝑟=
÷ = × =
9 27 9 25 5
5 3 1 1 3 1
Part B 𝑁𝑒𝑥𝑡 𝑇𝑤𝑜 𝑇𝑒𝑟𝑚𝑠: × = , × =
8 9 5 3 3 5 5
𝑟= =4
2 Part G
𝑁𝑒𝑥𝑡 𝑇𝑤𝑜 𝑇𝑒𝑟𝑚𝑠 = 512, 2048 1 1 1 1
𝑟=
÷ = × 10 =
Part C 100 10 100 10
1 1
2 8 2 5 1 𝑁𝑒𝑥𝑡 𝑇𝑤𝑜 𝑇𝑒𝑟𝑚𝑠 = ,
𝑟= ÷ = × = 10,000 100,000
25 5 25 8 20
1 1 Part I
𝑁𝑒𝑥𝑡 𝑇𝑤𝑜 𝑇𝑒𝑟𝑚𝑠 = , 1 𝑥
5,000 100,000
𝑥3 ÷ 𝑥2𝑦 = 𝑥3 ×
2
=
Part D 𝑥 𝑦 𝑦
36 12 4 𝑥5 𝑥6
= = ⇒𝑟=1 𝑁𝑒𝑥𝑡 𝑇𝑤𝑜 𝑇𝑒𝑟𝑚𝑠 = 2 ,
27 9 3 𝑦 𝑦3
4 4
𝑁𝑒𝑥𝑡 𝑇𝑤𝑜 𝑇𝑒𝑟𝑚𝑠 = , Part J
3 3 9𝑝 27𝑝2 9𝑝 64𝑞 3 2𝑞
Part E − 2
÷ 3
= − 2
× 2
=−
32𝑞 64𝑞 32𝑞 27𝑝 3𝑝
50 1
𝑟= =
250 5

Example 3.5
1 1 1
A. What is the sum of the next two terms in the geometric sequence 1, 2 , 4 , 8 , … ? Express your answer as a
common fraction. (MathCounts 2005 School Countdown)
1 1
B. Each successive term in the sequence 2048, 512, 128, 𝑥, 𝑦, 2, 2 , 8, is obtained by multiplying the previous
term by a constant. What is the value of 𝑥 + 𝑦? (MathCounts 2003 State Countdown)

Part A
1 1 3
+ =
16 32 32
Part B
1
𝐶𝑜𝑚𝑚𝑜𝑛 𝑟𝑎𝑡𝑖𝑜 = 𝑟 = ⇒ 𝑥 + 𝑦 = 32 + 8 = 40
4

3.6: Negative Common Ratio


If a geometric sequence has
𝐹𝑖𝑟𝑠𝑡 𝑇𝑒𝑟𝑚 = 𝑎, 𝐶𝑜𝑚𝑚𝑜𝑛 𝑅𝑎𝑡𝑖𝑜 = −𝑟, 𝑟>0
Then the sequence will be
𝑎, −𝑎𝑟, 𝑎𝑟 2 , −𝑎𝑟 3 , 𝑎𝑟 4

➢ If the common ratio is negative, then the terms will alternate between positive and negative.

P a g e 91 | 274
Get all the files at: https://bit.ly/azizhandouts
Aziz Manva (azizmanva@gmail.com)

➢ That is, consecutive terms will have opposite signs.

Example 3.7
A. 1, −1,1, −1,1, …. Is this sequence geometric? Is this sequence arithmetic. Determine the next two terms.
B. In the geometric sequence with a first term of 6 and a second term of −6, what is the 205th term?
(MathCounts 2008 Chapter Countdown)

Part A
𝑆𝑒𝑞𝑢𝑒𝑛𝑐𝑒 𝑖𝑠 𝑔𝑒𝑜𝑚𝑒𝑡𝑟𝑖𝑐, 𝑏𝑢𝑡 𝑛𝑜𝑡 𝑎𝑟𝑖𝑡ℎ𝑚𝑒𝑡𝑖𝑐.
−1
𝐶𝑜𝑚𝑚𝑜𝑛 𝑟𝑎𝑡𝑖𝑜 = 𝑟 = = −1
1
𝑁𝑒𝑥𝑡 𝑇𝑤𝑜 𝑇𝑒𝑟𝑚𝑠 = −1, 1
Part B
𝑟 = −1 ⇒ 205𝑡ℎ 𝑇𝑒𝑟𝑚 = 6

Example 3.8
Find the common ratio, and the next two terms of each geometric sequence below:

81
A. 2, −9,2
,…
1 1 1
B. 1, − , , − , …
3 9 27
C. 4, −8,16, −32
3 3 3
D. 3, − 2 , 4 , − 8
16 8
E. 25
, − 5 , 4, −10
1 1
𝑟 = − ÷1 = −
Part A 3 3
1 1
9 81 81 −1 9 ,−
𝑟=− , ÷ −9 = × =− 81 243
2 2 2 9 2
−729 6561 Part D
𝑁𝑒𝑥𝑡 𝑇𝑤𝑜 𝑇𝑒𝑟𝑚𝑠 = , −8
4 8 = −2
4
Part B 64, −128
10 5
𝑟=− =− Part E
4 2
5 3 3 1 1
10 × − = −25 𝑟= ÷3=− × =−
2 2 2 3 2
5 125 3 3
−25 × − = ,−
2 2 16 32
Part C

Example 3.9: Algebra with Consecutive Terms


A. If 𝑥, 2𝑥, 𝑥 2 is a geometric sequence, then find the possible values of 𝑥 2 + 3𝑥.
B. If 𝑥 and 𝑦 are distinct non-zero integers such that 𝑥, 𝑦, 𝑥 is a geometric sequence, then find the value of
the common ratio.
C. Find all possible value(s) of 𝑥 given that 𝑥 − 2, 𝑥 − 1 and 𝑥 + 1 are in a geometric sequence.
5
D. The positive number 𝑎 is chosen such that the terms 20, 𝑎, 4 are the first, second and third terms,
respectively, of a geometric sequence. What is the value of 𝑎, if 𝑎 is positive? (MathCounts 2004 State
Countdown)

P a g e 92 | 274
Part A 𝐶𝑜𝑚𝑚𝑜𝑛 𝑅𝑎𝑡𝑖𝑜 = 𝑟 = −1
Dividing any term by the previous term gives us the Part C
common ratio. Hence: 𝑥−1 𝑥+1
=
2𝑥 𝑥 2 𝑥−2 𝑥−1
𝑟= = ⇒ 𝑥 = 4 ⇒ 𝑥 2 + 3𝑥 = 16 + 12 = 28 (𝑥 − 1)2 = (𝑥 + 1)(𝑥 − 2)

𝑥 2𝑥
𝑬𝒒𝒖𝒂𝒕𝒊𝒐𝒏 𝑰 𝑥 2 − 2𝑥 + 1 = 𝑥 2 − 𝑥 − 2
Part B 𝑥=3
𝑦 𝑥 Part E
= ⇒ 𝑦 2 = 𝑥 2 ⇒ 𝑦 = ±𝑥
𝑥 𝑦 5
𝑎
Hence, the sequence can be: =4
𝑥, 𝑥, 𝑥 OR 𝑥, −𝑥, 𝑥 20 𝑎
5
The first sequence is not valid since we need 𝑎2 = 20 × = 52
distinct integers. 4
𝑎=5
Hence, the second sequence is the only valid
sequence, and it has

3.10: Restrictions on Common Ratio and First Term


The common ratio of a geometric sequence cannot be zero. That is:
𝑟≠0
The first term of a geometric sequence cannot be zero. That is:
𝑎≠0

Example 3.11
If 𝑥, 2𝑥 + 2,3𝑥 + 3, are in geometric progression, the fourth term is: (AHSME 1964/6)

2𝑥 + 2 3𝑥 + 3
=
𝑥 2𝑥 + 2
2
(2𝑥 + 2) = 𝑥(3𝑥 + 3)
4𝑥 2 + 8𝑥 + 4 = 3𝑥 2 + 3𝑥
𝑥 2 + 5𝑥 + 4 = 0
(𝑥 + 4)(𝑥 + 1) = 0
𝑥 ∈ {−4, −1}
Case I: 𝒙 = −𝟏
𝑥 = −1
2𝑥 + 2 = 2(−1) + 2 = 0
3𝑥 + 3 = 3(−1) + 3 = 0

Hence, the terms are:


−1,0,0 ⇒ 𝐶𝑜𝑚𝑚𝑜𝑛 𝑟𝑎𝑡𝑖𝑜 = 𝑟 = 0 ⇒ 𝑁𝑜𝑡 𝑣𝑎𝑙𝑖𝑑
𝑅𝑒𝑗𝑒𝑐𝑡 𝑥 = −1
Case II: 𝒙 = −𝟒
−4, −6, −9
−6 3 −9 3
𝑟= = , 𝑟= =
−4 2 −6 2
The next term in the sequence is
3 27
−9 × =− = −13.5
2 2

3.12: Non-Consecutive Terms


Get all the files at: https://bit.ly/azizhandouts
Aziz Manva (azizmanva@gmail.com)

If you are given terms


𝑎𝑛 , 𝑎𝑛+𝑘 , 𝑘 𝑖𝑠 𝑒𝑣𝑒𝑛
Then you will get two values for the common ratio.

Example 3.13
A. The first term of a geometric sequence is 1. The third term of the sequence is 4. Find the product of the
possible value of the common ratio.
3 2
B. The first term of a geometric sequence is . The fifth term of the sequence is . Find the product of the
5 7
possible value(s) of the common ratio.
1 1
C. (𝐶ℎ𝑎𝑙𝑙𝑒𝑛𝑔𝑒) The first term of a three-term geometric sequence is 𝑡1 = 2. The third term is 𝑡3 = 8.
William found the second term of the geometric sequence, and he found that 𝑡3 − 𝑡2 , 𝑡2 − 𝑡1 form an
1
arithmetic sequence with common difference 8. Harry also found a second term of the geometric
sequence, and he found that 𝑡3 − 𝑡2 , 𝑡2 − 𝑡1 form a geometric sequence with common ratio −1. Explain
with reasons, which of them is correct, and why?

Part A 1 2 1
𝑡3 − 𝑡2 = − =−

1, ⏟
𝑥, ⏟
4 8 8 8
𝑎 𝑎𝑟 𝑎𝑟 2 2 4 2
𝑥 4 𝑡2 − 𝑡1 = − = −
= ⇒ 𝑥 2 = 4 ⇒ 𝑥 = ±2 8 8 8
1 𝑥 1 2
Part B {− , − }
8 8
3 2 The above is
,

5 ⏟ 7 ➢ an arithmetic sequence with common
𝑎 𝑎𝑟 4 1
4 difference − .
𝑎𝑟 2 3 2 5 10 8
= ÷ = × = ➢ a geometric sequence with common ratio 2.
𝑎 7 5 7 3 21 1
If 𝑥 = − 4:
10 4 10
𝑟4 = ⇒ 𝑟 = ±√ 1 2 3
21 21 𝑡3 − 𝑡2 = − (− ) =
8 8 8
4 10 4 10 10 2 4 6
𝑃𝑟𝑜𝑑𝑢𝑐𝑡 = ( √ ) (− √ ) = −√ 𝑡2 − 𝑡1 = − − = −
21 21 21 8 8 8
3 6
{ ,− }
Part C 8 8
1 1
Let the terms be 2 , 𝑥, 8. Then: The above is
➢ an arithmetic sequence with common
1 9
𝑥 1 1 difference − .
= 8 ⇒ 𝑥2 = ⇒𝑥=± 8
1 𝑥 16 4 ➢ a geometric sequence with common ratio
2 −2.
1
If 𝑥 = 4: Hence, neither William nor Harry was correct.

Example 3.14
A. x and y are the first and the third terms of a geometric sequence. Find, with reasons, the sum of the
possible values of the second term.
B. The first term of a geometric sequence is 𝑥. The third term is 𝑦. Write the first four terms of the
sequence in terms of 𝑥 and 𝑦. You may have more than one answer.

Part A 𝑎 𝑦
= ⇒ 𝑎2 = 𝑥𝑦 ⇒ 𝑎 = ±√𝑥𝑦
𝑥, 𝑎, 𝑦 𝑥 𝑎
𝑆𝑢𝑚 = √𝑥𝑦 − √𝑥𝑦 = 0

P a g e 94 | 274
Get all the files at: https://bit.ly/azizhandouts
Aziz Manva (azizmanva@gmail.com)

Part B
𝑥, 𝑎, 𝑦 𝑦√𝑦
𝑎 𝑦 𝑟 = √𝑥𝑦 ⇒ 𝑥, √𝑥𝑦, 𝑦, 𝑦,
= ⇒ 𝑎2 = 𝑥𝑦 ⇒ 𝑎 = ±√𝑥𝑦 𝑥
𝑥 𝑎 𝑦√𝑦
𝑦 √𝑦 𝑦 𝑟 = −√𝑥𝑦 ⇒ 𝑥, −√𝑥𝑦, 𝑦, 𝑦, −
𝑟= =± = ±√ 𝑥
±√𝑥𝑦 √𝑥 𝑥

Example 3.15
The second and fourth terms of a geometric sequence are 2 and 6. Find the possible values of the first term?
(AMC 12B 2003/6, Adapted)

Method I
Let the three terms be 2, 𝑥, 6
𝑥 6
= ⇒ 𝑥 2 = 12 ⇒ 𝑥 = ±2√3
2 𝑥
6
𝐶𝑜𝑚𝑚𝑜𝑛 𝑅𝑎𝑡𝑖𝑜 = 𝑟 = = ±√3
±2√3
Method II
Divide the fourth term by the second:
𝑎𝑟 3 6
= 𝑟 2 = = 3 ⇒ 𝑟 = ±√3
𝑎𝑟 2
Find the possible values of the first term:
2 2√3
𝑎𝑟 = 2 ⇒ 𝑎(√3) = 2 ⇒ 𝑎 = =
√3 3
2 2√3
𝑎𝑟 = 2 ⇒ 𝑎(−√3) = 2 ⇒ 𝑎 = =−
−√3 3
B. Working with Terms

3.16: 𝑵𝒕𝒉 Term of a geometric sequence


For a geometric series with 𝑓𝑖𝑟𝑠𝑡 𝑡𝑒𝑟𝑚 = 𝑎, 𝑐𝑜𝑚𝑚𝑜𝑛 𝑟𝑎𝑡𝑖𝑜 = 𝑛, the 𝑛𝑡ℎ term is
𝑁 𝑡ℎ 𝑡𝑒𝑟𝑚 = 𝑇𝑛 = 𝑎𝑟 𝑛−1

If we equate to the value of the term, we can solve for 𝑛.

3.17: General Term of a geometric sequence


Substitute the values of 𝑎 and 𝑟 as constants, and leave 𝑛 as 𝑛.

Example 3.18
3,6, 12, … is a geometric sequence. Find the
A. Term Number for the term with value 96.
B. Tenth Term
C. General term
D. Smallest term that is larger than 1000

Part A
Substitute the known information (𝑎 = 3, 𝑟 = 2) in the formula for the 𝑛𝑡ℎ term:
96
𝑎𝑟 𝑛−1 = 96 ⇒ 3(2)𝑛−1 = 96 ⇒ (2)𝑛−1 = = 32 = 25 ⇒ 𝑛 − 1 = 5 ⇒ 𝑛 = 6
3
P a g e 95 | 274
Get all the files at: https://bit.ly/azizhandouts
Aziz Manva (azizmanva@gmail.com)

Part B
Substitute the values (𝑎 = 3, 𝑟 = 2, 𝑛 = 10) in the formula for the 𝑛𝑡ℎ term:
𝑎𝑟 𝑛−1 = 3(2)10−1 = 3 × 512 = 1536
Part C
Substitute the values (𝑎 = 3, 𝑟 = 2) in the formula for the 𝑛𝑡ℎ term:
𝑎𝑟 𝑛−1 = 3(2𝑛−1 )
Part D
We already know that the tenth term is 1536. We also know that the common ratio is 2.
Therefore:
1536
𝑎9 = ≈ 750 < 1000 ⇒ 𝑎9 < 1000
2
The answer is 1536.
3(2𝑛−1 ) > 1000
1000
2𝑛−1 >
3
𝑛−1
1
2 > 333 > 256 = 28
3
𝑛−1>8
𝑛>9
𝑆𝑚𝑎𝑙𝑙𝑒𝑠𝑡 𝑛 = 10

Example 3.19
81 27 9 4
In the geometric sequence , , , … , 9,
16 8 4
identify the common ratio, and the number of terms given. Then find
the tenth term and the general term.

Common Ratio: Logic


1
𝑁𝑢𝑚𝑒𝑟𝑎𝑡𝑜𝑟𝑠: 81 → 27 → 9 ⇒ 𝑃𝑎𝑡𝑡𝑒𝑟𝑛 𝑖𝑠 𝑑𝑖𝑣𝑖𝑑𝑒 𝑏𝑦 3 → 𝑚𝑢𝑙𝑡𝑖𝑝𝑙𝑦 𝑏𝑦
3
𝐷𝑒𝑛𝑜𝑚𝑖𝑛𝑎𝑡𝑜𝑟𝑠: 16 → 8 → 4 ⇒ 𝑃𝑎𝑡𝑡𝑒𝑟𝑛 𝑖𝑠 𝑚𝑢𝑙𝑡𝑖𝑝𝑙𝑦 𝑏𝑦 2
To get the final answer, we need both the numerator and the denominator patterns combined:
1 2
2× =
3 3
Common Ratio: Formula
To get the common ratio divide any term by the previous term:
𝑎2 27 81 27 16 2
𝑟= = 𝑎2 ÷ 𝑎1 = ÷ = × =
𝑎1 8 16 8 81 3
Number of Terms
4
𝑎𝑟 𝑛−1 =
9
81 2
Substitute 𝑓𝑖𝑟𝑠𝑡 𝑡𝑒𝑟𝑚 = 𝑎 = 16 , 𝑐𝑜𝑚𝑚𝑜𝑛 𝑟𝑎𝑡𝑖𝑜 𝑟 = 3 :
81 2 𝑛−1 4 2 𝑛−1 4 16 64 26 2 6
( ) = ⇒( ) = × = = 6=( )
16 3 9 3 9 81 729 3 3

Since the bases are the same, the exponents are also the same:
𝑛−1=6⇒𝑛 =7
Shortcut
This relies on the techniques used in Counting Lists.

P a g e 96 | 274
Get all the files at: https://bit.ly/azizhandouts
Aziz Manva (azizmanva@gmail.com)

81 34 4 22 3−2
= , = = ⇒ 4 − (−2) + 1 = 7
16 24 9 32 2−2

To go from 34 to 3−2 , the power of three must be decreased six times, and hence this is the seventh term.
Tenth Term
81 2
Substitute 𝑎 = 16 , 𝑟 = 3 , 𝑛 = 10 in 𝑎𝑟 𝑛−1 :

𝑛−1
81 2 10−1 34 2 9 34 × 29 25 32
𝑎𝑟 = ( ) = 4×( ) = 4 9
= 5=
16 3 2 3 2 ×3 3 243
General Term
81 2
Substitute 𝑎 = , 𝑟 = in 𝑎𝑟 𝑛−1 :
16 3

𝑛−1
81 2 𝑛−1 34 2𝑛−1 2𝑛−5
𝑎𝑟 = ( ) = 4 × 𝑛−1 = 𝑛−5
16 3 2 3 3

Example 3.20: Word Problems


16 8
A. Consider the geometric sequence , , 4, 6, 9 ….
9 3
What is the eighth term of the sequence? Express your
answer as a common fraction. (MathCounts 2004 Warm-Up 18)
1
B. What is the tenth term in the geometric sequence 9,3,1, 3 , … (MathCounts 2010 National Countdown)
C. The seventh and tenth terms of a geometric sequence are 7 and 21, respectively. What is the 13𝑡ℎ term
of this progression? (MathCounts 2010 Chapter Countdown)
D. The first term of a geometric sequence is 7 and the 7th term is 5103. What is the 5th term? (MathCounts
2000 Workout 9)
E. A geometric sequence of positive integers is formed for which the first term is 2 and the fifth term is
162. What is the sixth term of the sequence? (MathCounts 2007 Warm-Up 13)

Part A
16 3
Substitute 𝑎 = 9
,𝑟 = 2,𝑛 = 8

𝑛−1
16 3 8−1 24 × 37 35 243
𝑎𝑟 = ( )( ) = 2 = 3=
9 2 3 × 27 2 8
Part B
1
Substitute 𝑎 = 9, 𝑟 = 3 , 𝑛 = 10:
1 10−1 32 1 1
𝑎𝑟 𝑛−1 = (9) ( ) = 9= 7=
3 3 3 2187

Part C
𝑇𝑒𝑛𝑡ℎ 𝑇𝑒𝑟𝑚 𝑎𝑟 9 21
= 6 = 𝑟3 = =3
𝑆𝑒𝑣𝑒𝑛𝑡ℎ 𝑇𝑒𝑟𝑚 𝑎𝑟 7
13𝑡ℎ 𝑇𝑒𝑟𝑚 = 𝑎𝑟12 = (𝑎𝑟 9 )(𝑟 3 ) = (21)(3) = 63
Part D
𝑎=7
𝑎𝑟 6 = 5103 ⇒ 7𝑟 6 = 5103 ⇒ 𝑟 6 = 729
6
𝑟 = ± √729 = ±3
𝑎𝑟 4 = (7)(±3)4 = (7)(81) = 567
Part E
𝑎𝑟 4 = 162
2𝑟 4 = 162
𝑟 4 = 81

P a g e 97 | 274
Get all the files at: https://bit.ly/azizhandouts
Aziz Manva (azizmanva@gmail.com)
4
𝑟 = √81 = 3
𝑎𝑟 = (2)(35 ) = (2)(243) = 486
5

C. Finding the Term Number

Example 3.21
5
Which term of the sequence 320, −160,80, −40, … is equal to − 8?

5
𝑎𝑟 𝑛−1 = −
8
1 1
Substitute 𝑎 = 320 = 26 × 5, 𝑟 = − = :
2 −2
1 𝑛−1 5
(26 × 5) ( ) =− 3
−2 2
Collate numbers on one side:
1 1
=
2𝑛−1 (−2)9
Take reciprocals:
2𝑛−1 = (−2)9

Equate exponents, and solve for 𝑛:


𝑛−1=9
𝑛 = 10

D. Greatest Integer Function

3.22: Greatest Integer Function


The greatest integer function of 𝑥 is written
𝑦 = ⌊𝑥⌋
And it gives as its output the largest integer that is less than or equal to 𝑥.

⌊2.5⌋ = 2
⌊𝜋⌋ = 3
⌊3⌋ = 3

Example 3.23: Greatest Integer Function


Starting with the number 100, Shaffiq repeatedly divides his number by two and then takes the greatest integer
less than or equal to that number. How many times must he do this before he reaches the number 1?
(MathCounts 2003 Warm-Up 17)

100
𝑆𝑡𝑒𝑝 𝐼: ⌊⌋ = ⌊50⌋ = 50
2
50
𝑆𝑡𝑒𝑝 𝐼𝐼: ⌊ ⌋ = ⌊25⌋ = 25
2
25
𝑆𝑡𝑒𝑝 𝐼𝐼𝐼: ⌊ ⌋ = ⌊12.5⌋ = 12
2
12
𝑆𝑡𝑒𝑝 𝐼𝑉: ⌊ ⌋ = ⌊6⌋ = 6
2

P a g e 98 | 274
Get all the files at: https://bit.ly/azizhandouts
Aziz Manva (azizmanva@gmail.com)

6
𝑆𝑡𝑒𝑝 𝑉: ⌊ ⌋ = ⌊3⌋ = 3
2
3
𝑆𝑡𝑒𝑝 𝑉𝐼: ⌊ ⌋ = ⌊1.5⌋ = 1
2
No. of Times = 6

Try the above for 64, and then 128. Hence, what can you conclude for any number between 64 and 128.
Try the above for 32, and then 64. Hence, what can you conclude for any number between 32 and 64.
In general, what is the number of times you will apply this process to any number?

Step Number
1 64 128
⌊ ⌋ = 32 ⌊ ⌋ = 64
2 2
2 32 64
⌊ ⌋ = 16 ⌊ ⌋ = 32
2 2
3 16 32
⌊ ⌋=8 ⌊ ⌋ = 16
2 2
4 8 16
⌊ ⌋=4 ⌊ ⌋=8
2 2
5 4 8
⌊ ⌋=2 ⌊ ⌋=4
2 2
6 2 4
⌊ ⌋=1 ⌊ ⌋=2
2 2
7 2
⌊ ⌋=1
2

In general,
𝑎𝑛𝑦 𝑛𝑢𝑚𝑏𝑒𝑟 𝑏𝑒𝑡𝑤𝑒𝑒𝑛 64 𝑎𝑛𝑑 128 ⇒ 6 𝑆𝑡𝑒𝑝𝑠
𝑎𝑛𝑦 𝑛𝑢𝑚𝑏𝑒𝑟 𝑏𝑒𝑡𝑤𝑒𝑒𝑛 128 𝑎𝑛𝑑 256 ⇒ 7 𝑆𝑡𝑒𝑝𝑠
𝑎𝑛𝑦 𝑛𝑢𝑚𝑏𝑒𝑟 𝑏𝑒𝑡𝑤𝑒𝑒𝑛 2𝑛 𝑎𝑛𝑑 2𝑛+1 ⇒ 𝑛 𝑆𝑡𝑒𝑝𝑠
E. Number Theory

Example 3.24: Integer Solutions


A. A particular geometric sequence has strictly decreasing terms. After the first term, each successive term
𝑚
is calculated by multiplying the previous term by 7 . If the first term of the sequence is positive, how
many possible integer values are there for 𝑚? (MathCounts 2004 Warm-Up 16)
3𝑚
B. Redo Part A if each successive term is obtained by multiplying the previous term by 37
.

Part A
Note that the
𝑚
𝐶𝑜𝑚𝑚𝑜𝑛 𝑅𝑎𝑡𝑖𝑜 = , 𝑚∈ℤ
7
Since the terms are strictly decreasing:
𝑚
<1
7
If the common ratio is negative, the terms will alternate between positive and negative and that violates the
strictly decreasing condition:
𝑚
>0
7
Combine the above two conditions:

P a g e 99 | 274
Get all the files at: https://bit.ly/azizhandouts
Aziz Manva (azizmanva@gmail.com)

𝑚
0< <1
7
Multiply throughout by 7:
0<𝑚<7
𝑚 ∈ {1,2,3, … ,6} ⇒ 6 𝑉𝑎𝑙𝑢𝑒𝑠
Part B
As before,
3𝑚
0< < 1 ⇒ 𝑚 ∈ {1,2, … ,12} ⇒ 12 𝑉𝑎𝑙𝑢𝑒𝑠
37

Example 3.25: Decimal System


A. What is the second greatest three-digit number "𝑎𝑏𝑐" such 𝑎, 𝑏, 𝑐 form a geometric sequence?
B. What is the greatest three-digit number "𝑎𝑏𝑐" such that 4, 𝑎, 𝑏 forms a geometric sequence and 𝑏, 𝑐, 5
forms an arithmetic sequence? (MathCounts 2003 National Sprint)

Part A
The greatest three-digit number is 999, which forms a geometric sequence with common ratio= 1.
We need to find the second greatest three-digit number.
𝑎 is a digit in the decimal system. Largest possible value of 𝑎 is 9.

We need to find a common ratio such that 𝑏 and 𝑐 are also digits.
Suppose
8 8 64
𝑏 = 8 ⇒ 𝑎𝑟 = 8 ⇒ 9𝑟 = 8 ⇒ 𝑟 = ⇒ 𝑐 = 8 × = ⇒ 𝑁𝑜𝑡 𝑎𝑛 𝑖𝑛𝑡𝑒𝑔𝑒𝑟
9 9 9
7 7 49
𝑏 = 7 ⇒ 𝑎𝑟 = 7 ⇒ 9𝑟 = 7 ⇒ 𝑟 = ⇒ 𝑐 = 7 × = ⇒ 𝑁𝑜𝑡 𝑎𝑛 𝑖𝑛𝑡𝑒𝑔𝑒𝑟
9 9 9
6 2 2
𝑏 = 6 ⇒ 𝑎𝑟 = 6 ⇒ 9𝑟 = 6 ⇒ 𝑟 = = ⇒ 𝑐 = 6 × = 4 ⇒ 𝐼𝑛𝑡𝑒𝑔𝑒𝑟
9 3 3
𝑎𝑏𝑐 = 964
Part B
We get a geometric sequence from:
4, 𝑎, 𝑏
Let the common ratio of the geometric sequence be 𝑟.
4, 𝑎 = 4𝑟, 𝑏 = 4𝑟 2
We want 𝑏 to be as large as possible. But 𝑏 is a digit in the decimal system. Largest possible value is 9. Try:
9 3 3
𝑏 = 9 ⇒ 4𝑟 2 = 9 ⇒ 𝑟 2 = ⇒ 𝑟 = ⇒ 𝑎 = 4 × = 6
4 2 2
9 + 5 14
9, 𝑐, 5 𝑓𝑜𝑟𝑚 𝑎𝑛 𝑎𝑟𝑖𝑡ℎ𝑚𝑒𝑡𝑖𝑐 𝑠𝑒𝑞𝑢𝑒𝑛𝑐𝑒 ⇒ 𝑐 = = =7
2 2
𝑎𝑏𝑐 = 697
F. Exponents

Example 3.26
A. The sequence 1,000,000; 500,000; 250,000 and so on, is made by repeatedly dividing by 2. What is the
last integer in this sequence? (MathCounts 2007 Workout 5)
B. By starting with a million and alternatively dividing by 2 and multiplying by 5, Anisha created a
sequence of integers that starts 1000000, 500000, 2500000, 1250000, and so on. What is the last
integer in her sequence? Express your answer in the form 𝑎𝑏 , where 𝑎 and 𝑏 are positive integers and 𝑎
is as small as possible. (MathCounts 2008 National Sprint)
C. Answer Part B if Anisha started with 10𝑛 .

P a g e 100 | 274
Get all the files at: https://bit.ly/azizhandouts
Aziz Manva (azizmanva@gmail.com)

Part A
1,000,000 = 106 = 56 × 26
When we keep dividing by 2 the power of 2 will keep reducing, until there are no more powers of 2. In that
scenario, the last integer will be:
56 = 15625
Part B
1,000,000 = 106 = 56 × 26
Suppose Anisha divides by 2, and multiplies by 5:
5
56 × 26 × = 57 × 25
2
Each time we divide by 2 and multiply by 5:
𝑃𝑜𝑤𝑒𝑟 𝑜𝑓 2 𝑟𝑒𝑑𝑢𝑐𝑒𝑠 𝑏𝑦 1, 𝑝𝑜𝑤𝑒𝑟 𝑜𝑓 5 𝑖𝑛𝑐𝑟𝑒𝑎𝑠𝑒𝑠 𝑏𝑦 1

We can keep doing this so long as we have powers of 2, which is 6.


Hence, the last integer will be:
512
Part C
10𝑛 = 5𝑛 × 2𝑛
Each time we divide by 2 and multiply by 5:
𝑃𝑜𝑤𝑒𝑟 𝑜𝑓 2 𝑟𝑒𝑑𝑢𝑐𝑒𝑠 𝑏𝑦 1, 𝑝𝑜𝑤𝑒𝑟 𝑜𝑓 5 𝑖𝑛𝑐𝑟𝑒𝑎𝑠𝑒𝑠 𝑏𝑦 1
Hence, the last integer:
52𝑛

3.27: Terms in 𝑮𝑷 ⇒Exponents in AP


Consider a geometric progression given by 𝑥 𝑎 , 𝑥 𝑏 , 𝑥 𝑐 . Then, 𝑎, 𝑏, 𝑐 are in arithmetic progression.

It is given that we have a geometric progression:


𝑥𝑎, 𝑥𝑏, 𝑥𝑐
Let the common ratio be 𝑟:
𝑥 𝑎 , 𝑟𝑥 𝑎 , 𝑟 2 𝑥 𝑎
Let 𝑟 = 𝑥 𝑑 :
𝑥 𝑎 , 𝑥 𝑑 𝑥 𝑎 , 𝑥 2𝑑 𝑥 𝑎
Simplify using the multiplication rule of exponents:
𝑥 𝑎 , 𝑥 𝑎+𝑑 , 𝑥 𝑎+2𝑑

Example 3.28
3 6
The first three terms of a geometric progression are √2, √2, √2. Find the fourth term. (AHSME 1961/12)

3 6
1 1 1 3 2 1
√2, √2, √2 = 22 , 23 , 26 = 26 , 26 , 26

3 2 1 1
, , ⇒𝑑=𝑟=−
6 6 6 6
1 1
4𝑡ℎ 𝑇𝑒𝑟𝑚 = 26−6 = 20 = 1
G. Factorials

3.29: Factorials

P a g e 101 | 274
Get all the files at: https://bit.ly/azizhandouts
Aziz Manva (azizmanva@gmail.com)

𝑛! is defined to be the product of the first 𝑛 natural numbers:


𝑛! = 1 × 2 × 3 × … × 𝑛
By definition:
0! = 1

3! = 6
5! = 120

Example 3.30
The second and sixth term of a geometric sequence are 3! and 4! respectively. Find the first term.

𝑎, 𝑎𝑟 ⏟5
⏟ , … , 𝑎𝑟
𝑆𝑒𝑐𝑜𝑛𝑑 𝑆𝑖𝑥𝑡ℎ
𝑇𝑒𝑟𝑚 𝑇𝑒𝑟𝑚

𝑎𝑟 5 = 4! = 24
𝑎𝑟 = 3! = 6
𝑎𝑟 5 24 1 1
= ⇒ 𝑟 4 = 4 ⇒ 𝑟 = ±44 = ±22 = ±√2
𝑎𝑟 6
𝑎𝑟 6 6√2
𝑎= = =± = ±3√2
𝑟 ±√2 2
H. Geometry

Example 3.31
The volume of a certain rectangular solid is 8 𝑐𝑚3, its total surface area is 32 𝑐𝑚2, and its three dimensions are
in geometric progression. The sums of the lengths in cm of all the edges of this solid is (AHSME 1985/25)

Without loss of generality, let the dimensions be:



𝑎 , 𝑎𝑟 ⏟2
⏟ , 𝑎𝑟
𝑤 𝑙 ℎ
The sum of the edges is:
4(𝑙 + 𝑤 + ℎ) = 4(𝑎 + 𝑎𝑟 + 𝑎𝑟 2 )

The volume is the product of the three dimensions:


𝑉 = 𝑙𝑤ℎ = (𝑎)(𝑎𝑟)(𝑎𝑟 2 ) = 𝑎3 𝑟 3 = 8 ⇒ 𝑎𝑟 = 2

Substitute the dimensions in 𝑆𝐴 = 2(𝑙ℎ + 𝑤ℎ + 𝑙𝑤):


𝑆𝐴 = 2[(𝑎𝑟)(𝑎𝑟 2 ) + (𝑎)(𝑎𝑟 2 ) + (𝑎𝑟)(𝑎)] = 32
Factor 𝑎𝑟 on the LHS:
2𝑎𝑟[𝑎𝑟 2 + 𝑎𝑟 + 𝑎] = 32
Substitute 2𝑎𝑟 = 4:
4[𝑎𝑟 2 + 𝑎𝑟 + 𝑎] = 32

3.2 Exponential Growth and Decay


A. Exponential Growth

3.32: Exponential Growth


When a quantity increases every period by a fixed percentage of the value in its current time, it called

P a g e 102 | 274
Get all the files at: https://bit.ly/azizhandouts
Aziz Manva (azizmanva@gmail.com)

exponential growth.

➢ Compound Interest
➢ Appreciation: Increase in value of an asset
➢ Growth of an organism under ideal conditions

Example 3.33: Biology


A colony of bacteria in a petri dish grows by 50% each hour. If the colony is seeded with 64 bacteria by a lab
technician, what is the population of bacteria at the end of the fifth hour?

𝐹𝑖𝑟𝑠𝑡 𝑇𝑒𝑟𝑚 = 𝑎 = 64 No. of Bacteria 64 96


𝐺𝑟𝑜𝑤𝑠 𝑏𝑦 50% ⇒ 𝐵𝑒𝑐𝑜𝑚𝑒𝑠 150% 𝑜𝑓 𝑤ℎ𝑎𝑡 𝑖𝑡 𝑤𝑎𝑠 Expression 𝑎 𝑎𝑟 𝑎𝑟 2 𝑎𝑟 3 𝑎𝑟 4 𝑎𝑟 5
3 Hour(𝑡) 0 1 2 3 4 5
𝐶𝑜𝑚𝑚𝑜𝑛 𝑟𝑎𝑡𝑖𝑜 = 𝑟 = 150% =
2 Term(𝑛) 1 2 3 4 5 6
𝑛=6
3
Substitute 𝑎 = 64, 𝑟 = 2 , 𝑛 = 6
3 6−1 3 5 243
𝑎𝑟 𝑛−1 = (64) ( ) = 64 × ( ) = 64 × = 486
2 2 32

Logical Method
𝑆𝑡𝑎𝑟𝑡 𝑎𝑡 𝐻𝑜𝑢𝑟 𝑍𝑒𝑟𝑜
5
3 5 6
35
𝐻𝑜𝑢𝑟 𝐹𝑖𝑣𝑒 = (𝐻𝑜𝑢𝑟 𝑍𝑒𝑟𝑜)(𝐶𝑜𝑚𝑚𝑜𝑛 𝑅𝑎𝑡𝑖𝑜) = (64) ( ) = 2 × 5 = 2 × 243 = 486
2 2

Example 3.34: Biology


A certain organism begins as two cells. Each cell splits and becomes two cells at the end of three days. At the
end of another three days, every cell of the organism splits and becomes two cells. This process lasts for a total
of 15 days, and no cells die during this time. How many cells are there at the end of the 15𝑡ℎ day? (MathCounts
2006 School Sprint)

Logical Method
𝑁𝑜. 𝑜𝑓 𝐷𝑎𝑦𝑠 15
𝑁𝑜. 𝑜𝑓 𝑆𝑝𝑙𝑖𝑡𝑠 = = =5
𝐷𝑎𝑦𝑠 𝑝𝑒𝑟 𝑠𝑝𝑙𝑖𝑡 3
(𝑆𝑡𝑎𝑟𝑡𝑖𝑛𝑔 𝑣𝑎𝑙𝑢𝑒)(𝐺𝑟𝑜𝑤𝑡ℎ 𝐹𝑎𝑐𝑡𝑜𝑟)𝑁𝑜.𝑜𝑓 𝑆𝑝𝑙𝑖𝑡𝑠 = (2)(25 ) = 26 = 64

Geometric Sequences Value


Substitute 𝑎 = 2, 𝑟 = 2, 𝑛 = 6: Term 𝑎 𝑎𝑟 𝑎𝑟 2 𝑎𝑟 5
𝑎𝑟 𝑛−1 = (2)(26−1 ) = 2(25 ) = 26 = 64 Day 0 3 6 9 12 15
Splits 0 1 2 3 4 5
Term 1 2 3 4 5 6
Example 3.35: Biologyu Number
A tree doubled its height every year until it reached a height of 32 feet at the end of 6 years. What was the
height of the tree, in feet, at the end of 3 years? (MathCounts 2002 Chapter Sprint)

Logical Method
32 𝑓𝑒𝑒𝑡 = ⏟
⏟ 16 𝑓𝑒𝑒𝑡 = 8⏟𝑓𝑒𝑒𝑡 = 4⏟𝑓𝑒𝑒𝑡
6 𝑌𝑒𝑎𝑟𝑠 5 𝑌𝑒𝑎𝑟𝑠 4 𝑌𝑒𝑎𝑟𝑠 3 𝑌𝑒𝑎𝑟𝑠
Formal Method

P a g e 103 | 274
Get all the files at: https://bit.ly/azizhandouts
Aziz Manva (azizmanva@gmail.com)


𝑎 , 𝑎𝑟, 𝑎𝑟 2 , 𝑎𝑟
⏟3 , 𝑎𝑟 4 , 𝑎𝑟 5 , 𝑎𝑟
⏟6
𝑍𝑒𝑟𝑜𝑡ℎ 𝑇ℎ𝑖𝑟𝑑 𝑆𝑖𝑥𝑡ℎ
𝑌𝑒𝑎𝑟 𝑌𝑒𝑎𝑟 𝑌𝑒𝑎𝑟
Substitute 𝑎𝑟 6 = 32, 𝑟 = 2 ⇒ 𝑟 3 = 8 in
𝑎𝑟 6 32 32
3
= 𝑟 3 ⇒ 3 = 8 ⇒ 𝑎𝑟 3 = =4
𝑎𝑟 𝑎𝑟 8

Example 3.36: Finance


Mike paid 1.25 for a stamp three years ago. He was just offered double that amount for the stamp. Assuming the
stamp's offer price doubles every three years, how many dollars will he be offered in 12 more years?
(MathCounts 2006 State Countdown)

Logical Method
1.25
⏟ → 2.5
⏟ → ⏟
5 → 10
⏟ → 20
⏟ → 40

3 𝑌𝑒𝑎𝑟𝑠 𝑁𝑜𝑤 3 𝑌𝑒𝑎𝑟𝑠 6 𝑌𝑒𝑎𝑟𝑠 9 𝑌𝑒𝑎𝑟𝑠 12 𝑌𝑒𝑎𝑟
𝐴𝑔𝑜 𝐿𝑎𝑡𝑒𝑟 𝐿𝑎𝑡𝑒𝑟 𝐿𝑎𝑡𝑒𝑟 𝐿𝑎𝑡𝑒𝑟𝑠

Geometric Sequences:

𝑎 ⏟ , 𝑎𝑟 2 , 𝑎𝑟 3 , 𝑎𝑟 4 ,
, 𝑎𝑟 ⏟5
𝑎𝑟
3 𝑌𝑒𝑎𝑟𝑠 𝑁𝑜𝑤 𝑇𝑤𝑒𝑙𝑣𝑒
𝑃𝑟𝑖𝑜𝑟 𝑌𝑒𝑎𝑟𝑠 𝐿𝑎𝑡𝑒𝑟
Substitute 𝑎 = 1.25, 𝑟 = 2:
𝑎𝑟 5 = 1.25(25 ) = 1.25(32) = 5 × 8 = 40

B. Exponential Decay

3.37: Exponential Decay


When a quantity decreases every time period by a fixed percentage of the value in its current time, it called
exponential decay.

➢ Depreciation: Decrease in value of an asset


➢ Radioactive decay

Example 3.38: Everyday Scenarios


The doctor has told Cal O'Ree that during his ten weeks of working out at the gym, he can expect each week's
weight loss to be 1% of his weight at the end of the previous week. His weight at the beginning of the workouts
is 244 pounds. How many pounds does he expect to weigh at the end of the ten weeks? Express your answer as
a mathematical expression. (MathCounts 2003 Warm-Up 6, Adapted)

244 × 0.9910

Example 3.39: Multiple Triangles


Each triangle is a 30-60-90 triangle, and the hypotenuse of one triangle is the longer
leg of an adjacent triangle. The hypotenuse of the largest triangle is 8 centimeters.
What is the number of centimeters in the length of the longer leg of the smallest
triangle? Express your answer as a common fraction. (MathCounts 1999 School
Target)

All four triangles are 30 − 60 − 90 triangles. The longer leg of the largest triangle is:

P a g e 104 | 274
Get all the files at: https://bit.ly/azizhandouts
Aziz Manva (azizmanva@gmail.com)

√3

2
The longer leg of the second-largest triangle is:
2
√3
8×( )
2
The longer leg of the smallest triangle is:
4
√3 9 9
8×( ) =8× =
2 4 2
√3
Note that this forms a geometric sequence with 𝑎 = 8, 𝑟 = 2
:
2 𝑛
√3 √3 √3
8× ,8 × ( ) ,…,8 × ( )
⏟ 2 ⏟ 2 ⏟ 2
𝐿𝑎𝑟𝑔𝑒𝑠𝑡 2𝑛𝑑 𝐿𝑎𝑟𝑔𝑒𝑠𝑡
𝑛𝑡ℎ 𝐿𝑎𝑟𝑔𝑒𝑠𝑡
𝑇𝑟𝑖𝑎𝑛𝑔𝑙𝑒 𝑇𝑟𝑖𝑎𝑛𝑔𝑙𝑒 𝑇𝑟𝑖𝑎𝑛𝑔𝑙𝑒

Challenge 3.40: Finding the 𝒏𝒕𝒉 term


An equilateral triangle is originally painted black. Each time the triangle is changed, the middle fourth of each
black triangle turns white. After five changes, what fractional part of the original area of the black triangle
remains black? (AMC 8 1991/25)

Logic
Let the area of the original triangle be 1.

First change: The area of the triangle that is black is


3
4
3𝑡ℎ
Second change: The white area remains the same. But each triangle has 4 black area of what it had.
Hence, overall black area after second change
3 3 3 2
= × =( )
4 4 4

Fifth Change: Continuing the pattern, the black area after five changes
3 5 243
=( ) =
4 1024
Geometric Sequence
From the above, we can see that the area of the triangle forms a geometric sequence with
3
𝑎 = 1, 𝑟 =
4
And hence, we can find the area as

P a g e 105 | 274
Get all the files at: https://bit.ly/azizhandouts
Aziz Manva (azizmanva@gmail.com)

3 3 2 3 5

1 , , ( ) ,…, ( )

4 ⏟4 ⏟4
𝑶𝒓𝒊𝒈𝒊𝒏𝒂𝒍
𝑭𝒊𝒓𝒔𝒕 𝑺𝒆𝒄𝒐𝒏𝒅 𝑭𝒊𝒇𝒕𝒉
𝑪𝒉𝒂𝒏𝒈𝒆 𝑪𝒉𝒂𝒏𝒈𝒆 𝑪𝒉𝒂𝒏𝒈𝒆
2
3 3 3 5

1, ,( ) ,…,( )

4 ⏟ 4 ⏟4
𝒂
𝒂𝒓 𝒂𝒓𝟐 𝒂𝒓𝟓
Note that the:
➢ The First Change is represented by the Second Term
➢ The Fifth Change is represented by the Sixth Term

C. Radio Carbon Dating

3.41: Radio Carbon Dating


Radio carbon is used to estimate the age of fossils.

A living organism contains multiples isotopes of carbon, of which one is Carbon-14.


➢ When an organism is alive, it has the same of ratio of Carbon-14 as the atmosphere.
➢ When an organism dies, other carbon isotopes remains as is, but Carbon-14 decays into other carbon
isotopes.
➢ The rate of decay of Carbon-14, is such that approximately half of the Carbon-14 decays every 5730
years(approximate).
➢ Half-life is the time taken for a quantity to decay to half of its original value.
➢ This can be used to estimate the age of certain fossils.

Example 3.42: Radio Carbon Dating


For this question, approximate the half-life of Carbon-14 as 6000 years.
A. If Carbon-14 is kept for 12,000 years, find the quantity as a fraction of the original quantity.
1
B. Find the time taken for Carbon-14 to decay to 16 𝑡ℎ of its original value.
1
C. If Carbon-14 in a fossil is 𝑡ℎ of its original value, then find the age of the fossil.
8

1 1 1 1

1 → → → →
𝑂𝑟𝑖𝑔𝑖𝑛𝑎𝑙

2 ⏟
4 ⏟
8 ⏟
16
𝑄𝑢𝑎𝑛𝑡𝑖𝑡𝑦 𝐴𝑓𝑡𝑒𝑟 𝐴𝑓𝑡𝑒𝑟 𝐴𝑓𝑡𝑒𝑟 𝐴𝑓𝑡𝑒𝑟
6000 𝑌𝑒𝑎𝑟𝑠 12000 𝑌𝑒𝑎𝑟𝑠 18000 𝑌𝑒𝑎𝑟𝑠 24000 𝑌𝑒𝑎𝑟𝑠
Part A
1
4
Part B
24,000 𝑌𝑒𝑎𝑟𝑠
Part C
18,000 𝑌𝑒𝑎𝑟𝑠

Example 3.43: Radio Carbon Dating


1
Estimation techniques work reliably so long as Carbon-14 is greater than or equal to of its original value.
1024
Find the maximum age of a fossil that can be estimated using Carbon-14 dating. (Use 6000 years as the half-life
of Carbon-14).

P a g e 106 | 274
Get all the files at: https://bit.ly/azizhandouts
Aziz Manva (azizmanva@gmail.com)

1 1 1 1

1 → → → →⋯→

21 ⏟2
2 ⏟3
2 ⏟𝑛
2
𝑂𝑟𝑖𝑔𝑖𝑛𝑎𝑙
𝑄𝑢𝑎𝑛𝑡𝑖𝑡𝑦 𝐴𝑓𝑡𝑒𝑟 𝐴𝑓𝑡𝑒𝑟 𝐴𝑓𝑡𝑒𝑟 𝐴𝑓𝑡𝑒𝑟
6000 𝑌𝑒𝑎𝑟𝑠 6000×2 𝑌𝑒𝑎𝑟𝑠 6000×3 𝑌𝑒𝑎𝑟𝑠 6000×𝑛 𝑌𝑒𝑎𝑟𝑠
1 1
= 10 ⇒ 𝑛 = 10 ⇒ 6000 × 10 = 60,000 𝑦𝑒𝑎𝑟𝑠
1024 2

D. Finding the Term Number

Example 3.44: Finding the Term Number


A. The bacteria in a lab dish double in number every four hours. If 500 bacteria cells are in the dish now, in
how many hours will there be exactly 32,000 bacteria? (MathCounts 2006 National Countdown)
B. Jasmine had 3 paperclips on Monday, then she had 6 on Tuesday, and her number of paperclips
proceeded to double on each subsequent day. On what day of the week did she first have more than 100
paperclips? (MathCounts 2007 Chapter Countdown)
2
C. A ball bounces back up 3 of the height from which it falls. If the ball is dropped from a height of 243 cm,
after how many bounces does the ball first rise less than 30 cm? (MathCounts 2006 Chapter Team)
D. Zeno had to paint a 15′ × 15′ square floor. He decided that each day he would paint half of the
unpainted part from the previous day until there was only one square foot or less left, in which case he
would stay and finish the job that day. Using this strategy, how many days did it take Zeno to paint the
entire floor? (MathCounts 1998 Chapter Team)

Part A 2 𝑛−1
Calculate in thousands of bacteria: 243 × ( ) < 30
3
0.5 → 1 → 2 → 4 → 8 → 16 → 32 ⇒ 6 𝐷𝑜𝑢𝑏𝑙𝑖𝑛𝑔𝑠 2 𝑛−1
= 24 𝐻𝑜𝑢𝑟𝑠 35 × ( ) < 3 × 10
3
2𝑛−1
Substitute 𝑎 = 500, 𝑟 = 2, 𝑇𝑒𝑟𝑚 𝑉𝑎𝑙𝑢𝑒 = 32000: 34 × 𝑛−1 < 10
3
𝑎𝑟 𝑛−1 = 32000 2𝑛−1
(500)(2𝑛−1 ) = 32000 < 10
3𝑛−5
2𝑛−1 = 64 = 26 ⇒ 6 𝐷𝑜𝑢𝑏𝑙𝑖𝑛𝑔𝑠 = 24 𝐻𝑜𝑢𝑟𝑠 2𝑛−1 < 10 × 3𝑛−5
Part B Part D
Substitute 𝑎 = 3, 𝑟 = 2: Consider the unpainted floor at the end of each day:
3 × 2𝑛−1 > 100 1 1 1
1 , ,

2 ⏟
4 ⏟
8
2𝑛−1 > 33 𝐹𝑖𝑟𝑠𝑡 𝑆𝑒𝑐𝑜𝑛𝑑 𝑇ℎ𝑖𝑟𝑑
3 𝐷𝑎𝑦 𝐷𝑎𝑦 𝐷𝑎𝑦
2𝑛−1 = 64 = 26 1 1
𝐴𝑑𝑑 6 𝐷𝑎𝑦𝑠 = 𝑀𝑜𝑛 + 6 = 𝑆𝑢𝑛𝑑𝑎𝑦 This is a geometric sequence with 𝑎 = 2 , 𝑟 = 2
Part C We want the area of the unpainted floor to be
𝐷𝑟𝑜𝑝 𝐻𝑒𝑖𝑔ℎ𝑡 = 35 < 1 𝑓𝑡 2
𝐹𝑖𝑟𝑠𝑡 𝐵𝑜𝑢𝑛𝑐𝑒 = 2 × 34 Convert this to a fraction of the total area of the
𝑆𝑒𝑐𝑜𝑛𝑑 𝐵𝑜𝑢𝑛𝑐𝑒 = 4 × 33 floor:
𝑇ℎ𝑖𝑟𝑑 𝐵𝑜𝑢𝑛𝑐𝑒 = 8 × 32 1
<
𝐹𝑜𝑢𝑟𝑡ℎ 𝐵𝑜𝑢𝑛𝑐𝑒 = 16 × 3 = 48 > 30 225
𝐹𝑖𝑓𝑡ℎ 𝐵𝑜𝑢𝑛𝑐𝑒 = 32 > 30 And on the 𝑛𝑡ℎ day, the area left unpainted is given
64 by the 𝑛𝑡ℎ term of the geometric sequence:
𝑆𝑖𝑥𝑡ℎ 𝐵𝑜𝑢𝑛𝑐𝑒 = < 30 1 1 𝑛−1 1
3 𝑎𝑟 𝑛−1
= ( )( ) = 𝑛
2 2 2

P a g e 107 | 274
Get all the files at: https://bit.ly/azizhandouts
Aziz Manva (azizmanva@gmail.com)

And, we want the unpainted area to be less than 2𝑛 = 256 = 28


Zeno’s condition: 𝑛=8
1 1 8 𝐷𝑎𝑦𝑠
<

2 𝑛 ⏟
225
𝐴𝑐𝑡𝑢𝑎𝑙 𝐶𝑜𝑛𝑑𝑖𝑡𝑖𝑜𝑛 𝑓𝑜𝑟
𝑈𝑛𝑝𝑎𝑖𝑛𝑡𝑒𝑑 𝑈𝑛𝑝𝑎𝑖𝑛𝑡𝑒𝑑
𝐴𝑟𝑒𝑎 𝐴𝑟𝑒𝑎

Example 3.45: Geometric or Arithmetic


A. A condominium in New York costs a million dollars now (in 2020), and increases in price by a hundred
thousand dollars every year. Find the price of the condominium in 2025.
B. A car costs Rs. 20 Lakhs to purchase (Year 0), and depreciates 3 lakhs in value every year. Insurance
premium is payable on the car at 3% of the value of the car. For which year will the insurance premium
of the car first go below Rs. 30000 per year.

Part A
1,000,000 + 4 × 100,000 = 1,400,000
Part B
Calculate everything in lakhs:
0.03(20 − 3𝑦) < 0.3
3(20 − 3𝑦) < 30
20 − 3𝑦 < 10
20 − 10 < 3𝑦
10 < 3𝑦
10 1
𝑦> =3
3 3
𝑦=4
𝐹𝑖𝑓𝑡ℎ 𝑦𝑒𝑎𝑟

Example 3.46
A. Anton has two species of ants, Species A and Species B, in his ant farm. The two species are identical in
appearance, but Anton knows that every day, there are twice as many ants of Species A than before,
while there are three times as many ants of Species B. On Day 0, Anton counts that there are 30 ants in
his ant farm. On Day 5, Anton counts that there are 3281 ants in his ant farm. How many of these are of
Species A? (AOPS Alcumus, Algebra, Geometric Sequences)
B. A certain organization consists of five leaders and some number of regular members. Every year, the
current leaders are kicked out of the organization. Next, each regular member must find two new people
to join as regular members. Finally, five new people are elected from outside the organization to become
leaders. In the beginning, there are fifteen people in the organization total. How many people total will
be in the organization five years from now? (AOPS Alcumus, Algebra, Geometric Sequences)

Part A
(25 )𝑎 + (35 )(30 − 𝑎) = 3281
32𝑎 + (243)(30 − 𝑎) = 3281
32𝑎 + 7290 − 243𝑎 = 3281
4009 = 211𝑎
𝑎 = 19
32𝑎 = 608
Part B
𝑚 + 5 → 𝑚 → 3𝑚 → 3𝑚 + 5

P a g e 108 | 274
Get all the files at: https://bit.ly/azizhandouts
Aziz Manva (azizmanva@gmail.com)

𝐶𝑢𝑟𝑟𝑒𝑛𝑡 𝑀𝑒𝑚𝑏𝑒𝑟𝑠 = 15 − 5 = 10
After five years,
𝑀𝑒𝑚𝑏𝑒𝑟𝑠 = 10 × 35 = 10 × 243 = 2430
𝑇𝑜𝑡𝑎𝑙 = 𝑀𝑒𝑚𝑏𝑒𝑟𝑠 + 𝐿𝑒𝑎𝑑𝑒𝑟𝑠 = 2430 + 5 = 2435
⏟ 𝑝𝑒𝑜𝑝𝑙𝑒
𝐴𝑛𝑠𝑤𝑒𝑟
3.3 More Definitions
A. Geometric Mean

3.47: Geometric Mean of Two Numbers


The geometric mean of two numbers 𝑎 and 𝑏 is the square root of the product of the two numbers:
√𝑎𝑏

➢ Geometric mean is usually not defined if the numbers for which we are taking the mean are negative.

Example 3.48
Calculate the geometric mean:
A. 16, 81
B. 12, 75
C. 2002, 154

√16 × 81 = 4 × 9 = 36
√12 × 75 = √4 × 3 × 3 × 25 = 2 × 3 × 5 = 30
√2002 × 154 = √(2 × 7 × 11 × 13) × (2 × 7 × 11) = 154√13

3.49: Geometric Mean of 𝒏 Numbers


The geometric mean of 𝑛 numbers 𝑥1 , 𝑥2 , … 𝑥𝑛 is the 𝑛𝑡ℎ root of the product of the 𝑛 numbers:
Geometric Mean = 𝐺𝑀 = 𝑛√𝑥1 ⋅ … ⋅ 𝑥𝑛

Example 3.50
Calculate the geometric mean:
A. 6, 9,12

3 3 3 3
√6 × 9 × 12 = √2 × 3 × 32 × 22 × 3 = √23 × 33 × 3 = 6√3

3.51: Multiplying Factor


If a quantity grows by 𝑟%, then
𝑟
𝑀𝑢𝑙𝑡𝑖𝑝𝑙𝑦𝑖𝑛𝑔 𝐹𝑎𝑐𝑡𝑜𝑟 = 𝑀𝐹 = 1 +
100
𝑟
= 1 − 𝑀𝐹
100

Example 3.52: Multiplying Factor


A. Growth =5%

𝐺𝑟𝑜𝑤𝑡ℎ = 5% ⇒ 𝑀𝑢𝑙𝑡𝑖𝑝𝑙𝑦𝑖𝑛𝑔 𝐹𝑎𝑐𝑡𝑜𝑟 = 1.05

P a g e 109 | 274
Get all the files at: https://bit.ly/azizhandouts
Aziz Manva (azizmanva@gmail.com)

Example 3.53: Calculator Allowed


A. Ahana joined an actuarial firm at a salary of $𝑑 per year. She got a raise of 10% at the end of the first
year. She got a raise of 20% at the end of the second year. In the third, there was a recession, and she
took a salary cut of 5%. Find the average growth in her salary over the three years.

We convert the growth percentages into multiplying factors, and then multiply
𝑡0 = 𝑑
𝑡1 = 𝑑 × 1.1
𝑡2 = 𝑑 × 1.1 × 1.2
𝑡3 = 𝑑 × 1.1 × 1.2 × 0.95
Multiplying Factor for the last three years
𝑑 × 1.1 × 1.2 × 0.95
= = 1.1 × 1.2 × 0.95
𝑑
Average Multiplying Factor
3
𝑚 = √1.1 × 1.2 × 0.95 = 1.07836
Growth rate
= 1.07836 − 1 = 0.07836 = 7.836% = 𝑟

𝑑, 𝑑𝑚, 𝑑𝑚2 , 𝑑𝑚3


𝑚 = 1.07836 ⇒ 𝑚3 = 1.253982
B. Geometric Mean Property
Just like an arithmetic sequence has the arithmetic mean property, so a geometric sequence has the geometric
mean property.
This property is very useful since
➢ It can be applied to any geometric sequence.
➢ It can be used to show that a sequence is a geometric sequence.

Example 3.54
The fifth term of a geometric sequence of positive numbers is 11 and the eleventh term is 5. What is the eighth
term of the sequence? Express your answer in simplest radical form. (MathCounts 2010 State Sprint)

𝑎𝑟 4 = 11 ,
𝑡5 : ⏟ 𝑎𝑟10 = 5
𝑡11 : ⏟
𝑬𝒒𝒖𝒂𝒕𝒊𝒐𝒏 𝑰 𝑬𝒒𝒖𝒂𝒕𝒊𝒐𝒏 𝑰𝑰

Multiply Equations I and II:


𝑡5 𝑡11 = (𝑎𝑟 4 )(𝑎𝑟10 ) = 𝑎2 𝑟14 = (𝑎𝑟 7 )2 = (𝑡8 )2

Take square roots both sides:


𝑡8 = √𝑡5 𝑡11 = √11 × 5 = √55

3.55: Geometric Mean Property-I


Any term of a geometric sequence is the geometric mean of the terms that precede and follow it
𝑎𝑛−1 , 𝑎𝑛 , 𝑎𝑛+1 𝑎𝑟𝑒 𝑎 𝑔𝑒𝑜𝑚𝑒𝑡𝑟𝑖𝑐 𝑠𝑒𝑞𝑢𝑒𝑛𝑐𝑒 ⇒ 𝑎𝑛 = √𝑎𝑛−1 × 𝑎𝑛+1

Let the terms be:


𝑎𝑛−1 = 𝑎𝑟 𝑛−1 , 𝑎𝑛 = 𝑎𝑟, 𝑎𝑛+1 = 𝑎𝑟 𝑛+1

P a g e 110 | 274
Get all the files at: https://bit.ly/azizhandouts
Aziz Manva (azizmanva@gmail.com)

√𝑎𝑛−1 × 𝑎𝑛+1 = √𝑎𝑟 𝑛−1 × 𝑎𝑟 𝑛+1 = √𝑎2 𝑟 2𝑛 = 𝑎𝑟 𝑛 = 𝑎𝑛


Equivalently:
(𝑎𝑛 )2 = 𝑎𝑛−1 × 𝑎𝑛+1

Example 3.56
Show that:
A. 𝑡2 = √𝑡1 𝑡3 and 𝑡3 = √𝑡2 𝑡4 in the geometric sequence 𝑡1 = 2, 𝑡2 = 4, 𝑡3 = 8, 𝑡4 = 16
B. 𝑡2 = √𝑡1 𝑡3 and 𝑡3 = √𝑡2 𝑡4 in the general geometric sequence 𝑡1 = 𝑎, 𝑡2 = 𝑎𝑟, 𝑡3 = 𝑎𝑟 2 , 𝑡4 = 𝑎𝑟 3

Part A
√𝑡1 𝑡3 = √2 × 8 = √16 = 4 = 𝑡2
√𝑡2 𝑡4 = √4 × 16 = √64 = 8 = 𝑡3
Part B
√𝑡1 𝑡3 = √𝑎 × 𝑎𝑟 2 = √𝑎2 𝑟 2 = 𝑎𝑟 = 𝑡2
√𝑡2 𝑡4 = √𝑎𝑟 × 𝑎𝑟 3 = √𝑎2 𝑟 4 = 𝑎𝑟 2 = 𝑡3

Example 3.57
A. Suppose 𝑥, 𝑦, 𝑧 from a geometric sequence. Find the geometric mean of 𝑥 and 𝑧 in terms of 𝑦.
B. If 36, 𝑥, 9, 𝑦 form a decreasing geometric sequence, then find the value of 𝑦.
C. If 24,32, 𝑥 are three consecutive terms of a geometric sequence, then find the value of 𝑥.
D. If 𝑥 + 1, 𝑥 + 2, 𝑥 + 3 form a geometric progression, then find the possible value(s) of 𝑥.

Part C
Part A 32 = √24 × 𝑥
The geometric mean of 𝑥 and 𝑧: 210 = 24𝑥
√𝑥𝑧 = 𝑦 210 210 27 128
Part B 𝑥= = 3 = =
24 2 × 3 3 3
𝑥 = √36 × 9 = 6 × 3 = 18 Part D
9 = √18𝑦 (𝑥 + 2)2 = (𝑥 + 1)(𝑥 + 3)
81 = 18𝑦 𝑥 2 + 4𝑥 + 4 = 𝑥 2 + 4𝑥 + 3
81 9 4=3
𝑦= = 𝑁𝑜𝑡 𝑃𝑜𝑠𝑠𝑖𝑏𝑙𝑒
18 2
𝑥∈𝜙

Example 3.58
By adding the same constant to 20,50,100 a geometric progression results. The common ratio is: (AHSME
1959/12)

Suppose we add 𝑐 to each term:


20 + 𝑐, 50 + 𝑐, 100 + 𝑐
Since the above is a geometric progression, the middle term is the geometric mean of the other two terms:
(50 + 𝑐)2 = (20 + 𝑐)(100 + 𝑐)
2500 + 100𝑐 + 𝑐 2 = 2000 + 20𝑐 + 100𝑐 + 𝑐 2
500 = 20𝑐
𝑐 = 25
Progression is:

P a g e 111 | 274
Get all the files at: https://bit.ly/azizhandouts
Aziz Manva (azizmanva@gmail.com)

125 5
45,75,125 ⇒ 𝑟 = =
75 3

3.59: Symmetry
Consider a geometric sequence with middle term 𝑎 and common ratio 𝑟:
𝑎 𝑎
… , 2 , , 𝑎, 𝑎𝑟, 𝑎𝑟 2 , …
𝑟 𝑟
Skipping terms also gives a geometric sequence with the middle term 𝑎, and common ratio 𝑟 2 :
𝑎
, 𝑎, 𝑎𝑟 2
𝑟2

2, 𝑥, 8

Example 3.60
A. The fifth term of a geometric sequence of positive numbers is 11 and the eleventh term is 5. What is the
eighth term of the sequence? Express your answer in simplest radical form. (MathCounts 2010 State
Sprint)
B. The third term of a geometric sequence is 7. The thirteenth term of the geometric sequence is 8. Find
the product of the possible values of the eighth term of the geometric sequence.
C. The first, second and fifth term of an increasing geometric sequence are 𝑥, 𝑧 and 𝑦 respectively. Find the
fourth terms in terms 𝑥, 𝑦 and 𝑧 (all three variables must be present in your answer).

Part A in which case:


The terms under consideration are:
𝑎𝑟 4 , 𝑎𝑟 7 , 𝑎𝑟10 𝑥 = −√56
These form a geometric sequence with common Hence, the product of the possible values of the
ratio 𝑟 3 eighth term is:
𝑀𝑖𝑑𝑑𝑙𝑒 𝑇𝑒𝑟𝑚 = √𝑡5 𝑡11 = √5 × 11 = √55 (√56)(−√56) = −56
Part B Part C
The difference between the term numbers is 𝑥, 𝑧, 𝑡3 , 𝑡4 , 𝑦
13 − 3 = 10 → 𝐸𝑣𝑒𝑛 𝑡3 = √𝑡1 𝑡5 = √𝑥𝑦
The common ratio can be positive or negative.
𝑡3 = √𝑡2 𝑡4
We find the possible common ratio using the
geometric mean property: √𝑥𝑦 = √𝑧𝑡4
𝑎𝑟 2 , 𝑎𝑟 7 = 𝑥, 𝑎𝑟12 𝑥𝑦 = 𝑧𝑡4
𝑥𝑦
𝑥 = √56 𝑡4 =
𝑧
However, the common ratio could also be negative,

3.61: Geometric Mean Property-II


If the general term of a sequence is the geometric mean of the terms that precede and follow it, then the
sequence is a geometric sequence.
𝑎𝑛 = √𝑎𝑛−1 × 𝑎𝑛+1 ⇒ 𝑎𝑛−1 , 𝑎𝑛 , 𝑎𝑛+1 𝑎𝑟𝑒 𝑎 𝑔𝑒𝑜𝑚𝑒𝑡𝑟𝑖𝑐 𝑠𝑒𝑞𝑢𝑒𝑛𝑐𝑒

Suppose the equation given in the property above holds. Then:


𝑎𝑛 = √𝑎𝑛−1 × 𝑎𝑛+1
Square both sides:
(𝑎𝑛 )2 = 𝑎𝑛−1 × 𝑎𝑛+1
Let
𝑎𝑛−1 = 𝑎 ⇒ 𝑎𝑛 = 𝑟𝑎𝑛−1 = 𝑟𝑎, 𝑓𝑜𝑟 𝑠𝑜𝑚𝑒 𝑟

P a g e 112 | 274
Get all the files at: https://bit.ly/azizhandouts
Aziz Manva (azizmanva@gmail.com)

(𝑟𝑎)2 = 𝑎 × 𝑎𝑛+1
𝑎𝑟 2 = 𝑎𝑛+1
Hence, the terms are:

𝑎 , 𝑎𝑟 ⏟2 ⇒ 𝐺𝑒𝑜𝑚𝑒𝑡𝑟𝑖𝑐 𝑆𝑒𝑞𝑢𝑒𝑛𝑐𝑒
⏟ , 𝑎𝑟
𝑎𝑛−1 𝑎𝑛 𝑎𝑛+1

Example 3.62
Show that the geometric mean property holds in the following sequences. Hence, conclude that the sequences
are geometric sequences.
A. 2, 6, 18
48 16 16
B. , ,
60 30 45

In each case, we need to show that:


𝑎
⏟𝑛 =√
⏟𝑎𝑛−1 × 𝑎𝑛+1
𝑳𝑯𝑺 𝑹𝑯𝑺

Part A
𝐿𝐻𝑆 = 𝑎𝑛 = 6
𝑅𝐻𝑆 = √𝑎𝑛−1 × 𝑎𝑛+1 = √2 × 18 = √36 = 6 = 𝐿𝐻𝑆
Part B
16 8
𝐿𝐻𝑆 = 𝑎𝑛 = =
30 15
48 16 12 16 4 16 8 8 8 2 8
𝑅𝐻𝑆 = √ × =√ × =√ × =√ × = √( ) = = 𝐿𝐻𝑆
60 45 15 45 15 15 15 15 15 15

3.63: Inserting Geometric Means


Given values 𝑥 and 𝑦, the geometric mean property can be used to “insert” geometric means between them.

Example 3.64
A. If a geometric mean is inserted between 2 and 8, what is its value?
B. If two geometric means are inserted between 4 and 32, what is its value?
C. If five geometric means are inserted between 8 and 5832, the fifth term in the geometric series is:
(AHSME 1950/5)

Part A
𝑡1 = 2, 𝑡2 , 𝑡3 = 8
𝑡2 = √2 × 8 = √16 = 4
Part B
𝑎 = 4, 𝑎𝑟, 𝑎𝑟 2 , 𝑎𝑟 3 = 32
𝑎𝑟 3 32
= ⇒ 𝑟3 = 8 ⇒ 𝑟 = 2
𝑎 8
𝑎𝑟 = 8
𝑎𝑟 2 = 16
Part C
𝑎 = 8, 𝑎7 = 𝑎𝑟 6 = 5832
𝑎𝑟 6 5832
= ⇒ 𝑟 6 = 729 ⇒ 𝑟 = ±3
𝑎 8
𝑎5 = 𝑎𝑟 4 = 8 × 81 = 648

P a g e 113 | 274
Get all the files at: https://bit.ly/azizhandouts
Aziz Manva (azizmanva@gmail.com)

C. Recursive Definition

3.65: Geometric Sequence: Recursive Definition


A geometric sequence is defined as:
𝑎𝑛 = 𝑟𝑎𝑛−1 ,
⏟ 𝑎⏟1 = 𝑐
𝑅𝑒𝑐𝑢𝑟𝑠𝑖𝑣𝑒 𝐵𝑎𝑠𝑒 𝐶𝑎𝑠𝑒
𝐷𝑒𝑓𝑖𝑛𝑖𝑡𝑖𝑜𝑛

➢ A recursive definition is one that depends on the value that comes before it.
➢ That is, the 𝑛𝑡ℎ term of the sequence is 𝑟 times the (𝑛 − 1)𝑠𝑡 term of the sequence.
➢ And, we will also be given the starting value, which is the base case.

Example 3.66: Write Recursive Definitions


Write the definition of the following geometric sequences in recursive form.

Numbers 1 1 1
E. , , ,…
10 100 1000
A. 5,10,20,40, … 36 12 4
B. 2,8,32,128, … F. , , ,…
27 9 3
125 25 5
C. 1250,250,50, … G. , , ,…
81 27 9
Fractions
8 2 1
D. 5 , 25 , 250 , …

1 8
𝑎𝑛 = 𝑎 , 𝑎1 =
Numbers 20 𝑛−1 5
𝑎𝑛 = 2𝑎𝑛−1 , 𝑎1 = 5 1 1
𝑎𝑛 = 𝑎 , 𝑎1 =
𝑎𝑛 = 4𝑎𝑛−1 , 𝑎1 = 2 10 𝑛−1 10
1 4
𝑎𝑛 = 𝑎𝑛−1 , 𝑎1 = 1250 𝑎𝑛 = 𝑎𝑛−1 , 𝑎1 =
5 3
Fractions

Example 3.67
1
A sequence is given by 𝑎𝑛 = 5𝑎𝑛−1 , 𝑎1 = 1250
A. Find 𝑎2 , 𝑎3 , 𝑎4 , …
B. Find 𝑎8
C. Is the sequence geometric? If yes, find the common ratio and the 𝑛𝑡ℎ 𝑡𝑒𝑟𝑚. Use that formula to find 𝑎8 .

Part A
1 1 1
𝑎2 = 5𝑎1 = 5 × = =
2 × 625 2 × 125 250
1 1 1
𝑎3 = 5𝑎2 = 5 × = =
2 × 125 2 × 25 50
1 1 1
𝑎4 = 5𝑎3 = 5 × = =
2 × 25 2 × 5 10

Part B
𝑎𝑛 = 5𝑎𝑛−1 = 25𝑎𝑛−2 = 125𝑎𝑛−3 = ⋯
𝑎𝑛 = 5𝑎𝑛−1 = 52 𝑎𝑛−2 = 53 𝑎𝑛−3 = ⋯

P a g e 114 | 274
Get all the files at: https://bit.ly/azizhandouts
Aziz Manva (azizmanva@gmail.com)

1 53 125
𝑎8 = 57 𝑎8−7 = 57 𝑎1 = 57 × = =
2 × 54 2 2
Part C
Yes, it is geometric, because each successive term is 5 times greater than the previous term.

𝐶𝑜𝑚𝑚𝑜𝑛 𝑅𝑎𝑡𝑖𝑜 = 𝑟 = 5
1 5𝑛−1 5𝑛−5
𝑛𝑡ℎ 𝑡𝑒𝑟𝑚 = 𝑎𝑟 𝑛−1 = × 5𝑛−1 = =
1250 2 × 54 2
8−5 3
5 5 125
𝑎8 = = =
2 2 2

3.68: Converting Geometric Sequences from Recursive to Explicit


We can convert from the recursive definition to the explicit definition:
𝑎𝑛 = 𝑟𝑎𝑛−1 ,
⏟ 𝑎1 = 𝑎 ⇒ ⏟ 𝐶𝑜𝑚𝑚𝑜𝑛 𝑟𝑎𝑡𝑖𝑜 = 𝑟, 𝑛𝑡ℎ 𝑡𝑒𝑟𝑚 = 𝑎𝑟 𝑛−1
𝑹𝒆𝒄𝒖𝒓𝒔𝒊𝒗𝒆 𝑫𝒆𝒇𝒊𝒏𝒊𝒕𝒊𝒐𝒏 𝑬𝒙𝒑𝒍𝒊𝒄𝒊𝒕 𝑫𝒆𝒇𝒊𝒏𝒊𝒕𝒊𝒐𝒏

Example 3.69
Define a sequence of real numbers 𝑎1 , 𝑎2 , 𝑎3 , . .. by 𝑎1 = 1 and 𝑎𝑛3 + 1 = 99𝑎𝑛3 for all 𝑛 ≥ 1. Then 𝑎100 equals
(AHSME 1999/13)

We have been given that:


𝑎𝑛3 + 1 = 99 × 𝑎𝑛3
This relation is recursive, but it has the cubes of what we want to relate. Hence, take the cube root both sides:
1 1
𝑎𝑛+1 = (99)3 𝑎𝑛 ⇒ 𝐶𝑜𝑚𝑚𝑜𝑛 𝑅𝑎𝑡𝑖𝑜 = 𝑟 = (99)3

1 99
𝑎100 = 𝑟 99 × 𝑎1 = [(99)3 ] × 1 = 9933

Example 3.70
Consider the geometric sequence given by
3 1
98𝑠 2 14𝑠 2
𝑎1 = 4, 𝑎2 = 2 ,…
121𝑟 3 11𝑟 3
A. Find the common ratio
B. Find the 𝑛𝑡ℎ term
C. Write the definition of the geometric sequence in recursive form.

Part A
1 3 1 4 4 2 2
14𝑠 2 98𝑠 2 14𝑠 2 121𝑟 3 11𝑟 3−3 11𝑟 3
𝑟= 2 ÷ 4 = 2 × 3 = 3 1 =
7𝑠1
11𝑟 3 121𝑟 3 11𝑟 3 98𝑠 2 7𝑠 2−2
Part B
3 2 𝑛−1 2 2 4 2
98𝑠 2 11𝑟 3 2 × 11𝑛−3 × 𝑟 3𝑛−3−3 2 × 11𝑛−3 × 𝑟 3𝑛−2
𝑛𝑡ℎ 𝑡𝑒𝑟𝑚 = 𝑎𝑟 𝑛−1 = 4 × ( 7𝑠1 ) = 3 = 5
121𝑟 3 7𝑛−3 𝑠 𝑛−1−2 7𝑛−3 𝑠 𝑛−2
Part C

P a g e 115 | 274
Get all the files at: https://bit.ly/azizhandouts
Aziz Manva (azizmanva@gmail.com)

2 3
11𝑟 3 98𝑠 2
𝑎𝑛 = 𝑟𝑎𝑛−1 = ( 1 ) 𝑎𝑛−1 , 𝑎1 = 4
7𝑠
121𝑟 3

3.4 More Topics


A. Deciding whether a sequence is arithmetic or geometric
Given two terms
➢ You can determine both a common difference and a common ratio.
➢ Sequence can be either arithmetic or geometric
Given three terms
➢ You can determine whether it is arithmetic, geometric or neither.

Example 3.71: Finding the common difference / common ratio


A sequence which is known to be either arithmetic or geometric has 𝑛th term 40, and (𝑛 + 2)nd term 10. Find
the possible values that the common difference / common ratio can take.

𝑎 = 40, 𝑎 + 2𝑑 = 10 ⇒ 2𝑑 = −30 ⇒ 𝑑 = −15


1 1
𝑎 = 40, 𝑎𝑟 2 = 10 ⇒ 𝑟 2 = ⇒ ±
4 2

B. Arithmetic Sequences
Mixing arithmetic with geometric sequences often leads to quadratic equations.

Example 3.72
Three numbers 𝑎, 𝑏, 𝑐, none zero, form an arithmetic progression. Increasing 𝑎 by 1 or increasing 𝑐 by 2 results
in a geometric progression. Then 𝑏 equals: (AHSME 1963/16)

Let the common difference be 𝑑. Then the three numbers can be written:
𝑏 − 𝑑, 𝑏, 𝑏 + 𝑑
Increasing 𝑎 by 1 gives a geometric sequence:
𝑏 2 = (𝑏 + 𝑑)(𝑏 − 𝑑 + 1)
𝑏 2 = 𝑏 2 + 𝑏𝑑 − 𝑏𝑑 − 𝑑2 + 𝑏 + 𝑑
𝑑2 = 𝑏 + 𝑑

𝑬𝒒𝒖𝒂𝒕𝒊𝒐𝒏 𝑰
Increasing 𝑐 by 2 gives a geometric sequence:
𝑏 2 = (𝑏 − 𝑑)(𝑏 + 𝑑 + 2)
𝑏 2 = 𝑏 2 − 𝑑𝑏 + 𝑑𝑏 − 𝑑2 + 2𝑏 − 2𝑑
𝑑2 = 2𝑏 − 2𝑑

𝑬𝒒𝒖𝒂𝒕𝒊𝒐𝒏 𝑰𝑰
Equate the RHS of Equations I and II:
𝑏 + 𝑑 = 2𝑏 − 2𝑑
𝑏
𝑑=
3
𝑏
Substitute 𝑑 = in Equation I to get a quadratic in 𝑏:
3
𝑏 2 𝑏
( ) =𝑏+
3 3
𝑏 2 4𝑏
=
9 3

P a g e 116 | 274
Get all the files at: https://bit.ly/azizhandouts
Aziz Manva (azizmanva@gmail.com)

𝑏 = 12

Example 3.73
Suppose 𝑥, 𝑦, 𝑧 is a geometric sequence with common ratio 𝑟 and 𝑥 ≠ 𝑦. If 𝑥, 2𝑦, 3𝑧 is an arithmetic sequence,
then 𝑟 is (AHSME 1994/20)

Example 3.74
The positive integers 𝐴, 𝐵 and 𝐶 form an arithmetic sequence while the integers 𝐵, 𝐶 and 𝐷 form a geometric
𝐶 5
sequence. If 𝐵
= 3, what is the smallest possible value of 𝐴 + 𝐵 + 𝐶 + 𝐷? (MathCounts 2010 State Sprint)

Ans = 52

Example 3.75
Sequence A is a geometric sequence. Sequence B is an arithmetic sequence. Each sequence stops as soon as one
of its terms is greater than 300. What is the least positive difference between a number selected from sequence
A and a number selected from sequence B?

Sequence A:2,4,8,16,32,…
Sequence B:20,40,60,80,100,… (MathCounts 2004 Warm-Up 16)

2,4,8,16,32,64,128,256
20,40,60, … ,300

By looking at each difference:


𝑙𝑒𝑎𝑠𝑡 𝑝𝑜𝑠𝑖𝑡𝑖𝑣𝑒 𝑑𝑖𝑓𝑓𝑒𝑟𝑒𝑛𝑐𝑒 𝑖𝑠 4

Example 3.76
Find the positive difference in valuation under two methods if a three-year old car with original value $10,000
has reduction in value of $1000/year in Method A, and 10%/year on a reducing balance basis in Method B.

10000(0.9)3 − 10,000 × $1000 = 7290 − 7000 = 290

Example 3.77
A sequence of three real numbers forms an arithmetic progression with a first term of 9. If 2 is added to the
second term and 20 is added to the third term, the three resulting numbers form a geometric progression. What
is the smallest possible value for the third term in the geometric progression? (AMC 10A 2004/18, AMC 12A
2004/14)

9,
⏟ 9 + 𝑑, 9 + 2𝑑 ⇒ ⏟
9, 11 + 𝑑, 29 + 2𝑑
𝑶𝒓𝒊𝒈𝒊𝒏𝒂𝒍 𝑨𝑷 𝑨𝒇𝒕𝒆𝒓 𝑨𝒅𝒅𝒊𝒕𝒊𝒐𝒏

By the geometric mean property:


(11 + 𝑑)2 = 9(29 + 2𝑑)
Expanding:

P a g e 117 | 274
Get all the files at: https://bit.ly/azizhandouts
Aziz Manva (azizmanva@gmail.com)

𝑑2 + 22𝑑 + 121 = 261 + 18𝑑


Collect all terms on one side to get a quadratic, and then solve it:
𝑑2 + 4𝑑 − 140 = 0 ⇒ (𝑑 + 14)(𝑑 − 10) = 0 ⇒ 𝑑 ∈ {−14,10}

The third term will be smallest when 𝑑 is smallest:


29 + 2(−14) = 29 − 28 = 1

Example 3.78
Three numbers a, b, c add up to 15. a, b, c are successive terms in a geometric sequence. b, a, c are successive
terms in an arithmetic sequence. Find a, b, c

𝑏 𝑎 𝑐
𝑥−𝑑 𝑥 𝑥+𝑑

𝑎 + 𝑏 + 𝑐 = 15
𝑥 − 𝑑 + 𝑥 + 𝑥 + 𝑑 = 15
3𝑥 = 15
𝑥=5
𝑎=5
A B C
𝑥 𝑟 𝑥𝑟
𝑟

𝑎 + 𝑏 + 𝑐 = 15
𝑥
+ 𝑟 + 𝑥𝑟 = 15
𝑟
5
+ 𝑟 + 5𝑟 = 15
𝑟

Example 3.79
Given a geometric progression of five terms, each a positive integer less than 100. The sum of the five terms is
211. If 𝑆 is the sum of those terms in the progression which are squares of integers, then 𝑆 is: (AHSME
1967/36)

Example 3.80
There are two positive numbers that may be inserted between 3 and 9 such that the first three are in geometric
progression while the last three are in arithmetic progression. The sum of those two positive numbers is
(AHSME 1972/16)
Ans = 11 1/4

Example 3.81
If 𝑎1 , 𝑎2 , 𝑎3 , . .. is a sequence of positive numbers such that 𝑎𝑛+2 = 𝑎𝑛 𝑎𝑛+1 for all positive integers 𝑛, then the
sequence 𝑎1 , 𝑎2 , 𝑎3 , . .. is a geometric progression
A. for all positive values of a_1 and a_2
B. if and only if a_1=a_2
C. if and only if a_1=1
D. if and only if a_2=1
E. if and only if a_1=a_2=1 (AHSME 1977/13)

P a g e 118 | 274
Get all the files at: https://bit.ly/azizhandouts
Aziz Manva (azizmanva@gmail.com)

Example 3.82
If the distinct non-zero numbers 𝑥(𝑦 − 𝑧), 𝑦(𝑧 − 𝑥), 𝑧(𝑥 − 𝑦) form a geometric progression with common ratio
r, then r satisfies the equation
A. 𝑟 2 + 𝑟 + 1 = 0
B. 𝑟 2 − 𝑟 + 1 = 0
C. 𝑟 4 + 𝑟 2 − 1 = 0
D. (𝑟 + 1)4 + 𝑟 = 0
E. (𝑟 − 1)4 + 𝑟 = 0 (AHSME 1978/24)

Example 3.83
If 𝑎, 𝑏, 𝑐, 𝑑 are positive real numbers such that 𝑎, 𝑏, 𝑐, 𝑑 form an increasing arithmetic sequence and 𝑎, 𝑏, 𝑑 form
𝑎
a geometric sequence, then find the value of 𝑑 (AMC 12B 2002/9)

Example 3.84
In the sequence 0, 1, 1, 3, 6, 9, 27, ..., the first term is 0. Subsequent terms are produced by alternately adding
and multiplying by each successive integer beginning with 1. For instance, the second term is produced by
adding 1 to the first term; the third term is produced by multiplying the second term by 1; the fourth term is
produced by adding 2 to the third term; and so on. What is the value of the first term that is greater than 125?
(MathCounts 2006 Chapter Sprint)
Ans = 129

Example 3.85
If 𝑥, 𝑦 and 𝑧 are 𝑝𝑡ℎ , 𝑞 𝑡ℎ and 𝑟 𝑡ℎ terms of an arithmetic progression, and also of a geometric progression, then
𝑥 𝑦−𝑧 𝑦 𝑧−𝑥 𝑧 𝑥−𝑦 is equal to: (IIT JEE, 1981)

Express 𝑥, 𝑦 and 𝑧 as terms of an arithmetic progression:


𝑥 = 𝑎 + (𝑝 − 1)𝑑, 𝑦 = 𝑎 + (𝑞 − 1)𝑑, 𝑧 = 𝑎 + (𝑟 − 1)𝑑
Then:
⏟− 𝑧 = (𝑞 − 𝑟)𝑑,
𝑦 𝑧 − 𝑥 = (𝑟 − 𝑝)𝑑, 𝑥 − 𝑦 = (𝑝 − 𝑞)𝑑
𝑺𝒚𝒔𝒕𝒆𝒎 𝑰

Express 𝑥, 𝑦 and 𝑧 as terms of a geometric progression:


𝑥 = 𝑎𝑟 𝑝−1 ,
⏟ 𝑦 = 𝑎𝑟 𝑞−1 , 𝑧 = 𝑎𝑟 𝑟−1
𝑺𝒚𝒔𝒕𝒆𝒎 𝑰𝑰

Substitute the value from Systems I and II into 𝑥 𝑦−𝑧 𝑦 𝑧−𝑥 𝑧 𝑥−𝑦 :
= (𝑎𝑟 𝑝−1 )(𝑞−𝑟)𝑑 (𝑎𝑟 𝑞−1 )(𝑟−𝑝)𝑑 (𝑎𝑟 𝑟−1 )(𝑝−𝑞)𝑑

Open the brackets to get:


⏟(𝑞−𝑟)𝑑+(𝑟−𝑝)𝑑+(𝑝−𝑞)𝑑 𝑟 (𝑝−1)(𝑞−𝑟)𝑑+(𝑞−1)(𝑟−𝑝)𝑑+(𝑟−1)(𝑝−𝑞)𝑑
𝑎
𝑪𝒚𝒄𝒍𝒊𝒄𝒂𝒍
𝑹𝒆𝒅𝒖𝒄𝒆𝒔 𝒕𝒐 𝒛𝒆𝒓𝒐

Which reduces to:

𝑑 𝑝(𝑞−𝑟)+𝑞(𝑟−𝑝)+𝑟(𝑝−𝑞)−[(𝑞−𝑟)+(𝑟−𝑝)+(𝑝−𝑞)]

𝑪𝒚𝒄𝒍𝒊𝒄𝒂𝒍
0 [ ]
=𝑎 𝑟 𝑹𝒆𝒅𝒖𝒄𝒆𝒔 𝒕𝒐 𝒛𝒆𝒓𝒐

P a g e 119 | 274
Get all the files at: https://bit.ly/azizhandouts
Aziz Manva (azizmanva@gmail.com)

Which then reduces to:


= 𝑎0 𝑟 𝑑[𝑝𝑞−𝑝𝑟+𝑞𝑟−𝑞𝑝+𝑟𝑝−𝑟𝑞] = 𝑎0 𝑟 0 = 1

Example 3.86
Does there exist a geometric progression with 27, 8 and 12 as three of its terms. If so, how many such
progressions exist? (IIT JEE 1982)

3 3 2
8×= 12, 12 × ( ) = 27 ⇒ 8,12,18,27 ∈ 𝐺𝑃 3
2 2 𝑎=8,𝑟=
2
We can vary the common ratio = 𝑟 to get an infinite number of 𝐺𝑃’s:
𝑛
3 √3 √3
𝑟={ , ,…, 𝑛 ,𝑛 ∈ 𝑁 }
2 √2 √2

We can also vary 𝑓𝑖𝑟𝑠𝑡 𝑡𝑒𝑟𝑚 = 𝑎 to get an infinite number of 𝐺𝑃’s:


2 2 2 2 𝑛
𝑎 = {8 × , 8 × ( ) , … ,8 × ( ) , 𝑛∈𝑁}
3 3 3

3.5 Infinite Geometric Series


A. Infinite Series
Zeno was a Greek philosopher whose paradoxes, around 2500 years old, have been of great theoretical
importance for Maths. They can be resolved using the tools of modern Maths. We will approach these paradoxes
using the concept of geometric series. These same ideas will also come up again when we study the concept of a
limiting value (which gets explored in depth in Calculus).

Example 3.87: Zeno’s Paradox


Zeno is 1 foot away from his doorsill, and wants to reach there. He determines that Step I is to cover half the
1
distance, which is 2 feet. Step II is to go closer to the doorsill, covering half of the remaining distance in Step I,
1
that is 4 feet. How much distance does Zeno cover till:
A. Step III
B. Step IV
C. Step V
D. Step 𝑛
E. An infinite number of steps

1 1 1 7
+ + =
2 4 8 8
1 1 1 1 15
+ + + =
2 4 8 16 16
1 1 1 1 1 31
+ + + + =
2 4 8 16 32 32
1 1 1 1 2𝑛 − 1
+ 2 + 3 +⋯+ 𝑛 =
2 2 2 2 2𝑛
Suppose 𝑛 = 10
1023
𝐷𝑖𝑠𝑡𝑎𝑛𝑐𝑒 =
1024
When 𝑛 becomes a very large number (becomes infinity)

P a g e 120 | 274
Get all the files at: https://bit.ly/azizhandouts
Aziz Manva (azizmanva@gmail.com)

𝐷𝑖𝑠𝑡𝑎𝑛𝑐𝑒 = 1

Example 3.88
Zeno's cousin is like Zeno, but more conservative. Step I covers one-third of a foot, and each step covers only
one-third of the remaining distance. Answer the same questions as before.

1/3 + (1/3)(2/3)+ (1/3)(2/3)^2+...

3.89: A Warning
A word of warning: infinity is not a number. It is a concept. The rules of arithmetic and algebra do not
automatically apply. Not understanding how infinity works can lead to fundamental errors in arriving at
answers.
➢ When working with infinity you may use exactly the same techniques as used here.
➢ For a more general discussion, see the section on infinity in the Note on Logic, Proof and Games.

3.90: Convergence and Divergence


➢ A series which has a finite sum is called convergent.
➢ A series which does not have a finite sum is called divergent.

Example 3.91
Do the following series converge:
A. 1 + 2 + 3 + ⋯
B. 1 − 1 + 1 − 1 + 1 + ⋯

Part A
The value of the sum grows with restriction, hence, it goes to infinity.
𝐷𝑖𝑣𝑒𝑟𝑔𝑒𝑛𝑡

Part B
1=1
1−1=0
1−1+1=1

The series alternates between 1 and 0. You cannot assign a single value to the series.
𝐷𝑖𝑣𝑒𝑟𝑔𝑒𝑛𝑡.

3.92: Infinite Geometric Series


A infinite geometric series is a series where the successive terms are terms of a geometric sequence. That is, it is
a series of the form:
𝑎 + 𝑎𝑟 + 𝑎𝑟 2 + ⋯

Example 3.93
Visualization as a square of an infinite geometric series:
1 1 1 1
+ + + +⋯
2 4 8 16

P a g e 121 | 274
Get all the files at: https://bit.ly/azizhandouts
Aziz Manva (azizmanva@gmail.com)

When −1 < 𝑟 < 1, 𝑟 ≠ 0,


𝑟 → 0, as 𝑛 → ∞ and hence, 𝑆∞ is:
1 − 𝑟𝑛 1−0 𝑎
𝑆∞ = 𝑎 ( ) = 𝑎( )=
1−𝑟 1−𝑟 1−𝑟

B. Sum of an Infinite Geometric Series

3.94: Sum of an Infinite Geometric Series


The sum 𝑆 of an infinite geometric series with first term 𝑎, and common ratio 𝑟 is given by:
𝑎
𝑆=
1−𝑟

Assign a value to the sum of an infinite geometric series:


𝑆 = 𝑎 + 𝑎𝑟 + 𝑎𝑟 2 + ⋯

𝑬𝒒𝒖𝒂𝒕𝒊𝒐𝒏 𝑰
Multiply both sides by 𝑟:
𝑟𝑆 = 𝑟𝑎 + 𝑎𝑟 2 + ⋯

𝑬𝒒𝒖𝒂𝒕𝒊𝒐𝒏 𝑰𝑰
Subtract Equation II from Equation I, factor 𝑆 on the LHS, and then solve for S:
𝑎
𝑆 − 𝑟𝑆 = 𝑎 ⇒ 𝑆(1 − 𝑟) = 𝑎 ⇒ 𝑆 =
1−𝑟

Example 3.95
Find the sum of the following infinite geometric series:

Unit Fractions
1 1 1 1
A. 2
+ 4 + 8 + 16 + ⋯
1 1 1
B. + + + ⋯
3 9 27
Fractions
27 9 3
C. 64 + 32 + 16 + ⋯
Radicals
1 1 1
D. 2
+2 2
+4 +⋯

3 3 3
E. 5
+ +7
√35

Part A
1 1
Substitute 𝑎 = 2 , 𝑟 = 2:
1 1
𝑎 2
𝑆= = = 2 =1
1−𝑟 1−1 1
2 2
Part B
1 1
Substitute 𝑎 = 3 , 𝑟 = 3:
1 1
𝑎 3 1
𝑆= = =3=
1−𝑟 1−1 2 2
3 3
Part C

P a g e 122 | 274
Get all the files at: https://bit.ly/azizhandouts
Aziz Manva (azizmanva@gmail.com)
27 2
Substitute 𝑎 = 64 , 𝑟 = 3:
27 27
𝑎 64 27 81
𝑆= = = 64 = ×3=
1−𝑟 1−2 1 64 64
3 3
Part D
1 1 𝑎
Substitute 𝑎 = 2 , 𝑟 = in 𝑆 = 1−𝑟
√2
1 1
2 = 2
1 √2 − 1
1−
√2 √2
Multiply by the reciprocal of the denominator:
1 √2
×
2 √2 − 1
Rationalize the denominator:
1 √2 √2 + 1
× ×
2 √2 − 1 √2 + 1
1 2 + √2
×
2 2−1
2 + √2
2
Part E
3 3 3 5 5
÷ = × =
√35 5 √35 3 √35
3 5 𝑎
Substitute 𝑎 = 5
,𝑟 = in 𝑆 = :
√35 1−𝑟
3 3
= 5 5
5 √35 − 5
1−
√35 √35
Multiply by the reciprocal of the denominator:
3 √35
×
5 √35 − 5
Rationalize the denominator:
3 √35 √35 + 5
× ×
5 √35 − 5 √35 + 5
3 35 + 5√35
×
5 35 − 25
21 + 3√35
10

3.96: Convergence Conditions


➢ If the common ratio of a geometric series is between −1 and 1 (−1 < 𝑟 < 1), the series converges (has a
finite value).

If the condition is not met, then it does not have a finite value, and the series diverges.
If 𝑟 = 0, the series is not geometric.

P a g e 123 | 274
Get all the files at: https://bit.ly/azizhandouts
Aziz Manva (azizmanva@gmail.com)

Example 3.97
What can be said about the convergence or divergence of the geometric series below?
A. 2 + 4 + 8 + 16 + ⋯
1 1
B. 27 + 9 + 3 + 1 + 3 + 9 + ⋯
C. A geometric series which has second term greater than the first term.
D. A geometric series which has second term less than the first term.

Part A 1,2,4, … ⇒ 𝑟 = 2 ⇒ 𝐷𝑖𝑣𝑒𝑟𝑔𝑒


4 𝐶𝑎𝑛𝑛𝑜𝑡 𝑐𝑜𝑛𝑐𝑙𝑢𝑑𝑒 𝑎𝑛𝑦𝑡ℎ𝑖𝑛𝑔
𝑟 = = 2 > 1 ⇒ 𝐷𝑖𝑣𝑒𝑟𝑔𝑒𝑠
2 Part D
Part B −1, −2, −4 ⇒ 𝑟 = 2 ⇒ 𝐷𝑖𝑣𝑒𝑟𝑔𝑒
9 1 1 1 1
𝑟= = < 1 ⇒ 𝐶𝑜𝑛𝑣𝑒𝑟𝑔𝑒𝑠 1, , , … ⇒ 𝑟 = ⇒ 𝐶𝑜𝑛𝑣𝑒𝑟𝑔𝑒
27 3 2 4 2
Part C 𝐶𝑎𝑛𝑛𝑜𝑡 𝑐𝑜𝑛𝑐𝑙𝑢𝑑𝑒 𝑎𝑛𝑦𝑡ℎ𝑖𝑛𝑔
1
−2,1, −0.5,0.25 ⇒ 𝑟 = − ⇒ 𝐶𝑜𝑛𝑣𝑒𝑟𝑔𝑒
2

Example 3.98
If an infinite geometric progression has first term 𝑥 and sum 5, then find the range of values that 𝑥 can take. (IIT
JEE, 2001 Screening)

𝑎
Substitute 𝑎 = 𝑥, 𝑆 = 5 in the formula for the sum of an infinite geometric series = 𝑆:
1−𝑟
𝑥 𝑥 𝑥
=5⇒1−𝑟 = ⇒𝑟 =1−
1−𝑟 5 5
An infinite geometric series only converges when the common ratio is between −1 and 1:
−1 < 𝑟 < 1
𝑥
−1 < 1 − < 1
⏟ 5
𝑪𝒐𝒎𝒎𝒐𝒏
𝑹𝒂𝒕𝒊𝒐
Subtract 1:
𝑥
−2 < − < 0
5
Multiply by −5. Reverse the sign of the inequality since we are multiplying by a negative number.
10 > 𝑥 > 0
𝑥 ∈ (0,10)

3.99: Double Series


Suppose we have two geometric series given by:
𝑎 + 𝑎𝑟 + 𝑎𝑟 2 + ⋯.
𝑏 + 𝑏𝑞 + 𝑏𝑞 2 + ⋯
A double series consists of alternating terms from the above two series:
𝑎 + 𝑏 + 𝑎𝑟 + 𝑏𝑞 + 𝑎𝑟 2 + 𝑏𝑞 2 + ⋯

Example 3.100: Double Series


1 2 1 2
A. The sum to infinity of 7
+ 72 + 73 + 74 + ⋯ is: (AHSME 1950/43)
1 1 1 1 1 1 𝑏
B. The sum to infinity of 2 + 3 + 6 + 6 + 18 + 12 can be written as a mixed number in the form 𝑎 𝑐 , where 𝑎,
𝑏, 𝑐 are positive integers, 𝑏 < 𝑐 and 𝐻𝐶𝐹(𝑏, 𝑐) = 1. Find 𝑎 + 𝑏 + 𝑐.

P a g e 124 | 274
Get all the files at: https://bit.ly/azizhandouts
Aziz Manva (azizmanva@gmail.com)

Part A Part B
Think of this as two geometric series put together. The first, third, fifth….. terms comprise a geometric
The first, third, fifth….. terms comprise a geometric series:
series:
1 1 1 1 1 1 1 1 1 1
+ 3 + 5 + ⋯ ⇒ 𝑎 = ,𝑟 = 2 , , ⇒ 𝑎 = ,𝑟 =
7 7 7 7 7 2 6 18 2 3
1 1
𝑎 7 1 48 1 49 7 𝑎 1 3 3
𝑆= = = ÷ = × = 𝑆= = 2 = × =
1−𝑟 1− 1 7 49 7 48 48 1−𝑟 1−1 2 2 4
49 3
1 1 1 1 1
, , ⇒ 𝑎 = ,𝑟 =
The second, fourth, sixth…. Terms comprise another 3 6 12 3 2
geometric series: 1
𝑎 1 2
2 2 2 2 1 𝑆= = 3 = ×2=
2
+ 4 + 6… ⇒ 𝑎 = 2,𝑟 = 2 1−𝑟 1−1 3 3
7 7 7 7 7 2
2
𝑎 2 48 2 49 2
𝑆= = 49 = ÷ = × = Hence, the final answer
1−𝑟 1− 1 49 49 49 48 48
49 3 2 9 + 8 17 5
= + = = =1
We need the total of both: 4 3 12 12 12
7 2 9 3 𝑎 + 𝑏 + 𝑐 = 1 + 5 + 12 = 18
+ = =
48 48 48 16

Example 3.101
Find the sum of the following series:
A. 9(1+9)
B. 5(1+5(1+5))
C. 3(1+3(1+3(1+3)))
D. 𝑥 (1 + 𝑥(1 + 𝑥(1 + ⋯ )))

Parts A-C
9(1 + 9) = 9 + 92 = 9 + 81 = 90
5(1 + 5(1 + 5)) = 5 + 52 (1 + 5) = 5 + 52 + 53 = 5 + 25 + 125 = 155
3 (1 + 3(1 + 3(1 + 3))) = 3 + 32 (1 + 3(1 + 3)) = 3 + 32 + 33 (1 + 3) = 3 + 32 + 33 + 34
Part D
𝑥 (1 + 𝑥(1 + 𝑥(1 + ⋯ ))) = 𝑥 + 𝑥 2 (1 + 𝑥(1 + ⋯ )) = 𝑥 + 𝑥 2 + 𝑥 3 (1 + ⋯ )
= 𝑥 + 𝑥2 + 𝑥3 + ⋯
And the above is a geometric series with 𝑎 = 𝑥, 𝑟 = 𝑥, which has sum
𝑎 𝑥
𝑆= =
1−𝑟 1−𝑥

3.102: Limiting Value


➢ The limiting value of a series is the value as the number of terms reaches infinity.
➢ It is related to the concept of a limit in Calculus.

Example 3.103
1 1
Given the series 2 + 1 + 2 + 4 + ⋯ and the following five statements:
(1) the sum increases without limit

P a g e 125 | 274
Get all the files at: https://bit.ly/azizhandouts
Aziz Manva (azizmanva@gmail.com)

(2) the sum decreases without limit


(3) the difference between any term of the sequence and zero can be made less than any positive quantity no
matter how small
(4) the difference between the sum and 4 can be made less than any positive quantity no matter how small
(5) the sum approaches a limit
Of these statements, the correct ones are: (AHSME 1950/39)

1
Substitute 𝑎 = 2, 𝑟 = 2:
𝑎 2 2
𝑆= = = =4
1−𝑟 1− 1 1
2 2
𝑆1 = 2, 𝐷𝑖𝑓𝑓𝑒𝑟𝑒𝑛𝑐𝑒 = 4 − 2 = 2
𝑆2 = 3 ⇒ 𝐷𝑖𝑓𝑓 = 4 − 3 = 1
1 1 1
𝑆3 = 3 ⇒ 𝐷𝑖𝑓𝑓 = 4 − 3 =
2 2 2
3 3 1
𝑆4 = 3 ⇒ 𝐷𝑖𝑓𝑓 = 4 − 3 =
4 4 4
7 7 1
𝑆4 = 3 ⇒ 𝐷𝑖𝑓𝑓 = 4 − 3 =
8 8 8

Hence, the final answer is:


4 𝑎𝑛𝑑 5 𝑎𝑟𝑒 𝑐𝑜𝑟𝑟𝑒𝑐𝑡 𝑠𝑡𝑎𝑡𝑒𝑚𝑒𝑛𝑡𝑠

Example 3.104
8 16
Let 𝑠 be the limiting sum of the geometric series 4 − 3 + 9
− as the number of terms increases without bound.
Then 𝑠 equals: (AHSME 1962/14)

2
We need to find the sum on an infinite geometric series with 𝑎 = 4, 𝑟 = − :
3
𝑎 𝑎 4 4 3 12
𝑆= = = = =4× =
1 − 𝑟 1 − (− 2) 1 + 2 5 5 5
3 3 3

Example 3.105: Back Calculations


A. The sum to infinity of the terms of an infinite geometric progression is 6. The sum of the first two terms
1
is 4 2. The first term of the progression is: (AHSME 1952/12)
B. In a geometric progression whose terms are positive, any term is equal to the sum of the next two
following terms. Then the common ratio is: (AHSME 1953/25)

Part A Substitute the value of r from Equation I into


Let the geometric series be: Equation II:
𝑎 + 𝑎𝑟 + 𝑎𝑟 2 + ⋯ 6−𝑎 1
𝑎+𝑎( )=4
The sum is: 6 2
𝑎 6−𝑎 6𝑎 + 6𝑎 − 𝑎2 9
𝑆= = 6 ⇒ 𝑎 = 6 − 6𝑟 ⇒ 𝑟 = =
1−𝑟 ⏟ 6 6 2
2
𝑬𝒒𝒖𝒂𝒕𝒊𝒐𝒏 𝑰 12𝑎 − 𝑎 = 27
1
Also, the first two terms add up to 4 2: 𝑎2 − 12𝑎 + 27 = 0
1 (𝑎 − 3)(𝑎 − 9) = 0
𝑎 + 𝑎𝑟 = 4 𝑎 ∈ {3,9}
⏟ 2
𝑬𝒒𝒖𝒂𝒕𝒊𝒐𝒏 𝑰𝑰 Part B

P a g e 126 | 274
Get all the files at: https://bit.ly/azizhandouts
Aziz Manva (azizmanva@gmail.com)

Let the geometric series be: −1:


𝑎 + 𝑎𝑟 + 𝑎𝑟 2 + ⋯ −𝑏 ± √𝑏 2 − 4𝑎𝑐 −1 ± √1 − (4)(1)(−1)
The sum of the first term must equal the sum of the 𝑟= =
2𝑎 2𝑎
second and the third terms: −1 ± √5
𝑎 = 𝑎𝑟 + 𝑎𝑟 2 =
2
Divide both sides by 𝑎: −1 − √5
1 = 𝑟 + 𝑟2 𝑖𝑠 − 𝑣𝑒 ⇒ 𝑅𝑒𝑗𝑒𝑐𝑡
2
2
𝑟 +𝑟−1 = 0 −1 + √5 √5 − 1
Use the quadratic formula with 𝑎 = 1, 𝑏 = 1, 𝑐 = 𝑟= =
2 2

C. Recurring Decimals

3.106: Recurring Decimals


➢ A recurring decimal is a decimal where the digits of the decimal repeat infinitely many times.

Example 3.107
Some numbers are given below. For each number, identify the digits which repeat, and the number of digits
which repeat when the number is written as a recurring decimal.
A. 2. 3̅
B. 1.267̅̅̅̅
1
C.
7

Part A Part C
𝐷𝑖𝑔𝑖𝑡𝑠 𝑤ℎ𝑖𝑐ℎ 𝑟𝑒𝑝𝑒𝑎𝑡 = 3 1
= 0. ̅̅̅̅̅̅̅̅̅̅
142857
𝑁𝑜. 𝑜𝑓 𝑟𝑒𝑝𝑒𝑎𝑡𝑖𝑛𝑔 𝑑𝑖𝑔𝑖𝑡𝑠 = 1 7
Part B 𝐷𝑖𝑔𝑖𝑡𝑠 𝑤ℎ𝑖𝑐ℎ 𝑟𝑒𝑝𝑒𝑎𝑡 = 142857
𝐷𝑖𝑔𝑖𝑡𝑠 𝑤ℎ𝑖𝑐ℎ 𝑟𝑒𝑝𝑒𝑎𝑡 = 67 𝑁𝑜. 𝑜𝑓 𝑟𝑒𝑝𝑒𝑎𝑡𝑖𝑛𝑔 𝑑𝑖𝑔𝑖𝑡𝑠 = 6
𝑁𝑜. 𝑜𝑓 𝑟𝑒𝑝𝑒𝑎𝑡𝑖𝑛𝑔 𝑑𝑖𝑔𝑖𝑡𝑠 = 2

Example 3.108
A. Find the value of 0. 3̅ below using geometric series, and using equations.
B. Convert 0. 2̅ into a fraction written in lowest form.
C. For some integers 𝑎, 𝑏 such that 𝐻𝐶𝐹(𝑎, 𝑏) = 1, 0. 21 ̅̅̅̅ = 𝑎. Find 𝑎 + 𝑏.
𝑏
D. Convert 0. 𝑎̅ into a decimal fraction, where 𝑎 is a digit from 1 to 9.
E. Convert 0. 𝑥𝑦
̅̅̅ into a decimal fraction, where 𝑥, 𝑦 are digits from 1 to 9.

1
Part A 10
:
Using Geometric Series 3
𝑎 10 3 10 1
We wish to find the value of: 𝑆= = = × =
1−𝑟 1− 1 10 9 3
𝑥 = 0. 3̅ = 0.33333 …
10
Expand the recurring decimal, Using Equations
= 0.3 + 0.03 + 0.003 + ⋯ 1
Rewrite the decimals as decimal fractions: 𝑥 = 0.33 … ⇒ ⏟
⏟ 10𝑥 = 3.33 … ⇒ ⏟
9𝑥 = 3 ⇒ 𝑥 =
𝐼 𝐼𝐼 𝐼𝐼−𝐼
3
3 3 3
= + 2+ 3+⋯ Part B
10 10 10
3
This is an infinite geometric series with 𝑎 = 10 , 𝑟 = 2 2
+ +⋯
10 100

P a g e 127 | 274
Get all the files at: https://bit.ly/azizhandouts
Aziz Manva (azizmanva@gmail.com)

2 Part D
𝑎 10 2 10 2 𝑎 𝑎
𝑆= = = × = + +⋯
1−𝑟 1− 1 10 9 9 10 100
10 𝑎 𝑎
10 10 𝑎 10 𝑎
Part C 𝑆= = = × =
1−𝑟 1
21 21 1 − 10 10 9 9
+ +⋯
100 1002
21 Part E
21 100 21 7 𝑥𝑦 𝑥𝑦
𝑆= 100 = × = = + +⋯
1 100 1002
1 − 100 100 99 99 33 𝑥𝑦
𝑥𝑦 100 𝑥𝑦
𝑎 + 𝑏 = 7 + 33 = 40 𝑆 = 100 = × =
1 100 99 99
1−
100

Example 3.109
Let 𝐹 = .48181 be an infinite repeating decimal with the digits 8 and 1 repeating. When 𝐹 is written as a
fraction in lowest terms, the denominator exceeds the numerator by (AHSME 1970/10)

81 9
4 0. ̅81
̅̅̅ 4 99 4 11 4 9 53
̅̅̅̅ = 0.4 + 0.081
0.481 ̅̅̅̅ = + = + = + = + =
10 10 10 10 10 10 10 110 110

𝐷𝑖𝑓𝑓𝑒𝑟𝑒𝑛𝑐𝑒 = 110 − 53 = 57
D. Transformations
Geometric series can be manipulated in certain ways to still give a geometric series. Hence, questions using
these concepts can be done by combining our current knowledge of geometric series with algebra concepts. We
look at a few such manipulations:
➢ Squaring each term of the series
➢ Decomposing a series into odd and even powers of the common ratio
➢ Squaring an entire geometric series

3.110: Squares of Terms of Infinite Geometric Series


Squaring each term of the geometric series with 𝑓𝑖𝑟𝑠𝑡 𝑡𝑒𝑟𝑚 = 𝑎, and 𝑐𝑜𝑚𝑚𝑜𝑛 𝑟𝑎𝑡𝑖𝑜 𝑟:
𝑎 + 𝑎𝑟 + 𝑎𝑟 2 + ⋯ + 𝑎𝑟 𝑛−1
Gives a geometric series with 𝑓𝑖𝑟𝑠𝑡 𝑡𝑒𝑟𝑚 = 𝑎2 and 𝑐𝑜𝑚𝑚𝑜𝑛 𝑟𝑎𝑡𝑖𝑜 = 𝑟 2 :
𝑎2 + 𝑎2 𝑟 2 + 𝑎2 𝑟 4 + ⋯ + 𝑎2 𝑟 2(𝑛−1)

Example 3.111
𝑎
A. The limit of the sum of an infinite number of terms in a geometric progression is 1−𝑟 where 𝑎 denotes
the first term and −1 < 𝑟 < 1 denotes the common ratio. The limit of the sum of their squares is:
(AHSME 1951/11)
B. 𝐴 is an infinite geometric sequence. 𝐵 is the sequence obtained by squaring each term of A. What is the
ratio of the sum of B to the sum of A?
C. The sum of an infinite geometric series with common ratio 𝑟 such that |𝑟| < 1 is 15, and the sum of the
squares of the terms of this series is 45. The first term of the series is: (AHSME 1970/19)
D. An infinite geometric series has sum 2005. A new series, obtained by squaring each term of the original
𝑚
series, has 10 times the sum of the original series. The common ratio of the original series is 𝑛 where 𝑚
and 𝑛 are relatively prime integers. Find 𝑚 + 𝑛. (AIME 2005/II/3)

P a g e 128 | 274
Part A 2
2 2 𝑟=
𝐹𝑖𝑟𝑠𝑡 𝑇𝑒𝑟𝑚 = 𝑎 , 𝐶𝑜𝑚𝑚𝑜𝑛 𝑅𝑎𝑡𝑖𝑜 = 𝑟 3
𝑎2 2
𝑆= 𝑎 = 3 + 3𝑟 = 3 + 3 ( ) = 3 + 2 = 5
1 − 𝑟2 3
Part B Part D
The ratio is: Create an equation from the sum of the original
𝑎2 𝑎 𝑎 𝑎 𝑎 𝑎 series:
: =( )( ): = :1 𝑎
2
1−𝑟 1−𝑟 1+𝑟 1−𝑟 1−𝑟 1+𝑟 = 2005 ⇒ ⏟ 𝑎 = 2005 − 2005𝑟

1−𝑟 𝑬𝒒𝒖𝒂𝒕𝒊𝒐𝒏 𝑰𝑰
𝑬𝒒𝒖𝒂𝒕𝒊𝒐𝒏 𝑰
Part C Create an equation from the sum of the new series:
𝑎
= 15 ⇒ ⏟
𝑎 = 15 − 15𝑟 𝑎2
1−𝑟 𝑬𝒒𝒖𝒂𝒕𝒊𝒐𝒏 𝑰 = (10)(2005)
1 − 𝑟2
𝑎2 𝑎 𝑎 𝑎 𝑎
= 45 ⇒ ( )( ) = 45 ( )( ) = (10)(2005)
1−𝑟 2 1−𝑟 1+𝑟 ⏟1 − 𝑟 1 + 𝑟
𝑎 𝑬𝒒𝒖𝒂𝒕𝒊𝒐𝒏 𝑰𝑰𝑰
Substitute 1−𝑟 = 15:
𝑎 𝑎 Divide Equation III by Equation I:
15 ( ) = 45 ⇒ =3⇒⏟ 𝑎 = 3 + 3𝑟 𝑎
1+𝑟 1+𝑟 = 10 ⇒ ⏟𝑎 = 10 + 10𝑟
𝑬𝒒𝒖𝒂𝒕𝒊𝒐𝒏 𝑰𝑰 1+𝑟 𝑬𝒒𝒖𝒂𝒕𝒊𝒐𝒏 𝑰𝑽
Equate the RHS of Equation II and IV:
Note that the LHS of Equation I and II is the same.
10 + 10𝑟 = 2005 − 2005𝑟
Hence, the RHS must also be the same:
2015𝑟 = 1995
15 − 15𝑟 = 3 + 3𝑟 1995 399
12 = 18𝑟 𝑟= =
2015 403

3.112: Odd and Even Power Decomposition


For an infinite geometric series with first term 𝑎, and common ratio 𝑟, then:
𝑎
𝑆𝑢𝑚 𝑜𝑓 𝑒𝑣𝑒𝑛 𝑝𝑜𝑤𝑒𝑟𝑠 =
1 − 𝑟2
𝑎𝑟
𝑆𝑢𝑚 𝑜𝑓 𝑜𝑑𝑑 𝑝𝑜𝑤𝑒𝑟𝑠 =
1 − 𝑟2

We can decompose (break up) the geometric series with first term 𝑎 and common ratio 𝑟::
𝒂 + 𝑎𝑟 + 𝒂𝒓𝟐 + 𝑎𝑟 3 + 𝒂𝒓𝟒 + 𝑎𝑟 5 + ⋯ = ⏟
(𝑎𝑟 0 + 𝑎𝑟 2 + 𝑎𝑟 4 + ⋯ ) + ⏟
(𝑎𝑟 + 𝑎𝑟 3 + 𝑎𝑟 5 + ⋯ )
𝑬𝒗𝒆𝒏 𝑷𝒐𝒘𝒆𝒓𝒔 𝒐𝒇 𝒓 𝑶𝒅𝒅 𝑷𝒐𝒘𝒆𝒓𝒔 𝒐𝒇 𝒓
The even powers form a geometric series with first term 𝑎, and common ratio 𝑟 2 :
𝑎
𝑎𝑟 0 + 𝑎𝑟 2 + 𝑎𝑟 4 + ⋯ =
1 − 𝑟2
The odd powers form a geometric series with first term 𝑎𝑟, and common ratio 𝑟 2 :
𝑎𝑟
𝑎𝑟 0 + 𝑎𝑟 2 + 𝑎𝑟 4 + ⋯ =
1 − 𝑟2

Example 3.113
The geometric series 𝑎 + 𝑎𝑟 + 𝑎𝑟 2 + ⋯ has a sum of 7, and the terms involving odd powers of 𝑟 have a sum of 3.
What is 𝑎 + 𝑟? (AMC 12B 2007/15)

𝑆𝑢𝑚 𝑜𝑓 𝑔𝑒𝑜𝑚𝑒𝑡𝑟𝑖𝑐 𝑠𝑒𝑟𝑖𝑒𝑠 = 7 𝑎


×𝑟 =3
𝑆𝑢𝑚 𝑜𝑓 𝑂𝑑𝑑 𝑝𝑜𝑤𝑒𝑟𝑠 = 3 1 − 𝑟2
𝑎
𝑆𝑢𝑚 𝑜𝑓 𝑒𝑣𝑒𝑛 𝑝𝑜𝑤𝑒𝑟𝑠 = 7 − 3 = 4 Substitute = 4:
1−𝑟 2
3
4𝑟 = 3 ⇒ 𝑟 =
Sum of odd powers: 4
Get all the files at: https://bit.ly/azizhandouts
Aziz Manva (azizmanva@gmail.com)
3 𝑎 3 7 10 5
Substitute 𝑟 = 4 in 1−𝑟 = 7: 𝑎+𝑟 = + = =
𝑎 𝑎 7 4 4 4 2
=7⇒ =7⇒𝑎=
3 1 4
1−4 4

E. Geometry

Example 3.114
A. A line initially 1 inch long grows according to the following law, where the first term is the initial length.
1 1 1 1 1
1 + 4 √2 + 4 + 16 √2 + 16 + 64 √2 + ⋯
If the growth process continues forever, the limit of the length of the line is: (AHSME 1952/50)
B. An equilateral triangle is drawn with a side of length 𝑎. A new equilateral triangle is formed by joining
the midpoints of the sides of the first one. Then a third equilateral triangle is formed by joining the
midpoints of the sides of the second; and so on forever. The
limit of the sum of the perimeters of all the triangles thus
drawn is: (AHSME 1951/9)
C. (𝐴𝑛𝑠𝑤𝑒𝑟 𝑖𝑛 𝑡𝑒𝑟𝑚𝑠 𝑜𝑓 𝜋) The radius of the first circle is 1
inch, that of the second 1/2 inch, that of the third 1/4 inch
and so on indefinitely. The sum of the areas of the circles is:
(AHSME 1953/27)
D. The circles shown continue infinitely and have diameters 16 inches, 8
inches, 4 inches, and so on. The diameter of each circle is half the
diameter of the previous circle. What is the number of square inches in
the sum of the areas of all circles? Express your answer in terms of 𝜋.
(MathCounts 2001 Workout 7)
E. A square is divided into nine smaller squares of equal area. The center
square is then divided into nine smaller squares of equal area and the
pattern continues indefinitely. What fractional part of the figure is
shaded? (MathCounts 1994 Warm-Up 8)

Part A 3𝑎
=
Think of this as two geometric series put together: 2
1 1 1 1 1 We can continue to get:
(1 + + + ⋯ ) + ( √2 + √2 + √2 + ⋯ ) 3𝑎 3𝑎
⏟ 4 16 ⏟4 16 64 3𝑎 + + +⋯
𝑎=1,𝑟=
1 1
𝑎= √2,𝑟=
1 2 4
4 4 4 1
Substitute the above in the formula for the sum of a This is a geometric series with 𝑎 = 3𝑎, 𝑟 = 2
geometric series: 3𝑎 3𝑎
= = 6𝑎
1 1 1 1
1 √2 1 + 4 √2 4 + √2 4 4 + √2 1−
4 2 2
+ = = × = Part C
1 1 3 4 3 3
1− 1− 𝜋 𝜋
4 4 4
Part B ⏟ +
𝜋 + +⋯
𝐹𝑖𝑟𝑠𝑡

4 ⏟
16
Perimeter of the original triangle: 𝐶𝑖𝑟𝑐𝑙𝑒 𝑆𝑒𝑐𝑜𝑛𝑑 𝑇ℎ𝑖𝑟𝑑
𝐶𝑖𝑟𝑐𝑙𝑒 𝐶𝑖𝑟𝑐𝑙𝑒
= 𝑎 + 𝑎 + 𝑎 = 3𝑎 1
This is a geometric series with 𝑎 = 𝜋, 𝑟 =
By the Midpoint Theorem, the line formed by 4
𝜋 𝜋 4𝜋
joining the midpoints of two sides of a triangle is = =
parallel to the third side, and half of the third side. 1 3 3
1−
4 4
Hence, perimeter of second triangle: Part D
64𝜋 + 16𝜋 + 4𝜋 + ⋯

P a g e 130 | 274
Get all the files at: https://bit.ly/azizhandouts
Aziz Manva (azizmanva@gmail.com)
1
This is a geometric series with 𝑎 = 64𝜋, 𝑟 = 4 4 4 1 4 1 2
+ ( )( ) + ( )( ) + ⋯
64𝜋 𝜋 256𝜋 9 9 9 9 9
= = 4 1
This is a geometric series with 𝑎 = 9 , 𝑟 = 9
1 3 3
1−4 4
4
Part E 𝑎 4 9 1
𝑆= = 9 = × =
1−𝑟 1− 1 9 8 2
9

Example 3.115
Koch Snowflake
Quadrature of the parabola

Example 3.116
Each circle in an infinite sequence with decreasing radii is tangent externally to the one following it and to both
sides of a given right angle. The ratio of the area of the first circle to the sum of areas of all other circles in the
sequence, is (AHSME 1971/35)

Example 3.117
1 1 1
Find the sum of the real values of 𝑥 such that the infinite geometric series 𝑥 + 2 𝑥 3 + 4 𝑥 5 + 8 𝑥 7 + ⋯ is equal to
−12. (Alcumus)

Solve quadratic
Reject value outside of -1<r<1

Example 3.118
Consider two infinite geometric series. The first has leading term 𝑎, common ratio b, and sum S. The second has
1
a leading term 𝑏, common ratio 𝑎, and sum 𝑆. Find the value of 𝑎 + 𝑏. (AOPS Alcumus, Algebra, Geometric
Series)

The first geometric series has sum:


𝑎
𝑆=
⏟ 1−𝑏
𝐸𝑞𝑢𝑎𝑡𝑖𝑜𝑛 𝐼
The second geometric series has sum:
1 𝑏 1−𝑎
= ⇒𝑆=
𝑆 1−𝑎 ⏟ 𝑏
𝐸𝑞𝑢𝑎𝑡𝑖𝑜𝑛 𝐼𝐼
Equate the RHS of Equation I and II:
𝑎 1−𝑎
=
1−𝑏 𝑏
𝑎𝑏 = 1 − 𝑎 − 𝑏 + 𝑎𝑏
𝑎+𝑏=1

Example 3.119: Skipping Terms


Liu wanted to find the sum of an infinite geometric series. In each case, find the valid solution(s) for 𝑎 and 𝑟.
A. He forgot to add the first three terms, and his answer was negative one fourth of the correct answer.
B. He forgot to add the first two terms, and his answer was one fourth of the correct answer.
C. He forgot to add the first two terms, and his answer was negative one sixteenth of the correct answer.

P a g e 131 | 274
Get all the files at: https://bit.ly/azizhandouts
Aziz Manva (azizmanva@gmail.com)

D. He forgot to add the first two terms, and his answer was four less than the correct answer.

Part A 1
𝑆′ = − 𝑆
Let the original series be: 16
𝑎 𝑎𝑟 2 1 𝑎
𝑆= = ⏟ 𝑎 + 𝑎𝑟 + 𝑎𝑟 2 + 𝑎𝑟 3 + 𝑎𝑟 4 + ⋯ = (− ) ( )
1 − 𝑟 𝐹𝑖𝑟𝑠𝑡 𝑇ℎ𝑟𝑒𝑒 𝑇𝑒𝑟𝑚𝑠 1−𝑟 16 1 − 𝑟
Then, the series which Liu added was: 1
𝑟2 = −
𝑎𝑟 3 16
𝑆 ′ = 𝑎𝑟 3 + 𝑎𝑟 4 + ⋯ = No real solutions for 𝑟.
1−𝑟
Note that this is also a geometric series with first Part D
term 𝑎𝑟 3 , and common ratio 𝑟. 𝑎 𝑎𝑟 2
𝑆= , 𝑆−4=
Since the answer ⏟ 1−𝑟 ⏟ 1−𝑟
1 𝐸𝑞𝑢𝑎𝑡𝑖𝑜𝑛 𝐼 𝐸𝑞𝑢𝑎𝑡𝑖𝑜𝑛 𝐼𝐼
𝑆′ = − 𝑆 Subtract Equation II from Equation I:
4
𝑎𝑟 3 1 𝑎 𝑎 − 𝑎𝑟 2
= (− ) ( ) 4=
1−𝑟 4 1−𝑟 1−𝑟
1 4 = 𝑎(1 + 𝑟)
𝑟3 = − 4−𝑎
4 𝑟=
𝑎
3 1 1 But note that in an infinite geometric series
𝑟 = √− = − 3
4 √4 −1 < 𝑟 < 1
𝑎 can take any value. 4−𝑎
−1 < <1
Part B 𝑎
1 Case I: 𝑎 > 0
𝑆′ = 𝑆 −𝑎 < 4 − 𝑎 < 𝑎
4
𝑎𝑟 2 1 𝑎 0 < 4 < 2𝑎
= ( )( ) 2𝑎 > 4
1−𝑟 4 1−𝑟
1 𝑎>2
𝑟2 = Case I: 𝑎 < 0
4
1 −𝑎 > 4 − 𝑎 > 𝑎
𝑟=±
2 0 > 4 > 2𝑎
𝑎 can take any value. 2𝑎 < 4
Part C 𝑎<2
𝑎<0

3.6 Finite Geometric Series


A. Growth

3.120: Growth
In a geometric series:
𝑟 > 1 ⇒ 𝐺𝑟𝑜𝑤𝑡ℎ

➢ If the common ratio of a geometric series is greater than one, the resulting terms increase.
➢ An increasing geometric series (that is, one which exhibits growth) has many applications:
✓ (𝐹𝑖𝑛𝑎𝑛𝑐𝑒/𝐴𝑐𝑐𝑜𝑢𝑛𝑡𝑠) Money added to a bank account earns interest, and if the interest is
compounded, grows geometrically.
✓ (𝐹𝑖𝑛𝑎𝑛𝑐𝑒) The value of an asset, such land, or a stock market investment, or a rare stamp grows
as time passes. If the rate of growth every year is the same percentage, then the growth is
geometric.

P a g e 132 | 274
Get all the files at: https://bit.ly/azizhandouts
Aziz Manva (azizmanva@gmail.com)

✓ (𝐵𝑖𝑜𝑙𝑜𝑔𝑦) A population of animals (or insects, or bacteria) grows geometrically when the
conditions are ideal.
✓ (𝑅𝑒𝑎𝑙 𝐿𝑖𝑓𝑒 𝑆𝑐𝑒𝑛𝑎𝑟𝑖𝑜𝑠) The spread of an important piece of news can grow geometrically under
the right conditions.

Example 3.121
Plot the following geometric sequence:
1,2,4,8,16, …

𝐹𝑖𝑟𝑠𝑡 𝑇𝑒𝑟𝑚 = 𝑎 = 1
𝐶𝑜𝑚𝑚𝑜𝑛 𝑟𝑎𝑡𝑖𝑜 = 𝑟 = 2 > 1

Because 𝑟 > 1, the value of each term goes up.

Example 3.122
Let the sum of the first 𝑛 terms of the series 1 + 2 + 4 + 8 + 16 + ⋯ be 𝑆𝑛 .
A. Find 𝑆1 , 𝑆2 , 𝑆3 , 𝑆4 and 𝑆5

𝑆1 = 1
𝑆2 = 3
𝑆3 = 7
𝑆4 = 15
𝑆5 = 31

3.123: Sum of a Geometric Series


The sum of a finite geometric series 𝑎 + 𝑎𝑟 + 𝑎𝑟 2 + ⋯ + 𝑎𝑟 𝑛−1 with first term 𝑎, common ratio 𝑟 and 𝑛 terms is
given by:
𝑎(1 − 𝑟 𝑛 ) 𝑎(𝑟 𝑛 − 1)
𝑆𝑛 = =
1−𝑟 𝑟−1

We can define a finite geometric series with first term 𝑎, common ratio 𝑟 and 𝑛 terms as:
𝑆 = 𝑎 + 𝑎𝑟 + 𝑎𝑟 2 + ⋯ + 𝑎𝑟 𝑛−1

𝐸𝑞𝑢𝑎𝑡𝑖𝑜𝑛 𝐼
Multiply both sides by the common ratio = 𝑟:
𝑟𝑆 = 𝑎𝑟 + 𝑎𝑟 2 + ⋯ + 𝑎𝑟 𝑛−1 + 𝑎𝑟 𝑛

𝐸𝑞𝑢𝑎𝑡𝑖𝑜𝑛 𝐼𝐼

P a g e 133 | 274
Get all the files at: https://bit.ly/azizhandouts
Aziz Manva (azizmanva@gmail.com)

Subtract Equation I from Equation II:


𝑟𝑆 − 𝑆 = 𝑎𝑟 𝑛 − 𝑎

Factor out the 𝑆 on the LHS, and 𝑎on the RHS:


𝑆(𝑟 − 1) = 𝑎(𝑟 𝑛 − 1)

Isolate 𝑆:
𝑎(𝑟 𝑛 − 1) 𝑎(1 − 𝑟 𝑛 )
𝑆= =
𝑟−1 1−𝑟

Example 3.124: Basics


Identify 𝑎, 𝑟 and 𝑛 in each geometric series below. Hence, find the sum.
A. 1 + 2 + 4 + 8 + ⋯ + 1024
B. 3 + 9 + 27 + 81
C. 25 + 125 + 625

Part A
20 + 21 + 22 + ⋯ + 210 ⇒ 𝑛 = 11
𝑎(𝑟 𝑛 −1)
Substitute 𝑎 = 1, 𝑟 = 2, 𝑛 = 11 in 𝑆 = 𝑟−1
:
(1)(211 − 1) 2048 − 1
𝑆= = = 2047
2−1 2−1
Part B
3 + 32 + 33 + 34 ⇒ 𝑛 = 4
𝑎(𝑟 𝑛 −1)
Substitute 𝑎 = 3, 𝑟 = 3, 𝑛 = 4 in 𝑆 = :
𝑟−1
(3)(34 − 1) (3)(80) (3)(80)
𝑆= = = = 120
3−1 2 2
Part C
52 + 53 + 54 ⇒ 𝑛 = 3
𝑎(𝑟 𝑛 −1)
Substitute 𝑎 = 25, 𝑟 = 5, 𝑛 = 3 in 𝑆 = 𝑟−1
:
(25)(5 3
− 1) (25)(124)
𝑆= = = (25)(31) = 775
5−1 4

3.125: Sum of Powers of Two


1 + 2 + 4 + 8 + ⋯ + 2𝑥 = 2𝑥+1 − 1

20 + 21 + 22 + ⋯ + 2𝑥 ⇒ 𝑛 = 𝑥 + 1
Substitute 𝑎 = 1, 𝑟 = 2, 𝑛 = 𝑥 + 1:
𝑎(𝑟 𝑛 − 1) (1)(2𝑥+1 − 1)
𝑆= = = 2𝑥+1 − 1
𝑟−1 2−1

Example 3.126
What is the sum of the first seven whole number powers of two? the first seven natural number powers of two?

𝑊ℎ𝑜𝑙𝑒 𝑁𝑢𝑚𝑏𝑒𝑟𝑠 = 𝕎 = {0,1,2,3, … }


𝑁𝑎𝑡𝑢𝑟𝑎𝑙 𝑁𝑢𝑚𝑏𝑒𝑟𝑠 = ℕ = {1,2,3, … }
Whole Number Powers:
𝑆 = 20 + 21 + 22 + ⋯ + 27 = 256 − 1 = 255
Natural Number Powers:

P a g e 134 | 274
Get all the files at: https://bit.ly/azizhandouts
Aziz Manva (azizmanva@gmail.com)

21 + 22 + ⋯ + 27 = 𝑆 − 1 = 255 − 1 = 254

Example 3.127
A. On Monday, Jessica told two friends a secret. On Tuesday, each of those friends told the secret to two
other friends. Each time a student heard the secret, he or she told the secret to two other friends the
following day. On what day of the week will 1023 students know the secret? (MathCounts 2001 National
Sprint)
B. Krista put 1 cent into her new bank on a Sunday morning. On Monday she put 2 cents into her bank. On
Tuesday she put 4 cents into her bank, and she continued to double the amount of money she put into
her bank each day for two weeks. On what day of the week did the total amount of money in her bank
first exceed 2 dollars? (MathCounts 2006 Chapter Sprint)

Part A
The number of people who come to know the secret on a particular day is:

1 , ⏟ 2 , ⏟ 4 ,…, 2 ⏟𝑛
𝑆𝑢𝑛𝑑𝑎𝑦 𝑀𝑜𝑛𝑑𝑎𝑦 𝑇𝑢𝑒𝑠𝑑𝑎𝑦 𝐷𝑎𝑦 𝑛
𝐷𝑎𝑦 0 𝐷𝑎𝑦 1 𝐷𝑎𝑦 2
𝑛 𝑛+1
1 + 2 +4 +⋯+ 2 = 2 −1

2𝑛+1 − 1 = 1023
2𝑛+1 = 1024 = 210
𝑛 + 1 = 10
𝑛=9
𝐷𝑎𝑦 9 = 𝑆𝑢𝑛𝑑𝑎𝑦 + 9 = 𝑆𝑢𝑛𝑑𝑎𝑦 + 2 = 𝑇𝑢𝑒𝑠𝑑𝑎𝑦


1 , ⏟
3 , ⏟
7 , 15
⏟ , 31
⏟ , 63
⏟ , 127
⏟ , 255
⏟ , 511
⏟ , 1023

𝑆𝑢𝑛 𝑀𝑜𝑛 𝑇𝑢𝑒 𝑊𝑒𝑑 𝑇ℎ𝑢 𝐹𝑟𝑖 𝑆𝑎𝑡 𝑆𝑢𝑛 𝑀𝑜𝑛 𝑇𝑢𝑒
Part C

1 , ⏟
2 , ⏟
4 ⏟𝑛
,…, 2
𝑆𝑢𝑛𝑑𝑎𝑦 𝑀𝑜𝑛𝑑𝑎𝑦 𝑇𝑢𝑒𝑠𝑑𝑎𝑦 𝐷𝑎𝑦 𝑛
𝐷𝑎𝑦 0 𝐷𝑎𝑦 1 𝐷𝑎𝑦 2

20 + 21 + 22 + ⋯ + 27 = 28 − 1 = 255 ⇒ 7 𝐷𝑎𝑦𝑠 ⇒ 𝑆𝑢𝑛𝑑𝑎𝑦



1 , ⏟
3 , ⏟
7 , 15
⏟ , 31
⏟ , 63
⏟ , 127
⏟ , 255

𝑆𝑢𝑛 𝑀𝑜𝑛 𝑇𝑢𝑒 𝑊𝑒𝑑 𝑇ℎ𝑢 𝐹𝑟𝑖 𝑆𝑎𝑡 𝑆𝑢𝑛

Example 3.128
A. Sam decides to start a rumor. Sam tells the rumor to her three friends. Each of Sam's three friends then
tells the rumor to three friends who have not heard the rumor. This continues for five total cycles. Sam
telling her three friends was the first cycle. How many people, not including Sam, will have heard the
rumor when the fifth cycle is complete?


3 , ⏟
9 , 27
⏟ , 81
⏟ , 243

𝐶𝑦𝑐𝑙𝑒 1 𝐶𝑦𝑐𝑙𝑒 2 𝐶𝑦𝑐𝑙𝑒 3 𝐶𝑦𝑐𝑙𝑒 4 𝐶𝑦𝑐𝑙𝑒 5
3 + 9 + 27 + 81 + 243 = 363

Substitute 𝑎 = 3, 𝑟 = 3, 𝑛 = 5
𝑎(𝑟 𝑛 − 1) 3(3𝑛 − 1) 3(242)
𝑆= = = = 363
𝑟−1 3−1 2
B. Decay

P a g e 135 | 274
Get all the files at: https://bit.ly/azizhandouts
Aziz Manva (azizmanva@gmail.com)

3.129: Decay
In a geometric series:
0 < 𝑟 < 1 ⇒ 𝐷𝑒𝑐𝑎𝑦

➢ If the common ratio of a geometric series is positive but less than than one, the resulting terms
decreases.
➢ A decreasing geometric series (that is, one which exhibits decay) has a number of real-life applications:
✓ (𝑃ℎ𝑦𝑠𝑖𝑐𝑠) A bouncing ball rises back up to a fraction of the height that it was dropped from.
✓ (𝑅𝑒𝑐𝑦𝑐𝑙𝑖𝑛𝑔) If a fraction of a commodity can be recycled, then the amount available to recycle
will exhibit decay.

Example 3.130
Plot the geometric sequence:
1
4,2,1,
2

Example 3.131
1 1 1
A. Find the sum of the first six terms in the geometric sequence , , , … Express your answer as a
2 4 8
common fraction. (MathCounts 1996 School Countdown)
1 1 1 1 1 1
B. What is the following value when expressed as a common fraction: + + + ⋯+ + + ?
21 22 23 28 29 210
(MathCounts 2007 Warm-Up 10)
1 1 1 1 1
C. What is the value of the following expression: − + − + ? Express your answer as a common
3 9 27 81 243
fraction. (MathCounts 2007 Chapter Sprint)

Part A
1 1 𝑎(1−𝑟 𝑛 )
Substitute 𝑎 = 2 , 𝑟 = 2 , 𝑛 = 6 in 𝑆 = 1−𝑟
:
1 1 6
( ) (1 − ( ) ) (1) (1 − 1 )
2 2 64 = 63
𝑆= = 2
1 1 64
1−2 2
Part B
1 1 𝑎(1−𝑟 𝑛 )
Substitute 𝑎 = 2 , 𝑟 = 2 , 𝑛 = 10 in 𝑆 = 1−𝑟
:
1 1 10
(2) (1 − (2) ) (1) (1 − 1 )
𝑆= = 2 1024 = 1023
1 1 1024
1−2 2
Part C
1 1 𝑎(1−𝑟 𝑛 )
Substitute 𝑎 = , 𝑟 = − , 𝑛 = 5 in 𝑆 = :
3 3 1−𝑟
5
1 1 1 1
(3) (1 − (− 3) ) (3) (1 + 243) 244 1 61
= = × =
1 4 243 4 243
1 − (− 3) 3

P a g e 136 | 274
Get all the files at: https://bit.ly/azizhandouts
Aziz Manva (azizmanva@gmail.com)

3.132: Sum of Negative Powers of Two


1 1 1 2𝑛 − 1
+ 2 + ⋯+ 𝑛 =
2 2 2 2𝑛

1 1 𝑎(1−𝑟 𝑛 )
Substitute 𝑎 = , 𝑟 = , 𝑛 = 𝑛 in 𝑆 = :
2 2 1−𝑟
1 1 𝑛
(2) (1 − (2) ) (1) (1 − 1𝑛 ) 2𝑛 − 1
𝑆= = 2 2 =
1 1 2𝑛
1−2 2

Example 3.133
Identify 𝑎, 𝑟 and 𝑛 in each geometric series below. Hence, find the sum.
1 1 1 1
A. 3 + 9 + 27 + 81
1 1
B. 25 − 5 + 1 − 5 + 25
1 1 1 2 4
C. 4
+ 6 + 9 + 27 + 81
1 1 1 1
D. 10
+ 100 + 1000 + 10,000
1 1 1
E. 10
+ 1000 + 100,000
1 1 1 1
F. 10
− 100 + 1000 − 10,000

Part A
1 1 𝑎(𝑟 𝑛 −1)
Substitute 𝑎 = 3 , 𝑟 = 3 , 𝑛 = 4 in 𝑆 = 𝑟−1
:
1 1 4
(3) (1 − (3) ) (1) (1 − 1 ) 80
𝑆= = 3 81 = 81 = 40
1 2 2 81
1−3 3
Part B
1 𝑎(𝑟 𝑛 −1)
Substitute 𝑎 = 25, 𝑟 = − , 𝑛 = 5 in 𝑆 = :
5 𝑟−1
1 5
(25) (1 − (− ) ) (25) (1 + 1 ) (25) (
3126
) (
3126
) 3126 5 521
5 3125 3125
𝑆= = = = 125 = × =
1 6 6 6 125 6 25
1 − (− )
5 5 5 5
Part C
1 2 𝑎(𝑟 𝑛 −1)
Substitute 𝑎 = 4 , 𝑟 = 3 , 𝑛 = 5 in 𝑆 = 𝑟−1
:
5
1 2
( ) (1 − ( ) ) (1) (1 − 32 )
4 3 243 = (1) (211) × 3 = 211
𝑆= = 4
2 1 4 243 324
1−3 3
Part D
1 1 𝑎(𝑟 𝑛 −1)
Substitute 𝑎 = 10 , 𝑟 = 10 , 𝑛 = 4 in 𝑆 = 𝑟−1
:
1 1 4
( ) (1 − ( ) ) ( 1 ) (1 − 1 )
10 10 10000 = 9999 × 1 = 1111 = 0.1111
𝑆= = 10
1 9 10000 9 10000
1 − 10 10

P a g e 137 | 274
Get all the files at: https://bit.ly/azizhandouts
Aziz Manva (azizmanva@gmail.com)

1 1 1 1
+ + + = 0.1 + 0.01 + 0.001 + 0.0001 = 0.1111
10 100 1000 10,000
Part E
1 1 1
+ + = 0.1 + 0.001 + 0.00001 = 0.10101
10 1000 100,000
Part F
1 1 𝑎(𝑟 𝑛 −1)
Substitute 𝑎 = 10 , 𝑟 = − 10 , 𝑛 = 4 in 𝑆 = 𝑟−1
:
1 1 4
( ) (1 − (− ) ) ( 1 ) (1 − 1 ) 9999
10 10 10 10000 10000 9999 1 909
𝑆= = = = × =
1 11 11 10000 11 10000
1 − (− 10) 10

1 1 1 1
− + −
10 100 1000 10,000

Example 3.134
If 𝑛 is a multiple of 4, the sum 𝑠 = 1 + 2𝑖 + 3𝑖 2 +. . . +(𝑛 + 1)𝑖 𝑛 , where 𝑖 = √−1, equals: (AHSME 1964/34)

Ans=(1/2)(n+2-ni)

Example 3.135: Recursive Definition


A. The first term of a given sequence is 1 and each successive term is the sum of all the previous terms of
the sequence. What is the value of the first term which exceeds 5000? (MathCounts 2003 National
Sprint)
B. Find the value of the general term.
C. In the above, find the term number of the term whose value you found.


1 , ⏟
1 , ⏟
2 , ⏟
4 , ⏟
8
𝑇𝑒𝑟𝑚 𝑇𝑒𝑟𝑚 𝑇𝑒𝑟𝑚 𝑇𝑒𝑟𝑚 𝑇𝑒𝑟𝑚
1 2 3 4 5

𝑡 = 1, 𝑛 = 1
𝑡𝑛 = { 𝑛 𝑛−2
2 ,𝑛 ≥ 2

2𝑛−2 > 5000


𝑛−2
2 = 8192 = 213
𝑛 − 2 = 13
𝑛 = 15

Example 3.136
(𝑆𝑢𝑚 𝑜𝑓 𝑆𝑒𝑞𝑢𝑒𝑛𝑐𝑒: 𝐸𝑎𝑐ℎ 𝑇𝑒𝑟𝑚 𝑖𝑠 𝑎 𝑆𝑒𝑟𝑖𝑒𝑠) The sum of the first 𝑛 terms of the sequence 1, (1 + 2), (1 + 2 +
22 ), . . . , (1 + 2 + 22 + ⋯ + 2𝑛−1 ) in terms of 𝑛 is: (AHSME 1972/19)

The first few terms of the sequence are:


1,3,7,15,31, …
We can write them as:
𝑆1 = 1 = 21 − 1
𝑆2 = 1 + 2 = 3 = 22 − 1
𝑆3 = 1 + 2 + 4 = 7 = 23 − 1

P a g e 138 | 274
Get all the files at: https://bit.ly/azizhandouts
Aziz Manva (azizmanva@gmail.com)

.
.
.
𝑆𝑛 = 1 + 2 + ⋯ + 2𝑛−1 = 2𝑛 − 1

Add the RHS of each of the above:


21 + 22 + ⋯ + 2𝑛 − 𝑛

20 + 21 + 22 + ⋯ + 2𝑛 = 2𝑛 − 1
21 + 22 + ⋯ + 2𝑛 = 2𝑛 − 2

= 2𝑛+1 − 𝑛 − 2
..
Example 3.137: Decay
A ball is dropped straight down from a height of 16 feet. If it bounces back each time to a height
A. one-half the height from which it last fell, how far will the ball have traveled when it hits the floor for the
sixth time, in feet? (MathCounts 2009 Warm-up 3)
B. one-half the height from which it last fell, how far will the ball have traveled when it hits the floor for the
𝑛𝑡ℎ time, in feet?
C. 𝑟 times (where 0 < 𝑟 < 1) the height from which it last fell, how far will the ball have traveled when it
hits the floor for the sixth time, in feet?

Part A

16 + ⏟
16 + 8 + 4 + 2 + 1 = 16 + 31 = 47
𝐺𝑒𝑜𝑚𝑒𝑡𝑟𝑖𝑐 𝑆𝑒𝑟𝑖𝑒𝑠
1
𝑎=16,𝑟=
2
1
Substitute 𝑎 = 16, 𝑟 = 2 , 𝑛 = 5:
1 5 1 1
16 (1 − (2) ) 16 (1 − 32) 16 (1 − 32)
𝑎(1 − 𝑟 𝑛 ) 31
𝑆= = = = = 32 ( ) = 31
1−𝑟 1 1 1 32
1−2 2 2
Part B
1
Substitute 𝑎 = 16, 𝑟 = 2 , 𝑛 = 𝑛 − 1:

P a g e 139 | 274
Get all the files at: https://bit.ly/azizhandouts
Aziz Manva (azizmanva@gmail.com)

1 𝑛−1 1 2𝑛−1 − 1
16 (1 − (2) ) 16 (1 − 16 ( )
𝑛−1 )
𝑛)
𝑎(1 − 𝑟 2 2𝑛−1 2𝑛−1 − 1 2𝑛−1 − 1
𝑆= = = = = 32 ( 𝑛−1 ) =
1−𝑟 1 1 1 2 2𝑛−6
1−2 2 2

And hence the final answer is:


2𝑛−1 − 1
16 + 𝑆 = 16 +
2𝑛−6
Part C
Substitute 𝑎 = 16, 𝑟 = 𝑟, 𝑛 = 5:
𝑎(1 − 𝑟 𝑛 ) 16(1 − (𝑟)5 ) 16(1 − 𝑟 5 )
𝑆= = = = 32(1 − 𝑟 5 )
1−𝑟 1 1
1−2 2
And hence the final answer is:
16 + 𝑆 = 16 + 32(1 − 𝑟 5 )

Example 3.138: Decay


A. A ball is dropped straight down from a height of ℎ feet. If it bounces back each time to a height 𝑟 times
(where 0 < 𝑟 < 1) the height from which it last fell, how far will the ball have traveled when it hits the
floor for the 𝑛𝑡ℎ time, in feet?

ℎ + ℎ𝑟 + ℎ𝑟 2 + ⋯ =
ℎ+⏟
𝐺𝑒𝑜𝑚𝑒𝑡𝑟𝑖𝑐 𝑆𝑒𝑟𝑖𝑒𝑠
𝑎=ℎ,𝑟=𝑟
Substitute 𝑎 = ℎ, 𝑟 = 𝑟, 𝑛 = 𝑛 − 1:
𝑎(1 − 𝑟 𝑛 ) ℎ(1 − (𝑟)𝑛−1 ) ℎ(1 − 𝑟 𝑛−1 )
𝑆= = =
1−𝑟 1−𝑟 1−𝑟

And hence the final answer is:


ℎ(1 − 𝑟 𝑛−1 ) ℎ(1 − 𝑟 𝑛−1 )
16 + = 16 +
1−𝑟 1−𝑟

Example 3.139: Decay


A. A ''super ball'' is dropped from a window 16 meters above the ground. On each bounce it rises 3/4 the
distance of the preceding high point. The ball is caught when it reached the high point after hitting the
ground for the third time. How far has it travelled? (MathCounts 1993 School Countdown)
B. A super ball is dropped from 100 feet and rebounds half the distance it falls each time it bounces. How
many feet will the ball have traveled when it hits the ground the fourth time? (MathCounts 1997 School
Target)

Part A r=3/4
16+12+... n=3
a=16 (16+28) (1-(3/4)^3)/(1-3/4) =259/4
r=3/4 Type equation here.
n=3 Part B
100+50+25+12.5=187.5
12+9+... 50+25+12.5=87.5
a=12 Total=275

P a g e 140 | 274
Example 3.140: Decay
A. It took Lara five days to read a novel. Each day after the first day, Lara read half as many pages as the day
before. If the novel was 248 pages long, how many pages did she read on the first day? (MathCounts
2004 School Sprint)
B. Five aluminum cans can be recycled to make a new can. How many new cans can eventually be made
from 125 aluminum cans? (Remember that the first new cans that are made can then be recycled into
even newer cans!) (MathCounts 2005 Warm-Up 1)
C. Candle stubs to make candles

Part A Part B
Ans=128 25+5+1

3.141: Negative Common Ratio


𝑟 < −1 ⇒ 𝑆𝑖𝑔𝑛𝑠 𝑎𝑙𝑡𝑒𝑟𝑛𝑎𝑡𝑒, 𝑎𝑏𝑠𝑜𝑙𝑢𝑡𝑒 𝑣𝑎𝑙𝑢𝑒 𝑖𝑛𝑐𝑟𝑒𝑎𝑠𝑒𝑠
−1 < 𝑟 < 0 ⇒ 𝑆𝑖𝑔𝑛𝑠 𝑎𝑙𝑡𝑒𝑟𝑛𝑎𝑡𝑒, 𝑎𝑏𝑠𝑜𝑙𝑢𝑡𝑒 𝑣𝑎𝑙𝑢𝑒 𝑑𝑒𝑐𝑟𝑒𝑎𝑠𝑒𝑠

Example 3.142
Let the sum of the first 𝑛 terms of the series 1 − 2 + 4 − 8 + 16 + ⋯ be 𝑆𝑛 .
B. Find 𝑆1 , 𝑆2 , 𝑆3 , 𝑆4 and 𝑆5
C. Find a general formula for 𝑆𝑛 .

𝑆1 = 1
𝑆2 = −1
𝑆3 = 3
𝑆4 = −5
𝑆5 = 11

1 1 1 1
Repeat the above question with 4 − 2 + 1 − 2 + 4 − 8 + 16 + ⋯ be 𝑆𝑛 .
Grapojh

𝑆1 = 4
𝑆2 = 2
𝑆3 = 3
1
𝑆4 = 2
2
3
𝑆5 = 2
4

1 From here

Example 3.143: Back Calculations


A. In a geometric series of positive terms the difference between the fifth and fourth terms is 576, and the
difference between the second and first terms is 9. What is the sum of the first five terms of this series?
(AHSME 1974/21)
Get all the files at: https://bit.ly/azizhandouts
Aziz Manva (azizmanva@gmail.com)

B. If the first term of an infinite geometric series is a positive integer, the common ratio is the reciprocal of
a positive integer, and the sum of the series is 3, then the sum of the first two terms of the series is
(AHSME 1975/16)

Part A
Ans=1023
Part B

Example 3.144: Transformations


A. Let a geometric progression with 𝑛 terms have first term one, common ratio 𝑟 and sum 𝑠, where 𝑟 and 𝑠
are not zero. The sum of the geometric progression formed by replacing each term of the original
progression by its reciprocal is (AHSME 1976/4)
B. If 𝑃 is the product of 𝑛 quantities in Geometric Progression, 𝑆 their sum, and 𝑆′ the sum of their
reciprocals, then 𝑃 in terms of 𝑆, 𝑆′, and 𝑛 is: (AHSME 1971/33)

Part A
Ans = s/(r^n-1)
Part B

C. Mixing Arithmetic and Geometric Series

Example 3.145
Given a geometric sequence with the first term ≠ 0 and 𝑟 ≠ 0 and an arithmetic sequence with the first term =
0. A third sequence 1,1,2, . .. is formed by adding corresponding terms of the two given sequences. The sum of
the first ten terms of the third sequence is: (AHSME 1955/45)

Let the geometric sequence be 𝑎, 𝑎𝑟, 𝑎𝑟 2 and the arithmetic sequence be 0, 𝑑, 2𝑑. Then:
𝑎+0=1
𝑎𝑟 + 𝑑 = 1
𝑎𝑟 2 + 2𝑑 = 2

Example 3.146
A high school basketball game between the Raiders and Wildcats was tied at the end of the first quarter. The
number of points scored by the Raiders in each of the four quarters formed an increasing geometric sequence,
and the number of points scored by the Wildcats in each of the four quarters formed an increasing arithmetic
sequence. At the end of the fourth quarter, the Raiders had won by one point. Neither team scored more than
100 points. What was the total number of points scored by the two teams in the first half? (AMC 10B 2010/24,
AMC 12B 2010/19)

Let the score at the end of the first quarter for both 15𝑎 = 4𝑎 + 6𝑑 + 1
Raiders and Wildcats be 𝑎. Then, the scores will be: 11𝑎
⏟ = 6𝑑 + 1

Since Raiders won by 1 point: 𝑀𝑢𝑙𝑡𝑖𝑝𝑙𝑒 𝑂𝑛𝑒 𝑚𝑜𝑟𝑒 𝑡ℎ𝑎𝑛
𝑜𝑓 11 𝑎 𝑚𝑢𝑙𝑡𝑖𝑝𝑙𝑒 𝑜𝑓 6
𝑎 (1 + 𝑟 + 𝑟 2 + 𝑟 3 ) = ⏟
⏟ 4𝑎 + 6𝑑 + 1
Check the first few multiples of 11:
𝐼𝑛𝑡𝑒𝑔𝑒𝑟 𝐼𝑛𝑡𝑒𝑔𝑒𝑟
Since points scored in basketball are integers, the 11 = 6 + 5
RHS and the first term on the left must be an 22 = 18 + 4
integer. This means 1 + 𝑟 + 𝑟 2 + 𝑟 3 must also be an 33 = 30 + 3
integer. 44 = 42 + 2
Check 𝑟 = 2: 55 = 54 + 1 ⇒ 𝑊𝑜𝑟𝑘𝑠

P a g e 142 | 274
Get all the files at: https://bit.ly/azizhandouts
Aziz Manva (azizmanva@gmail.com)

11𝑎 = 55 ⇒ 𝑎 = 5 𝑅𝑎𝑖𝑑𝑒𝑟𝑠 𝑇𝑜𝑡𝑎𝑙 = 5 + 10 + 20 + 40 = 75 = 74 + 1


6𝑑 + 1 = 55 ⇒ 𝑑 = 9 Total scored by the two teams in the first half
We can confirm this works: = 5 + 14 + 5 + 10 = 34
𝑊𝑖𝑙𝑑𝑐𝑎𝑡 𝑇𝑜𝑡𝑎𝑙 = 5 + 14 + 23 + 32 = 74

D. Geometry

3.147: Golden Ratio in a Right-Angled Triangle3


If the sides of a right-angled triangle form a geometric sequence, then the common ratio is

𝟏 + √𝟓
𝒓 = √𝜙 = √
𝟐
𝟏+√𝟓
➢ The golden ratio is 𝜙 = 𝟐
.
➢ You should have seen the golden ratio in the Note on Quadratics. (If you haven’t, then do it now.)

Let the sides be:



𝑎 , 𝑎𝑟
⏟ , ⏟2
𝑎𝑟
𝑺𝒉𝒐𝒓𝒕𝒆𝒓 𝑳𝒆𝒈 𝑳𝒐𝒏𝒈𝒆𝒓 𝑳𝒆𝒈 𝑯𝒚𝒑𝒐𝒕𝒆𝒏𝒖𝒔𝒆
Then, by Pythagoras Theorem:
⏟2 + ⏟
𝑎 (𝑎𝑟)2 = (𝑎𝑟
⏟ 2 )2 ⇒ 𝑎2 + 𝑎2 𝑟 2 = 𝑎2 𝑟 4 ⇒ 𝑎2 𝑟 4 − 𝑎2 𝑟 2 − 𝑎2 = 0
𝑺𝒉𝒐𝒓𝒕𝒆𝒓 𝑳𝒐𝒏𝒈𝒆𝒓 𝑯𝒚𝒑𝒐𝒕𝒆𝒏𝒖𝒔𝒆
𝑳𝒆𝒈 𝑳𝒆𝒈
2
Divide both sides by 𝑎 , which we can since 𝑎 ≠ 0:
𝑟4 − 𝑟2 − 1 = 0
This is a disguised quadratic. Substitute 𝜙 = 𝑟 2 :
𝜙2 − 𝜙 − 1 = 0
Apply the Quadratic Formula, and reject the negative value since we know that the ratio must be positive:

1 + √5 𝟏 − √𝟓 1 + √5 1 + √5
𝜙∈{ , }⇒𝜙= ⇒ 𝑟 = √𝜙 = √
2 ⏟ 𝟐 2 2
𝑵𝒆𝒈𝒂𝒕𝒊𝒗𝒆

Example 3.148: Midpoint of Triangles


Points 𝐵, 𝐷, and 𝐽 are midpoints of the sides of right triangle 𝐴𝐶𝐺. Points 𝐾, 𝐸, 𝐼 are midpoints of the sides of
triangle 𝐽𝐷𝐺, etc. If the dividing and shading process is done 100 times (the first three are shown) and 𝐴𝐶 =
𝐶𝐺 = 6, then the total area of the shaded triangles is nearest (AMC 8 1999/25)

Midpoint Theorem
By the Midpoint Theorem:
Since J and D are midpoints of AG and CG respectively,
1
𝐽𝐷 ∥ 𝐵𝐶, 𝐽𝐷 = 𝐵𝐶
2
Since B and J are midpoints of AC and AG respectively,
1
𝐵𝐽 ∥ 𝐶𝐺, 𝐵𝐽 = 𝐶𝐺
2
Also,

3These triangles are called Kepler Triangles, after Kepler, who discussed them in a letter written in 1597. However, they
were known much earlier.

P a g e 143 | 274
Get all the files at: https://bit.ly/azizhandouts
Aziz Manva (azizmanva@gmail.com)

∠𝐵𝐽𝐷 = 90°
Hence
Δ𝐴𝐽𝐵 ≅ Δ𝐷𝐽𝐵 ≅ Δ𝐷𝐶𝐵
Hence,
1
[𝐷𝐶𝐵] = [𝐴𝐽𝐷𝐶]
3
This same logic is applicable to each of the 100 quadrilaterals which will be created.
And hence
1 1 1 1 1
𝑆ℎ𝑎𝑑𝑒𝑑 𝐴𝑟𝑒𝑎 ≈ [𝐴𝐵𝐶] = ( ℎ𝑏) = × × 6 × 6 = 6
3 3 2 3 2

Geometric Series
1
[𝐵𝐶𝐷] = [𝐴𝐵𝐶]
4
1 1
[𝐾𝐷𝐸] = [𝐵𝐶𝐷] = [𝐴𝐵𝐶]
4 16

1 1 1
+ + +⋯
4 16 64

1 1
1 1 4 1
𝑎 = ,𝑟 = ⇒ =4=
4 4 1 3 3
1−
4 4

Example 3.149
Consider a square with side length 1.
Step 1: Draw the diagonals of a square with side length 1, dividing the square into four triangles, which have
vertices at the intersection of the diagonals.
Step 2: Draw the altitude for the triangle facing downward, dividing it into two triangles, hence reaching 𝑃2 .
Step 3: Select one of the two triangles from Step 2, and draw the altitude from 𝑃2 , reaching 𝑃3 .

Steps 1, 2 and 3 are illustrated above.


Continue this process till you complete Step 2021, each time drawing the altitude from 𝑃𝑛 to reach 𝑃𝑛+1

A. What is the length of the line drawn in Step 2021?


B. What is the length of all the lines drawn from Step 1 onwards?
The four equal triangles formed in Step 1 are 𝑇1 , 𝑇2 , 𝑇3 , 𝑇4 . Step 2, gives us two equal triangles: 𝑇5 , 𝑇6 . Step 3 gives
us two equal triangles: 𝑇7 , 𝑇8 . And so on…..From Step 1 to Step 2021, pick exactly one triangle from the triangles
created at each step.
C. Find the total area of all triangles picked.

P a g e 144 | 274
Get all the files at: https://bit.ly/azizhandouts
Aziz Manva (azizmanva@gmail.com)

D. What is the perimeter of all triangles picked?

Part A
Let the length of the line in Step 𝑛 be 𝐿𝑛
1 1 1 1 1 1 1
𝐿1 = 2√2, 𝐿2 = , 𝐿3 = × = , 𝐿4 = =
2 2 √2 2√2 2√2 √2 4
If you ignore Step 1, you get a geometric sequence with:
1 1
𝑎= , 𝑟=
2 √2
We want to find the length of Line in Step 2021, which is the 2020th Term.
1 1
Substitute 𝑎 = 2 , 𝑟 = 2 , 𝑛 = 2020 in the formula for the 𝑛𝑡ℎ term:

1 1 2019 1 1 1
𝑎𝑟 𝑛−1 = ( ) ( ) = 2 2019 = 2020 =
2 √2 (√2) (√2) (√2)(√2) √2 × 21010
Part B
1 1
Substitute 𝑎 = 2 , 𝑟 = ,𝑛 = 2020 in the formula for the sum to 𝑛 terms:
√2
1 1 2020 1010
𝑛) 2 (1 − ( ) ) 1 (1 − 1 ) 1 (2 −1
)
𝑎(1 − 𝑟 √2 2 2 1010 2 21010 21010 − 1 √2
= = = = ( 1011 ) ×
1−𝑟 1 √2 − 1 √2 − 1 2 √2 − 1
1−
√2 √2 √2
Rationalize the denominator:
21010 − 1 √2 √2 + 1 21010 − 1
= ( 1011 ) × × = (2 + √2) ( 1011 )
2 √2 − 1 √2 + 1 2
And the hence the final answer is:
21010 − 1
2√2 + (2 + √2) ( 1011 )
2
Part C
1 1 1 1 1 1 1 1
[𝑇4 ] = [𝑆𝑞𝑢𝑎𝑟𝑒] = , [𝑇5 ] = [𝑇2 ] = , [𝑇7 ] = [𝑇5 ] = × =
4 4 2 8 2 2 8 16
The above is a geometric sequence.
1 1
Substitute 𝑎 = 4 , 𝑟 = 2 , 𝑛 = 2021:

P a g e 145 | 274
Get all the files at: https://bit.ly/azizhandouts
Aziz Manva (azizmanva@gmail.com)

1 1 2021
4 (1 − (2) ) 1 (1 − 1 )
𝑎(1 − 𝑟 𝑛 ) 4 22021 = 1 (2
2021
−1 22021 − 1
𝑆= = = ) =
1−𝑟 1 1 2 22021 22022
1−2 2
Part D
1 1
Substitute 𝑎 = , 𝑟 = , 𝑛 = 2021:
4 2
1 1 2021
𝑛) 4 (1 − (2) ) 1 (1 − 1 )
𝑎(1 − 𝑟 4 2 2021 1 22021 − 1 22021 − 1
𝑆= = = = ( 2021 ) =
1−𝑟 1 1 2 2 22022
1−2 2

Example 3.150
30-60-90 Triangles

Example 3.151: Midpoint of a Square


Start with a square (call it 𝑆1 ) with side length 𝑥. 𝑆2 is a square
formed by joining the midpoints of 𝑆1 . 𝑆3 is a square formed by
joining the midpoints of 𝑆2 , and so on.
Let 𝐴1 be the area of 𝑆1 , 𝐴2 be the area of 𝑆2 , and so on.
A. Find the nature of the sequence 𝐴1 , 𝐴2 , 𝐴3 , …
B. Find the definition of the sequence.

Black Square to Blue Square


The side length of 𝑆1 is 𝑥.
Hence, the midpoint of any side of 𝑆1 will divide it into two segments,
𝑥
each of length .
2

By the property of 45° − 45° − 90° triangle, the hypotenuse is √2 times the length of the legs.
1 √2
𝐻𝑦𝑝 = √2 × 𝑥 = 𝑥
2 2
Area of the inner square
√2 2 1
= 𝑆𝑖𝑑𝑒 2 = 𝑥= 𝑥= 𝑥
2 4 2
The ratio of area is
1
𝐴1 : 𝐴2 = 𝑥: 𝑥 = 2: 1
2

Blue Square to Red Square


Using similar calculations, it can be shown that
𝐴𝑛 : 𝐴𝑛−1 = 2: 1
1
Hence, the sequence is geometric, and the common ratio is 2.

Example 3.152
Square 𝑆1 is 1 × 1. For 𝑖 ≥ 1, the lengths of the sides of square 𝑆𝑖+1 are half the lengths of the sides of square 𝑆𝑖 ,
two adjacent sides of square 𝑆𝑖 are perpendicular bisectors of two adjacent sides of square 𝑆𝑖+1 , and the other

P a g e 146 | 274
Get all the files at: https://bit.ly/azizhandouts
Aziz Manva (azizmanva@gmail.com)

two sides of square 𝑆𝑖+1 , are the perpendicular bisectors of two adjacent sides of square 𝑆𝑖+2 . The total area
𝑚
enclosed by at least one of 𝑆1 , 𝑆2 , 𝑆3 , 𝑆4 , 𝑆5 can be written in the form 𝑛 , where 𝑚 and 𝑛 are relatively prime
positive integers. Find 𝑚 − 𝑛. (AIME 1995/1)
(Ans = 255)

Example 3.153
A. Sectors in a Circle
B. One hundred concentric circles with radii 1, 2, 3, . . . ,100 are drawn in a plane. The interior of the circle
of radius 1 is colored red, and each region bounded by consecutive circles is colored either red or green,
with no two adjacent regions the same color. The ratio of the total area of the green regions to the area
𝑚
of the circle of radius 100 can be expressed as , where 𝑚 and 𝑛 are relatively prime positive integers.
𝑛
Find 𝑚 + 𝑛. (AIME 2003/I/2)

(Ans = 301)

Example 3.154
A saint has a magic pot. He puts one gold ball of radius 1 mm daily inside it for 10 days. If the weight of the first
ball is 1 gm, and if the radius of a ball inside the pot doubles every day, how much gold has the saint made due
to his magic pot? (JMET 2011/82)

Radius of balls on 10th Day:


𝑅𝑎𝑑𝑖𝑢𝑠: ⏟0
2 , ⏟1
2 , 22 , … , ⏟9
2
𝐵𝑎𝑙𝑙 𝐴𝑑𝑑𝑒𝑑: 𝐵𝑎𝑙𝑙 𝐴𝑑𝑑𝑒𝑑: 𝐵𝑎𝑙𝑙 𝐴𝑑𝑑𝑒𝑑:
10𝑡ℎ 𝐷𝑎𝑦 9𝑡ℎ 𝐷𝑎𝑦 𝐹𝑖𝑟𝑠𝑡 𝐷𝑎𝑦

By similarity, the volume of balls is proportional to the cube of the radius, and so is the weight:
𝑊𝑒𝑖𝑔ℎ𝑡: ⏟1 , ⏟(21 )3 , (22 )3 , … , ⏟
(29 )3
𝐵𝑎𝑙𝑙 𝐴𝑑𝑑𝑒𝑑: 𝐵𝑎𝑙𝑙 𝐴𝑑𝑑𝑒𝑑: 𝐵𝑎𝑙𝑙 𝐴𝑑𝑑𝑒𝑑:
10𝑡ℎ 𝐷𝑎𝑦 9𝑡ℎ 𝐷𝑎𝑦 𝐹𝑖𝑟𝑠𝑡 𝐷𝑎𝑦
Total Weight
= 20 + 23 + 26 + ⋯ + 227
3
Substitute 𝑎 = 1, 𝑟 = 2 , 𝑛 = 10 in
𝑎(𝑟 𝑛 − 1) 1((23 )10 − 1) 230 − 1
𝑆𝑛 = = =
𝑟−1 23 − 1 7
Gain in Gold:
230 − 1 230 − 71
= − 10⏟ = 𝑔𝑚
⏟ 7 𝑂𝑟𝑖𝑔𝑖𝑛𝑎𝑙
7
𝐹𝑖𝑛𝑎𝑙 𝐺𝑜𝑙𝑑 𝐺𝑜𝑙𝑑

E. Applications in Finance4 (Optional)

3.155: Present Value


The present value of 𝐷 dollars received n time periods from now considering a discount rate of 𝑟 is
𝐷
(1 + 𝑟)𝑛

Example 3.156
𝑟 = 0.09

4 This section is optional for school level competitions.

P a g e 147 | 274
Get all the files at: https://bit.ly/azizhandouts
Aziz Manva (azizmanva@gmail.com)

𝐶𝑜𝑛𝑡𝑟𝑎𝑐𝑡 𝑉𝑎𝑙𝑢𝑒 = $121,000,000


𝐶𝑜𝑛𝑡𝑟𝑎𝑐𝑡 𝑃𝑒𝑟𝑖𝑜𝑑 = 7 𝑌𝑒𝑎𝑟𝑠
121,000,000
𝐸𝑎𝑐ℎ 𝑝𝑎𝑦𝑚𝑒𝑛𝑡 = 𝑎 = = 17,285,714.29
7
𝑎 𝑎 𝑎
𝑆=𝑎+ + 2
+ ⋯+
1.09 1.09 1.096

But note that the above is a finite geometric series with


1
𝐹𝑖𝑟𝑠𝑡 𝑡𝑒𝑟𝑚 = 𝑎, 𝑟= , 𝑛=7
1.09
Substitute the above in the formula for the sum of a finite geometric series:
𝑎(𝑟 𝑛 − 1) 1.096 − 1
𝑆= = 17,285,714.29 ( )=
𝑟−1 1.09 − 1

3.157: Annuity
An annuity is a series of payments, each of same dollar value. The present value of an annuity of 𝐷 dollars,
received for 𝑛 periods, with the first payment received after one time period is
𝐷 × 𝐴𝐹
Where
1 𝑛
1 − (1 + 𝑖 )
𝐴𝐹 = 𝐴𝑛𝑛𝑢𝑖𝑡𝑦 𝐹𝑎𝑐𝑡𝑜𝑟 =
𝑖

Time 0 1 2 3 . . . 𝑛
Period
Payment D D D . . . D
Present 𝐷 𝐷 𝐷 . . . 𝐷
Value 1+𝑖 (1 + 𝑖)2 (1 + 𝑖)3 (1 + 𝑖)𝑛

The sum of the present values is a geometric series with


𝐷 1
𝑎= , 𝑟=
1+𝑖 1+𝑖

Use the formula for the sum of a geometric series:


𝐷 1 𝑛 𝐷 1 𝑛 1 𝑛
𝑎(1 − 𝑟 𝑛) ( 1+𝑖 ) [1 − ( 1+𝑖 ) ] (1+𝑖 ) [1 − (1+𝑖 ) ] 𝐷 [1 − ( 1 + 𝑖) ]
𝑆= = = =
1−𝑟 1 𝑖 𝑖
1−1+𝑖 1+𝑖

Example 3.158
You are 30 years old and have $150,000 in your retirement account. You want to save a constant amount every
year, so that you will have a total of $1.5 million in your retirement account in 20 years from today. Assume that
the rate of return in the retirement account is 4.5% per year and that the deposits into the account are made at
the end of each year (i.e. the first deposit will be made in a year from now). How much would you need to
deposit each year to meet your retirement goal?

Discount all amount to present time.

Required Amount in Retirement Account

P a g e 148 | 274
Get all the files at: https://bit.ly/azizhandouts
Aziz Manva (azizmanva@gmail.com)

1,500,000
= = 621,964.28
(1 + 0.045)20
Money to be saved
= 621,964.28 − 150,000 = 471,964.28

This must be the value of the annuity that your deposits add up to. Calculate the annuity factor with 𝑖 =
4.5% , 𝑛 = 20:
1 𝑛 1 20
1 − (1 + 𝑖 ) 1−( )
𝐴𝑛𝑛𝑢𝑖𝑡𝑦 𝐹𝑎𝑐𝑡𝑜𝑟 = = 1.045 = 13
𝑖 0.045
IF you were to invest $1 every year, for 20 years, then at a discounting factor of 4.5% per year, the present value
of the stream of payments would be $13.
And hence the required yearly deposit is:
471964.28
471,964.28 = 𝐷(13) ⇒ 𝐷 = =
13

3.159: Perpetuity
A perpetuity is an annuity that goes on forever. The present value of a perpetuity of 𝐷 dollars with the first
payment received after one time period is
𝐷
𝑖

Start with the formula for an annuity


1 𝑛
𝐷 [1 − (1 + 𝑖 ) ]
𝑖
1 𝑛
And note that as 𝑛 → ∞, (1+𝑖) → 0, ad hence the expression simplifies to:
𝐷
𝑖

3.160: EMI
𝐴0 (𝑟)(1 + 𝑟)𝑛
𝐸𝑀𝐼 =
(1 + 𝑟)𝑛 − 1

Example 3.161
If 𝑃 = 87.17, 𝑟 = 7%, 𝐴0 = $1000 then find 𝑛

1000(0.007)(1 + 0.007)𝑛
87.17 =
(1 + 0.007)𝑛 − 1
87.17𝑛
Example 3.162
𝑟 = 0.003, 𝐴0 = 10,000, 𝑛 = 60

𝐴0 (𝑟)(1 + 𝑟)𝑛 (10,000)(0.003)(1.003)60


𝐸𝑀𝐼 = = =
(1 + 𝑟)𝑛 − 1 (1.003)60 − 1

P a g e 149 | 274
Get all the files at: https://bit.ly/azizhandouts
Aziz Manva (azizmanva@gmail.com)

4. FURTHER TOPICS
4.1 Summation Notation
A. Basics
Σ is the capital Greek letter sigma. We use it to write summation notation.

4.1: Sigma Notation


Sigma notation is a compact notation used to write the sum of a series.

10

∑𝑥
𝑥=1
𝐿𝑜𝑤𝑒𝑟 𝐿𝑖𝑚𝑖𝑡 𝑜𝑓 𝑆𝑢𝑚𝑚𝑎𝑡𝑖𝑜𝑛: 𝑥 = 1
𝑈𝑝𝑝𝑒𝑟 𝐿𝑖𝑚𝑖𝑡 𝑜𝑓 𝑆𝑢𝑚𝑚𝑎𝑡𝑖𝑜𝑛: 𝑥 = 10

To evaluate an expression written in sigma notation, you replace the given variable by its first value, then its
second value, and so on.
10

∑𝑥 = ⏟
1 + ⏟
2 + ⏟
3 + ⋯ + 10

𝑥=1 𝑥=1 𝑥=2 𝑥=3 𝑥=10

𝑁𝑜. 𝑜𝑓 𝑇𝑒𝑟𝑚𝑠 = 10
Notes:
➢ In this case, the number of terms is equal to the upper limit, but that need not always be the case.
➢ The variable used for summation is called a 𝑑𝑢𝑚𝑚𝑦 𝑣𝑎𝑟𝑖𝑎𝑏𝑙𝑒 because when you substitute the limits of
summation, the variable disappears. You can use any letter for the dummy variable, and the value of the
expression does not change.
10 10

∑ 𝑥 = ∑ 𝑛 = 1 + 2 + 3 + ⋯ + 10
𝑥=1 𝑛=1

Example 4.2: Sum of Natural Numbers


Write the summations below in summation notation
A. 1 + 2 + 3 + ⋯ + 100
B. 1 + 2 + 3 + ⋯ + 𝑥

Part A
100

∑𝑛
𝑛=1

Part B
𝑥

∑𝑛
𝑛=1

4.3: Alternating Series


An alternating series is a series whose terms alternate between positive and negative terms.

P a g e 150 | 274
Get all the files at: https://bit.ly/azizhandouts
Aziz Manva (azizmanva@gmail.com)

Note that:
𝑛 𝑖𝑠 𝑎𝑛 𝑜𝑑𝑑 𝑖𝑛𝑡𝑒𝑔𝑒𝑟 ⇒ (−1)𝑛 = −1
𝑛 𝑖𝑠 𝑎𝑛 𝑒𝑣𝑒𝑛 𝑖𝑛𝑡𝑒𝑔𝑒𝑟 ⇒ (−1)𝑛 = 1
Hence, we can use the parity of powers of (−1) to make the terms of a series alternate between negative and
positive.

Example 4.4
Write the expressions below in summation notation:
A. −1 + 2 − 3 + 4 + ⋯ + 100
B. 1 − 2 + 3 − 4 + ⋯ − 100
1 3 5 97
C. − 1 + − 2 + + ⋯+
2 2 2 2

100

∑(−1)𝑛 𝑛 = −1 + 2 − 3 + ⋯ + 100
𝑛=1

100

∑(−1)𝑛+1 𝑛 = 1 − 2 + 3 − 4 + ⋯ − 100
𝑛=1

97
1 2 3 4 5 97 1 𝑛
− + − + + ⋯+ = (1 − 2 + 3 − 4 + ⋯ + 97) = ∑(−1)𝑛+1
2 2 2 2 2 2 2 2
𝑛=1

Example 4.5
Expand
50 50

∑(1 − 2𝑛) + ∑ 2𝑛
𝑛=1 𝑛=1

−1 − 3 − 5 − ⋯ − 99 + 2 + 4 + 8 + ⋯ + 100

4.6: Multiplication
If you can recognize multiplication in the series, it will naturally translate into multiplication in the summation
notation.

Example 4.7
Write the expressions below in summation notation:
2 + 4 + 6 + ⋯ + 28

14

2 + 4 + 6 + ⋯ + 28 = ∑ 2𝑥
𝑥=1

4.8: Summation independent of the variable


𝑘

∑ 𝑥 = 𝑘𝑥
𝑛=1

P a g e 151 | 274
Get all the files at: https://bit.ly/azizhandouts
Aziz Manva (azizmanva@gmail.com)

∑𝑥 = ⏟
𝑥 + 𝑥 + ⋯ + 𝑥 = 𝑘𝑥
𝑛=1 𝑘 𝑡𝑖𝑚𝑒𝑠

Example 4.9
14

𝐸𝑣𝑎𝑙𝑢𝑎𝑡𝑒 ∑ 𝑥
𝑛=1

𝑛=1⇒𝑥
𝑛=2⇒𝑥
𝑛=3⇒𝑥
.
.
.
𝑛 = 14 ⇒ 𝑥


𝑥 + ⏟
𝑥 + ⏟
𝑥 + ⋯+ ⏟
𝑥 =⏟
𝑥 + 𝑥 + ⋯ + 𝑥 = 14𝑥
𝑛=1 𝑛=2 𝑛=3 𝑛=14 14 𝑡𝑖𝑚𝑒𝑠

Example 4.10
A grocery store clerk gets paid 𝑥 dollars for every hour that she works. Last week, she worked for 14 hours.
Write her total remuneration in summation notation, and then evaluate the expression that you wrote.

Let the salary of the grocery store clerk for the 𝑖 𝑡ℎ hour be ℎ𝑖 :
𝑆𝑎𝑙𝑎𝑟𝑦 𝑓𝑜𝑟 1𝑠𝑡 ℎ𝑜𝑢𝑟 = ℎ1
𝑆𝑎𝑙𝑎𝑟𝑦 𝑓𝑜𝑟 2𝑛𝑑 ℎ𝑜𝑢𝑟 = ℎ2
𝑆𝑎𝑙𝑎𝑟𝑦 𝑓𝑜𝑟 𝑖 𝑡ℎ ℎ𝑜𝑢𝑟 = ℎ𝑖
14

𝑇𝑜𝑡𝑎𝑙 𝑆𝑎𝑙𝑎𝑟𝑦 = ℎ1 + ℎ2 + ℎ3 + ⋯ + ℎ14 = ∑ ℎ𝑖


𝑖=1

But note that the salary for each hour:


= ℎ1 = ℎ2 = ℎ𝑖 = 𝑥
Substitute ℎ𝑖 = 𝑥
14

∑𝑥 = ⏟
𝑥 + ⏟
𝑥 + ⏟
𝑥 + ⋯+ ⏟
𝑥 =⏟
𝑥 + 𝑥 + ⋯ + 𝑥 = 14𝑥
𝑖=1 𝑖=1 𝑖=2 𝑖=3 𝑖=14 14 𝑡𝑖𝑚𝑒𝑠

4.11: Property I: Constant Property


A constant can be moved out of, or into the summation sign without changing its value.
𝑛 𝑛

∑ 𝑐𝑥 = 𝑐 ∑ 𝑥
𝑥=1 𝑥=1

Expand ∑𝑛𝑥=1 𝑐𝑥 using the definition:


= 𝑐 + 2𝑐 + ⋯ + 𝑛𝑐
Factor 𝑐 from each term in the expression:
𝑐(1 + 2 + ⋯ + 𝑛)
Write the second term as a summation:

P a g e 152 | 274
Get all the files at: https://bit.ly/azizhandouts
Aziz Manva (azizmanva@gmail.com)
𝑛

= 𝑐∑𝑥
𝑥=1

Example 4.12
𝑥 − 2𝑥 + 3𝑥 − 4𝑥 + ⋯ + 𝑘𝑥, 𝑘 𝑖𝑠 𝑜𝑑𝑑

100

∑(−1)𝑛+1 𝑛 = 1 − 2 + 3 − 4 + ⋯ − 100
𝑛=1
Multiply both sides of the above by 𝑥:
100

𝑥 (∑(−1)𝑛+1 𝑛) = 𝑥(1 − 2 + 3 − 4 + ⋯ − 100)


𝑛=1
On the LHS, carry out the multiplication. On the RHS, move the constant inside the multiplication sign:
𝑛=𝑘

∑(−1)𝑛+1 𝑛𝑥 = 𝑥 − 2𝑥 + 3𝑥 − 4𝑥 + ⋯ + 𝑘𝑥, 𝑘 𝑖𝑠 𝑜𝑑𝑑


𝑛=1

4.13: Index of Summation


It is not necessary that the index of summation starts with 1 or 0. It can start with any number that you want.

Example 4.14
A. 3 + 4 + ⋯ + 17
B. 50 − 51 + 52 − 53 + ⋯ + 104
C. 4 + 8 + 12 + ⋯ + 396
25 30 35 40 175
D. 3
− 3
+ 3
− 3 +⋯+ 3

17

3 + 4 + ⋯ + 17 = ∑ 𝑥
𝑥=3
104 55

50 − 51 + 52 − 53 + ⋯ + 104 = ∑ (−1)𝑥 𝑥 = ∑(−1)𝑥+1 (𝑥 + 49)


𝑥=50 𝑥=1
4 + 8 + 12 + ⋯ + 396 = 4( )

4.15: Reindexing
Using a change of variable, you can change the index of summation of a series without changing the value of the
expression. This is called reindexing.

Example 4.16
Write each of the above series using 𝑛 = 1 as the lower limit of summation:
A. 3 + 4 + ⋯ + 17
B. 50 − 51 + 52 − 53 + ⋯ + 104
C. 4 + 8 + 12 + ⋯ + 396
25 30 35 40 175
D. − + − +⋯+
3 3 3 3 3

17

3 + 4 + ⋯ + 17 = ∑ 𝑥
𝑥=3

P a g e 153 | 274
Get all the files at: https://bit.ly/azizhandouts
Aziz Manva (azizmanva@gmail.com)

Use a change of variable. Let 𝑥 = 𝑛 + 2 ⇒ 𝑛 = 𝑥 − 2:


𝑛+2=17 𝑛=15

= ∑ 𝑛+2= ∑ 𝑛+2
𝑛+2=3 𝑛=1

Since 𝑥 is a dummy variable, we substitute 𝑛 = 𝑥 without changing the value of the expression:
𝑛=15 𝑥=15

∑ 𝑛+2= ∑ 𝑥+2
𝑛=1 𝑥=1
Hence, finally, we can say that:
17 𝑥=15

∑𝑥 = ∑ 𝑥+2
𝑥=3 𝑥=1

Example 4.17: Changing the index of Multiplication


In the questions below, do not change the value of the final sum

𝑛 𝑛−5

Change ∑ 𝑖 to start from 𝑖 = 0 ⇒ ∑ 𝑖 + 5


𝑖=5 𝑖=0
𝑛 𝑛−9 𝑛−9

Change ∑ 2𝑖 + 4 to start from 𝑖 = −2 ⇒ ∑ 2(𝑖 − 9) + 4 = ∑ 2𝑖 − 14


𝑖=7 𝑖=−2 𝑖=−2

Example 4.18: Arithmetic Series


Write the following series in sigma notation
A. 1 + 3 + ⋯ + 21
B. 5 + 7 + ⋯ + 101
C. 3 − 6 + 9 − ⋯ + 81
D. 4 − 7 + 10 − ⋯ + 82

Part A
10

∑ 2𝑛 + 1
𝑛=0
1 + 3 + ⋯ + 21
Each term of the series form part of an arithmetic sequence with
𝑓𝑖𝑟𝑠𝑡 𝑡𝑒𝑟𝑚 = 𝑎 = 1, 𝑐𝑜𝑚𝑚𝑜𝑛 𝑑𝑖𝑓𝑓𝑒𝑟𝑒𝑛𝑐𝑒 = 𝑑 = 2

The 𝑛𝑡ℎ term of the sequence is given by:


1 + (𝑛 − 1)2 = 1 + 2𝑛 − 2 = 2𝑛 − 1
Which we can convert into sigma notation as:
11

∑ 2𝑛 − 1
𝑛=1
Part B
When the numbers do not start from 1, the lower limit of summation will generally not start from one:
48

5 + 7 + ⋯ + 101 = ∑ 2𝑥 + 5
𝑥=0

P a g e 154 | 274
Get all the files at: https://bit.ly/azizhandouts
Aziz Manva (azizmanva@gmail.com)
𝑛=20

∑ (𝑛 + 1) , 𝑛 ∈ 2𝑘, 𝑘 ∈ 𝕎
𝑛=0
𝑘 = 0 ⇒ 𝑛 = 2(0) = 0 ⇒ 𝑛 + 1 = 0 + 1 = 1
𝑘 = 1 ⇒ 𝑛 = 2(1) = 2 ⇒ 𝑛 + 1 = 2 + 1 = 3
.
.
.
𝑘 = 10 ⇒ 𝑛 = 2(10) = 20 ⇒ 𝑛 + 1 = 20 + 1 = 21

27

3 − 6 + 9 − ⋯ + 81 = ∑(−1)𝑥−1 3𝑥
𝑥=1

27

4 − 7 + 10 − ⋯ + 82 = ∑(−1)𝑥−1 (3𝑥 + 1)
𝑥=1

Example 4.19: Geometric Series


Write the following series in sigma notation
A. 3 + 6 + 12 + 24
B. 2 + 6 + 18 + 54 + ⋯ + 1458
1 1 1 1
C. 1 − 2 + 4 − 8 + 16

Part A
𝑛=3

= 3(2 + 2 + 2 + 2 ) = 3 ∑ 2𝑛
0 1 2 3

𝑛=0
Part B
𝑛=6
0 1 2 3 6)
= 2(3 + 3 + 3 + 3 + ⋯ + 3 = 2 ∑ 3𝑛
𝑛=0

B. Evaluating Sigma Notation

4.20: Arithmetic Series


The sum of 𝑛 terms of an arithmetic series with 𝑓𝑖𝑟𝑠𝑡 𝑡𝑒𝑟𝑚 = 𝑎, and 𝑐𝑜𝑚𝑚𝑜𝑛 𝑑𝑖𝑓𝑓𝑒𝑟𝑒𝑛𝑐𝑒 = 𝑑 is given by:
𝑛
𝑛
∑ 𝑎 + (𝑘 − 1)𝑑 = [2𝑎 + (𝑛 − 1)𝑑]
2
𝑘=1

4.21: Geometric Series


The sum of n terms of a geometric series with 𝑓𝑖𝑟𝑠𝑡 𝑡𝑒𝑟𝑚 = 𝑎, and 𝑐𝑜𝑚𝑚𝑜𝑛 𝑟𝑎𝑡𝑖𝑜 = 𝑟 is given by:
𝑛
𝑎(𝑟 𝑛 − 1)
∑ 𝑎𝑟 𝑥−1 =
𝑟−1
𝑥=1

Example 4.22

P a g e 155 | 274
Get all the files at: https://bit.ly/azizhandouts
Aziz Manva (azizmanva@gmail.com)

Determine the sum of the following summation:


20

∑ 32𝑖
𝑖=0

20

∑ 32𝑖 = 32(0) + 32(1) + ⋯ + 32(20) = 30 + 32 + 34 + ⋯ + 320


𝑖=0

Geometric Series with 𝑎 = 1, 𝑟 = 9, 𝑛 = 21:


𝑎(𝑟 𝑛 − 1) 1(921 − 1) 342 − 1
𝑆21 = = =
𝑟−1 9−1 8

Example 4.23: Geometric Series


Find the value of 𝑋, 𝑌 and 𝑍 given that:
4 7 30
𝑥 𝑥
𝑋 = ∑2 , 𝑌 = ∑2 , 𝑍 = ∑ 2𝑥
𝑥=1 𝑥=4 𝑥=4

Part A
4

∑ 2𝑥 = 21 + 22 + 23 + 24 = 2 + 4 + 8 + 16 = 30
𝑥=1
Part B
7

∑ 2𝑥 = 24 + 25 + 26 + 27 = 16 + 32 + 64 + 128 = 240
𝑥=4
This is a geometric series with 𝐹𝑖𝑟𝑠𝑡 𝑇𝑒𝑟𝑚 = 𝑎 = 24 = 16, 𝐶𝑜𝑚𝑚𝑜𝑛 𝑅𝑎𝑡𝑖𝑜 = 𝑟 = 2, 𝑁𝑢𝑚𝑏𝑒𝑟 𝑜𝑓 𝑇𝑒𝑟𝑚𝑠 = 𝑛 = 4
The sum of this series is:
𝑎(𝑟 𝑛 − 1) 24 (24 − 1) 16(15)
= = = = 240
𝑟−1 2−1 1
Part C
30

∑ 2𝑥 = 24 + 25 + ⋯ + 230
𝑥=4
This is a geometric series with 𝐹𝑖𝑟𝑠𝑡 𝑇𝑒𝑟𝑚 = 𝑎 = 24 = 16, 𝐶𝑜𝑚𝑚𝑜𝑛 𝑅𝑎𝑡𝑖𝑜 = 𝑟 = 2, 𝑁𝑢𝑚𝑏𝑒𝑟 𝑜𝑓 𝑇𝑒𝑟𝑚𝑠 = 𝑛 =
30 − 3 = 27
The sum of this series is:
𝑎(𝑟 𝑛 − 1) 24 (227 − 1)
= = = 24 (227 − 1)
𝑟−1 2−1

4.24: Property II: Distribution of Summation over an Expression


You can distribute the summation operator over individual terms (for addition or subtraction)
𝑛 𝑛 𝑛

∑𝑥 + 𝑦 = ∑𝑥 + ∑𝑦
𝑖=1 𝑖=1 𝑖=1
𝑛 𝑛 𝑛

∑𝑥 − 𝑦 = ∑𝑥 − ∑𝑦
𝑖=1 𝑖=1 𝑖=1

P a g e 156 | 274
Get all the files at: https://bit.ly/azizhandouts
Aziz Manva (azizmanva@gmail.com)

4.25: Sum of Natural Numbers


𝑛
𝑛(𝑛 + 1)
∑𝑘 =
2
𝑥=1

And this formula also works when 𝑛 is odd, since


➢ This is the sum of the first 𝑛 natural the remaining number has value exactly half of the
numbers. value of each pair. Hence, we can think of the
➢ It generates the triangular numbers. remaining number as representing half a pair.
1+2+3+4+5
Proof I: By forming Pairs 5+1=6
1 + 2 + ⋯+ 𝑛 2+4=6
𝑛 + (𝑛 − 1) + ⋯ + 1 6
3=
2
Make pairs. Each pair has sum 𝑛 + 1. And there are Proof II: By forming Pairs
𝑛 Substitute 𝑓 = 𝐹𝑖𝑟𝑠𝑡 𝑇𝑒𝑟𝑚 = 1, 𝑙 = 𝐿𝑎𝑠𝑡 𝑇𝑒𝑟𝑚 =
2
pairs, giving us a sum of:
𝑛 𝑛, 𝑛 = 𝑁𝑜. 𝑜𝑓 𝑡𝑒𝑟𝑚𝑠 in:
(𝑛 + 1)
⏟ 𝑓+𝑙 𝑛+1

2 𝑆𝑢𝑚 𝑜𝑓 𝑆𝑛 = 𝑛 × ( )=𝑛×( )
𝑁𝑜.𝑜𝑓
𝑒𝑎𝑐ℎ 𝑝𝑎𝑖𝑟
2 2
𝑃𝑎𝑖𝑟𝑠

Example 4.26
What is the sum of all integers from 80 through 90, inclusive? (MathCounts 2004 State Sprint)

Method I: Without Summation Notation 𝑥=10

80 + 81 + 82 + ⋯ + 90 = ∑ 80 + 𝑥
= 80 + (80 + 1) + (80 + 2) + ⋯ + 90 𝑥=0
= (11)(80) + [0 + 1 + 2 + ⋯ + 10] Split the sum:
𝑥=10 𝑥=10
= 880 + 55 = 935
= ∑ 80 + ∑ 𝑥
𝑥=0 𝑥=0
Method I: With Summation Notation
𝑥=90 Evaluate each sum:
10 × 11
∑ 𝑥 = (11)(800) + ( )
2
𝑥=80
Reindex the sum: = 880 + 55
= 935

4.27: Sum of Squares of Natural Numbers


𝑛
𝑛(𝑛 + 1)(2𝑛 + 1)
∑ 𝑘2 =
6
𝑥=1

Example 4.28
10

If 𝑢𝑖 = −3 + 4𝑖, and 𝑣𝑖 = 12 − 3𝑖, find ∑ 𝑢𝑖 𝑣𝑖


𝑖=1

Substitute the values of 𝑢𝑖 and 𝑣𝑖 , and carry out the multiplication:

P a g e 157 | 274
Get all the files at: https://bit.ly/azizhandouts
Aziz Manva (azizmanva@gmail.com)
10 10 10 10

∑ 𝑢𝑖 𝑣𝑖 = ∑(−3 + 4𝑖)(12 − 3𝑖) = ∑ −36 + 48𝑖 + 9𝑖 − 12𝑖 = ∑ −36 + 57𝑖 − 12𝑖 2 2

𝑖=1 𝑖=1 𝑖=1 𝑖=1

Break up each summation term, and move the constant out of the summation:
10 10 10 10 10

∑ −36 + ∑ 57𝑖 + ∑ −12𝑖 = −360 + 57 ∑ 𝑖 − 12 ∑ 𝑖 2


2

𝑖=1 𝑖=1 𝑖=1 𝑖=1 𝑖=1


𝑛(𝑛+1)
Use the formula 1 + 2 + 3 + ⋯ + 𝑛 = 2 to evaluate the sum of the first 𝑛 natural numbers:
10
10 × 11 5 × 11
57 ∑ 𝑖 = 57 × = 57 × = 57 × 55 = 3135
2 1
𝑖=1

𝑛(𝑛+1)(2𝑛+1)
Use the formula 1 + 2 + ⋯ + 𝑛2 = 6
to evaluate the sum of the squares of the first 𝑛 natural numbers:
10
(10)(11)(21)
−12 ∑ 𝑖 2 = (−12) = 4620
6
𝑖=1
= −360 + 3135 − 4620 = −1845

C. Averages
Summation notation can be used in many different parts of Maths. We look at some applications.

4.29: Arithmetic Mean


The arithmetic mean of a set of 𝑛 values {𝑥1 , 𝑥2 , … , 𝑥𝑛 } is
𝑖=𝑛
𝑥1 + 𝑥2 + ⋯ + 𝑥𝑛 1
𝑥̅ = = ∑ 𝑥𝑖
𝑛 𝑛
𝑖=1

Note that
➢ The average of {𝑥1 , 𝑥2 , … , 𝑥𝑛 } is generally written as 𝑥̅ .
➢ We can write the summation of 𝑥 values in a number of different ways:
𝑖=𝑛

∑ 𝑥𝑖 = ∑ 𝑥𝑖 = ∑ 𝑥𝑖
𝑖 𝑖=1

Example 4.30
The average weight of 14 students and their teacher in a class is 60 kg.
A. Find the sum of the weights of all 15 people in the class.
B. If the teacher weighs 65 kg, find the weight of all the students.

Part A
1
∑ 𝑥 = 60 ⇒ ∑ 𝑥 = 15 × 60 = 900
15
Part B
(∑ 𝑥) − 𝑥𝑇𝑒𝑎𝑐ℎ𝑒𝑟 = 900 − 65 = 935

4.31: Change of Scale

P a g e 158 | 274
Get all the files at: https://bit.ly/azizhandouts
Aziz Manva (azizmanva@gmail.com)

Example 4.32
Show that the average of a set of numbers is invariant to change of scale

𝑛
Consider the numbers: 𝑥1 + 𝑥2 + ⋯ + 𝑥𝑛
∑ 𝑥𝑖
𝑖=1
Divide each number by 𝑞. (Change 𝑥1 𝑥2 𝑥𝑛 𝑛
+ + ⋯+ 1
of scale) 𝑞 𝑞 𝑞 ∑ 𝑥𝑖
𝑞
𝑖=1
𝑥1 + 𝑥2 + ⋯ + 𝑥𝑛 𝑛
Find the average 1
𝑛𝑞 ∑ 𝑥𝑖
𝑛𝑞
𝑖=1
𝑥1 + 𝑥2 + ⋯ + 𝑥𝑛 𝑛
Multiply the average by 𝑞: 1
(Reverse change of scale) 𝑛 ∑ 𝑥𝑖
𝑛
𝑖=1

Example 4.33
Find the average of 𝑥1 , 𝑥2 , 𝑥3 , 𝑥4 , 𝑥5 given that 𝑥𝑖 = 1000 + 𝑖.

Method I: Work with the numbers


1001 + 1002 + 1003 + 1004 + 1005
5
1000 + 1 + 1000 + 2 + 1000 + 3 + 1000 + 4 + 1000 + 5
=
5
5 × 1000 + 1 + 2 + 3 + 4 + 5
=
5
15
= 1000 +
5
= 1003
Method II: Use a change of variable
Define a new variable
𝑋𝑖 = 𝑥𝑖 − 1000 = 1000 + 𝑖 − 1000 = 𝑖
Calculate the average of the new variable:
1 + 2 + 3 + 4 + 5 15
𝑋̅ = = =3
5 5
Change back to the original variable:
𝑥̅ = 𝑋̅ + 1000 = 1000 + 3 = 1003

4.34: Change of Origin

Example 4.35
Show that the average of a set of numbers is invariant to change of origin.

𝑛
Consider the sum of the 𝑥1 + 𝑥2 + ⋯ + 𝑥𝑛
numbers ∑ 𝑥𝑖
𝑖=1
𝑛
Subtract 𝑝 from each 𝑥1 − 𝑝, 𝑥2 − 𝑝, … , 𝑥𝑛 − 𝑝
number. (Change of origin) ∑ 𝑥𝑖 − 𝑝
𝑖=1

P a g e 159 | 274
Get all the files at: https://bit.ly/azizhandouts
Aziz Manva (azizmanva@gmail.com)

(𝑥1 − 𝑝) + (𝑥2 − 𝑝) + ⋯ + (𝑥𝑛 − 𝑝) 𝑛


Find the average and 1
simplify it. 𝑛 ∑(𝑥𝑖 − 𝑝)
𝑥1 + 𝑥2 + ⋯ + 𝑥𝑛 − 𝑛𝑝 𝑛
𝑖=1
= Split the summation using the sum and
𝑛
𝑥1 + 𝑥2 + ⋯ + 𝑥𝑛 difference property:
= −𝑝 𝑛 𝑛
𝑛 1 1
= ∑ 𝑥𝑖 − ∑ 𝑝
𝑛 𝑛
𝑖=1 𝑖=1
Use the constant property in the second
term:
𝑛
1 1
= ( ∑ 𝑥𝑖 ) − ( 𝑛𝑝)
𝑛 𝑛
𝑖=1
𝑛
1
= ( ∑ 𝑥𝑖 ) − 𝑝
𝑛
𝑖=1
𝑥1 + 𝑥2 + ⋯ + 𝑥𝑛 𝑛
Add back 𝑝. 1
= = 𝑥̅ ∑ 𝑥𝑖
𝑛
𝑛
𝑖=1

Example 4.36
Find the average of
5700,5900,6400

5700 + 5900 + 6400


3
Use a change of scale:
57 + 59 + 64
= 100 ×
3
Use a change of origin:
60 × 3 − 3 − 1 + 4
= 100 ×
3
= 100 × 60
Simplify:
= 6000

Example 4.37
Show that the average of a set of numbers is invariant to change of origin and change of scale (provided the
operations are performed in the correct order).

𝑥1 − 𝑝 𝑥2 − 𝑝 𝑥𝑛 − 𝑝 𝑥1 + 𝑥2 + ⋯ + 𝑥𝑛 − 𝑛𝑝
𝑥1 , 𝑥2 , … , 𝑥𝑛 → , ,…, →
⏟𝑞 𝑞 𝑞 ⏟ 𝑛𝑞
𝐶ℎ𝑎𝑛𝑔𝑒 𝑜𝑓 𝑂𝑟𝑖𝑔𝑖𝑛: 𝑆𝑢𝑏𝑡𝑟𝑎𝑐𝑡 𝑝 𝐴𝑑𝑑 𝑎𝑙𝑙 𝑡ℎ𝑒 𝑛𝑢𝑚𝑏𝑒𝑟𝑠,𝑎𝑛𝑑 𝑡ℎ𝑒𝑛
Change of Scale: Divide by 𝑞 𝑑𝑖𝑣𝑖𝑑𝑒 𝑏𝑦 𝑛
To get our average, we will reverse the steps on the average:
Multiply by 𝑞, and add back 𝑝:
𝑥1 + 𝑥2 + ⋯ + 𝑥𝑛 𝑝 𝑥1 + 𝑥2 + ⋯ + 𝑥𝑛 𝑥1 + 𝑥2 + ⋯ + 𝑥𝑛
[( − ) × 𝑞] + 𝑝 = −𝑝+𝑝=
𝑛𝑞 𝑞 𝑛 𝑛
Which is the same as the average of the original numbers.

Start with the sum of a set of numbers:

P a g e 160 | 274
Get all the files at: https://bit.ly/azizhandouts
Aziz Manva (azizmanva@gmail.com)
𝑛
Split the summation:
∑ 𝑥𝑖 𝑛 𝑛
1 𝑥𝑖 𝑝
𝑖=1 = (∑ − ∑ )
Subtract 𝑝 from each number (Change of origin) 𝑞 𝑛 𝑛
𝑖=1 𝑖=1
𝑛
Use the constant multiple property:
∑ 𝑥𝑖 − 𝑝 𝑛
1 𝑥𝑖
𝑖=1 = [(∑ ) − 𝑝]
Divide each number by 𝑞. (Change of scale) 𝑞 𝑛
𝑖=1
𝑛
𝑥𝑖 − 𝑝 Reverse the operations we performed. Start by
∑ multiplying by 𝑞:
𝑞 𝑛
𝑖=1
1 1
Divide by 𝑛 to find the average of this new set of [ ( ∑ 𝑥𝑖 − 𝑝) × 𝑞]
numbers: 𝑞 𝑛
𝑖=1
𝑛
𝑥𝑖 − 𝑝 Continue by adding back 𝑝:
∑ 𝑛
𝑛𝑞 1
𝑖=1 = ∑ 𝑥𝑖 − 𝑝 + 𝑝
1 𝑛
Move 𝑞 outside of the summation sign 𝑖=1
𝑛
𝑛 1
1 𝑥𝑖 − 𝑝 = ∑ 𝑥𝑖
∑ 𝑛
𝑞 𝑛 𝑖=1
𝑖=1

D. Number Theory

Example 4.38
Find the probability that the value of the units digit of the expression below is 7, for some random four-digit
number 𝑛.
𝑥=𝑛

∑ 𝑥!
𝑥=1

𝑥=𝑛

∑ 𝑥! = 1! + 2! + 3! + 4! + 5! + ⋯ + 𝑛! = 1 + 2 + 6 + 24 + 120 + ⋯ + 𝑛! = 153 + ⋯ + 𝑛!
𝑥=1

𝑛 ≥ 5! ⇒ 𝑈𝑛𝑖𝑡𝑠 𝑑𝑖𝑔𝑖𝑡 𝑜𝑓 𝑛! 𝑖𝑠 𝑧𝑒𝑟𝑜.

Hence, there is no change in the units digit after we reach 5!.


The units digit is always 3.
𝑃(𝑈𝑛𝑖𝑡𝑠 𝐷𝑖𝑔𝑖𝑡 = 7) = 0

E. Imaginary Numbers

4.39: Sum of four consecutive powers of 𝒊 is zero


𝑖 = √−1
𝑖 2 = −1
𝑖 3 = −𝑖
𝑖4 = 1

4.40: Sum of four consecutive powers of 𝒊 is zero


P a g e 161 | 274
Get all the files at: https://bit.ly/azizhandouts
Aziz Manva (azizmanva@gmail.com)

The sum of any four consecutive, integral powers of 𝒊 is zero


𝒊𝒙 + 𝒊𝒙+𝟏 + 𝒊𝒙+𝟐 + 𝒊𝒙+𝟑 = 𝟎, 𝒙∈ℤ

𝑖⏟𝑥 (1 + 𝑖 + 𝑖 2 + 𝑖 3 ) = 𝑖⏟𝑥 (1 + 𝑖 − 1 − 𝑖 ) = 𝑖 𝑥 (0) = 0


𝑭𝒂𝒄𝒕𝒐𝒓𝒊𝒏𝒈 𝒐𝒖𝒕 𝒊𝒙 𝑺𝒖𝒃𝒔𝒕𝒊𝒕𝒖𝒕𝒊𝒏𝒈 𝒗𝒂𝒍𝒖𝒆𝒔
Since 𝑥 can be negative, this above identity is true for negative values of 𝑥 as well.

Example 4.41
Find the sum of the first 2021 powers of iota. That is, evaluate:
2021

∑ 𝑖𝑥
𝑥=1

Method I: Using the properties of 𝒊


𝑖 1 + 𝑖 2 + ⋯ + 𝑖 2021
Group the terms of the series, with each group having four consecutive powers of 𝒊, and one 𝒊 left over:
=⏟(𝑖 1 + 𝑖 2 + 𝑖 3 + 𝑖 4 ) + ⏟
(𝑖 5 + 𝑖 6 + 𝑖 7 + 𝑖 8 ) + ⋯ + ⏟
(𝑖 2017 + 𝑖 2018 + 𝑖 2019 + 𝑖 2020 ) + 𝑖 2021
=𝟎 =𝟎 =𝟎
All groups have a value of zero, except for the lone 𝑖 2021 , which can be simplified as:
𝑖 2021 = 𝑖 2020 × 𝑖 = 𝑖
Method II: Using the properties of Geometric Series
Note that
2021

∑ 𝑖 𝑥 = 𝑖 1 + 𝑖 2 + ⋯ + 𝑖 2021
𝑥=1
Is a geometric series with 𝐹𝑖𝑟𝑠𝑡 𝑡𝑒𝑟𝑚 = 𝑎 = 𝑖 and 𝐶𝑜𝑚𝑚𝑜𝑛 𝑅𝑎𝑡𝑖𝑜 = 𝑖, 𝑁𝑜. 𝑜𝑓 𝑇𝑒𝑟𝑚𝑠 = 𝑛 = 2021

Substitute 𝑎 = 𝑖 and 𝑟 = 𝑖 in the formula for the sum of a finite geometric series:
𝑎(1 − 𝑟 𝑛 ) 𝑖(1 − 𝑖 2021 ) 𝑖(1 − 𝑖)
𝑆= = = =𝑖
1−𝑟 1−𝑖 1−𝑖

F. Nested Sums
In a nested summation, you have a summation that is itself inside a summation sign.

Example 4.42
𝑏=5 𝑎=5

∑ (∑ 𝑎𝑏)
𝑏=1 𝑎=1

Expand the inner summation operator:


𝑏=5 𝑏=5

∑ 𝑏 + 2𝑏 + 3𝑏 + 4𝑏 + 5𝑏 = ∑ 15𝑏
𝑏=1 𝑏=1
Move the 15 outside of the summation sign:
𝑏=5

= 15 ∑ 𝑏
𝑏=1
Expand the inner summation operator:
= 15(1 + 2 + 3 + 4 + 5) = 152 = 225

P a g e 162 | 274
Get all the files at: https://bit.ly/azizhandouts
Aziz Manva (azizmanva@gmail.com)

4.43: Sum of Cubes of Natural Numbers


𝑛 2
𝑛(𝑛 + 1)
3
∑𝑘 = [ ]
2
𝑥=1

Example 4.44
𝑛(𝑛+1)(2𝑛+1)(3𝑛2 +3𝑛−1)
Given that ∑𝑛𝑥=1 𝑥 4 = 30
find the value of
𝑥=𝑛 𝑎=𝑥

∑ (∑ 𝑎3 )
𝑥=1 𝑎=1

𝑥=𝑛 2
𝑥(𝑥 + 1)
∑( )
2
𝑥=1
Carry out the multiplication:
𝑥=𝑛
𝑥 4 + 2𝑥 3 + 𝑥 2

4
𝑥=1
Split the terms:
𝑥=𝑛 𝑥=𝑛 𝑥=𝑛
𝑥4 2𝑥 3 𝑥2
∑ +∑ +∑
4 4 4
𝑥=1 𝑥=1 𝑥=1
Substitute:
2
𝑛(𝑛 + 1)
𝑛(𝑛 + 1)(2𝑛 + 1)(3𝑛 + 3𝑛 − 1) 2
2 ] [
𝑛(𝑛 + 1)
+ +
120 2 8

Example 4.45
Evaluate
𝑘=3

∑ ∑ 𝑥 2𝑘+1 + 5 , −5 ≤ 𝑥 ≤ 5, 𝑥 ∈ ℤ, 𝑥 ≠ 0
𝑥 𝑘=−3

Split the inside summation:


𝑘=3 𝑘=3
2𝑘+1
∑( ∑ 𝑥 + ∑ 5)
𝑥 𝑘=−3 𝑘=−3
Distribute the outer summation to each inner summation:
𝑘=3 𝑘=3

∑ ∑ 𝑥 2𝑘+1 + ∑ ∑ 5
𝑥 𝑘=−3 𝑥 𝑘=−3
Simplify the First Term
𝑘=3
1 1 1
∑ 𝑥 2𝑘+1 = 5
+ 3 + + 𝑥1 + 𝑥 3 + 𝑥 5 + 𝑥 7
𝑥 𝑥 𝑥
𝑘=−3
Note that:
𝑘=3 𝑘=3 𝑘=3
(−𝑥)2𝑘+1 = −𝑥 2𝑘+1
⇒ ∑ (−5)2𝑘+1 = ∑ −5 2𝑘+1
= − ∑ 52𝑘+1
𝑘=−3 𝑘=−3 𝑘=−3
Since the terms cancel out, the final value is zero.
Simplify the Second Term

P a g e 163 | 274
Get all the files at: https://bit.ly/azizhandouts
Aziz Manva (azizmanva@gmail.com)

𝑘=3

∑ ∑ 5 = ∑ 35 = 350
𝑥 𝑘=−3 𝑥

Example 4.46: Imaginary Numbers


The expression below can be written in the form 𝑎 + 𝑏𝑖 where 𝑎 and 𝑏 are real numbers and 𝑖 = √−1. Find 𝑎 +
𝑏.
𝑛=2022 𝑛

∑ ∑ 𝑖𝑥
𝑛=1 𝑥=1

1 4𝑛+1

∑ 𝑖𝑥 = ∑ 𝑖𝑥 = 𝑖
𝑥=1 𝑥=1
2 4𝑛+2

∑ 𝑖𝑥 = ∑ 𝑖𝑥 = 𝑖 + 𝑖2 = 𝑖 − 1
𝑥=1 𝑥=1
3 4𝑛+3

∑ 𝑖 𝑥 = ∑ 𝑖 𝑥 = 𝑖 + 𝑖 2 + 𝑖 3 = 𝑖 − 1 − 𝑖 = −1
𝑥=1 𝑥=1
4 4𝑛

∑ 𝑖𝑥 = ∑ 𝑖𝑥 = 𝑖 + 𝑖2 + 𝑖3 + 𝑖4 = 𝑖 − 1 − 𝑖 + 1 = 0
𝑥=1 𝑥=1

1 2 3 4

𝑛 = 4 ⇒ ∑ 𝑖 + ∑ 𝑖 + ∑ 𝑖 + ∑ 𝑖 𝑥 = 𝑖 + 𝑖 − 1 − 1 = 2𝑖 − 2
𝑥 𝑥 𝑥

𝑥=1 𝑥=1 𝑥=1 𝑥=1

The entire summation is:


𝑛=2022 𝑛 1 2 3 4 2017 2018 2019 2020 2021 2022

∑ ∑ 𝑖 = ∑ 𝑖 + ∑ 𝑖 + ∑ 𝑖 + ∑ 𝑖 + ⋯ + ∑ 𝑖 + ∑ 𝑖 + ∑ 𝑖 + ∑ 𝑖 + ∑ 𝑖 + ∑ 𝑖𝑥
𝑥 𝑥 𝑥 𝑥 𝑥 𝑥 𝑥 𝑥 𝑥 𝑥

𝑛=1 𝑥=1 ⏟
𝑥=1 𝑥=1 𝑥=1 𝑥=1 ⏟
𝑥=1 𝑥=1 𝑥=1 𝑥=1 𝑥=1 𝑥=1
𝟐𝒊−𝟐 𝟐𝒊−𝟐

The number of groups formed is:


2022
⌊ ⌋ = 505
4
2021 2022

505(2𝑖 − 2) + ∑ 𝑖 + ∑ 𝑖 𝑥 𝑥

𝑥=1 𝑥=1
1010𝑖 − 1010 + 𝑖 + 𝑖 − 1
−1011 + 1012𝑖

𝑎 + 𝑏 = −1011 + 1012 = 1

4.2 Product and Other Notations


A. Evaluating Product Notation
Product notation is less frequently used as compared to summation notation. It does come up, however.

4.47: Product Notation

P a g e 164 | 274
Get all the files at: https://bit.ly/azizhandouts
Aziz Manva (azizmanva@gmail.com)
𝑛

∏𝑖 = 1 × 2 × 3 × … × 𝑛 = 𝑛!

𝑖=1
𝑃𝑟𝑜𝑑𝑢𝑐𝑡 𝑜𝑣𝑒𝑟 𝑖

Example 4.48
Evaluate
A. Find the sum of digits of ∏5𝑖=1 𝑖
B. Find the product of the digits of ∏4𝑖=1 2𝑖
C. Evaluate ∏5𝑖=1(2𝑖 − 1)

Part A

1 × 2 × … × 5 = 5! = 120 ⇒ 1 + 2 + 0 = 3
Part B
2 × 4 × 6 × 8 = 24 (4!) = 16 × 24 = 384 ⇒ 3 × 8 × 4 = 96
Part C
1 × 3 × 5 × 7 × 9 = 945

4.49: Laws of Exponents


∏ 𝑥𝑛 = 𝑥∑ 𝑛
𝑛

𝐿𝐻𝑆 = ∏ 𝑥 𝑛 = 𝑥 𝑛1 × 𝑥 𝑛2 × … × 𝑥 𝑛𝑖 = 𝑥 𝑛1 +𝑛2 +⋯+𝑛𝑖 = 𝑥 ∑ 𝑛 = 𝑅𝐻𝑆


𝑛
Exponents convert multiplication into addition. Hence, we can convert the product operation into a summation
operator over the exponents.

Example 4.50
A. ∏10 𝑛
𝑛=1 𝑥 = 𝑥
𝑝×𝑞
where 𝑝 and 𝑞 are prime numbers with 𝑞 > 𝑝. Find 𝑞 − 𝑝.
𝑏
B. The product ∏4𝑛=2 √𝑥 can be written 𝑥 𝑎 𝑐 , where 𝑎, 𝑏, 𝑐 ∈ ℕ and 𝐻𝐶𝐹(𝑏, 𝑐) = 1. Find 𝑎 + 𝑏 + 𝑐.
𝑛

Part A
10 10 𝑛(𝑛+1) 10(11)
∏ 𝑥 𝑛 = 𝑥 ∑𝑛=1 𝑛 = 𝑥 2 =𝑥 2 = 𝑥 55 = 𝑥 5×11 ⇒ 𝑞 − 𝑝 = 11 − 5 = 6
𝑛=1
Part B
10 4 1 1 1 1 1 6+4+3 13 1
∑4𝑛=2
= 𝑥 2+3+4 = 𝑥 = 𝑥 12 = 𝑥 112 ⇒ 𝑎 + 𝑏 + 𝑐 = 1 + 1 + 12 = 14
𝑛
∏ √𝑥 = ∏ 𝑥𝑛 = 𝑥 𝑛 12
𝑛=2 𝑛=2

Example 4.51
𝑛

∑ log 𝑥 𝑎
𝑎=1

Method I(Shorter) Use the product rule for exponents:


Expand the summation: = log(𝑥 1+2+⋯+𝑛 )
log 𝑥 + log 𝑥 2 + ⋯ + log 𝑥 𝑛 Use the formula for the sum of the first 𝑛 natural
Use the product rule for logarithms: numbers:
= log(𝑥 × 𝑥 2 × … 𝑥 𝑛 )

P a g e 165 | 274
Get all the files at: https://bit.ly/azizhandouts
Aziz Manva (azizmanva@gmail.com)

𝑛(𝑛+1) operator:
= log (𝑥 2 ) 𝑛 𝑛

Use the power rule for logarithms: ∑ log 𝑥 = log (∏ 𝑥 𝑎 )


𝑎

𝑛(𝑛 + 1) 𝑎=1 𝑎=1


= log 𝑥 Moving the product operator into the exponent
2
Method II: Using Summation and Product Notation converts into a summation operator
Interchanging the operators results in the 𝑎=𝑛 𝑛(𝑛+1) 𝑛(𝑛 + 1)
log(𝑥 ∑𝑎=1 𝑎 ) = log (𝑥 2 ) = log 𝑥
summation operator converting into a product 2

Example 4.52
Write the finite value of the product below with infinite terms as a radical: (AMC 12 2022/8, Adapted) 5
∞ 3
3 3
∏ √… √ √10
𝑛=1 ⏟
𝑛 𝑟𝑎𝑑𝑖𝑐𝑎𝑙𝑠

Expand the product


Write out the infinite product:
3 3 3
√10 × √ √10 × √ √ √10 × …
3 3 3

Convert the radicals into fractional exponents:


1 1 1
= 103 × 109 × 1027 × …
Multiply the terms:
1 1 1
= 103+9+27+⋯
The series in the exponent is a geometric series. Use the formula for the sum of a geometric series with
1 1
𝑓𝑖𝑟𝑠𝑡 𝑡𝑒𝑟𝑚 = 𝑎 = 3 , 𝑐𝑜𝑚𝑚𝑜𝑛 𝑟𝑎𝑡𝑖𝑜 = 𝑟 = 3:
1
3
𝑎 1 1 2 1 3 1
1−
= 101−𝑟 = 10 3 = 103÷3 = 103×2 = 102
Convert the fractional exponent into a radical:
= √10
Product Notation
We can recognize the pattern, and convert the product into a summation over the exponents to get:

1 1
∑𝑛 𝑛
= ∏ 103𝑛 = 10 3
𝑛=1
And then the solution can proceed as before.
B. Converting to Product Notation

Example 4.53: Converting to Product Notation


Write the following in Product Notation
A. 𝑛(𝑛 − 1)(𝑛 − 2) … (𝑛 − 𝑘)
B. 𝑛(𝑛 + 1)(𝑛 + 2) … (𝑛 + 𝑘)
C. ⏟ 1×3×5×…
𝑛 𝑡𝑒𝑟𝑚𝑠

5 The math remains the same from the original question, but it is presented in product notation.

P a g e 166 | 274
Get all the files at: https://bit.ly/azizhandouts
Aziz Manva (azizmanva@gmail.com)

Part A
This is called a falling factorial:
𝑛

∏ 𝑖

𝑖=𝑛−𝑘
𝐹𝑎𝑙𝑙𝑖𝑛𝑔 𝐹𝑎𝑐𝑡𝑜𝑟𝑖𝑎𝑙
Part B
This is called a rising factorial:
𝑛+𝑘

∏𝑖

𝑖=𝑛
𝑅𝑖𝑠𝑖𝑛𝑔 𝐹𝑎𝑐𝑡𝑜𝑟𝑖𝑎𝑙
Part C
𝑛

∏ 2𝑖 − 1

𝑖=1
𝑂𝑑𝑑 𝑁𝑢𝑚𝑏𝑒𝑟𝑠

Example 4.54
A. The number of divisors 𝜏(𝑥) of a number 𝑥 with prime factorisation 𝑎𝑝 𝑏 𝑞 𝑐 𝑟 is 𝜏(𝑥) =
(𝑝 + 1)(𝑞 + 1)(𝑟 + 1). Write 𝜏(𝑥) of a number 𝑥 with prime factorization 𝑝1𝑎1 𝑝2𝑎2 𝑝3𝑎3 … 𝑝𝑛𝑎𝑛 in product
notation.
B. A number 𝑥 with prime factorization 𝑝𝑎 𝑞𝑏 𝑟 𝑐 has sum of its factors 𝑠(𝑛) = (1 + 𝑝 + 𝑝2 + ⋯ + 𝑝𝑎 )(1 +
𝑞 + 𝑞 2 + ⋯ + 𝑞 𝑏 )(1 + 𝑟 + 𝑟 2 + ⋯ + 𝑟 𝑐 ). Write the expression for the sum of factors of 𝑋 =
𝑎 𝑎 𝑎 𝑎
𝑝1 1 𝑝2 2 𝑝3 3 … 𝑝𝑛 𝑛 using the formula for the sum of a finite geometric series.

Part A
𝑛

𝜏(𝑥) = (𝑎1 + 1)(𝑎2 + 1) … (𝑎𝑛 + 1) = ∏(𝑎𝑖 + 1)


𝑖=1
Part B
𝑎 𝑎 𝑎
𝑠(𝑋) = (1 + 𝑝1 + 𝑝12 + ⋯ + 𝑝1 1 )(1 + 𝑝2 + 𝑝22 + ⋯ + 𝑝2 2 ) … (1 + 𝑝𝑛 + 𝑝𝑛2 + ⋯ + 𝑝𝑛 𝑛 )
To find the sum of the first parentheses substitute 𝑓𝑖𝑟𝑠𝑡 𝑡𝑒𝑟𝑚 = 𝑎 = 1, 𝑐𝑜𝑚𝑚𝑜𝑛 𝑟𝑎𝑡𝑖𝑜 = 𝑟 =
𝑝1 , 𝑛𝑢𝑚𝑏𝑒𝑟 𝑜𝑓 𝑡𝑒𝑟𝑚𝑠 = 𝑎 + 1 in the formula for the sum of a finite geometric series:
𝑎 +1
𝑎(𝑟 𝑛 − 1) 𝑝1 1 − 1
=
𝑟−1 𝑝1 − 1
And the entire expression is:
𝑎 +1 𝑎 +1 𝑎 +1 𝑛 𝑎 +1
𝑝1 1 − 1 𝑝2 2 − 1 𝑝𝑛 𝑛 − 1 𝑝𝑖 𝑖 − 1
( )( )…( )=∏
𝑝1 − 1 𝑝2 − 1 𝑝𝑛 − 1 𝑝𝑖 − 1
𝑖=1

C. Properties

4.55: Product over a Constant


A product over a constant can be determined by exponentiating the constant by the number of terms.
𝑛

𝑐 × 𝑐 × … × 𝑐 = 𝑐𝑛
∏𝑐 = ⏟
𝑖=𝑖 𝑛 𝑡𝑖𝑚𝑒𝑠

Example 4.56
What is the minimum value of 𝑛 for which the expression below is greater than 1000.

P a g e 167 | 274
Get all the files at: https://bit.ly/azizhandouts
Aziz Manva (azizmanva@gmail.com)
𝑛

∏2
𝑖=𝑖

∏ 2 > 1000
𝑖=𝑖
𝑛
2 > 1000
𝑛 ≥ 10

4.57: Moving a constant out


A constant can be moved out of a product sign by exponentiating it the number of terms.
𝑛 𝑛
𝑛
∏ 𝑐𝑓(𝑖) = 𝑐 ∏ 𝑓(𝑖)
𝑖=𝑖 𝑖=𝑖

𝑛 𝑛
𝑛 [𝑓(1) 𝑛
∏ 𝑐𝑓(𝑖) = 𝑐𝑓(1) × 𝑐𝑓(2) × … 𝑐𝑓(𝑛) = 𝑐 × 𝑓(2) × … 𝑓(𝑛)] = 𝑐 ∏ 𝑓(𝑖)
𝑖=𝑖 𝑖=𝑖

Example 4.58
Prove that the product of the first
A. 𝑛 even numbers is 2𝑛 𝑛!.
(2𝑛)!
B. 𝑛 odd numbers is 2𝑛 𝑛!

Part A
Without product notation
2 × 4 × 6 × … × 2𝑛 = 2𝑛 (1 × 2 × 3 × … × 𝑛) = 2𝑛 𝑛!
With product notation
𝑖=𝑛 𝑖=𝑛
2 × 4 × 6 × … × 2𝑛 = ∏ 2𝑖 = 2𝑛 ∏ 𝑖 = 2𝑛 𝑛!
𝑖=1 𝑖=1
Part B
Without product notation
2×4×6×…

𝐹𝑖𝑟𝑠𝑡 𝑛 𝐸𝑣𝑒𝑛 𝑁𝑢𝑚𝑏𝑒𝑟𝑠 𝑃𝑟𝑜𝑑𝑢𝑐𝑡 𝑜𝑓 𝐹𝑖𝑟𝑠𝑡 2𝑛 𝑁𝑢𝑚𝑏𝑒𝑟𝑠 (2𝑛)!
1×3×5×…×
⏟ = = 𝑛
𝐹𝑖𝑟𝑠𝑡 𝑛
2×4×6×…
⏟ 2×4×6×…
⏟ 2 𝑛!
𝑂𝑑𝑑 𝑁𝑢𝑚𝑏𝑒𝑟𝑠 𝐹𝑖𝑟𝑠𝑡 𝑛 𝐸𝑣𝑒𝑛 𝑁𝑢𝑚𝑏𝑒𝑟𝑠 𝐹𝑖𝑟𝑠𝑡 𝑛 𝐸𝑣𝑒𝑛 𝑁𝑢𝑚𝑏𝑒𝑟𝑠
With product notation
Multiply and divide by the product of the first 𝑛 even numbers:
𝑛 𝑛
𝑛 (∏
⏟ 𝑖=1 2𝑖 − 1) × (∏
⏟ 𝑖=1 2𝑖 )
𝑂𝑑𝑑 𝑁𝑢𝑚𝑏𝑒𝑟𝑠 𝐸𝑣𝑒𝑛 𝑁𝑢𝑚𝑏𝑒𝑟𝑠 ∏2𝑛
𝑖=1 𝑖 (2𝑛)!
1 × 3 × 5 × … = ∏ 2𝑖 − 1 =
⏟ = = 𝑛
∏𝑛𝑖=1 2𝑖 𝑛
2 𝑛 2 𝑛
𝑛 𝑡𝑒𝑟𝑚𝑠 ⏟
𝑖=1
𝑂𝑑𝑑 𝑁𝑢𝑚𝑏𝑒𝑟𝑠
Using Mathematical Induction
When 𝑛 = 1:
(2𝑛)! (2)! 2
= = = 1 ⇒ 𝐵𝑎𝑠𝑒 𝑐𝑎𝑠𝑒 𝑖𝑠 𝑝𝑟𝑜𝑣𝑒𝑑
2𝑛 𝑛! 21 1! 2
Now, if it is true for some integer 𝑘, then
(2𝑘)!
⏟× 3 × 5 × … × (2𝑘 − 1) = 𝑘
1
𝑘 𝑡𝑒𝑟𝑚𝑠
2 𝑘!

P a g e 168 | 274
Get all the files at: https://bit.ly/azizhandouts
Aziz Manva (azizmanva@gmail.com)

Multiply both sides of the above by (2𝑘 + 1):


1 × 3 × 5 × … × (2𝑘 + 1)
𝐿𝐻𝑆 = ⏟
𝑘+1 𝑡𝑒𝑟𝑚𝑠
(2𝑘)! (2𝑘 + 1) (2𝑘 + 1)! 2𝑘 + 2 (2𝑘 + 2)! (2𝑘 + 2)!
𝑅𝐻𝑆 = 𝑘
= 𝑘
× = 𝑘 = 𝑘+1
2 𝑘! 2 𝑘! 2𝑘 + 2 2 𝑘! 2(𝑘 + 1) 2 (𝑘 + 1)!
And we get the equality below which proves the inductive case:
(2(𝑘 + 1))!
⏟× 3 × 5 × … × (2𝑘 + 1) = 𝑘+1
1
𝑘+1 𝑡𝑒𝑟𝑚𝑠
2 (𝑘 + 1)!

4.59: Changing the Index of Multiplication


𝑘=𝑦 𝑘=𝑦−𝑛 𝑘=𝑦+𝑛
∏ 𝑘=∏ 𝑘+𝑛 =∏ 𝑘−𝑛
𝑘=𝑥 𝑘=𝑥−𝑛 𝑘=𝑥+𝑛

Changing the index of multiplication for product notation is like changing the index of summation for
summation notation

Example 4.60
Find 𝑥 and 𝑦 if:
𝑛=𝑚 𝑛=𝑦
∏ (𝑛 + 𝑘) = ∏ 𝑛
𝑛=1 𝑛=𝑥

𝑛=𝑚 𝑛=𝑚+𝑘
∏ (𝑛 + 𝑘) = ∏ 𝑛
𝑛=1 𝑛=1+𝑘
𝑥 = 1 + 𝑘, 𝑦 = 𝑚 + 𝑘

4.61: Interchanging Log and Product Operators6


log (∏ 𝑥𝑛 ) = ∑ log 𝑥𝑛
𝑛 𝑛

Expand the LHS:


log(𝑥1 × 𝑥2 × … × 𝑥𝑛 )
Use the product rule for logarithms:
= log 𝑥1 + log 𝑥2 + ⋯ + log 𝑥𝑛
Write the expression as a summation:
= ∑ log 𝑥𝑛
𝑛

Example 4.62
Evaluate
𝑖=100
1
∑ (𝑿𝑨𝑻 𝟐𝟎𝟏𝟒)
log 𝑖 100!
𝑖=1

Use the change of base rule to move the logarithm from denominator to numerator:

6 This generalizes the property log 𝑎𝑏 = log 𝑎 + log 𝑏

P a g e 169 | 274
Get all the files at: https://bit.ly/azizhandouts
Aziz Manva (azizmanva@gmail.com)

𝑖=100

∑ log100! 𝑖
𝑖=1
Interchange the summation and the log operators:
𝑖=100
log100! ∏ 𝑖
𝑖=2
The product is a factorial:
= log100! 100! = 1

D. Applications

4.63: Vieta’s Formulas


For a quadratic 𝑎𝑥 2 + 𝑏𝑥 + 𝑐, 𝑎 ≠ 0
𝑏 𝑐
𝑆𝑢𝑚 𝑜𝑓 𝑅𝑜𝑜𝑡𝑠 = − , 𝑃𝑟𝑜𝑑𝑢𝑐𝑡 𝑜𝑓 𝑅𝑜𝑜𝑡𝑠 =
𝑎 𝑎

A quadratic has real roots when


𝐷𝑖𝑠𝑐𝑟𝑖𝑚𝑖𝑛𝑎𝑛𝑡 ≥ 0 ⇒ 𝑏 2 − 4𝑎𝑐 ≥ 0

Example 4.64: Vieta’s Formulas/Discriminant


𝑀𝑎𝑟𝑘 𝑎𝑙𝑙 𝑐𝑜𝑟𝑟𝑒𝑐𝑡 𝑜𝑝𝑡𝑖𝑜𝑛𝑠
If
999
𝑓(𝑥) = ∏ (𝑥 2 − 47𝑥 + 𝑘)
𝑘=1
then the product of all real roots of 𝑓(𝑥) = 0 is: ( , Adapted)7
552
𝐴. ∏ 𝑛
𝑛=1
552
𝐵. ∏ 𝑛!
𝑛=1
999
𝐶. ∏ 𝑛
𝑛=1
∏999
𝑛=1 𝑛
𝐷. 447
∏𝑛=1(𝑛 + 552)
E. None of these

We have 999 quadratics, for a total for 999 × 2 roots.

Consider each quadratic. For the roots to be real, the discriminant must be non-negative.
𝑏 2 − 4𝑎𝑐 ≥ 0 ⇒ 472 − 4𝑘 ≥ 0 ⇒ 𝑘 ≤ 552.25 ⇒ 𝑘 ∈ {1,2, … ,552}

Using Vieta’s Formulas, for a single quadratic:


𝑐 𝑘
𝑃𝑟𝑜𝑑𝑢𝑐𝑡 𝑜𝑓 𝑅𝑜𝑜𝑡𝑠 = = =𝑘
𝑎 1
And hence the final answer is
999
552 (∏552
𝑛=1 𝑛 )(∏𝑛=553 𝑛 ) ∏999
𝑛=1 𝑛
1 × 2 × … × 552 = 552! = ∏ 𝑛 = 999 = 447
⏟ 𝑛=1 ∏𝑛=553 𝑛 ∏𝑛=1(𝑛 + 552)

𝑶𝒑𝒕𝒊𝒐𝒏 𝑨 𝑶𝒑𝒕𝒊𝒐𝒏 𝑫

7 The original question had options in terms of factorials only. This has options in terms of products.

P a g e 170 | 274
Get all the files at: https://bit.ly/azizhandouts
Aziz Manva (azizmanva@gmail.com)

Example 4.65
Let 𝑥𝑛 be the 𝑛𝑡ℎ distinct root of:
100
𝑓(𝑥) = ∏ (𝑥 2 − 𝑘)
𝑘=1
𝑎 is the number of zeroes at the end of ∏𝑛 𝑥𝑛 . Let 𝑟 be the number of roots of 𝑓(𝑥). Find 𝑎 + 𝑟.

𝑁𝑜. 𝑜𝑓 𝑅𝑜𝑜𝑡𝑠 = 200


𝑐 1
𝑘 = 1 ⇒ 𝑥 2 − 1 ⇒ 𝑃𝑟𝑜𝑑𝑢𝑐𝑡 𝑜𝑓 𝑅𝑜𝑜𝑡𝑠 = = − = −1
𝑎 1
𝑐 2
𝑘 = 2 ⇒ 𝑥 2 − 2 ⇒ 𝑃𝑟𝑜𝑑𝑢𝑐𝑡 𝑜𝑓 𝑅𝑜𝑜𝑡𝑠 = = − = −2
𝑎 1
The product of roots of all the quadratics is:
(−1)(−2)(−3) … (−100) = (−1)100 100! = 100!
Any zero is basically generated by
10 = 2 × 5
There are many 2′𝑠 (for example, consider 64 = 26 ), so the 5′𝑠 are in short supply. Count the number of 5′𝑠:
{5,10,15, … ,100} = 5 × {1,2,3, … ,20} ⇒ 20 𝑉𝑎𝑙𝑢𝑒𝑠
And we get two 5′ 𝑠 each from the multiples of 25:
{25, 50, 75, 100} ⇒ 4 𝑉𝑎𝑙𝑢𝑒𝑠
The total is:
20 + 4 = 24
E. Nested Products and Sums

4.66: Nested Products


∏ ∏ 𝑥 = ∏ (∏ 𝑥 )
𝑛 𝑚 𝑛 𝑚

➢ Like nested summations, nested products are evaluating by first finding the “inside” product.

Example 4.67
Find the sum of digits of
𝑚=100 𝑛=𝑚
log 2 (∏ ∏ 2)
𝑚=1 𝑛=1

Evaluate the inner product using the constant property:


𝑚=100
log 2 (∏ 2𝑚 )
𝑚=1
Interchange the log and the summation operators:
𝑚=100

= ∑ log 2 2𝑚
𝑚=1
Use the power rule for logarithms
𝑚=100

= ∑ 𝑚
𝑚=1
Use the formula for the sum of the first 𝑛 natural numbers:
100(101)
= = 5050
2
P a g e 171 | 274
Get all the files at: https://bit.ly/azizhandouts
Aziz Manva (azizmanva@gmail.com)

Which has sum of digits:


10

Example 4.68
Find the number of zeros at the end of
𝑚=100 𝑛=𝑚
∏ ∏ 𝑛
𝑚=1 𝑛=1

The inside product is a factorial, giving us:


𝑚=100
∏ 𝑚!
𝑚=1
Count the number of 5′𝑠:

5! 10! 15! . . 100! Total


5 5 5 . 5 20
10 10 . . 10 19
15 . . 15 18
.
.
.
1

20(21)
1 + 2 + ⋯ + 20 = = 210
2
Count the number of 25′𝑠:

25! 50! 75! 100! Total


25 25 25 25 4
50 50 50 3
75 75 2
100 1

10

1 + 2 + 3 + 4 = 10
The final answer is
210 + 10 = 220

Example 4.69
A number 𝑥 with prime factorization 𝑝𝑎 𝑞𝑏 𝑟 𝑐 has sum of its factors 𝑠(𝑛) = (1 + 𝑝 + 𝑝2 + ⋯ + 𝑝𝑎 )(1 + 𝑞 + 𝑞 2 +
𝑎 𝑎 𝑎 𝑎
⋯ + 𝑞 𝑏 )(1 + 𝑟 + 𝑟 2 + ⋯ + 𝑟 𝑐 ). Write the expression for the sum of factors of 𝑋 = 𝑝1 1 𝑝2 2 𝑝3 3 … 𝑝𝑛 𝑛 using a
summation nested in a product.

𝑎1 𝑎2 𝑎𝑛

𝑠(𝑋) = ∑ 𝑝1𝑥 × ∑ 𝑝2𝑥 × …× ∑ 𝑝𝑛𝑥



𝑥=0 ⏟
𝑥=0 ⏟
𝑥=0
𝑎 𝑎 𝑎
2 +⋯+𝑝 𝑛 )
(1+𝑝1 +𝑝12 +⋯+𝑝1 1 ) (1+𝑝2 +𝑝22 +⋯+𝑝2 2 ) (1+𝑝𝑛 +𝑝𝑛 𝑛
And now we can write this using product notation:

P a g e 172 | 274
Get all the files at: https://bit.ly/azizhandouts
Aziz Manva (azizmanva@gmail.com)

𝑛 𝑎1

∏ ∑ 𝑝𝑖𝑥
𝑖=1 𝑥=0

F. Union and Intersection Notation

4.70: Union Notation


𝑛
⋃ 𝑆𝑖 = 𝑆1 ∪ 𝑆2 ∪ … ∪ 𝑆𝑛
𝑖=1

➢ Union notation is used to indicate a union of intervals.


➢ It generalizes the union operator you would have seen in Set Theory.

4.71: Length of an Interval


The length of an interval (𝑎, 𝑏), [𝑎, 𝑏], (𝑎, 𝑏] 𝑜𝑟 [𝑎. 𝑏) is defined to be |𝑏 − 𝑎|.

For example
𝐿𝑒𝑛𝑔𝑡ℎ 𝑜𝑓(2,8) = |8 − 2| = 6

Example 4.72
A. Define the “length” of a union of intervals to be the sum of the length of the disjoint intervals that make
up the union. Why is are the intervals 𝑑𝑖𝑠𝑗𝑜𝑖𝑛𝑡?
B. Find the length of ⋃𝑛𝑖=1(2𝑖 , 2𝑖+1 )
C. Find the length of ⋃𝑛𝑖=0(22𝑖 , 22𝑖+1 )

Part A
(1,5) ∪ (4,6) ⇒ 𝐿𝑒𝑛𝑔𝑡ℎ = 4 + 2 = 6
(1,5) ∪ (5,6) ⇒ 𝐿𝑒𝑛𝑔𝑡ℎ = 4 + 1 = 5
We should get the same length in both cases since the:
(1,5) ∪ (4,6) = (1,5) ∪ (5,6) = (1,6) ⇒ 𝐿𝑒𝑛𝑔𝑡ℎ = 6 − 1 = 5
Part B
(21 , 22 ) ∪ (22 , 23 ) ∪ … ∪ (2𝑛 , 2𝑛+1 )
=1+
1 𝑛+1 )
= (2 ,2 ⇒ 𝐿𝑒𝑛𝑔𝑡ℎ = 2𝑛+1 − 2
Part C
Write the union in set notation:
(20 , 21 ) ∪ (22 , 23 ) ∪ (24 , 25 ) ∪ … ∪ (22𝑛 , 22𝑛+1 )
= (1,2) ∪ (4,8) ∪ (16,32) ∪ … ∪ (22𝑛 , 22𝑛+1 )
Determine the length of each interval, add the resulting geometric series:
1(4𝑛+1 − 1) 4𝑛+1 − 1
= 1 + 4 + 16 + ⋯ + 4𝑛 = =
4−1 3

4.73: Intersection Notation


𝑛
⋂ 𝑆𝑖 = 𝑆1 ∩ 𝑆2 ∩ … ∩ 𝑆𝑛
𝑖=1

➢ Union notation is used to indicate an intersection of intervals.


➢ It generalizes the intersection operator you would have seen in Set Theory.

P a g e 173 | 274
Get all the files at: https://bit.ly/azizhandouts
Aziz Manva (azizmanva@gmail.com)

Example 4.74
Determine the smallest value of 𝑛 for which ⋂𝑛𝑖=1 𝑆𝑖 is a null set.

G. “AND” Notation; “OR” Notation

4.75: AND Notation

Example 4.76

4.77: OR Notation

Example 4.78

4.3 Log Sequences; AP-GP General Revision


A. General Sequences

4.79: Sequence
A sequence 𝑎1 , 𝑎2 , 𝑎3 , … , 𝑎𝑛 is a set of terms.

4.80: General Term of a Sequence


➢ The expression for the 𝑛𝑡ℎ of a sequence, written 𝑎𝑛 is called the general term.
➢ The general term can be written in explicit form, which is a direct formula for the 𝑛𝑡ℎ term.
➢ It can also be written in recursive form, where the 𝑛𝑡ℎ term depends on one or more prior terms.

Example 4.81
Each sequence below is a sequence of a standard type. Identify the type of sequence, and give its general term in
explicit and recursive form.
A. 𝑎, 𝑎 + 𝑑, 𝑎 + 2𝑑, 𝑎 + 3𝑑, … , 𝑎 + 𝑛𝑑
B. 𝑎, 𝑎𝑟, 𝑎𝑟 2 , … 𝑎𝑟 𝑛
C. 𝑎 + (𝑎 + 𝑑)𝑟 + (𝑎 + 2𝑑)𝑟 2 + ⋯ + [𝑎 + (𝑛 − 1)𝑑]𝑟 𝑛−1
D. 1,1 + 2, 1 + 2 + 3, … ,1 + 2 + 3 + ⋯ + 𝑛
E. 1,1 + 4,1 + 4 + 9, … ,1 + 4 + ⋯ + 𝑛2
F. 1,1 + 8,1 + 8 + 27, … ,1 + 8 + ⋯ + 𝑛3
G. 2,4,6,8, …
H. 1,3,5,7 …

Part A
This is an arithmetic sequence, where each term increases by a common difference compared to the previous
term.
In this sequence:
𝐹𝑖𝑟𝑠𝑡 𝑇𝑒𝑟𝑚 = 𝑎, 𝐶𝑜𝑚𝑚𝑜𝑛 𝐷𝑖𝑓𝑓𝑒𝑟𝑒𝑛𝑐𝑒 = 𝑑
𝐸𝑥𝑝𝑙𝑖𝑐𝑖𝑡: 𝑎𝑛 = 𝑎 + (𝑛 − 1)𝑑
𝑅𝑒𝑐𝑢𝑟𝑠𝑖𝑣𝑒: 𝑎𝑛 = 𝑎𝑛−1 + 𝑑
Part B

P a g e 174 | 274
Get all the files at: https://bit.ly/azizhandouts
Aziz Manva (azizmanva@gmail.com)

This is a geometric sequence, where each term is multiplied by the common the ratio to get the next term,
In this sequence:
𝐹𝑖𝑟𝑠𝑡 𝑇𝑒𝑟𝑚 = 𝑎, 𝐶𝑜𝑚𝑚𝑜𝑛 𝑅𝑎𝑡𝑖𝑜 = 𝑟
𝐸𝑥𝑝𝑙𝑖𝑐𝑖𝑡: 𝑎𝑛 = 𝑎𝑟 𝑛−1
𝑅𝑒𝑐𝑢𝑟𝑠𝑖𝑣𝑒: 𝑎𝑛 = 𝑎𝑛−1 𝑟
Part C
An arithmetico-geometric sequence has ideas which combine arithmetic and geometric sequences. The first
term is 𝑎. Each term increases 𝑎 by the common difference 𝑑, and the increased value is multiplied by the
common ratio 𝑟.
𝐹𝑖𝑟𝑠𝑡 𝑇𝑒𝑟𝑚 = 𝑎, 𝐶𝑜𝑚𝑚𝑜𝑛 𝐷𝑖𝑓𝑓𝑒𝑟𝑒𝑛𝑐𝑒 = 𝑑, 𝐶𝑜𝑚𝑚𝑜𝑛 𝑅𝑎𝑡𝑖𝑜 = 𝑟
𝐸𝑥𝑝𝑙𝑖𝑐𝑖𝑡: 𝑎𝑛 = [𝑎 + (𝑛 − 1)𝑑]𝑟 𝑛−1
Part D
This is the sequence of triangular numbers, where the 𝑛𝑡ℎ triangular number is the sum of the first n numbers:
𝑛(𝑛 + 1)
1 + 2 + 3 + ⋯+ 𝑛 =
2
Part E
The 𝑛𝑡ℎ term of this sequence is the sum of the squares of the first 𝑛 numbers.
𝑛(𝑛 + 1)(2𝑛 + 1)
𝑎𝑛 = 1 + 2 + ⋯ + 𝑛2 =
6
Part F
The 𝑛𝑡ℎ term of this sequence is the sum of the cubes of the first 𝑛 numbers.
2
𝑛(𝑛 + 1)
𝑎𝑛 = 1 + 8 + ⋯ + 𝑛3 = [ ]
2
Part G
𝑛𝑡ℎ even natural number:
𝐸𝑣𝑒𝑛 𝑁𝑢𝑚𝑏𝑒𝑟𝑠 = 2𝑛
Part H
𝑛𝑡ℎ odd natural number:
𝑂𝑑𝑑 𝑁𝑢𝑚𝑏𝑒𝑟𝑠 = 2𝑛 − 1

4.82: Partial Sums


The sequence 𝑆𝑛 formed by the sum of the first 𝑛 terms of a sequence 𝑎𝑛 is called the sequence of partial sums.

Example 4.83
A. 𝑎𝑛 = 𝑎𝑟 𝑛−1

𝑎𝑛 𝑎 𝑎𝑟 𝑎𝑟 2 . 𝑎𝑟 𝑛−1
𝑆𝑛 𝑎 𝑎 + 𝑎𝑟 𝑎 + 𝑎𝑟 + 𝑎𝑟 2 . 𝑎 + 𝑎𝑟 + ⋯ + 𝑎𝑟 𝑛−1

4.84: Sequence Notation


{𝑎𝑛 }
{𝑎𝑛 }1∞

4.85: Alternating Sequence


An alternating sequence is a sequence where the terms alternate between positive and negative.

P a g e 175 | 274
Get all the files at: https://bit.ly/azizhandouts
Aziz Manva (azizmanva@gmail.com)

𝑎1 , −𝑎2 , 𝑎3 , −𝑎4 , 𝑎5 , …

Example 4.86
Given the sequence 𝑎𝑛 write the general term of the alternating sequence
A. 𝑏1 = 𝑎1 , 𝑏2 = −𝑎2 , 𝑏3 = 𝑎3 , 𝑏4 = −𝑎4 , 𝑏5 = 𝑎5 , …
B. −𝑎1 , +𝑎2 , −𝑎3 , +𝑎4 , −𝑎5 ,

Part A
We can define it piece-wise for 𝑛 ≥ 1:
𝑏2𝑛 = −𝑎2𝑛
𝑏2𝑛−1 = 𝑎2𝑛+1

We can also define it in a single general term:


𝑏𝑛 = (−1)𝑛+1 𝑎𝑛

4.87: Limit
The end behavior of a sequence as it approaches infinity is called the limit of the sequence.

4.88: Convergent vs. Divergent Sequences


➢ If the sum of the terms of a sequence has a finite value as the number of terms goes to infinity, then the
sequence is called convergent.
➢ If a sequence is not convergent, then it is called divergent.

Example 4.89
A. Is an arithmetic sequence 𝑆𝑛 convergent?
B. Is a geometric sequence 𝑆𝑛 convergent?

Part A
An arithmetic sequence either keeps increasing, or it keeps decreasing. It has no finite value.
lim 𝑆𝑛 𝑑𝑜𝑒𝑠 𝑛𝑜𝑡 𝑒𝑥𝑖𝑠𝑡 (𝐷𝑁𝐸)
𝑛→∞
Part B
A geometric sequence 𝑎, 𝑎𝑟, . . 𝑎𝑟 𝑛−1 is convergent if and only if
lim 𝑆𝑛 𝑒𝑥𝑖𝑠𝑡𝑠 𝑖𝑓 𝑎𝑛𝑑 𝑜𝑛𝑙𝑦 𝑖𝑓 − 1 < 𝑟 < 1
𝑛→∞

Example 4.90
A. Investigate the convergence or divergence of 𝑆𝑛 = ⏟
1−1+1−1+⋯
𝑛 𝑡𝑒𝑟𝑚𝑠
B. Even though the sequence is divergent, and hence does not have a sum, it has a 𝐶𝑒𝑠𝑎𝑟𝑜 − 𝑠𝑢𝑚.

Part A
𝑆1 = 1, 𝑆2 = 0
𝑆2𝑛+1 = 1, 𝑆2𝑛 = 0

lim 𝑆𝑛 𝐷𝑁𝐸 ⇒ 𝐷𝑖𝑣𝑒𝑟𝑔𝑒𝑛𝑡


𝑛→∞
Part B
𝑆2𝑛 = 0, 𝑆2𝑛−1 = 1

P a g e 176 | 274
Get all the files at: https://bit.ly/azizhandouts
Aziz Manva (azizmanva@gmail.com)

0+1 1
𝐶𝑎𝑒𝑠𝑎𝑟𝑜 − 𝑆𝑢𝑚 = =
2 2

Example 4.91

1−1+1−1+1−1+1−1+⋯


1 + ⏟
1 − ⏟
1 + ⏟
1 + ⏟
1 − ⏟
1
1𝑠𝑡 3𝑟𝑑 2𝑛𝑑 5𝑡ℎ 7𝑡ℎ 4𝑡ℎ
𝑇𝑒𝑟𝑚 𝑇𝑒𝑟𝑚 𝑇𝑒𝑟𝑚 𝑇𝑒𝑟𝑚 𝑇𝑒𝑟𝑚 𝑇𝑒𝑟𝑚

𝑆1 = 1
𝑆2 = 2
𝑆3 = 1
𝑆4 = 2
𝑆5 = 3
𝑆6 = 2

4.92: Increasing Sequence


An increasing sequence is a sequence where the next term is greater than the one before it.
a_(n+1)> a_n

Nondecreasing sequence
An nondecreasing sequence is a sequence where the next term is greater than or equal to the one before it.
a_(n+1) >= a_n

Decreasing Sequence
An decreasing sequence is a sequence where the next term is less than the one before it.
a_(n+1) < a_n

Nonincreasing Sequence
An nonincreasing sequence is a sequence where the next term is less than the one before it.
a_(n+1) <= a_n

Example 4.93

My profits have been increasing year after year.


(Increasing)

My marks in any test are never less than the test before it.
(Nondecreasing)

Example 4.94

A sequence is decreasing when the next term is greater than the previous term.
𝑎𝑛+1 < 𝑎𝑛
Rearrange to get the equivalent condition that a sequence is decreasing when the difference of successive terms
is negative:
𝑎𝑛+1 − 𝑎𝑛 < 0

P a g e 177 | 274
Get all the files at: https://bit.ly/azizhandouts
Aziz Manva (azizmanva@gmail.com)

B. Log Sequences

4.95: Log Rules


𝑃𝑟𝑜𝑑𝑢𝑐𝑡 𝑅𝑢𝑙𝑒: log 𝑎 + log 𝑏 = log 𝑎𝑏
𝑎
𝑄𝑢𝑜𝑡𝑖𝑒𝑛𝑡 𝑅𝑢𝑙𝑒: log 𝑎 − log 𝑏 = log
𝑏
𝑃𝑜𝑤𝑒𝑟 𝑅𝑢𝑙𝑒: log 𝑎𝑏 = 𝑏 log 𝑎
𝑥 log𝑥 𝑎 = 𝑎

Example 4.96
Find the range of 𝑥 for which 𝑆 is a two-digit positive integer, if:
𝑆𝑛 = log 𝑥 𝑥 + log 𝑥 𝑥 2 + log 𝑥 𝑥.3 + ⋯ + log 𝑥 𝑥.𝑥 , 𝑥 ∈ ℕ

Use the power rule on each term:


𝑆𝑛 = 1 log 𝑥 𝑥 + 2 log 𝑥 𝑥 + 3 log 𝑥 𝑥 + ⋯ + 𝑛 log 𝑥 𝑥
Substitute log 𝑥 𝑥 = 1
𝑥(𝑥 + 1)
𝑆 = 1 + 2 +3 +⋯+ 𝑥 =
2
𝑥(𝑥 + 1)
9< < 100
2
Splitting the inequality and solving it will result in two quadratic inequalities, which we must solve and then
find the intersection.

Instead, since 𝑆 is an integer, 𝑥 must also be an integer. Using trial and error, the smallest value that works is 4,
and the largest value 13.
𝑥 ∈ {4, 5, 6, … ,13}

Example 4.97
Show that:
log 𝑥 + log 2𝑥 + ⋯ + log 𝑛𝑥 = log 𝑛! + 𝑛 log 𝑥

A common method of handling series is to break them up logically into two different series that are easier to
handle.

Use the product rule


𝐿𝐻𝑆 = ⏟
log 1 + log 𝑥 + ⏟
log 2 + log 𝑥 + ⋯ + ⏟
log 𝑛 + log 𝑥
𝐹𝑖𝑟𝑠𝑡 𝑇𝑒𝑟𝑚 𝑆𝑒𝑐𝑜𝑛𝑑 𝑇𝑒𝑟𝑚 𝑛𝑡ℎ 𝑡𝑒𝑟𝑚
Rearrange:
= log 1 + log 2 + ⋯ + log 𝑛 + log
⏟ 𝑥 + ⋯ + log 𝑥
𝑛 𝑡𝑖𝑚𝑒𝑠
Use the reverse of the product rule:
= log 1 × 2 × … × 𝑛 + log 𝑥 𝑛
= log 𝑛! + 𝑛 log 𝑥 = 𝑅𝐻𝑆

Example 4.98
If 1, log_9 (3^(1-x)+2), log_3 (4\bullet3^x -1) are in AP, then x can be written in the form 1- log_3 y. Find the
value of y. (JEE Main 2002, Adapted)

P a g e 178 | 274
Get all the files at: https://bit.ly/azizhandouts
Aziz Manva (azizmanva@gmail.com)

Example 4.99
𝑛

∑ log 𝑖
𝑖=1

∑ log 𝑖 = log 1 + log 2 + ⋯ + log 𝑛 = log(1 × 2 × … × 𝑛) = log 𝑛!


𝑖=1

Example 4.100
𝑖
𝑛
∑ log
𝑛+1
𝑛=1

Method I: Shorter Method


𝑖
𝑛 1 2 𝑖 1 2 𝑖 1
∑ log = log ( ) + log ( ) + ⋯ + log ( ) = log ( × × … × ) = log
𝑛+1 2 3 𝑖+1 2 3 𝑖+1 𝑖+1
𝑛=1

Method I
Split the term inside the summation sign:
𝑖

∑ log 𝑛 − log(𝑛 + 1)
𝑛=1
Split the summation:
𝑖 𝑖

= ∑ log 𝑛 − ∑ log(𝑛 + 1)
𝑛=1 𝑛=1
Change the index of summation
𝑖 𝑖+1

= ∑ log 𝑛 − ∑ log(𝑛)
𝑛=1 𝑛=2
𝑖 𝑖

= log 1 + ∑ log 𝑛 − ∑ log(𝑛) − log(𝑖 + 1)


𝑛=2 𝑛=2
1
= log 1 − log(𝑖 + 1) = log ( )
𝑖+1

C. Arithmetic Sequences
In the next section, we will study harmonic sequences, where each term is the reciprocals of an arithmetic
sequence. Hence, we will revise the important properties of arithmetic sequences and series.

4.101: Arithmetic Sequence


An arithmetic sequence with first term 𝑎 has a common difference 𝑑 between successive elements.
𝑎, 𝑎 + 𝑑, 𝑎 + 2𝑑, … , 𝑎 + (𝑛 − 1)𝑑

Some of examples of arithmetic sequences are:


12, 17,22,27, … ⇒ 𝑅𝑢𝑙𝑒 𝑖𝑠: 𝐴𝑑𝑑 5 ⇒ 𝑎 = 12, 𝑑 = 5
31,28,25, … ⇒ 𝑅𝑢𝑙𝑒 𝑖𝑠: 𝑆𝑢𝑏𝑡𝑟𝑎𝑐𝑡 3 ⇒ 𝑎 = 31, 𝑑 = −3

P a g e 179 | 274
Get all the files at: https://bit.ly/azizhandouts
Aziz Manva (azizmanva@gmail.com)

Example 4.102
If 𝑎, 𝑏, 𝑐 form an arithmetic progression with 𝑓𝑖𝑟𝑠𝑡 𝑡𝑒𝑟𝑚 = 𝑎 = 2, and common difference= 2, then find 𝑥: 𝑦
given:
log 𝑎 {log 𝑏 [log 𝑐 𝑥]} = log 𝑐 {log 𝑏 [log 𝑎 𝑦]} = 0

log 𝑎 {log𝑎+2 [log 𝑎+4 𝑥]} = log 𝑎+4 {log 𝑎+2 [log 𝑎 𝑦]}
This is very difficult to do anything with directly because the bases are not the same.
Key Idea: Create Simple Equations
Equating the first and third terms creates an easy equation. Then, equate the second and third terms.
Equating the first and the third term: Equating the second and the third term: Substituting the
log 2{log 4 [log 6 𝑥]} = 0 log 6 {log 4[log 2 𝑦]} = 0 values of 𝒙 and 𝒚:
log 4 [log 6 𝑥] = 1 log 4 [log 2 𝑦] = 1 𝑥: 𝑦 = 64 : 24 = 81: 1
log 6 𝑥 = 4 log 2 𝑦 = 4
𝑥 = 64 𝑦 = 24

4.103: Sub Sequences


Given an arithmetic sequence
𝑎𝑛−𝑘 , … , 𝑎𝑛−1 , 𝑎𝑛 , 𝑎𝑛+1 , … , 𝑎𝑛+𝑘
the terms
𝑎𝑛−𝑘 , 𝑎𝑛 , 𝑎𝑛+𝑘
also form an arithmetic sequence

Given an arithmetic sequence with common difference 5, and middle term 50:
35,40,45, 𝟓𝟎, 55,60,65

The following symmetrical sequences are also arithmetic sequences:


45, 50, 55
40, 50, 60
35, 50, 65

4.104: General Term


For an arithmetic sequence, the general term (also the called the 𝑛𝑡ℎ term) is given by:
𝑡𝑛 = 𝑎 + (𝑛 − 1)𝑑

𝐹𝑖𝑟𝑠𝑡 𝑇𝑒𝑟𝑚 = 𝑎 = 𝑎 + 0𝑑
𝑆𝑒𝑐𝑜𝑛𝑑 𝑇𝑒𝑟𝑚 = 𝑎 + 𝑑 = 𝑎 + 1𝑑
𝑇ℎ𝑖𝑟𝑑 𝑇𝑒𝑟𝑚 = 𝑎 + 2𝑑

And in general, 𝑑 is added one less than the term number:


𝑎 + (𝑛 − 1)𝑑

4.105: Common Difference Property


If the difference between two terms of a sequence is constant, then the sequence is an arithmetic sequence.

43,45,47

P a g e 180 | 274
Get all the files at: https://bit.ly/azizhandouts
Aziz Manva (azizmanva@gmail.com)

47 − 45 = 2 = 45 − 43 ⇒ 𝐴𝑟𝑖𝑡ℎ𝑚𝑒𝑡𝑖𝑐

If the sequence 𝑥 + 5, 2𝑥 + 4, 3𝑥 − 7 is arithmetic:


(2𝑥 + 4) − (𝑥 + 5) = (3𝑥 − 7) − (2𝑥 + 4)
2(2𝑥 + 4) − (𝑥 + 5) = 3𝑥 − 7
4𝑥 + 8 − 𝑥 − 5 = 3𝑥 − 7
3𝑥 + 3 = 3𝑥 − 7

4.106: Arithmetic Mean Property and its converse


Any term of an arithmetic sequence is the arithmetic mean of the terms that precede and follow it.
𝑎𝑛−1 + 𝑎𝑛+1
𝑎𝑛 =
2

𝑎𝑛−1 + 𝑎𝑛+1 [𝑎 + (𝑛 − 2)𝑑] + [𝑎 + 𝑛𝑑] 2𝑎 + 2𝑛𝑑 − 2𝑑


𝑅𝐻𝑆 = = = = 𝑎 + 𝑛𝑑 − 𝑑 = 𝑎 + (𝑛 − 1)𝑑 = 𝐿𝐻𝑆
2 2 2

Converse: If the general term of a sequence is the arithmetic mean of the terms that precede and follow it, then
that sequence is an arithmetic sequence.

𝑎𝑛−1 + 𝑎𝑛+1
𝑎𝑛 =
2
2𝑎𝑛 = 𝑎𝑛−1 + 𝑎𝑛+1
𝑎𝑛 − 𝑎𝑛−1 = 𝑎𝑛+1 − 𝑎𝑛
Since the common difference is the same, the sequence is arithmetic.

The arithmetic mean property has both theoretical and practical importance. We can apply it:
➢ To find missing values when we know that a sequence is arithmetic.
➢ To prove that a sequence is arithmetic

4.107: Middle term is average of all the terms (for odd number of Terms)
The average of an arithmetic sequence with an odd number of terms is equal to its middle term.
𝑎1 + 𝑎2 + ⋯ + 𝑎𝑛
= 𝑀𝑖𝑑𝑑𝑙𝑒 𝑇𝑒𝑟𝑚
𝑛

Three Term Case


𝑎1 = 𝑎 − 𝑑, 𝑎2 = 𝑎, 𝑎3 = 𝑎 + 𝑑
Suppose we consider three terms:
𝑎1 + 𝑎2 + 𝑎3 (𝑎 − 𝑑) + 𝑎 + (𝑎 + 𝑑) 3𝑎
= = = 𝑎 = 𝑀𝑖𝑑𝑑𝑙𝑒 𝑇𝑒𝑟𝑚
3 3 3
General Case
Let 𝑎𝑛−𝑘 , … , 𝑎𝑛−1 , 𝑎𝑛 , 𝑎𝑛+1 , … , 𝑎𝑛+𝑘 be an arithmetic sequence with middle term 𝑎 and common difference 𝑑.
Then, the terms are:
𝑎 − 𝑘𝑑, … , 𝑎 − 𝑑, 𝑎, 𝑎 + 𝑑, … , 𝑎 + 𝑘𝑑
(𝑎 − 𝑘𝑑) + ⋯ + 𝑎 + ⋯ + (𝑎 + 𝑘𝑑) (2𝑘 + 1)𝑎
= = 𝑎 = 𝑀𝑖𝑑𝑑𝑙𝑒 𝑇𝑒𝑟𝑚
2𝑘 + 1 2𝑘 + 1

4.108: Average for Even Number of Terms


The average of an arithmetic sequence with an even number of terms is equal to the average of its two
middlemost terms:

P a g e 181 | 274
Get all the files at: https://bit.ly/azizhandouts
Aziz Manva (azizmanva@gmail.com)

𝒂𝟏 + ⋯ + 𝒂𝒌 + 𝒂𝒌 + ⋯ + 𝒂𝟐𝒌 𝒂𝒌 + 𝒂𝒌
+𝟏 +𝟏
𝟐 𝟐 𝟐 𝟐
=
𝒌 𝟐

11,13,15,17
Average
11 + 13 + 15 + 17 (14 − 3) + (14 − 1) + (14 + 1) + (14 + 3) 14 × 4
= = = = 14
4 4 4

4.109: Symmetry in Average and Median


➢ When a sequence is arranged in ascending (or descending) order, the median is the middle term if there
are an odd number of terms.
➢ An arithmetic sequence has a constant difference. Hence, its arithmetic mean is also its median.

Odd Term Case


𝑎 − 𝑑, 𝑎, 𝑎 + 𝑑 ⇒ 𝑀𝑒𝑑𝑖𝑎𝑛 = 𝑎
𝑎 − 𝑑, 𝑎, 𝑎 + 𝑑 ⇒ 𝑀𝑒𝑎𝑛 = 𝑎
Even Term Case
𝑎 − 3𝑑, 𝑎 − 𝑑, 𝑎 + 𝑑, 𝑎 + 3𝑑 ⇒ 𝑀𝑒𝑑𝑖𝑎𝑛 = 𝐴𝑣𝑔(𝑎 − 𝑑, 𝑎 + 𝑑) = 𝑎
4𝑎
𝑎 − 3𝑑, 𝑎 − 𝑑, 𝑎 + 𝑑, 𝑎 + 3𝑑 ⇒ 𝑀𝑒𝑎𝑛 = =𝑎
4

4.110: Recursive Definition


An arithmetic sequence can be defined as:
𝑎𝑛 = 𝑎𝑛−1 + 𝑑, 𝑎1 = 𝑐

An arithmetic sequence can be defined either recursively, or explicitly, and both definitions are equivalent.
It is also possible to convert from one form to another, and this is useful.

4.111: Converting from Recursive Definition to Explicit Definition


An arithmetic sequence given in recursive form can be converted into explicit form
𝑎𝑛 = 𝑎𝑛−1 + 𝑑,
⏟ 𝑎𝑛 = 𝑐 + (𝑛 − 1)𝑑
𝑎1 = 𝑐 ⇔ ⏟
𝑅𝑒𝑐𝑢𝑟𝑠𝑖𝑣𝑒 𝐷𝑒𝑓𝑖𝑛𝑖𝑡𝑖𝑜𝑛 𝐸𝑥𝑝𝑙𝑖𝑐𝑡 𝐷𝑒𝑓𝑖𝑛𝑖𝑡𝑖𝑜𝑛

𝑎1 = 𝑐
𝑎2 = 𝑐 + 𝑑
𝑎3 = 𝑐 + 𝑑 + 𝑑 = 𝑐 + 2𝑑
𝑎𝑛 = 𝑐 + (𝑛 − 1)𝑑
D. Arithmetic Series

4.112: Sum of an Arithmetic Series


The sum of an arithmetic series with 𝑓 = 𝐹𝑖𝑟𝑠𝑡 𝑇𝑒𝑟𝑚, 𝑙 = 𝐿𝑎𝑠𝑡 𝑇𝑒𝑟𝑚, 𝑛 = 𝑁𝑜. 𝑜𝑓 𝑡𝑒𝑟𝑚𝑠 is:

𝑓+𝑙
𝑆𝑛 = 𝑛 × ( ) = No. of Terms(𝐴𝑣𝑔. 𝑜𝑓 𝐹𝑖𝑟𝑠𝑡 𝑎𝑛𝑑 𝐿𝑎𝑠𝑡 𝑇𝑒𝑟𝑚)
2

4.113: Sum of an arithmetic Series


𝑛
𝑆𝑛 = [2𝑎 + (𝑛 − 1)𝑑]
2
P a g e 182 | 274
Get all the files at: https://bit.ly/azizhandouts
Aziz Manva (azizmanva@gmail.com)

𝑎 = 𝐹𝑖𝑟𝑠𝑡 𝑇𝑒𝑟𝑚, 𝑛 = 𝑁𝑜. 𝑜𝑓 𝑡𝑒𝑟𝑚𝑠, 𝑑 = 𝐶𝑜𝑚𝑚𝑜𝑛 𝐷𝑖𝑓𝑓𝑒𝑟𝑒𝑛𝑐𝑒

We write the sum twice, first in regular order, and then back to front.

First Term Second Term 𝒏𝒕𝒉 Term


S 𝑎 𝑎+𝑑 . . . 𝑎 + (𝑛 − 1)𝑑

S 𝑎 + (𝑛 − 1)𝑑 𝑎 + (𝑛 − 2)𝑑 𝑎

2S 2𝑎 + (𝑛 − 1)𝑑 2𝑎 + (𝑛 − 1)𝑑 . . . 2𝑎 + (𝑛 − 1)𝑑

𝑛
2𝑆 = 𝑛[2𝑎 + (𝑛 − 1)𝑑] ⇒ 𝑆 = [2𝑎 + (𝑛 − 1)𝑑]
2
E. Geometric Sequences
When considering questions with harmonic sequences, they also require concepts from geometric sequences
and series. Hence, we revise these as well.

4.114: Geometric Sequence


A geometric sequence with first term 𝑎 ≠ 0 has a common ratio 𝑟 ≠ 0 between successive elements, and a
geometric sequence of 𝑛 terms is represented by:
𝑎, 𝑎𝑟, 𝑎𝑟 2 , … , 𝑎𝑟 𝑛−1

For example
2,4,8, …
1 1 1
, , ,…
3 9 27

4.115: Common Ratio: Consecutive Terms


Any two consecutive terms of a geometric sequence will be
𝑎𝑟 𝑥 , 𝑎𝑟 𝑥+1
If we divide any two consecutive terms of a geometric sequence, we get
𝑎𝑟 𝑥+1 𝑟 𝑥+1−𝑥
= =𝑟
𝑎𝑟 𝑥 1

➢ The common ratio of a geometric sequence can also be negative.


➢ If you are given terms 𝑎𝑛 , 𝑎𝑛+𝑘 , 𝑘 𝑖𝑠 𝑒𝑣𝑒𝑛, then you will get two values for the common ratio.

4.116: 𝑵𝒕𝒉 Term of a geometric sequence


➢ The 𝑛𝑡ℎ term is given by 𝑎𝑟 𝑛−1 .
➢ If we equate to the value of the term, we can solve for 𝑛.

To find the 𝑛𝑡ℎ term of a geometric sequence, substitute the values of 𝑎 and 𝑟 as constants, and leave 𝑛 as 𝑛.

4.117: Subsequences
Consider a geometric sequence with middle term 𝑎 and common ratio 𝑟
𝑎 𝑎
… , 2 , , 𝑎, 𝑎𝑟, 𝑎𝑟 2 , …
𝑟 𝑟

If 𝑎𝑛 is the middle term, then 𝑎𝑛−𝑘 , 𝑎𝑛 , 𝑎𝑛+𝑘 are also in geometric sequence with common ratio

P a g e 183 | 274
Get all the files at: https://bit.ly/azizhandouts
Aziz Manva (azizmanva@gmail.com)

𝑟𝑘

𝑎
𝑘=1⇒ , 𝑎, 𝑎𝑟
𝑟
𝑎
𝑘 = 2 ⇒ 2 , 𝑎, 𝑎𝑟 2
𝑟

4.118: Terms in 𝑮𝑷 ⇒Exponents/Logs in AP


Consider a geometric progression given by 𝑥 𝑎 , 𝑥 𝑏 , 𝑥 𝑐 . Then, 𝑎, 𝑏, 𝑐 are in arithmetic progression.

A geometric progression 𝑥 𝑎 , 𝑥 𝑏 , 𝑥 𝑐 with common ratio 𝑟 = 𝑥 𝑑 can be written:


𝑥 𝑎 , 𝑥 𝑏 , 𝑥 𝑐 = 𝑥 𝑎 , 𝑟𝑥 𝑎 , 𝑟 2 𝑥 𝑎 = 𝑥 𝑎 , 𝑥 𝑑 𝑥 𝑎 , 𝑥 2𝑑 𝑥 𝑎 = 𝑥 𝑎 , 𝑥 𝑎+𝑑 , 𝑥 𝑎+2𝑑

And taking the logs gives us:


log 𝑥 𝑎 , log 𝑥 𝑎+𝑑 , log 𝑥 𝑎+2𝑑 = 𝑎 log 𝑥 , (𝑎 + 𝑑) log 𝑥 , (𝑎 + 2𝑑) log 𝑥

4.119: Logs of the Terms in 𝑨𝑷 ⇒Exponents are in GP

ln 𝑎 , ln 𝑏 , ln 𝑐 → 𝐴𝑃
2 ln 𝑏 = ln 𝑎 + ln 𝑐
ln 𝑏 2 = ln 𝑎𝑐
𝑏 2 = 𝑎𝑐
𝑏 = √𝑎𝑐
𝑏 is the geometric mean of 𝑎 and 𝑐.
𝑎, 𝑏, 𝑐 𝑎𝑟𝑒 𝑖𝑛 𝑔𝑒𝑜𝑚𝑒𝑡𝑟𝑖𝑐 𝑝𝑟𝑜𝑔𝑟𝑒𝑠𝑠𝑖𝑜𝑛

4.120: Properties
log 𝑎, log 𝑏, log 𝑐 are in arithmetic progression if and only if 𝑎, 𝑏, 𝑐 are in geometric progression.

log 𝑎, log 𝑏, log 𝑐 ⇔ 2 log 𝑏 = log 𝑎 + log 𝑐 ⇔ log 𝑏 2 = log 𝑎𝑐 ⇔ 𝑏 2 = 𝑎𝑐 ⇔ ⏟


⏟ 𝑏 = √𝑎𝑐
𝑖𝑛 𝐴𝑃 𝑖𝑛 𝐺𝑃

Method II
𝑞
Consider quantities 𝑎, 𝑏, 𝑐 in geometric progression with first term 𝑟 and common ratio 𝑟. Then:
𝑞
𝑎 = , 𝑏 = 𝑞, 𝑐 = 𝑞𝑟
𝑟
Take the log of each term above:
𝑞
log , log 𝑞 , log 𝑞𝑟
𝑟
Use the product and quotient rules:
log 𝑞 − log 𝑟 , log 𝑞 , log 𝑞 + log 𝑟
This is an arithmetic progression with first term log 𝑞 − log 𝑟 and common difference log 𝑟.

𝑞
log 𝑟 + log 𝑞𝑟 1
= log 𝑞 2 = log 𝑞
2 2

Example 4.121
P a g e 184 | 274
Get all the files at: https://bit.ly/azizhandouts
Aziz Manva (azizmanva@gmail.com)

𝑎2 𝑎3
Show that log 𝑎 , log 𝑏
, log 𝑏2 , … form an arithmetic progression.

Method I
𝑎2 𝑎3
Rewrite log 𝑎 , log 𝑏
, log 𝑏2
, … to get:
𝑎 𝑎 2
log 𝑎 , log 𝑎 ( ) , log 𝑎 ( )
𝑏 𝑏
𝑎 𝑎
log 𝑎 , log
⏟𝑎 + log ( ) , log 𝑎 + 2 log ( )
⏟ 𝑏 𝑏
𝑭𝒊𝒓𝒔𝒕
𝑻𝒆𝒓𝒎 𝑪𝒐𝒎𝒎𝒐𝒏
𝑫𝒊𝒇𝒇𝒆𝒓𝒆𝒏𝒄𝒆
Method II
𝑎2 𝑎2 𝑎
log
− log 𝑎 = log = log ( )
𝑏 𝑎𝑏 𝑏
𝑎3 𝑎2 𝑎3 𝑎2 𝑎3 𝑏 𝑎
log 2 − log = log 2 ÷ = log 2 × 2 = log
𝑏 𝑏 𝑏 𝑏 𝑏 𝑎 𝑏
Method III
𝑎3 1
log 𝑎 + log 4 4 2 2
𝑏 = log = log ( ) = log 𝑎
2 1 𝑎 𝑎
2 2 𝑏2 𝑏2 𝑏
Method IV
The general term is
𝑎𝑛
𝑎𝑛 = log
𝑏 𝑛−1
We show that this sequence is an arithmetic progression by finding the difference between consecutive terms:
𝑎𝑛+1 𝑎𝑛 𝑎𝑛+1 𝑎𝑛 𝑎𝑛+1 𝑏 𝑛−1 𝑎
𝑎𝑛+1 − 𝑎𝑛 = log 𝑛 − log 𝑛−1 = log 𝑛 ÷ 𝑛−1 = log 𝑛 × 𝑛 = log ⇒ 𝐴𝑃
𝑏 𝑏 𝑏 𝑏 𝑏 𝑎 ⏟𝑏
𝑪𝒐𝒏𝒔𝒕𝒂𝒏𝒕
Since the difference between two terms is a constant, the sequence is an arithmetic progression.

Example 4.122
If 𝑎, 𝑏, and 𝑐 are in geometric progression (G.P.) with 1 < 𝑎 < 𝑏 < 𝑐 and 𝑛 > 1 is an integer, then log 𝑎 𝑛,
log 𝑏 𝑛 , log 𝑐 𝑛 form a sequence
A. which is a G.P
B. which is an arithmetic progression (A.P)
C. in which the reciprocals of the terms form an A.P
D. in which the second and third terms are the n^th powers of the first and second respectively
E. none of these (AHSME 1973/28)

Since a, b, c are in geometric progression, for a ratio 𝑟 > 1:


𝑏 = 𝑎𝑟, 𝑐 = 𝑎𝑟 2
Hence, the sequence becomes
log 𝑎 𝑛 , log 𝑎𝑟 𝑛 , log 𝑎𝑟2 𝑛
1
Use the property that log 𝑎 𝑥 = , and hence the reciprocals will interchange the number and the base:
log𝑥 𝑎
log 𝑛 𝑎 , log 𝑛 𝑎𝑟 , log 𝑛 𝑎𝑟 2
Expand:
log 𝑛 𝑎 , log 𝑛 𝑎 + log 𝑛 𝑟 , log 𝑛 𝑎 + 2 log 𝑛 𝑟
And this is now an arithmetic sequence with 𝑓𝑖𝑟𝑠𝑡 𝑡𝑒𝑟𝑚 = log 𝑛 𝑎 and 𝑐𝑜𝑚𝑚𝑜𝑛 𝑑𝑖𝑓𝑓𝑒𝑟𝑒𝑛𝑐𝑒 = 𝑑 = log 𝑛 𝑟
Option C.

P a g e 185 | 274
Get all the files at: https://bit.ly/azizhandouts
Aziz Manva (azizmanva@gmail.com)

Example 4.123
7
If log 3 2, log 3 (2𝑥 − 5) and log 3 (2𝑥 − ) are in arithmetic progression, then determine the value of 𝑥. (JEE Adv.
2
1990)

Use a change of variable. Let 𝑦 = 2𝑥 .


Also, if the logs of the terms are in arithmetic progression, then the terms themselves must be in geometric
progression:
7
(𝑦 − 5)2 = 2 (𝑦 − )
2

This is a quadratic. Expand and collate all terms on one side:


𝑦 2 − 12𝑦 + 32 = 0

Change back to the original variable


𝑦 = {4,8} ⇒ 2𝑥 = {𝟐𝟐 , 23 }
However:
log 3 (2𝑥 − 5) = log 3 (4 − 5) = log 3 (−1) ⇒ 𝑁𝑜𝑡 𝑑𝑒𝑓𝑖𝑛𝑒𝑑

𝑥=3

4.124: Geometric Mean Property-I


Any term of a geometric sequence is the geometric mean of the terms that precede and follow it:
𝑎𝑛−1 , 𝑎𝑛 , 𝑎𝑛+1 𝑎𝑟𝑒 𝑎 𝑔𝑒𝑜𝑚𝑒𝑡𝑟𝑖𝑐 𝑠𝑒𝑞𝑢𝑒𝑛𝑐𝑒 ⇒ 𝑎𝑛 = √𝑎𝑛−1 × 𝑎𝑛+1

√𝑎𝑛−1 × 𝑎𝑛+1 = √𝑎𝑟 𝑛−2 × 𝑎𝑟 𝑛 = √𝑎2 𝑟 2𝑛−2 = 𝑎𝑟 𝑛−1 = 𝑎𝑛


Equivalently:
(𝑎𝑛 )2 = 𝑎𝑛−1 × 𝑎𝑛+1

Note: The geometric mean of two numbers 𝑎 and 𝑏 is given by the square root of the product of the two
numbers. Geometric mean is usually not defined if the numbers for which we are taking the mean are negative.

4.125: Geometric Mean Property-II


If the general term of a sequence is the geometric mean of the terms that precede and follow it, then the
sequence is a geometric sequence.
𝑎𝑛 = √𝑎𝑛−1 × 𝑎𝑛+1 ⇒ 𝑎𝑛−1 , 𝑎𝑛 , 𝑎𝑛+1 𝑎𝑟𝑒 𝑎 𝑔𝑒𝑜𝑚𝑒𝑡𝑟𝑖𝑐 𝑠𝑒𝑞𝑢𝑒𝑛𝑐𝑒

Suppose the equation given in the property above holds. Then:


𝑎𝑛 = √𝑎𝑛−1 × 𝑎𝑛+1
Square both sides:
(𝑎𝑛 )2 = 𝑎𝑛−1 × 𝑎𝑛+1
Let
𝑎𝑛−1 = 𝑎 ⇒ 𝑎𝑛 = 𝑟𝑎𝑛−1 = 𝑟𝑎, 𝑓𝑜𝑟 𝑠𝑜𝑚𝑒 𝑟
(𝑟𝑎)2 = 𝑎 × 𝑎𝑛+1
𝑎𝑟 2 = 𝑎𝑛+1
2
Hence, the terms are: ⏟
𝑎 , 𝑎𝑟
⏟ , 𝑎𝑟
⏟ ⇒ 𝐺𝑒𝑜𝑚𝑒𝑡𝑟𝑖𝑐 𝑆𝑒𝑞𝑢𝑒𝑛𝑐𝑒
𝑎𝑛−1 𝑎𝑛 𝑎𝑛+1

P a g e 186 | 274
Get all the files at: https://bit.ly/azizhandouts
Aziz Manva (azizmanva@gmail.com)

4.126: Product of Terms

𝑎 𝑎
A geometric sequence … , 𝑟⏟2 , ⏟
𝑟
, ⏟
𝑎 , 𝑎𝑟
⏟ ⏟2 , … with an even number of terms has two
, 𝑎𝑟
𝑴𝒊𝒅𝒅𝒍𝒆=𝒂 𝑴𝒊𝒅𝒅𝒍𝒆=𝒂 𝒂𝒏+𝟐
𝒂𝒏−𝟐 𝒂𝒏−𝟏 𝑻𝒆𝒓𝒎 𝒏 𝑻𝒆𝒓𝒎 𝒏+𝟏

middle terms. And each product below is equal to the product of the two middle terms.
𝑎𝑛 𝑎𝑛+1 = 𝑎 × 𝑎𝑟 = 𝑎2 𝑟
𝑎
𝑎𝑛−1 𝑎𝑛+2 = × 𝑎𝑟 2 = 𝑎2 𝑟
𝑟
𝑎𝑛−𝑘+1 𝑎𝑛+𝑘 = 𝑎2 𝑟

4.127: Inserting Geometric Means


Given values 𝑥 and 𝑦, the geometric mean property can be used to “insert” geometric means between them.

4.128: Geometric Sequence: Recursive Definition


A geometric sequence is defined as:
𝑎𝑛 = 𝑟𝑎𝑛−1 ,
⏟ 𝑎⏟1 = 𝑐
𝑅𝑒𝑐𝑢𝑟𝑠𝑖𝑣𝑒 𝐵𝑎𝑠𝑒 𝐶𝑎𝑠𝑒
𝐷𝑒𝑓𝑖𝑛𝑖𝑡𝑖𝑜𝑛

➢ A recursive definition is one that depends on the value that comes before it.
➢ That is, the 𝑛𝑡ℎ term of the sequence is 𝑟 times the (𝑛 − 1)𝑠𝑡 term of the sequence.
➢ And, we will also be given the starting value, which is the base case.

We can convert from the recursive definition to the explicit definition:


𝑎
⏟𝑛 = 𝑟𝑎𝑛−1 , 𝑎1 = 𝑎 ⇒ ⏟ 𝐶𝑜𝑚𝑚𝑜𝑛 𝑟𝑎𝑡𝑖𝑜 = 𝑟, 𝑛𝑡ℎ 𝑡𝑒𝑟𝑚 = 𝑎𝑟 𝑛−1
𝑹𝒆𝒄𝒖𝒓𝒔𝒊𝒗𝒆 𝑫𝒆𝒇𝒊𝒏𝒊𝒕𝒊𝒐𝒏 𝑬𝒙𝒑𝒍𝒊𝒄𝒊𝒕 𝑫𝒆𝒇𝒊𝒏𝒊𝒕𝒊𝒐𝒏

F. Geometric Series

4.129: Sum of Infinite Geometric Series


The sum 𝑆 of an infinite geometric series (𝑎 + 𝑎𝑟 + 𝑎𝑟 2 + ⋯ ) with first term 𝑎, and common ratio 𝑟 is given by:
𝑎
𝑆=
1−𝑟

−1 < 𝑟 < 1 is the convergence condition. If the condition is not met, the series diverges (has an infinite value).

Example 4.130
log10 √√√𝑥
log10(𝑥 log10 √𝑥 ) + log10 (𝑥 log10 √√𝑥 ) + log10 (𝑥 )+ ⋯∞ = 4
𝑎
If 𝑥 ∈ 𝑆, the sum of the elements of 𝑆 can be written as , where 𝑎, 𝑏 ∈ ℝ, 𝐻𝐶𝐹(𝑎, 𝑏) = 1, then find the sum of
𝑏
digits of 𝑎 + 𝑏.

Change the radicals in the expression to fractional exponents:


1 1 1
log10 (𝑥 log10 𝑥 2 ) + log10 (𝑥 log10 𝑥 4 ) + log10 (𝑥 log10 𝑥 8 ) + ⋯ ∞ = 4
Use the product rule log 𝑎 + log 𝑏 = log 𝑎𝑏 in the given equation:

P a g e 187 | 274
Get all the files at: https://bit.ly/azizhandouts
Aziz Manva (azizmanva@gmail.com)
1 1 1
log10 (𝑥 log10 𝑥 2 ∙ 𝑥 log10 𝑥 4 ∙ 𝑥 log10 𝑥 8 … ) = 4
Use the laws of exponents 𝑥 𝑎 𝑥 𝑏 𝑥 𝑐 …
1 1 1
log10 (𝑥 log10 𝑥 2 +log10 𝑥 4 +log10 𝑥 8 +⋯ ) = 4
Use the product rule one more time:
1 1 1
+⋯
log10 (𝑥 log10 𝑥 2 ∙𝑥 4 ∙𝑥8 )=4
Use the exponent rule one more time:
1 1 1
+ + +⋯
log10 (𝑥 log10 𝑥 2 4 8
)=4
1 1
1 1 𝑎 2 2
The exponent has an infinite geometric series with sum 𝑎 = , 𝑟 = ⇒ 𝑆𝑢𝑚 = = 1 = 1 =1
2 2 1−𝑟 1−
2 2
log10 (𝑥 log10 𝑥 ) = 4
Use the power rule:
(log10 𝑥) = 4
(log10 𝑥)2 = 4

(log10 𝑥) = 2 ⇒ 𝑥 = 102 = 100


1
(log10 𝑥) = −2 ⇒ 𝑥 = 10−2 =
100

1
𝑆 ∈ {100, }
100
𝑎 1 10001
𝑆𝑢𝑚 𝑜𝑓 𝑒𝑙𝑒𝑚𝑒𝑛𝑡𝑠 𝑜𝑓 𝑆 = = 100 + =
𝑏 100 100
𝑎 + 𝑏 = 10001 + 100 = 10101
𝑆𝑢𝑚 𝑜𝑓 𝐷𝑖𝑔𝑖𝑡𝑠 = 1 + 1 + 1 = 3

Example 4.131
If 𝑥, 𝑦 ∈ 𝑅, 𝑥 > 0 such that:
1 1
𝑦 = log10 𝑥 + log10 𝑥 3 + log10 𝑥 9 + ⋯ ∞
And
2 + 4 + 6 + ⋯ + 2𝑦 4
=
3 + 6 + 9 + ⋯ + 3𝑦 log10 𝑥
then the ordered pair (𝑥, 𝑦) is equal to (JEE 2021, 27 Aug, Shift-I)

This looks like a system of equations in 𝑥 and 𝑦, but 𝑦 can be eliminated from the second equation directly.
2(1 + 2 + ⋯ + 𝑦) 4
=
3(1 + 2 + ⋯ + 𝑦) log10 𝑥
Cancel (1 + 2 + ⋯ + 𝑦) from numerator and denominator:
2 4
=
3 log10 𝑥
Cross-multiply:
log10 𝑥 = 6

𝑬𝒒𝒖𝒂𝒕𝒊𝒐𝒏 𝑰
6
𝑥 = 10

1 1
𝑦 = log10 𝑥 1 + log10 𝑥 3 + log10 𝑥 9 + ⋯ ∞

P a g e 188 | 274
Get all the files at: https://bit.ly/azizhandouts
Aziz Manva (azizmanva@gmail.com)

Use the power rule log 𝑥 𝑎 = 𝑎 log 𝑥 in the first equation to get:
1 1
𝑦 = 1 ∙ log10 𝑥 + log10 𝑥 + log10 𝑥 + ⋯
3 9
Factor log10 𝑥:
1 1
𝑦 = log10 𝑥 (1 + + + ⋯ )
3 9
Substitute Equation in the above:
1 1
𝑦 = 6 (1 + + + ⋯ )
3 9
1 𝑎
The expression in parenthesis is an infinite geometric series. Substitute 𝑎 = 1, 𝑟 = in :
3 1−𝑟

1 1 3
𝑦 = 6( ) = 6( ) = 6 × = 9
1 2 2
1−3 3

(𝑥, 𝑦) = (106 , 9)

Example 4.132
Find 𝛼 𝛽 given that
1 1 1
𝛼 = 0.16, 𝛽 = log 2.5 ( + 2 + 3 + ⋯ ) (𝑱𝑬𝑬 𝑴𝒂𝒊𝒏 𝟐𝟎𝟐𝟎, 𝟑 𝑺𝒆𝒑, 𝑺𝒉𝒊𝒇𝒕 − 𝑰)
3 3 3

Find the value of 𝜷 Substitute 𝛽 = − log 5 2:


2
The expression in 𝛽 is an infinite geometric series.
1 1 𝑎 4 − log52 2
Substitute 𝑎 = 3 , 𝑟 = 3 in 1−𝑟: =( )
25
1 1 𝑎 −𝑥 𝑏 𝑥
3 1 Use ( ) =( ) :
= log 10 ( ) = log 5 ( 3 ) = log 5 ( ) 𝑏 𝑎
4 1 2 2 2 2 25 log52 2 5 2 log52 2
1−3 3 =( ) =( )
Rewrite using the rules of exponents: 4 2
= log 5 (2−1 ) Use the power rule for logs:
2
2 5 log52 2 5 log52 4
Use the power rule for logarithms: =( ) =( )
2 2
= − log 5 2 Use 𝑎log𝑎 𝑥 = 𝑥:
2
Find the value of 𝜶𝜷 =4
16 𝛽 4 𝛽
𝛼 𝛽 = (0.16)𝛽 = ( ) =( )
100 25

Example 4.133
1 1 1 1
The product 24 ∙ 416 ∙ 848 ∙ 16128 ∙ … is equal to (JEE Main 2020, 9 Jan, Shift-I)

Convert all the bases to powers of 2:


1 1 1 1
24 ∙ (22 )16 ∙ (23 )48 ∙ (24 )128 ∙ …
Use the exponent rule (𝑎𝑚 )𝑛 = 𝑎𝑚𝑛 :
1 1 1 1
24 ∙ 28 ∙ 216 ∙ 232 ∙ …
Use the exponent rule 𝑎𝑚 ∙ 𝑎𝑛 = 𝑎𝑚+𝑛 :
1 1 1 1
24+8+16+32
1 1 𝑎
The exponent is a geometric series with 𝑎 = 4 , 𝑟 = 2. Substitute in 1−𝑟:

P a g e 189 | 274
Get all the files at: https://bit.ly/azizhandouts
Aziz Manva (azizmanva@gmail.com)
1 1
4 4
1 1 1
1−
2 2 = 22 = 22 = √2

4.134: Double Series


Suppose we have two geometric series given by:
𝑎 + 𝑎𝑟 + 𝑎𝑟 2 + ⋯.
𝑏 + 𝑏𝑞 + 𝑏𝑞 2 + ⋯
A double series consists of alternating terms from the above two series:
𝑎 + 𝑏 + 𝑎𝑟 + 𝑏𝑞 + 𝑎𝑟 2 + 𝑏𝑞 2 + ⋯

4.135: Squares of Terms of Infinite Geometric Series


Squaring each term of the geometric series with 𝑓𝑖𝑟𝑠𝑡 𝑡𝑒𝑟𝑚 = 𝑎, and 𝑐𝑜𝑚𝑚𝑜𝑛 𝑟𝑎𝑡𝑖𝑜 𝑟:
𝑎 + 𝑎𝑟 + 𝑎𝑟 2 + ⋯ + 𝑎𝑟 𝑛−1
Gives a geometric series with 𝑓𝑖𝑟𝑠𝑡 𝑡𝑒𝑟𝑚 = 𝑎2 and 𝑐𝑜𝑚𝑚𝑜𝑛 𝑟𝑎𝑡𝑖𝑜 = 𝑟 2 :
𝑎2 + 𝑎2 𝑟 2 + 𝑎2 𝑟 4 + ⋯ + 𝑎2 𝑟 2(𝑛−1)

4.136: Odd and Even Power Decomposition


If I have an infinite geometric series with first term 𝑎, and common ratio 𝑟, then:
𝑎
𝑆𝑢𝑚 𝑜𝑓 𝑒𝑣𝑒𝑛 𝑝𝑜𝑤𝑒𝑟𝑠 =
1 − 𝑟2
𝑎𝑟
𝑆𝑢𝑚 𝑜𝑓 𝑜𝑑𝑑 𝑝𝑜𝑤𝑒𝑟𝑠 =
1 − 𝑟2

We can decompose (break up) the geometric series with first term 𝑎 and common ratio 𝑟::
𝒂 + 𝑎𝑟 + 𝒂𝒓𝟐 + 𝑎𝑟 3 + 𝒂𝒓𝟒 + 𝑎𝑟 5 + ⋯ = ⏟
(𝑎𝑟 0 + 𝑎𝑟 2 + 𝑎𝑟 4 + ⋯ ) + ⏟
(𝑎𝑟 + 𝑎𝑟 3 + 𝑎𝑟 5 + ⋯ )
𝑬𝒗𝒆𝒏 𝑷𝒐𝒘𝒆𝒓𝒔 𝒐𝒇 𝒓 𝑶𝒅𝒅 𝑷𝒐𝒘𝒆𝒓𝒔 𝒐𝒇 𝒓
The even powers form a geometric series with first term 𝑎, and common ratio 𝑟 2 :
𝑎
𝑎𝑟 0 + 𝑎𝑟 2 + 𝑎𝑟 4 + ⋯ =
1 − 𝑟2
The odd powers form a geometric series with first term 𝑎𝑟, and common ratio 𝑟 2 :
𝑎𝑟
𝑎𝑟 + 𝑎𝑟 4 + 𝑎𝑟 5 + ⋯ =
1 − 𝑟2

4.137: Sum of a Finite Geometric Series


The sum of a finite geometric series 𝑎 + 𝑎𝑟 + 𝑎𝑟 2 + ⋯ + 𝑎𝑟 𝑛−1 with first term 𝑎, common ratio 𝑟 and 𝑛 terms is
given by:
𝑎(1 − 𝑟 𝑛 ) 𝑎(𝑟 𝑛 − 1)
𝑆𝑛 = =
1−𝑟 𝑟−1

We can define a finite geometric series with first term 𝑎, common ratio 𝑟 and 𝑛 terms as:
𝑆 = 𝑎 + 𝒂𝒓 + 𝒂𝒓𝟐 + ⋯ + 𝒂𝒓𝒏−𝟏

𝐸𝑞𝑢𝑎𝑡𝑖𝑜𝑛 𝐼
Multiply both sides by 𝑟:
𝑟𝑆 = 𝒂𝒓 + 𝒂𝒓𝟐 + ⋯ + 𝒂𝒓𝒏−𝟏 + 𝑎𝑟 𝑛

𝐸𝑞𝑢𝑎𝑡𝑖𝑜𝑛 𝐼𝐼
Subtract Equation I from Equation II:
𝑟𝑆 − 𝑆 = 𝑎𝑟 𝑛 − 𝑎
𝑆(𝑟 − 1) = 𝑎(𝑟 𝑛 − 1)
Solve for 𝑆:

P a g e 190 | 274
Get all the files at: https://bit.ly/azizhandouts
Aziz Manva (azizmanva@gmail.com)

𝑎(𝑟 𝑛 − 1) 𝑎(1 − 𝑟 𝑛 )
𝑆= =
𝑟−1 1−𝑟

4.4 Harmonic Sequences and Series8


A. Basics

4.138: Harmonic Sequence


If the reciprocals of the terms of a sequence form an arithmetic sequence, then that sequence is harmonic.
1 1 1
𝑎
⏟− 𝑑, 𝑎, 𝑎 + 𝑑 ⇔ , ,

𝑎 − 𝑑 𝑎 𝑎+𝑑
𝑨𝒓𝒊𝒕𝒉𝒎𝒆𝒕𝒊𝒄 𝑺𝒆𝒒𝒖𝒆𝒏𝒄𝒆
𝑯𝒂𝒓𝒎𝒐𝒏𝒊𝒄 𝑺𝒆𝒒𝒖𝒆𝒏𝒄𝒆

A standard technique with harmonic sequences is


➢ Take the reciprocals of the terms. This results in an arithmetic sequence
➢ Use the properties of arithmetic sequences to manipulate the sequence as required.
➢ Take the reciprocal again to convert the sequence back to the original sequence.

Example 4.139
1 1 1
A. The first three terms of a harmonic sequence are 2 , 3 and 4. Find the fourth term.
1 1 1
B. Find the 𝑛𝑡ℎ term of the harmonic sequence: 𝑥 , 𝑥+2 , 2𝑥−2
C. A harmonic progression is a sequence of numbers such that their reciprocals are in arithmetic
progression. Let 𝑆𝑛 represent the sum of the first 𝑛 terms of the harmonic progression; for example, 𝑆3
represents the sum of the first three terms. If the first three terms of a harmonic progression are 3,4,6,
then 𝑆4 = (AHSME 1959/33, Adapted)

2𝑥 + 4 = 3𝑥 − 2
Part A 𝑥=6
Convert the given sequence into arithmetic by The given terms are:
taking the reciprocal: 1 1 1
, , ⇒ 6,8,10 ⇒ 𝑑 = 2, 𝑎 = 6
2,3,4
⏟ 6 8 10
𝐴𝑟𝑖𝑡ℎ𝑚𝑒𝑡𝑖𝑐 The general term is:
This has common difference = 𝑑 = 1 1 1 1
𝑡4 = 5 = =
(𝑛
6 + − 1)2 6 + 2𝑛 − 2 4 + 2𝑛
Find the fourth term of the harmonic sequence by Part C
again taking the reciprocal: 1 1 1
1 1 𝐻𝑃 = 3,4,6 ⇒ 𝐴𝑃 = , ,
= 3 4 6
𝑡4 5 1
𝑑=−
Part B 12
Convert into an arithmetic sequence: 1
𝑡4 =
𝑥, 𝑥 + 2, 2𝑥 − 2 12
Apply the arithmetic mean property: 𝑆4 = 3 + 4 + 6 + 12 = 25
2(𝑥 + 2) = 𝑥 + 2𝑥 − 2

Example 4.140
Let 𝑎1 , 𝑎2 , … , 𝑎10 be in AP, and ℎ1 , ℎ2 , . . . , ℎ10 be in HP. If 𝑎1 = ℎ1 = 2 and 𝑎10 = ℎ10 = 3, then 𝑎4 ℎ7 is: (JEE Adv.
1992)

8 Many exam questions make use of AP/GP. The previous section has a summary of the properties of AP/GP.

P a g e 191 | 274
Get all the files at: https://bit.ly/azizhandouts
Aziz Manva (azizmanva@gmail.com)

Arithmetic Sequence
The common difference of the arithmetic sequence is:
𝑎10 − 𝑎1 3 − 2 1
𝑑= = =
10 − 1 9 9
The fourth term is:
1 1 7
𝑎4 = 𝑎1 + (𝑛 − 1)𝑑 = 2 + (4 − 1) = 2 =
9 3 3
Harmonic Sequence
1 1 1 1 1
The reciprocals of the harmonic sequence ℎ1 , ℎ2 , . . . , ℎ10 form an arithmetic sequence ℎ = 2 , ℎ , … , ℎ = 3,
1 2 10
which has common difference:
1 1 1 1

ℎ10 ℎ1 3 − 2 2 − 3 1 1
𝑑= = = × =−
10 − 1 9 6 9 54
And hence the seventh term of the arithmetic sequence is:
1 1 1 1 1 1 7
= + (𝑛 − 1)𝑑 = + (7 − 1) (− ) = − =
ℎ7 ℎ1 2 54 2 9 18
The required product is then:
7 18
𝑎4ℎ7 = ( ) ( ) = 6
3 7

Example 4.141
If a harmonic sequence of five terms has first term = ℎ1 = 2, and fifth term = ℎ5 = 3, then:
A. Find ℎ2 , ℎ3 , ℎ4
B. An infinite geometric sequence is formed by taking any two values of the harmonic sequence as its first
two terms. Find the number of such sequences that are convergent.

Part A
1 1 1 1 1
Since ℎ1 , ℎ2 , ℎ3 , ℎ4 , ℎ5 are in harmonic sequence, their reciprocals ( , , , , ) are in arithmetic sequence
ℎ1 ℎ2 ℎ3 ℎ4 ℎ 5
with common difference is:
1 1 1 1 1

ℎ5 ℎ1 3 − 2 − 6 1
𝑑= = = =−
4 4 4 24
1 1 1 1 12 1 11 24
= +𝑑 = − = − = ⇒ ℎ2 =
ℎ2 ℎ1 2 24 24 24 24 11
1 1 1 1 12 2 10 5 12
= + 2𝑑 = − 2 ∙ = − = = ⇒ ℎ3 =
ℎ3 ℎ1 2 24 24 24 24 12 5
1 1 1 1 12 3 9 3 8
= + 3𝑑 = − 3 ∙ = − = = ⇒ ℎ3 =
ℎ4 ℎ1 2 24 24 24 24 8 3
Part B
If we pick two terms ℎ𝑎 and ℎ𝑏 with 𝑏 > 𝑎, then ℎ𝑏 > ℎ𝑎 and the common ratio is greater than 1. the series is
not convergent. For example:
3
𝑎 = 1, 𝑏 = 5 ⇒ ℎ1 + ℎ5 = 2 + 3 + ⋯ ⇒ 𝑎 = 2, 𝑟 = > 1 ⇒ 𝑁𝑜𝑡 𝐶𝑜𝑛𝑣𝑒𝑟𝑔𝑒𝑛𝑡
2

If we pick two terms ℎ𝑎 and ℎ𝑏 with 𝑎 = 𝑏, then the series is not convergent. For example:
𝑎 = 𝑏 = 1 ⇒ ℎ1 + ℎ1 = 2 + 2 + ⋯ ⇒ 𝑎 = 2, 𝑟 = 1 ⇒ 𝑁𝑜𝑡 𝐶𝑜𝑛𝑣𝑒𝑟𝑔𝑒𝑛𝑡

Hence, we need to pick two distinct terms with the second term having a smaller value than the first term. For

P a g e 192 | 274
Get all the files at: https://bit.ly/azizhandouts
Aziz Manva (azizmanva@gmail.com)

example:
2
ℎ5 + ℎ1 = 3 + 2 + ⋯ ⇒ 𝑎 = 3, 𝑟 = < 1 ⇒ 𝐶𝑜𝑛𝑣𝑒𝑟𝑔𝑒𝑛𝑡
3
This can be done in:
5 5! 5×4
( )= = = 10 𝑊𝑎𝑦𝑠
2 2! 3! 2

Example 4.142
Let 𝑎1 , 𝑎2 , 𝑎3 , … be in harmonic progression with 𝑎1 = 5 and 𝑎20 = 25. The least positive integer 𝑛 for which
𝑎𝑛 < 0 is: (JEE Adv. 2012)

1 1 4
Substituting 𝑎 = 5 and 𝑑 = − 475:
1
Taking the reciprocals of the given sequence gives
1 4
us the arithmetic sequence: + (𝑛 − 1) (− )<0
1 1 1 1 1 5 475
= , ,…, = Clearing fractions:
𝑎1 5 𝑎2 𝑎20 20 95 − 4𝑛 + 4
with common difference: <0
475
1 1 1 1 Multiplying both sides by 475:
𝑎20 − 𝑎1 20 − 5 4
𝑑= = =− 95 − 4𝑛 + 4 < 0
20 − 1 19 475
Collating variables one side:
We need 𝑎𝑛 < 0 and since the reciprocal of a 99 < 4𝑛
number does not change sign, we calculate the least Solving for 𝑛:
99 3
𝑛 such that: 𝑛> = 24
1 1 4 4
< 0 ⇒ + (𝑛 − 1)𝑑 < 0 Hence the least value of 𝑛 that works is:
𝑎𝑛 𝑎1
𝑛 = 25

MCQ 4.143
𝑀𝑎𝑟𝑘 𝑡ℎ𝑒 𝑐𝑜𝑟𝑟𝑒𝑐𝑡 𝑜𝑝𝑡𝑖𝑜𝑛
If 𝑎1 , 𝑎2 , . . . , 𝑎𝑛 are in HP, then the expression 𝑎1 𝑎2 + 𝑎2 𝑎3 +. . . 𝑎𝑛−1 𝑎𝑛 is equal to
A. 𝑛(𝑎1 − 𝑎𝑛 )
B. (𝑛 − 1)(𝑎1 − 𝑎𝑛 )
C. 𝑛𝑎1 𝑎𝑛
D. (𝑛 − 1)(𝑎1 𝑎𝑛 ) (JEE Main 2002)

If this property is true in general, then it should be true for specific values. Recall that
40,48, 60 = 4(10,12,15) → 𝐻𝑃
𝑎1 𝑎2 + 𝑎2 𝑎3 = (10)(12) + (12)(15) = 120 + 180 = 300
(𝑛 − 1)(𝑎1 𝑎𝑛 ) = (2)(10)(15) = 300 ⇒ 𝑂𝑝𝑡𝑖𝑜𝑛 𝐷

4.144: Sum of Product of Consecutive Terms of a Harmonic Sequence


If 𝑎1 , 𝑎2 , . . . , 𝑎𝑛 are in HP:
𝑎1 𝑎2 + 𝑎2 𝑎3 +. . . +𝑎𝑛−1 𝑎𝑛 = (𝑛 − 1)(𝑎1 𝑎𝑛 )

1 1 1
The reciprocals of the given HP ( , , … , ) are in AP with common difference:
𝑎1 𝑎2 𝑎𝑛
1 1 𝑎1 − 𝑎2 𝑎1 − 𝑎2
− =𝑑⇒ = 𝑑 ⇒ 𝑎1 𝑎2 =
𝑎2 𝑎1 𝑎1 𝑎2 𝑑
In general

P a g e 193 | 274
Get all the files at: https://bit.ly/azizhandouts
Aziz Manva (azizmanva@gmail.com)

1 1 𝑎𝑛−1 − 𝑎𝑛 𝑎𝑛−1 − 𝑎𝑛
− =𝑑⇒ = 𝑑 ⇒ 𝑎𝑛 𝑎𝑛−1 =
𝑎𝑛 𝑎𝑛−1 𝑎𝑛 𝑎𝑛−1 𝑑
Hence:
𝑎1 − 𝑎2 𝑎2 − 𝑎3 𝑎𝑛−1 − 𝑎𝑛
𝐿𝐻𝑆 = + + ⋯+
⏟ 𝑑 ⏟ 𝑑 ⏟ 𝑑
𝑎1 𝑎2 𝑎2 𝑎3 𝑎𝑛−1 𝑎𝑛

The expression above telescopes:


𝑎1 − 𝑎𝑛
=
𝑑
1 1 𝑎1 −𝑎𝑛
Substitute − = (𝑛 − 1)𝑑 ⇒ = (𝑛 − 1)𝑑 ⇒ 𝑎1 − 𝑎𝑛 = 𝑎1 𝑎𝑛 (𝑛 − 1)𝑑
𝑎𝑛 𝑎1 𝑎1 𝑎𝑛
𝑎1 𝑎𝑛 (𝑛 − 1)𝑑
= = (𝑛 − 1)𝑎1 𝑎𝑛 = 𝑅𝐻𝑆
𝑑

Example 4.145
If ℎ1 , ℎ2 , … , ℎ10 are in harmonic sequence with first term 3 and tenth term five, then ℎ2 ℎ3 + ℎ3 ℎ4 + ⋯ + ℎ8 ℎ9
2𝑎 ×3𝑏 ×5𝑐
can be written in the form . Find 𝑎 + 𝑏 + 𝑐 + 𝑑 + 𝑒.
29𝑑 ×43𝑒

1 1 1 1

ℎ10 ℎ1 5 − 3 2 1 2
𝑑= = =− × =−
10 − 1 9 15 9 135
1 1 1 2 45 2 43 135
= +𝑑 = − = − = ⇒ ℎ2 =
ℎ2 ℎ1 3 135 135 135 135 43
1 1 1 16 45 16 29 135
= + 8𝑑 = − = − = ⇒ ℎ2 =
ℎ9 ℎ1 3 135 135 135 135 29

135 135 23 × 36 × 52
(9 − 1)ℎ2 ℎ9 = (8) ( )( )=
43 29 291 × 431
𝑎 + 𝑏 + 𝑐 + 𝑑 + 𝑒 = 3 + 6 + 2 + 1 + 1 = 13

B. Converting Series

MCQ 4.146
𝑀𝑎𝑟𝑘 𝑡ℎ𝑒 𝑐𝑜𝑟𝑟𝑒𝑐𝑡 𝑜𝑝𝑡𝑖𝑜𝑛
Let the positive numbers 𝑎, 𝑏, 𝑐, 𝑑 be in AP. Then 𝑎𝑏𝑐, 𝑎𝑏𝑑, 𝑎𝑐𝑑, 𝑏𝑐𝑑 are:
A. NOT in AP/GP/HP
B. in AP
C. in GP
D. in HP (JEE Adv. 2001 Screening)

𝑎, 𝑏, 𝑐, 𝑑 → 𝐴𝑃
Divide each term by 𝑎𝑏𝑐𝑑, and note that it is still an arithmetic sequence:
1 1 1 1
, , ,
𝑏𝑐𝑑 𝑎𝑐𝑑 𝑎𝑏𝑑 𝑎𝑏𝑐
Write the terms in reverse order, and note that it is still an arithmetic sequence:
1 1 1 1
, , ,
𝑎𝑏𝑐 𝑎𝑏𝑑 𝑎𝑐𝑑 𝑏𝑐𝑑
Since the above terms are an arithmetic sequence, their reciprocals are a harmonic sequence.

P a g e 194 | 274
Get all the files at: https://bit.ly/azizhandouts
Aziz Manva (azizmanva@gmail.com)

𝑂𝑝𝑡𝑖𝑜𝑛 𝐷

MCQ 4.147
𝑀𝑎𝑟𝑘 𝑡ℎ𝑒 𝑐𝑜𝑟𝑟𝑒𝑐𝑡 𝑜𝑝𝑡𝑖𝑜𝑛
1 1 1
If 𝑥 > 1, 𝑦 > 1, 𝑧 > 1 are in 𝐺𝑃, then 1+𝑙𝑛 𝑥 , 1+𝑙𝑛 𝑦 , 1+𝑙𝑛𝑧
are in:
A. AP
B. HP
C. GP
D. None of these (JEE Adv. 1998)

If 𝑥, 𝑦, 𝑧 are in GP, then the logs of the terms are in AP:


ln 𝑥 , ln 𝑦 , ln 𝑧
Adding 1 to each term keeps it an AP:
1 + ln 𝑥 , 1 + ln 𝑦 , 1 + ln 𝑧
Taking the reciprocal of an AP makes it an HP:
1 1 1
, , ⇒ 𝑂𝑝𝑡𝑖𝑜𝑛 𝐵
1 + 𝑙𝑛 𝑥 1 + 𝑙𝑛 𝑦 1 + 𝑙𝑛𝑧
C. The Harmonic Series

4.148: The Harmonic Sequence


The reciprocals of the natural numbers form the harmonic sequence:
1 1 1
1, , , , …
2 3 4

4.149: The Harmonic Series


The sum of the terms of the harmonic sequence is called the harmonic series:
1 1 1
1+ + + +⋯
2 3 4

4.150: Divergence of the Harmonic Series


1 1 1 1 1 1 1
1+ + + + + + + +⋯
2 3 4 5 6 7 8
1 1 1 1 1 1 1 1
< + + + + + + + +⋯

2 2 ⏟
4 4 ⏟
8 8 8 8
𝐺𝑟𝑜𝑢𝑝 1 𝐺𝑟𝑜𝑢𝑝 2 𝐺𝑟𝑜𝑢𝑝 3
1 1
𝐺𝑟𝑜𝑢𝑝 1 = + = 1
2 2
1 1 1
𝐺𝑟𝑜𝑢𝑝 2 = + =
4 4 2
1 1 1 1 1
𝐺𝑟𝑜𝑢𝑝 3 = + + + =
8 8 8 8 2
1
𝐺𝑟𝑜𝑢𝑝 48 𝑇𝑒𝑟𝑚𝑠 =
2
D. Harmonic Mean

4.151: Harmonic Mean of Two Numbers


The harmonic mean of 𝑎 and 𝑏 is twice the reciprocal of the sum of the reciprocals of the two numbers.

P a g e 195 | 274
Get all the files at: https://bit.ly/azizhandouts
Aziz Manva (azizmanva@gmail.com)

2𝑎𝑏
𝐻𝑎𝑟𝑚𝑜𝑛𝑖𝑐 𝑀𝑒𝑎𝑛(𝑎, 𝑏) =
𝑎+𝑏

The sum of the reciprocals


1 1 𝑎+𝑏
= + =
𝑎 𝑏 𝑎𝑏
Twice the reciprocal of the sum of the reciprocals
𝑎𝑏 2𝑎𝑏
=2× =
𝑎+𝑏 𝑎+𝑏

Challenge 4.152: Harmonic Mean


Given that 𝑎 and 𝑏 are constants, determine 𝑟 in terms of the harmonic mean of 𝑝 and 𝑞 if:
𝑎𝑝 = 𝑏 𝑞 = (𝑎𝑏)𝑟

Equate the first part and the second to get an equation. We want the answer in terms of 𝑝 and 𝑞. So, we solve for
𝑎 (or log 𝑎), in this case:
𝑞
𝑎𝑝 = 𝑏 𝑞 ⇒ 𝑝 log 𝑎 = 𝑞 log 𝑏 ⇒ log 𝑎 = log 𝑏
⏟ 𝑝
𝑬𝒒𝒖𝒂𝒕𝒊𝒐𝒏 𝑰
Also, we can say:
𝑏 𝑞 = (𝑎𝑏)𝑟 ⇒ 𝑞 log 𝑏 = 𝑟 log 𝑎 + 𝑟 log 𝑏
Substitute the value of log 𝑎 from Equation I:
𝑞
𝑞 log 𝑏 = 𝑟 ( log 𝑏) + 𝑟 log 𝑏
𝑝
Divide both sides by log 𝑏:
𝑞 𝑞𝑝 1 2𝑞𝑝 1
𝑞 = 𝑟 + 𝑟 ⇒ 𝑞𝑝 = 𝑟𝑞 + 𝑟𝑝 ⇒ 𝑟 = = ( ) = [𝐻𝑎𝑟𝑚𝑜𝑛𝑖𝑐 𝑀𝑒𝑎𝑛(𝑝, 𝑞)]
𝑝 𝑞+𝑝 2 𝑞+𝑝 2

4.153: Harmonic Mean


Given 𝑛 numbers (𝑦1 , 𝑦2 , … , 𝑦𝑛 ), their harmonic mean (𝐻𝑀) is 𝑛 times the reciprocal of the sum of the
reciprocals of the numbers.
𝑛
𝐻𝑀(𝑦1 , 𝑦2 , … , 𝑦𝑛 ) =
1 1 1
𝑦1 + 𝑦2 + ⋯ + 𝑦𝑛

The sum of the reciprocals


1 1 1
= + + ⋯+
𝑦1 𝑦2 𝑦𝑛
The reciprocal of the sum of the reciprocals
1
=
1 1 1
𝑦1 + 𝑦2 + ⋯ + 𝑦𝑛
𝑛 times the reciprocal of the sum of the reciprocals
𝑛
=
1 1 1
𝑦1 + 𝑦2 + ⋯ + 𝑦𝑛

Example 4.154: Harmonic Mean


Find the harmonic mean of:
log 𝑥 𝑥, log 𝑥2 𝑥 , … , log 𝑥 𝑛 𝑥

P a g e 196 | 274
Get all the files at: https://bit.ly/azizhandouts
Aziz Manva (azizmanva@gmail.com)

1 1 1 𝑛 𝑛 2
𝐻𝑀(log 𝑥 𝑥, log 𝑥 2 𝑥 , … , log 𝑥𝑛 𝑥) = 𝐻𝑀 (1, , , … , ) = = =
2 3 𝑛 1 + 2 + 3 + ⋯ + 𝑛 𝑛(𝑛 + 1) 𝑛 + 1
2

4.155: Harmonic Mean Property


If three numbers are in a harmonic progression, then the middle term is the harmonic mean of the other two.
2𝑎𝑐
𝑎, 𝑏, 𝑐 𝑎𝑟𝑒 𝑖𝑛 𝐻𝑃 ⇒ 𝑏 = 𝐻𝑀(𝑎, 𝑐) =
𝑎+𝑐

Let
1 1 1
𝑎, 𝑏, 𝑐
⏟ ⇔ , ,
𝐻𝑎𝑟𝑚𝑜𝑛𝑖𝑐
𝑎⏟ 𝑏 𝑐
𝑃𝑟𝑜𝑔𝑟𝑒𝑠𝑠𝑖𝑜𝑛 𝐴𝑟𝑖𝑡ℎ𝑚𝑒𝑡𝑖𝑐
𝑃𝑟𝑜𝑔𝑟𝑒𝑠𝑠𝑖𝑜𝑛
Since the above is an arithmetic progression, it must satisfy the arithmetic mean property:
1 1
1 𝑎+𝑐
=
𝑏 2
Take the reciprocals of the above:
2 2 2𝑎𝑐
𝑏= = 𝑎+𝑐 = = 𝐻𝑀(𝑎, 𝑐)
1 1 𝑎 + 𝑐
𝑎+𝑐 𝑎𝑐

Example 4.156
1 1 1
A. Show that 2 , 3 , 4 form a harmonic sequence.
B. Show that 40, 48 and 60 form a harmonic sequence.

Part A
The reciprocals have a common difference:
𝑑 = 3−2 =4−3
Since the reciprocals form an arithmetic sequence, the sequence itself is harmonic.
Part B
2(40)(60) 4800
𝐻𝑀(40, 60) = = = 48 = 𝑀𝑖𝑑𝑑𝑙𝑒 𝑇𝑒𝑟𝑚
40 + 60 100
Since the middle term is the harmonic mean of the other two, the sequence is harmonic.

E. Applications of Harmonic Mean Property

MCQ 4.157
𝑀𝑎𝑟𝑘 𝑡ℎ𝑒 𝑐𝑜𝑟𝑟𝑒𝑐𝑡 𝑜𝑝𝑡𝑖𝑜𝑛
If 𝑎, 𝑏, 𝑐 are the terms of a harmonic sequence, then the equality 2𝑎𝑏𝑐 = 𝑎𝑏 2 + 𝑏 2 𝑐
A. Never holds.
B. Always holds.
C. Is true if and only if 𝑎, 𝑏, 𝑐 are the terms of an arithmetic sequence.
D. Is true if and only if 𝑎, 𝑏, 𝑐 are the terms of a geometric sequence.

Method I
2𝑎𝑐
Substitute 𝑏 = 𝑎+𝑐 in each side of the given equality:
2𝑎𝑐 4𝑎2 𝑐 2
𝐿𝐻𝑆 = 2𝑎𝑏𝑐 = 2𝑎 ∙ ∙𝑐 =
𝑎+𝑐 𝑎+𝑐

P a g e 197 | 274
Get all the files at: https://bit.ly/azizhandouts
Aziz Manva (azizmanva@gmail.com)

2𝑎𝑐 2 4𝑎2 𝑐 2
𝑅𝐻𝑆 = 𝑏 2 (𝑎 + 𝑐) = ( ) (𝑎 + 𝑐) = = 𝐿𝐻𝑆
𝑎+𝑐 𝑎+𝑐
Method II
1 1 1
If 𝑎, 𝑏, 𝑐 are the terms of a harmonic sequence, then their reciprocals 𝑎 , 𝑏 , 𝑐 form an arithmetic sequence with
common difference:
1 1 1 1
− = −
𝑏 𝑎 𝑐 𝑏
Combine fractions:
𝑎−𝑏 𝑏−𝑐
=
𝑎𝑏 𝑏𝑐
Cross-multiply:
𝑎𝑏𝑐 − 𝑏 2 𝑐 = 𝑎𝑏 2 − 𝑎𝑏𝑐
Rearrange:
2𝑎𝑏𝑐 = 𝑎𝑏 2 + 𝑏 2 𝑐
𝑂𝑝𝑡𝑖𝑜𝑛 𝐵 𝑖𝑠 𝑐𝑜𝑟𝑟𝑒𝑐𝑡

Example 4.158
Find the number of ordered triplets (𝑎, 𝑏, 𝑐) such that:
1. 𝑎, 𝑏, 𝑐 are each two digit prime numbers
2. 𝑎, 𝑏, 𝑐 (in that order) form a harmonic sequence
3. 𝑐, 𝑏, 𝑎 (in that order) form an arithmetic sequence

1 1 1
If 𝑎, 𝑏, 𝑐 are the terms of a harmonic sequence, then their reciprocals , , form an arithmetic sequence with
𝑎 𝑏 𝑐
common difference:
1 1 1 1 𝑎−𝑏 𝑏−𝑐
𝑑1 = − = − ⇒ =
𝑏 𝑎 𝑐 𝑏 𝑎𝑏 𝑏𝑐
Since 𝑐, 𝑏, 𝑎 from an arithmetic sequence, 𝑑2 = 𝑎 − 𝑏 = 𝑏 − 𝑐:
1 1
= ⇒𝑎=𝑐
𝑎 𝑐
And we can find the value of 𝑏 since 𝑏 is the arithmetic mean of 𝑎 and 𝑐:
𝑎+𝑐 𝑎+𝑎
𝑏= = =𝑎
2 2
Hence:
𝑎 = 𝑏 = 𝑐 ⇒ 𝐶𝑜𝑛𝑠𝑡𝑎𝑛𝑡 𝑆𝑒𝑞𝑢𝑒𝑛𝑐𝑒
The number of two-digit prime numbers is:
25 − 4 = 21
Hence, the answer is:
(11,11,11), (13,13,13), … , (97,97,97) → 21 𝑆𝑒𝑞𝑢𝑒𝑛𝑐𝑒𝑠

Example 4.159
𝑀𝑎𝑟𝑘 𝑎𝑙𝑙 𝐶𝑜𝑟𝑟𝑒𝑐𝑡 𝑂𝑝𝑡𝑖𝑜𝑛𝑠
An 𝑛 (𝑛 ≥ 3) term sequence which is both harmonic and arithmetic is:
A. a geometric sequence.
B. a constant sequence.
C. a non-constant sequence.
D. does not exist.

Consider terms 𝑇1 , 𝑇2 , 𝑇3 of the sequence. By the logic of the previous example:

P a g e 198 | 274
Get all the files at: https://bit.ly/azizhandouts
Aziz Manva (azizmanva@gmail.com)

𝑇1 = 𝑇2 = 𝑇3

Similarly,
𝑇2 = 𝑇3 = 𝑇4

In general, by combining results:


𝑇1 = 𝑇2 = ⋯ = 𝑇𝑛 ⇒ 𝐶𝑜𝑛𝑠𝑡𝑎𝑛𝑡 𝑆𝑒𝑞𝑢𝑒𝑛𝑐𝑒 ⇒ 𝑂𝑝𝑡𝑖𝑜𝑛 𝐵

A constant sequence is also a geometric sequence with common ratio 1 ⇒ 𝑂𝑝𝑡𝑖𝑜𝑛 𝐴

4.160: Sequences which are both Harmonic and Arithmetic


If a sequence is both harmonic and arithmetic, then it is a constant sequence.

Example 4.161
𝑀𝑎𝑟𝑘 𝑡ℎ𝑒 𝑐𝑜𝑟𝑟𝑒𝑐𝑡 𝑜𝑝𝑡𝑖𝑜𝑛
If ln(𝑎 + 𝑐) , ln(𝑎 − 𝑐) , ln(𝑎 − 2𝑏 + 𝑐) are in AP, then:
A. 𝑎, 𝑏, 𝑐 are in AP.
B. 𝑎2 , 𝑏 2 , 𝑐 2 are in AP.
C. 𝑎, 𝑏, 𝑐 are in GP.
D. 𝑎, 𝑏, 𝑐 are in HP. (JEE Adv. 1994)

ln(𝑎 + 𝑐) , ln(𝑎 − 𝑐) , ln(𝑎 − 2𝑏 + 𝑐) → 𝐴𝑃


If the natural logs of the terms form an arithmetic sequence, then the terms themselves form a geometric
sequence:
𝑎 + 𝑐, 𝑎 − 𝑐, 𝑎 − 2𝑏 + 𝑐 → 𝐺𝑃
The above geometric sequence satisfies the geometric mean property:
(𝑎 − 𝑐)2 = (𝑎 + 𝑐)(𝑎 − 2𝑏 + 𝑐)
Expand:
𝑎2 − 2𝑎𝑐 + 𝑐 2 = 𝑎2 − 2𝑎𝑏 + 𝑎𝑐 + 𝑎𝑐 − 2𝑏𝑐 + 𝑐 2
Simplify:
−2𝑎𝑐 = −2𝑎𝑏 + 2𝑎𝑐 − 2𝑏𝑐
Collate all 𝑏 terms on LHS:
2𝑎𝑐
2𝑎𝑏 + 2𝑏𝑐 = 4𝑎𝑐 ⇒ 𝑏(𝑎 + 𝑐) = 2𝑎𝑐 ⇒ 𝑏 =
𝑎+𝑐
𝑏 is the harmonic mean of 𝑎 and 𝑐.
𝑂𝑝𝑡𝑖𝑜𝑛 𝐷

Example 4.162
If 𝑥, 𝑦, 𝑧 are in HP, then prove that log(𝑥 + 𝑧) + log(𝑥 + 𝑧 − 2𝑦) = 2 log(𝑥 − 𝑧) (JEE Adv. 1978)

If we let 𝑥 = 𝑎, 𝑦 = 𝑏, 𝑧 = 𝑐, then this requires proving the converse of the previous example.
Each of the steps in the example above is reversible.

Since ln(𝑎 + 𝑐) , ln(𝑎 − 𝑐) , ln(𝑎 − 2𝑏 + 𝑐) form an arithmetic sequence, the terms satisfy the arithmetic mean
property, and hence:
2 log(𝑎 − 𝑐) = log(𝑎 + 𝑐) + log(𝑎 + 𝑐 − 2𝑏)
Changing back to the original variables gives us the result we want.

P a g e 199 | 274
Get all the files at: https://bit.ly/azizhandouts
Aziz Manva (azizmanva@gmail.com)

F. Equality of Means

4.163: Equality of Means


𝐻𝑀(𝑎, 𝑏) = 𝐺𝑀(𝑎, 𝑏) = 𝐴𝑀(𝑎, 𝑏) if and only if 𝑎 = 𝑏

If 𝑎 = 𝑏:
2𝑎𝑏 2𝑎2 𝑎 + 𝑏 2𝑎
= = 𝑎, √𝑎𝑏 = √𝑎2 = 𝑎,
⏟ = =𝑎

𝑎+𝑏 2𝑎 ⏟2 𝑎
𝑮𝒆𝒐𝒎𝒆𝒕𝒓𝒊𝒄 𝑴𝒆𝒂𝒏
𝑯𝒂𝒓𝒎𝒐𝒏𝒊𝒄 𝑴𝒆𝒂𝒏 𝑨𝒓𝒊𝒕𝒉𝒎𝒆𝒕𝒊𝒄 𝑴𝒆𝒂𝒏

If 𝐻𝑀 = 𝐺𝑀:
2𝑎𝑏 𝑎+𝑏
= √𝑎𝑏 ⇒ √𝑎𝑏 =
𝑎+𝑏 ⏟ 2
⇒𝑮𝑴=𝑨𝑴
Square both sides of the above:
4𝑎𝑏 = 𝑎2 + 2𝑎𝑏 + 𝑏 2
0 = (𝑎 − 𝑏)2
0=𝑎−𝑏
𝑎=𝑏

Example 4.1649
𝑀𝑎𝑟𝑘 𝑎𝑙𝑙 𝐶𝑜𝑟𝑟𝑒𝑐𝑡 𝑂𝑝𝑡𝑖𝑜𝑛𝑠
If the first and the (2𝑛– 1)𝑠𝑡 terms of an AP, a GP, and an HP (all of positive terms) are equal and their 𝑛𝑡ℎ terms
are 𝑎, 𝑏 and 𝑐 respectively, then, for 𝑛 ≥ 2:
A. 𝑎 = 𝑏 = 𝑐
B. 𝑎 ≥ 𝑏 ≥ 𝑐
C. 𝑎 + 𝑐 = 𝑏
D. 𝑎𝑐 − 𝑏 2 = 0 (JEE Adv. 1988, Adapted)

Let
𝐹𝑖𝑟𝑠𝑡 𝑇𝑒𝑟𝑚 = 𝐿𝑎𝑠𝑡 𝑇𝑒𝑟𝑚 = 𝑥
Arithmetic Sequence
Let the terms of the arithmetic, geometric and harmonic
sequences be:
𝐴𝑃: 𝑎1 = 𝑥, 𝑎2 , … , 𝑎𝑛 = 𝑎, … , 𝑎2𝑛−1 = 𝑥
𝐺𝑃: 𝑏1 = 𝑥, 𝑏2 , … , 𝑏𝑛 = 𝑏, … , 𝑏2𝑛−1 = 𝑥
𝐻𝑃: 𝑐1 = 𝑥, 𝑏2 , … , 𝑏𝑛 = 𝑏, … , 𝑐2𝑛−1 = 𝑥
Notice the symmetry. By the subsequence property, which holds for each sequence above:
𝑎1 = 𝑥, 𝑎𝑛 , 𝑎2𝑛−1 = 𝑥 → 𝐴𝑃
𝑏1 = 𝑥, 𝑏𝑛 = 𝑏, 𝑏2𝑛−1 = 𝑥 → 𝐺𝑃
𝑐1 = 𝑥, 𝑐𝑛 = 𝑥, 𝑐2𝑛−1 = 𝑥 → 𝐻𝑃
From the arithmetic, geometric and harmonic sequences, we use the property that the middle term is the
arithmetic, geometric and harmonic mean respectively:
𝑎 = 𝑎𝑛 = 𝐴𝑀(𝑥, 𝑥) = 𝑥
𝑏 = 𝑏𝑛 = 𝐺𝑀(𝑥, 𝑥) = 𝑥

9 The original question does not mention:


A. Positive, allowing 𝐴𝑃 = 𝐻𝑃 = (1,1,1) and 𝐺𝑃 = (1, −1,1), and then options A and B are incorrect. Several other
sources mark options A, B correct without applying the positive restriction.
B. 𝑛 ≥ 2. If we consider 𝑛 = 1 as valid, then the sequences have a single-term and Options A, B and D remain correct.

P a g e 200 | 274
Get all the files at: https://bit.ly/azizhandouts
Aziz Manva (azizmanva@gmail.com)

𝑐 = 𝑐𝑛 = 𝐻𝑀(𝑥, 𝑥) = 𝑥
Hence:
𝑎 = 𝑏 = 𝑐 ⇒ 𝑂𝑝𝑡𝑖𝑜𝑛 𝐴
Since option A is a special case of Option B, that is also correct:
𝑎 ≥ 𝑏 > 𝑐 ⇒ 𝑂𝑝𝑡𝑖𝑜𝑛 𝐵
And finally,
𝑎𝑐 − 𝑏 2 = 𝑎2 − 𝑎2 = 0 ⇒ 𝑂𝑝𝑡𝑖𝑜𝑛 𝐷

4.5 𝑹𝑴𝑺 ≥ 𝑨𝑴 ≥ 𝑮𝑴 ≥ 𝑯𝑴 (In)equality


A. Trivial Inequality

4.165: Trivial Inequality


The square of any real quantity is greater than or equal to zero:
𝑥2 ≥ 0

The trivial inequality forms the basis for proving many, many inequalities and solving many questions.

Example 4.166
What is the least possible value of (𝑥𝑦 − 1)2 + (𝑥 + 𝑦)2 for real numbers 𝑥 and 𝑦? (AMC 12A 2021/7, AMC 10A
2021/9)

𝑥 2 𝑦 2 − 2𝑥𝑦 + 1 + 𝑥 2 + 2𝑥𝑦 + 𝑦 2 = 𝑥 2 + 𝑦 2 + 𝑥 2 𝑦 2 + 1
𝑀𝑖𝑛𝑖𝑚𝑢𝑚 𝑜𝑓 𝑥 2 = 0 ⇒ 𝐴𝑐ℎ𝑖𝑒𝑣𝑒𝑑 𝑎𝑡 𝑥 = 0
𝑀𝑖𝑛𝑖𝑚𝑢𝑚 𝑜𝑓 𝑦 2 = 0 ⇒ 𝐴𝑐ℎ𝑖𝑒𝑣𝑒𝑑 𝑎𝑡 𝑦 = 0
𝐴𝑡 𝑥 = 𝑦 = 0 ⇒ 𝑥 2 𝑦 2 = 0

By the trivial inequality, we know that this is the best we can do for the first three terms.
Hence, the fourth term, which is a constant, is also the minimum.

Example 4.167
Identify the mistake in the “proof” below:
4 = 5 ⇒ 𝑀𝑢𝑙𝑡𝑖𝑝𝑙𝑦 𝑏𝑜𝑡ℎ 𝑠𝑖𝑑𝑒𝑠 𝑏𝑦 0 ⇒ 0 = 0

You cannot begin the proof by assuming the truth of what needs to be proved.

Example 4.168
Prove that:
A. 𝑎2 + 𝑏 2 ≥ 2𝑎𝑏
B. 𝑎2 + 𝑏 2 + 𝑐 2 ≥ 2𝑎𝑏 + 2𝑏𝑐 − 2𝑎𝑐
C. 𝑎2 + 𝑏 2 + 𝑐 2 ≥ 𝑎𝑏 + 𝑏𝑐 + 𝑎𝑐

Part A
Subtract 2𝑎𝑏 from both sides of the given inequality:
⇔ 𝑎2 − 2𝑎𝑏 + 𝑏 2 ≥ 0
⇔ (𝑎 − 𝑏)2 ≥ 0
Each step above is reversible, and hence the original equality holds.

Part B:

P a g e 201 | 274
Get all the files at: https://bit.ly/azizhandouts
Aziz Manva (azizmanva@gmail.com)

Expand, and move the required terms to RHS:


(𝑎 − 𝑏 + 𝑐)2 ≥ 0 ⇔ 𝑎2 + 𝑏 2 + 𝑐 2 ≥ 2𝑎𝑏 + 2𝑏𝑐 − 2𝑎𝑐
Part C:
(𝑎 − 𝑏)2 + (𝑏 − 𝑐)2 + (𝑐 − 𝑎)2 ≥ 0 ⇔ 𝑎2 + 𝑏 2 + 𝑐 2 ≥ 𝑎𝑏 + 𝑏𝑐 + 𝑎𝑐

4.169: Sum of a positive number and its reciprocal is always greater than 2.
For positive real numbers 𝑎 and 𝑏:
𝑎 𝑏
+ ≥2
𝑏 𝑎

Add fractions:
𝑎2 + 𝑏 2
⇔ ≥2
𝑎𝑏
Clear the denominator. Note that we can multiply both sides by 𝒂, and 𝒃 only because both are positive:
⇔ 𝑎2 + 𝑏 2 ≥ 2𝑎𝑏
Collate all terms on one side and factor:
(𝑎 − 𝑏)2 ≥ 0
And the above is true by the trivial inequality.
(All steps are reversible, and hence the equality we started with is true.)

In the above, where was the condition that 𝑎 and 𝑏 are both positive used?

In one step, we multiplied by 𝑎𝑏.


If 𝑎𝑏 < 0, then the inequality will flip.

In the above, can 𝑎 and 𝑏 be both negative?

Yes.

4.170: Minimum Value of an expression


The minimum value of 𝑎2 + 𝑏 is:
𝑏

This is because, by the trivial inequality:


𝑎2 ≥ 0

Example 4.171: Completing the Square


Find the minimum and maximum value of 𝑙 2 + 𝑤 2 given that 𝑙 and 𝑤 are nonnegative real numbers and 𝑙 + 𝑤 =
2.

Let
𝑋 = 𝑙2 + 𝑤 2
Substitute 𝑙 + 𝑤 = 2 ⇒ 𝑙 = 2 − 𝑤 which is valid for 0 ≤ 𝑙, 𝑤 ≤ 2 and simplify:
(2 − 𝑤)2 + 𝑤 2 = 4 − 4𝑤 + 𝑤 2 + 𝑤 2 = 2𝑤 2 − 4𝑤 + 4 = 2(𝑤 2 − 2𝑤 + 2)
Complete the square to get:
2[(𝑤 2 − 1)2 + 1]
𝑀𝑖𝑛 𝑤ℎ𝑒𝑛: 𝑤 = 1 ⇒ 𝑋 = 2
𝑀𝑎𝑥 𝑤ℎ𝑒𝑛: 𝑤 = 2 ⇒ 𝑋 = 4

P a g e 202 | 274
Get all the files at: https://bit.ly/azizhandouts
Aziz Manva (azizmanva@gmail.com)

Example 4.172: Change of Variable


Let 𝑥 ≥ 0, 𝑦 ≥ 0 be real numbers with 𝑥 + 𝑦 = 2. Prove that 𝑥 2 𝑦 2 (𝑥 2 + 𝑦 2 ) ≤ 2. (Irish Math Olympiad
2000/P2/Q1)

We start with a change of variable that is not immediately recognizable as useful.


Let 𝑥 = 1 + 𝑡. Then
𝑥+𝑦 = 2⇒ 1+𝑡+𝑦 = 2⇒ 𝑦 =1−𝑡

Substitute 𝑥 = 1 + 𝑡, 𝑦 = 1 − 𝑡 in 𝑥 2 𝑦 2 (𝑥 2 + 𝑦 2 ):
= (1 + 𝑡)2 (1 − 𝑡)2 [(1 + 𝑡)2 + (1 − 𝑡)2 ]
= (1 − 𝑡 2 )2 [2 + 2𝑡 2 ]
= 2(1 − 𝑡 2 )(1 − 𝑡 2 )[1 + 𝑡 2 ]
Since 𝑡 2 , 𝑡 4 ≥ 0:
=2 ⏟ (1 − 𝑡 2 ) (1 − 𝑡 4 )

𝑩𝒆𝒕𝒘𝒆𝒆𝒏 𝟎 𝒂𝒏𝒅 𝟏 𝑩𝒆𝒕𝒘𝒆𝒆𝒏 𝟎 𝒂𝒏𝒅 𝟏

B. AM-GM Inequality: Two Variables

4.173: Geometric Mean and Arithmetic Mean


𝐺𝑀(𝑎1 , 𝑎2 , … 𝑎𝑛 ) = 𝑛√𝑎1 × 𝑎2 × … × 𝑎𝑛
𝑎1 + 𝑎2 + ⋯ + 𝑎𝑛
𝐴𝑀(𝑎1 , 𝑎2 , … 𝑎𝑛 ) =
𝑛

The geometric mean is the 𝑛𝑡ℎ root of the product of 𝑛 numbers.


Square roots (and geometric mean) are only defined for positive numbers.
Hence, you cannot apply the AM-GM inequality to expressions that can also be negative.

Example 4.174
Consider a rectangle with perimeter 20 and integer side lengths. Find the value of the side lengths that
maximizes the area.

𝑃 = 2(𝑙 + 𝑤) = 20 ⇒ 𝑙 + 𝑤 = 10

Length 1 2 3 4 5 6 7 8 9
Width 9 8 7 6 5 4 3 2 1
Area 9 18 21 24 25 24 21 16 9

4.175: AM-GM Inequality


𝑎+𝑏
≥ √𝑎𝑏

⏟2
𝑮𝒆𝒐𝒎𝒆𝒕𝒓𝒊𝒄 𝑴𝒆𝒂𝒏
𝑨𝒓𝒊𝒕𝒉𝒎𝒆𝒕𝒊𝒄 𝑴𝒆𝒂𝒏

By the trivial inequality:


(𝑎 − 𝑏)2 ≥ 0
Square both sides:
𝑎2 − 2𝑎𝑏 + 𝑏 2 ≥ 0
Add 4𝑎𝑏 to both sides:
𝑎2 + 2𝑎𝑏 + 𝑏 2 ≥ 4𝑎𝑏
Take the square root both sides:

P a g e 203 | 274
Get all the files at: https://bit.ly/azizhandouts
Aziz Manva (azizmanva@gmail.com)

𝑎 + 𝑏 ≥ 2√𝑎𝑏
Divide by 2 both sides:
𝑎+𝑏
≥ √𝑎𝑏
2

4.176: AM-GM Equality


Equality of AM and GM is achieved 𝑖𝑓 𝑎𝑛𝑑 𝑜𝑛𝑙𝑦 𝑖𝑓 𝑎 = 𝑏.

We first prove that 𝑎 = 𝑏 ⇒ 𝐴𝑀 = 𝐺𝑀:


𝑎 + 𝑎 2𝑎
𝐿𝐻𝑆 = = =𝑎
2 2
𝑅𝐻𝑆 = √𝑎𝑎 = √𝑎2 = 𝑎
Hence, equality is achieved when 𝑎 = 𝑏.

Now we prove the converse. If 𝐴𝑀 = 𝐺𝑀 ⇒ 𝑎 = 𝑏:


𝑎+𝑏
= √𝑎𝑏
2
Eliminate fractions:
𝑎 + 𝑏 = 2√𝑎𝑏
Square both sides:
(𝑎 + 𝑏)2 = 4𝑎𝑏
Collate all terms on one side:
𝑎2 − 2𝑎𝑏 + 𝑏 2 = 0
Factor:
(𝑎 − 𝑏)2 = 0
Take the square root:
𝑎−𝑏=0
𝑎=𝑏

Example 4.177
Find the minimum and maximum value of 𝑙 2 + 𝑤 2 given that 𝑙 and 𝑤 are nonnegative real numbers and 𝑙 + 𝑤 =
2.

Since 𝑙 and 𝑤 are nonnegative real numbers, we must have:


𝑙 ≥ 0, 𝑤 ≥ 0 ⇒ ⏟ 𝑙𝑤 ≥ 0
𝑰𝒏𝒆𝒒𝒖𝒂𝒍𝒊𝒕𝒚 𝑰
Use AM-GM on the given equality:
𝑙+𝑤 2
√𝑙𝑤 ≤ = =1⇒ ⏟𝑙𝑤 ≤ 1
2 2 𝑰𝒏𝒆𝒒𝒖𝒂𝒍𝒊𝒕𝒚 𝑰𝑰
Combine Inequality I and II:
0 ≤ 𝑙𝑤 ≤ 1
Complete the square on the expression we want to minimize:
𝑙 2 + 𝑤 2 = (𝑙 + 𝑤)2 − 2𝑙𝑤 = 22 − 2𝑙𝑤 = 4 − 2𝑙𝑤

This will be maximum when 𝑙𝑤 is minimum:


4−0=4
And it will be minimum when 𝑙𝑤 is maximum:
4 − 2𝑙𝑤 = 4 − 2(1) = 2

P a g e 204 | 274
Get all the files at: https://bit.ly/azizhandouts
Aziz Manva (azizmanva@gmail.com)

4.178: Finding Minimum


𝑎+𝑏
≥ √𝑎𝑏
2
Equality is achieved when
𝑎=𝑏

In the above inequality, if you are able to get a value on the RHS, it can be used to establish a lower bound for
the LHS.

Example 4.179
A. Two positive real numbers 𝑎 and 𝑏 have a product of 100. What is the minimum value of their sum?
B. The area of a rectangle is 256 square meters. Find the minimum perimeter of the rectangle, in 𝑐𝑚.

Part A
𝑎+𝑏
≥ √𝑎𝑏 = √100 = 10 ⇒ 𝑎 + 𝑏 ≥ 20
2
Hence, the minimum value of the sum is:
20. 𝑤ℎ𝑒𝑛 𝑎 = 𝑏 = 10
Part B
𝑙+𝑤
≥ √𝑙𝑤 = √256 = 16 ⇒ 2(𝑙 + 𝑤) = 64
2
Minimum perimeter is 64, when
𝑙 = 𝑤 = 16

Example 4.180
9𝑥 2 sin2 𝑥+4
If 0 < 𝑥 < 𝜋, find the minimum value of (AIME 1983/9)
𝑥 sin 𝑥

Use a change of variable. Let 𝑦 = 𝑥 sin 𝑥:


9𝑥 2 sin2 𝑥 + 4 9𝑦 2 + 4 4
= = 9𝑦 +
𝑥 sin 𝑥 𝑦 𝑦
The domain gives us a clue. Both 𝑥 and sin 𝑥 are positive in the interval 0 < 𝑥 < 𝜋. So, when we take the
product, the 𝑦’s will cancel.

Apply 𝑨𝑴 − 𝑮𝑴
4 4
9𝑦 + ≥ 2√9𝑦 × = 12
𝑦 𝑦
Equality holds when the two terms are equal:
4 2 2
9𝑦 = ⇒ 𝑦 = ⇒ 𝑥 sin 𝑥 =
𝑦 3 3
When 𝑥 = 0
𝜋 𝜋
𝑥 = 0 ⇒ 𝑥 sin 𝑥 = 0, 𝑥= ⇒ 𝑥 sin 𝑥 = ≈ 1.57
2 2
𝑥 sin 𝑥 is continuous.
2 𝜋
𝐻𝑒𝑛𝑐𝑒, 𝑖𝑡 𝑎𝑐ℎ𝑖𝑒𝑣𝑒𝑠 𝑏𝑒𝑡𝑤𝑒𝑒𝑛 𝑥 = 0, 𝑎𝑛𝑑 𝑥 =
3 2

Example 4.181

P a g e 205 | 274
Get all the files at: https://bit.ly/azizhandouts
Aziz Manva (azizmanva@gmail.com)

Find the dimensions of a rectangle with perimeter 𝑝 that maximize its area. Find that area.

Let
𝑝 𝑝
2(𝑙 + 𝑤) = 𝑝 ⇒ 𝑤 = − 𝑙 ⇒ 𝐴𝑟𝑒𝑎 = 𝑙 ( − 𝑙)
2 2
By AM-GM Inequality:
𝑝
𝑝 𝑙 + (2 − 𝑙) 𝑝
√𝑙 ( − 𝑙) ≤ =
2 2 4
Square both sides:
𝑝 𝑝2
𝑙 ( − 𝑙) ≤
2 16
𝑝2
The maximum area (16) is achieved when:
𝑝 𝑝
𝑙=
−𝑙 ⇒𝑙 =
2 4
In other words, for a given perimeter, the area of a rectangle is maximized when it is a square.

Example 4.182
𝑦
1 𝑦
If log 2 𝑎 + log 2 𝑏 ≥ , then the minimum value of 𝑎 + 𝑏 can be written as 𝑥 𝑧 , where 𝑥 is a prime number, and is
4 𝑧
an irreducible fraction. Find 𝑥 + 𝑦 + 𝑧.

Combine the logarithms using the product rule:


1
log 2 𝑎𝑏 ≥
4
Convert from logarithmic form to exponential form:
1
𝑎𝑏 ≥ 24
From the AM-GM inequality, we know that:
1 1 9
𝑎 + 𝑏 ≥ 2√𝑎𝑏 = 2√24 = 2 × 28 = 28
Finally:
𝑥 + 𝑦 + 𝑧 = 2 + 9 + 8 = 19

Example 4.183
Let 𝑥 ≥ 0, 𝑦 ≥ 0 be real numbers with 𝑥 + 𝑦 = 2. Prove that 𝑥 2 𝑦 2 (𝑥 2 + 𝑦 2 ) ≤ 2. (Irish Math Olympiad
2000/P2/Q1)10

Apply AM-GM on the given equality condition:


𝑥+𝑦
√𝑥𝑦 ≤ ⏟ 2 ≤ 𝑥𝑦 ≤ 1
= 1 ⇒ √𝑥𝑦 ≤ 1 ⇒ 𝑥𝑦 ≤ 1 ⇒ (𝑥𝑦)
2
𝑰𝒏𝒆𝒒𝒖𝒂𝒍𝒊𝒕𝒚 𝑰
The last inequality holds since the square of a number between 0 and 1 is less than the number (for example
0.92 = 0.81).

Work with the LHS of the inequality to be proved. Complete the square:
𝐿𝐻𝑆 = 𝑥 2 𝑦 2 (𝑥 2 + 𝑦 2 ) = 𝑥 2 𝑦 2 [(𝑥 + 𝑦)2 − 2𝑥𝑦]
Substitute 𝑥 + 𝑦 = 2, and factor:
= 𝑥 2 𝑦 2 [(2)2 − 2𝑥𝑦] = 2(𝑥𝑦)2 [2 − 𝑥𝑦]

10 Also see the alternate solution that does not use the AM-GM inequality earlier in this document.

P a g e 206 | 274
Get all the files at: https://bit.ly/azizhandouts
Aziz Manva (azizmanva@gmail.com)

Use Inequality I to create a chain of inequalities:


2(𝑥𝑦)2 [2 − 𝑥𝑦] ≤ 2𝑥𝑦[2 − 𝑥𝑦]
𝑎+𝑏 2
By AM-GM 𝑎𝑏 ≤ ( ) , which we apply to the last expression:
2
2
𝑥𝑦 + (2 − 𝑥𝑦) 2 2
𝑥𝑦
⏟⏟ [2 − 𝑥𝑦] ≤ ( ) = ( ) = (1)2 = 1
2 2
𝑎 𝑏
2𝑥𝑦[2 − 𝑥𝑦] ≤ 2
Since 0 ≤ 𝑥𝑦 ≤ 1, we must have (𝑥𝑦)2 ≤ 𝑥𝑦:
2(𝑥𝑦)2 [2 − 𝑥𝑦] ≤ 2
And finally, we conclude:
𝑥 2 𝑦 2 (𝑥 2 + 𝑦 2 ) ≤ 2

Example 4.184
The number of elements in the set below is: (JEE Main, July 29, 2022-II)
𝑥2 + 𝑥
𝑆 = {𝑥 ∈ ℝ: 2 cos ( ) = 4𝑥 + 4−𝑥 }
6
Is:

𝑥2 + 𝑥 4𝑥 + 4−𝑥
cos ( )=
6 2

By AM-GM, the RHS has minimum value 1 since:


4𝑥 + 4−𝑥
≥ √4𝑥 ∙ 4−𝑥 = √4𝑥−𝑥 = √40 = √1 = 1
2

This value is achieved when:


4𝑥 = 4−𝑥 ⇒ 𝑥 = −𝑥 ⇒ 𝑥 = 0

The LHS has maximum value 1, and hence the only solution for equation is when:
𝐿𝐻𝑆 = 𝑅𝐻𝑆 = 1

The only solution that works for RHS is 𝑥 = 1, and this also work for the LHS.
𝑥2 + 𝑥 02 + 0
cos ( ) = cos ( ) = cos(0) = 1
6 6

Hence, there is exactly one element in S.

4.185: Cauchy Schwarz Inequality


𝑛 2 𝑛 𝑛

(∑ 𝑢𝑖 𝑣𝑖 ) ≤ (∑ 𝑢𝑖 ) (∑ 𝑣𝑖2 )
2

𝑖=1 𝑖=𝑖 𝑖=𝑖

➢ This is one of the most important and widely used inequalities in Maths.11
➢ It was first proved in the 19𝑡ℎ century.

11 This inequality has a Vector form, and an integral form as well.

P a g e 207 | 274
Get all the files at: https://bit.ly/azizhandouts
Aziz Manva (azizmanva@gmail.com)

Example 4.186: Proving the Two Variable Cauchy Schwarz12


A. Substitute 𝑛 = 2 in the Cauchy Schwarz equality to obtain the two-variable case of the inequality. Then
make the substitution 𝑢1 = 𝑎, 𝑣1 = 𝑏, 𝑢2 = 𝑐, 𝑣2 = 𝑑 to make the variables easier to work with.
B. Expand both sides of the inequality from Part A, and simplify, to get an equivalent inequality.
C. Apply the 𝐴𝑀 − 𝐺𝑀 inequality to prove Part B.

Part A
Substitute 𝑛 = 2 to get:
(𝑢1 𝑣1 + 𝑢2 𝑣2 )2 ≤ (𝑢12 + 𝑢22 )(𝑣12 + 𝑣22 )
Substitute 𝑢1 = 𝑎, 𝑣1 = 𝑏, 𝑢2 = 𝑐, 𝑣2 = 𝑑
⇔ (𝑎𝑏 + 𝑐𝑑)2 ≤ (𝑎2 + 𝑐 2 )(𝑏2 + 𝑑2 )
Part B
⇔ 𝑎2 𝑏 2 + 2𝑎𝑏𝑐𝑑 + 𝑐 2 𝑑2 ≤ 𝑎2 𝑏 2 + 𝑎2 𝑑2 + 𝑐 2 𝑏2 + 𝑐 2 𝑑2
⇔ 2𝑎𝑏𝑐𝑑 ≤ 𝑎2 𝑑2 + 𝑐 2 𝑏2
Part C
𝑎2 𝑑 2 + 𝑐 2 𝑏 2
√𝑎2 𝑑2 𝑐 2 𝑏2 ≤
2
2|𝑎𝑏𝑐𝑑| ≤ 𝑎2 𝑑2 + 𝑐 2 𝑏2
2𝑎𝑏𝑐𝑑 ≤ 𝑎2 𝑑2 + 𝑐 2 𝑏 2

C. AM-GM Inequality: Three Variables

4.187: AM-GM: Three Variables


For positive real numbers 𝑎, 𝑏 and 𝑐:
𝑎+𝑏+𝑐 3
≥ √𝑎𝑏𝑐
3

We assume the four-variable case here.13


1 𝑤+𝑥+𝑦+𝑧
Substitute 𝑤 = (𝑥 + 𝑦 + 𝑧) in ≥ 4√𝑤𝑥𝑦𝑧:
3 4
1 4
(𝑥 + 𝑦 + 𝑧) + 𝑥 + 𝑦 + 𝑧 (𝑥 + 𝑦 + 𝑧) 𝑥 + 𝑦 + 𝑧
𝐿𝐻𝑆 = 3 =3 =
4 4 3

And hence the inequality becomes:


𝑥+𝑦+𝑧 4 1
≥ √ (𝑥 + 𝑦 + 𝑧)𝑥𝑦𝑧
3 3
4 1
Divide both sides by √3 (𝑥 + 𝑦 + 𝑧):
3
𝑥+𝑦+𝑧 4 4
( ) ≥ √𝑥𝑦𝑧
3
4
Raise both sides to the power :
3
𝑥+𝑦+𝑧 3
≥ √𝑥𝑦𝑧
3

12 There are other ways of proving the general Cauchy Schwarz, which are not necessarily difficult. But our objective here is
to 𝑎𝑝𝑝𝑙𝑦 the AM-GM inequality.
13 The four variable case is proved below.

P a g e 208 | 274
Get all the files at: https://bit.ly/azizhandouts
Aziz Manva (azizmanva@gmail.com)

Example 4.188
Let 𝑥, 𝑦, and 𝑧 be positive real numbers such that (𝑥 ∙ 𝑦) + 𝑧 = (𝑥 + 𝑧) ∙ (𝑦 + 𝑧). What is the maximum possible
value of 𝑥𝑦𝑧? (HMMT 2005/Guts/2)

Expand the given expression on the RHS:


𝑥𝑦 + 𝑧 = 𝑥𝑦 + 𝑥𝑧 + 𝑧𝑦 + 𝑧 2
𝑧 = 𝑥𝑧 + 𝑧𝑦 + 𝑧 2
Since 𝑧 ≠ 0, divide both sides by z:
𝑥+𝑦+𝑧 = 1
But we know that, by the three variable AM-GM Inequality:
𝑥+𝑦+𝑧 3
≥ √𝑥𝑦𝑧
3
Substitute 𝑥 + 𝑦 + 𝑧 = 1:
1 3
≥ √𝑥𝑦𝑧
3
Cube both sides:
1 1 1
≥ 𝑥𝑦𝑧 ⇒ 𝑥𝑦𝑧 ≤ ⇒ 𝑀𝑎𝑥(𝑥𝑦𝑧) =
27 27 27

Example 4.189
Let 𝐴, 𝑀, and 𝐶 be nonnegative integers such that 𝐴 + 𝑀 + 𝐶 = 12. What is the maximum value of 𝐴 ∙ 𝑀 ∙ 𝐶 + 𝐴 ∙
𝑀 + 𝑀 ∙ 𝐶 + 𝐴 ∙ 𝐶? (AMC 12 2000/12)

Shortcut Method
Since the expression is symmetric, make the values of the variables equal:
12
𝐴=𝑀=𝐶= =4
3
Formal Method
Note that:
(𝐴 + 1)(𝑀 + 1)(𝐶 + 1) = 𝐴𝑀𝐶 + 𝐴𝑀 + 𝑀𝐶 + 𝐴𝐶 + 𝐴 + 𝑀 + 𝐶 + 1
Substitute 𝐴 + 𝑀 + 𝐶 = 12:
𝐴𝑀𝐶 + 𝐴𝑀 + 𝑀𝐶 + 𝐴𝐶 = (𝐴 + 1)(𝑀 + 1)(𝐶 + 1) − 13
Maximizing the LHS is the same as maximizing the RHS.
3 (𝐴 + 1) + (𝑀 + 1) + (𝐶 + 1)
√(𝐴 + 1)(𝑀 + 1)(𝐶 + 1) ≤
3
Cube both sides:
𝐴+𝑀+𝐶+3 3 12 + 3 3 15 3
(𝐴 + 1)(𝑀 + 1)(𝐶 + 1) ≤ ( ) =( ) = ( ) = 53 = 125
3 3 3
Equality is achieved when:
𝐴+1=𝑀+1=𝐶+1=5⇒𝐴 =𝑀 =𝐶 =4

4.190: Manipulating the Expression


➢ You may need to manipulate the expression to get into a form where AM-GM can be applied.
➢ This manipulation may not be obvious until you have experience with such expressions.

Example 4.191
A square sheet of metal of side length 𝑠 has smaller squares of side length 𝑥 cut at each of its corners, and the
resulting “flaps” folded to form an open-top box.
A. Show that the volume of the box is 𝑉 = (𝑠 − 2𝑥)2 (𝑥).

P a g e 209 | 274
Get all the files at: https://bit.ly/azizhandouts
Aziz Manva (azizmanva@gmail.com)

B. Apply AM-GM on 𝑉.
C. Apply AM-GM on 4𝑉. Find the value of 𝑥 in terms of 𝑠.

Part A By AM-GM Inequality:


𝑉 = (𝑠 − 2𝑥)2 (𝑥) = (𝑠 − 2𝑥)(𝑠 − 2𝑥)(𝑥) (𝑠 − 2𝑥) + (𝑠 − 2𝑥) + 4𝑥 3
≥ √(𝑠 − 2𝑥)2 (4𝑥)
3
Simplify:
2𝑠 3
≥ √(𝑠 − 2𝑥)2 (4𝑥)
3
Cube:
8𝑠 3
≥ (𝑠 − 2𝑥)2 (4𝑥)
27
Divide both sides by 4:
Part B 2𝑠 3
Apply AM-GM to the above: ≥ (𝑠 − 2𝑥)2 (𝑥)
27
(𝑠 − 2𝑥) + (𝑠 − 2𝑥) + 𝑥 3 Hence, the maximum is:
≥ √(𝑠 − 2𝑥)2 (𝑥)
3 2𝑠 3
2𝑠 − 3𝑥 3
≥ √(𝑠 − 2𝑥)2 (𝑥) 27
3 And this maximum is achieved when the three
Now if we cube the expression, the LHS is the cube
terms are equal:
of a binomial making the expression very messy. 𝑠
Part C 𝑠 − 2𝑥 = 4𝑥 ⇒ 𝑥 =
6
4𝑉 = (𝑠 − 2𝑥)(𝑠 − 2𝑥)(4𝑥)

D. AM-GM Inequality: General Case

4.192: AM-GM: Four Variables


𝑤+𝑥+𝑦+𝑧 4
≥ √𝑤𝑥𝑦𝑧
4

By AM-GM, we must have:


𝑤+𝑥 𝑦+𝑧
≥ √𝑤𝑥, ≥ √𝑦𝑧
2 2
Add the two, and then divide by two to get:
𝑤 + 𝑥 + 𝑦 + 𝑧 √𝑤𝑥 + √𝑦𝑧

⏟ 4 2
𝑰𝒏𝒆𝒒𝒖𝒂𝒍𝒊𝒕𝒚 𝑰
Apply AM-GM on the RHS one more time to get:
√𝑤𝑥 + √𝑦𝑧 4
≥ √𝑤𝑥𝑦𝑧
⏟ 2
𝑰𝒏𝒆𝒒𝒖𝒂𝒍𝒊𝒕𝒚 𝑰𝑰𝑰
And then combine Inequalities I and II to get:
𝑤+𝑥+𝑦+𝑧 4
≥ √𝑤𝑥𝑦𝑧
4

4.193: AM-GM: General Case


𝑥1 + 𝑥2 + ⋯ + 𝑥𝑛 𝑛
≥ √𝑥1 𝑥2 … 𝑥𝑛
𝑛

P a g e 210 | 274
Get all the files at: https://bit.ly/azizhandouts
Aziz Manva (azizmanva@gmail.com)

One way to prove the general case of the AM-GM inequality is by forward-backward induction.
➢ Prove the base case for 𝑛 = 2. We did this above.
➢ Forward Step: Prove that if it is true for 𝑛 = 𝑘, then it is true for 𝑛 = 2𝑘
➢ Backward: Prove that if it is true for 𝑛 = 𝑘, then it is true for 𝑛 = 𝑘 − 1

A. Relation between Means

4.194: AM, HM and GM: Two Variable Case


For two positive variables 𝑎 and 𝑏:
𝐺𝑀2 = 𝐴𝑀 ∙ 𝐻𝑀

2
𝐿𝐻𝑆 = 𝐺𝑀2 = (√𝑎𝑏) = 𝑎𝑏
𝑎 + 𝑏 2𝑎𝑏
𝑅𝐻𝑆 = 𝐴𝑀 ∙ 𝐻𝑀 = ∙ = 𝑎𝑏 = 𝐿𝐻𝑆
2 𝑎+𝑏

Example 4.195
The harmonic mean of two numbers is 4. Their arithmetic mean 𝐴 and the geometric mean 𝐺 satisfy the
relation 2𝐴 + 𝐺 2 = 27. Find the two numbers. (JEE Adv. 1979)

Let the numbers be 𝑥 and 𝑦. Since their harmonic mean is 4:


2𝑥𝑦
= 4 ⇒ 2𝑥𝑦 = 4𝑥 + 4𝑦 ⇒ ⏟
𝑥𝑦 = 2𝑥 + 2𝑦
𝑥+𝑦
𝑬𝒒𝒖𝒂𝒕𝒊𝒐𝒏 𝑰
Since 2𝐴 + 𝐺 2 = 27, we must have:
2(𝑥 + 𝑦) 2
+ (√𝑥𝑦) = 27 ⇒ 𝑥 + 𝑦 + 𝑥𝑦 = 27
2
Substitute the value of 𝑥𝑦 from Equation I in the above:
𝑥 + 𝑦 + 2𝑥 + 2𝑦 = 27 ⇒ 3𝑥 + 3𝑦 = 27 ⇒ ⏟𝑥+𝑦 =9
𝑬𝒒𝒖𝒂𝒕𝒊𝒐𝒏 𝑰𝑰
𝑥𝑦 = 2(𝑥 + 𝑦) = 18 ⇒ ⏟
𝑥𝑦 = 18
𝑬𝒒𝒖𝒂𝒕𝒊𝒐𝒏 𝑰𝑰𝑰
18
Solve the system of equations. Substitute 𝑥𝑦 = 18 ⇒ 𝑦 = in Equation II
to get a quadratic and solve:
𝑥
18
𝑥+ = 9 ⇒ 𝑥 2 − 9𝑥 + 18 = 0 ⇒ (𝑥 − 6)(𝑥 − 3) = 0 ⇒ 𝑥 ∈ {3,6}
𝑥

Challenge 4.196
𝑀𝑎𝑟𝑘 𝑎𝑙𝑙 𝑐𝑜𝑟𝑟𝑒𝑐𝑡 𝑜𝑝𝑡𝑖𝑜𝑛𝑠
Let 𝑎, 𝑏 be positive real numbers. If 𝑎, 𝐴1 , 𝐴2 , 𝑏 are in arithmetic progression, 𝑎, 𝐺1 , 𝐺2 , 𝑏 are in geometric
progression and 𝑎, 𝐻1 , 𝐻2 , 𝑏 are in harmonic progression, and let:
𝐺1 𝐺2 𝐴1 + 𝐴2 (2𝑎 + 𝑏)(𝑎 + 2𝑏)
𝑋= , 𝑌= , 𝑍=
𝐻1 𝐻2 𝐻1 + 𝐻2 9𝑎𝑏
Then:
A. 𝑋 = 𝑌
B. 𝑌 = 𝑍
C. 𝑋 = 𝑍
D. None of the above (JEE Adv. 2002, Adapted)

P a g e 211 | 274
Get all the files at: https://bit.ly/azizhandouts
Aziz Manva (azizmanva@gmail.com)

If the geometric progression is 𝑎, 𝐺1 = 𝑎𝑟, 𝐺2 = 𝑎𝑟 2 , 𝑏 = 𝑎𝑟 3 for some common ratio 𝑟:


𝐺1 𝐺2 = (𝑎𝑟)(𝑎𝑟 2 ) = 𝑎2 𝑟 2 = 𝑎(𝑎𝑟 3 ) = 𝑎𝑏

If the arithmetic progression is 𝑎, 𝐴1 = 𝑎 + 𝑑, 𝐴2 = 𝑎 + 2𝑑, 𝑏 = 𝑎 + 3𝑑 for some common difference then:


𝐴1 + 𝐴2 = (𝑎 + 𝑑) + (𝑎 + 2𝑑) = 𝑎 + 𝑎 + 3𝑑 = 𝑎 + 𝑏

Finally, since 𝑎, 𝐻1 , 𝐻2 , 𝑏 are in harmonic progression, their reciprocals are in arithmetic progression:
1 1 1 1 1 1 1 𝑎−𝑏
, , , ⇒ 𝐶𝑜𝑚𝑚𝑜𝑛 𝐷𝑖𝑓𝑓𝑒𝑟𝑒𝑛𝑐𝑒 = 𝐷 = ( − ) =
𝑎 𝐻1 𝐻2 𝑏 3 𝑏 𝑎 3𝑎𝑏
Then:
1 1 1 𝑎 − 𝑏 3𝑏 + 𝑎 − 𝑏 2𝑏 + 𝑎 3𝑎𝑏
= +𝐷 = + = = ⇒ 𝐻1 =
𝐻1 𝑎 𝑎 3𝑎𝑏 3𝑎𝑏 3𝑎𝑏 2𝑏 + 𝑎
1 1 1 𝑎−𝑏 3𝑏 + 2𝑎 − 2𝑏 2𝑎 + 𝑏 3𝑎𝑏
= + 2𝐷 = + 2 ( )= = ⇒ 𝐻2 =
𝐻2 𝑎 𝑎 3𝑎𝑏 3𝑎𝑏 3𝑎𝑏 2𝑎 + 𝑏
Hence:
3𝑎𝑏 3𝑎𝑏 9𝑎2 𝑏 2
𝐻1 𝐻2 = × =
2𝑏 + 𝑎 2𝑎 + 𝑏 (2𝑏 + 𝑎)(2𝑎 + 𝑏)
3𝑎𝑏 3𝑎𝑏 (3𝑎𝑏)(2𝑎 + 𝑏) + (3𝑎𝑏)(2𝑏 + 𝑎) (9𝑎𝑏)(𝑎 + 𝑏)
𝐻1 + 𝐻2 = + = =
2𝑏 + 𝑎 2𝑎 + 𝑏 (2𝑏 + 𝑎)(2𝑎 + 𝑏) (2𝑏 + 𝑎)(2𝑎 + 𝑏)

𝐺1 𝐺2 𝑎𝑏 (2𝑏 + 𝑎)(2𝑎 + 𝑏) (2𝑏 + 𝑎)(2𝑎 + 𝑏)


𝑋= = 2 2 = 𝑎𝑏 ∙ = =𝑍
𝐻1 𝐻2 9𝑎 𝑏 9𝑎2 𝑏 2 9𝑎𝑏
(2𝑏 + 𝑎)(2𝑎 + 𝑏)
𝐴1 + 𝐴2 𝑎+𝑏 (2𝑏 + 𝑎)(2𝑎 + 𝑏) (2𝑏 + 𝑎)(2𝑎 + 𝑏)
𝑌= = = (𝑎 + 𝑏) ∙ = =𝑍
𝐻1 + 𝐻2 (9𝑎𝑏)(𝑎 + 𝑏) (9𝑎𝑏)(𝑎 + 𝑏) 9𝑎𝑏
(2𝑏 + 𝑎)(2𝑎 + 𝑏)

Hence:
𝑋 = 𝑌 = 𝑍 ⇒ 𝑂𝑝𝑡𝑖𝑜𝑛𝑠 𝐴, 𝐵, 𝐶

B. 𝑯𝑴 ≤ 𝑮𝑴 ≤ 𝑨𝑴 < 𝑹𝑴𝑺 Inequality

4.197: HM-GM Inequality: Two Variable Case


2𝑎𝑏
≤ √𝑎𝑏
𝑎+𝑏

Divide both sides by √𝑎𝑏:


2√𝑎𝑏
⇔ ≤1
𝑎+𝑏
Multiply both sides by 𝑎 + 𝑏:
⇔ 2√𝑎𝑏 ≤ 𝑎 + 𝑏
Square both sides:
⇔ 4𝑎𝑏 ≤ 𝑎2 + 2𝑎𝑏 + 𝑏 2
Subtract 4𝑎𝑏 from both sides:
⇔ 0 ≤ 𝑎2 − 2𝑎𝑏 + 𝑏 2
Factor the RHS:
⇔ 0 ≤ (𝑎 − 𝑏)2
Which is true by the trivial inequality.

P a g e 212 | 274
Get all the files at: https://bit.ly/azizhandouts
Aziz Manva (azizmanva@gmail.com)

And since each of the above steps is reversible, the inequality above is true.

Example 4.198
Show that if the harmonic mean and the geometric mean of two numbers are equal, then the numbers are also
equal.

2𝑎𝑏
= √𝑎𝑏
𝑎+𝑏
Divide both sides by √𝑎𝑏:
2√𝑎𝑏
=1
𝑎+𝑏
Multiply both sides by 𝑎 + 𝑏:
2√𝑎𝑏 = 𝑎 + 𝑏
Square both sides:
4𝑎𝑏 = 𝑎2 + 2𝑎𝑏 + 𝑏 2
Subtract 4𝑎𝑏 from both sides:
0 = 𝑎2 − 2𝑎𝑏 + 𝑏 2
Factor the RHS:
0 = (𝑎 − 𝑏)2
𝑎−𝑏=0
𝑎=𝑏
Challenge 4.199
𝐶ℎ𝑜𝑜𝑠𝑒 𝑒𝑥𝑎𝑐𝑡𝑙𝑦 𝑜𝑛𝑒 𝑠𝑡𝑎𝑡𝑒𝑚𝑒𝑛𝑡 𝑝𝑒𝑟 𝑔𝑟𝑜𝑢𝑝
Let 𝐴1 , 𝐺1 , 𝐻1 denote the arithmetic, geometric and harmonic means, respectively, of two distinct positive
numbers. For 𝑛 ≥ 2, Let 𝐴𝑛–1 and 𝐻𝑛–1 have arithmetic, geometric, and harmonic means as 𝐴𝑛 , 𝐺𝑛 , 𝐻𝑛
respectively.

Group I
A. 𝐺1 > 𝐺2 > 𝐺3 >. . .
B. 𝐺1 < 𝐺2 < 𝐺3 <. . .
C. 𝐺1 = 𝐺2 = 𝐺3 =. . .
D. 𝐺1 < 𝐺3 < 𝐺5 < ⋯ 𝑎𝑛𝑑 𝐺2 > 𝐺4 > 𝐺6 > ⋯

Group II
A. 𝐴1 > 𝐴2 > 𝐴3 >. . .
B. 𝐴1 < 𝐴2 < 𝐴3 <. . .
C. 𝐴1 > 𝐴3 > 𝐴5 >. . . 𝑎𝑛𝑑 𝐴2 < 𝐴4 < 𝐴6 <. . .
D. 𝐴1 < 𝐴3 < 𝐴5 < ⋯ 𝑎𝑛𝑑 𝐴2 > 𝐴4 > 𝐴6 > ⋯

Group III
A. 𝐻1 > 𝐻2 > 𝐻3 >. . .
B. 𝐻1 < 𝐻2 < 𝐻3 <. . .
C. 𝐻1 > 𝐻3 > 𝐻5 >. . . 𝑎𝑛𝑑 𝐻2 < 𝐻4 < 𝐻6 <. . .
D. 𝐻1 < 𝐻3 < 𝐻6 < ⋯ 𝑎𝑛𝑑 𝐻2 > 𝐻4 > 𝐻6 > ⋯ (JEE Adv. 2007)

Let the numbers be:


𝑎 𝑎𝑛𝑑 𝑏 ⇒ 𝑎 < 𝐻1 < 𝐺1 < 𝐴1 < 𝑏
Group II
𝐴2 = 𝐴𝑀(𝐻1 , 𝐴1 ) lies between 𝐻1 and 𝐴1

P a g e 213 | 274
Get all the files at: https://bit.ly/azizhandouts
Aziz Manva (azizmanva@gmail.com)

Similarly, 𝐴3 = 𝐴𝑀(𝐻2 , 𝐴2 ) lies between 𝐻2 and 𝐴2


In general:
𝐴1 > 𝐴2 > 𝐴3 ⇒ 𝑂𝑝𝑡𝑖𝑜𝑛 𝐴
Group III
𝐻2 = 𝐻𝑀(𝐻1 , 𝐴1 ) lies between 𝐻1 and 𝐴1
Similarly, 𝐻3 = 𝐻𝑀(𝐻2 , 𝐴2 ) lies between 𝐻2 and 𝐴2
In general:
𝐻1 < 𝐻2 < 𝐻3 ⇒ 𝑂𝑝𝑡𝑖𝑜𝑛 𝐵
Group I

𝐺12 = 𝑎𝑏 = 𝐴1 𝐻1
𝐺𝑛2 = 𝐴𝑛 𝐻𝑛
𝐺𝑛2 = 𝐴𝑛−1 𝐻𝑛−1

𝐴𝑛 𝐻𝑛 = 𝐴𝑛−1 𝐻𝑛−1

𝐴𝑛 𝐻𝑛 = 𝐴𝑛−1 𝐻𝑛−1 = ⋯ = 𝐴1 𝐻1 = 𝑎𝑏

𝐺𝑛2 = 𝐴𝑛 𝐻𝑛 = 𝑎𝑏 ⇒ 𝑂𝑝𝑡𝑖𝑜𝑛 𝐶

4.200: Root Mean Square


The root mean square of 𝑛 numbers is the square root of the arithmetic mean of the squares of the 𝑛 numbers.

Example 4.201
Find

4.202: AM-RMS Inequality: Two Variable Case


𝑎+𝑏 𝑎2 + 𝑏 2
≤√
2 2

𝑎2 + 2𝑎𝑏 + 𝑏 2 𝑎2 + 𝑏 2
⇔ ≤
4 2
⇔ 𝑎 + 2𝑎𝑏 + 𝑏 ≤ 2𝑎2 + 2𝑏 2
2 2

⇔ 0 ≤ 𝑎2 − 2𝑎𝑏 + 𝑏 2
⇔ 0 ≤ (𝑎 − 𝑏)2
Which is true by the trivial inequality.
And since each of the above steps is reversible, the inequality above is true.

4.203: Cascading Inequalities: 𝑯𝑴 ≤ 𝑮𝑴 ≤ 𝑨𝑴 ≤ 𝑹𝑴𝑺


2𝑎𝑏 𝑎+𝑏 𝑎2 + 𝑏 2
≤ √𝑎𝑏 ≤ ≤√
𝑎+𝑏 2 2

Example 4.204

P a g e 214 | 274
Get all the files at: https://bit.ly/azizhandouts
Aziz Manva (azizmanva@gmail.com)

The harmonic mean of 75, 85, 65 is between


A. 65 to 70
B. 70 to 75
C. 75 to 80
D. 80 to 85

AM = middle value = 75.


HM has to be less than 75, and can be approximated to be not so far away from AM14.
𝑜𝑝𝑡𝑖𝑜𝑛 𝐵

C. Minkowski Inequality

4.205: Minkowski Inequality: 𝒏 = 𝟐, 𝒑 = 𝟐


√𝑎12 + 𝑎22 + √𝑏12 + 𝑏22 ≥ √(𝑎1 + 𝑏1 )2 + (𝑎1 + 𝑏1 )2

(Calculator) Example 4.206


A main gas line runs through 𝑃 and Q. From some point 𝑇 on 𝑃𝑄, a supply line runs to a house at point 𝑀. A
second supply line from 𝑇 runs to a house at point 𝑁. What is the minimum total length of pipe required for the
two supply lines? (CEMC Cayley 1999/22)

Drop a perpendicular from N to MP. Then,


𝑀𝑅 = 105 − 55 = 50
By a Pythagorean Triplet
𝑁𝑅 = 120

Reflect 𝑁 across 𝑃𝑄 to get S.


𝑁𝑄 = 𝑄𝑆, ∠𝑇𝑄𝑁 = ∠𝑇𝑄𝑆 ⇒ Δ𝑇𝑄𝑁 ≅ 𝑇𝑄𝑆 ⇒ 𝑇𝑁 = 𝑇𝑆

We need to minimize
𝑀𝑇 + 𝑇𝑁 = 𝑀𝑇 + 𝑇𝑆
Which will get minimized when 𝑀, 𝑇 and S are collinear.

𝑀𝑆 = √(105 + 55)2 + 1202 = 200

3 71
14 Actual value of 𝐻𝑀(65, 75, 85) = 1 1 1 = 74 is not a good expression to work with.
+ + 671
65 75 85

P a g e 215 | 274
Get all the files at: https://bit.ly/azizhandouts
Aziz Manva (azizmanva@gmail.com)

Example 4.207
A main gas line runs through 𝑃 and Q. From some point 𝑇 on 𝑃𝑄, a supply line runs to a house at point 𝑀. A
second supply line from 𝑇 runs to a house at point 𝑁. What is the minimum total length of pipe required for the
two supply lines? (CEMC Cayley 1999/22)

𝑃𝑖𝑝𝑒 𝐿𝑒𝑛𝑔𝑡ℎ = 𝑁𝑇 + 𝑀𝑇 = √552 + 𝑥 2 + √1052 + (120 − 𝑥)2

√𝑎12 + 𝑎22 + √𝑏12 + 𝑏22 ≥ √(𝑎1 + 𝑏1 )2 + (𝑎1 + 𝑏1 )2

√(𝑎1 + 𝑏1 )2 + (𝑎1 + 𝑏1 )2 = √(55 + 105)2 + (𝑥 + 120 − 𝑥)2


= √1602 + 1202 = 200

4.208: Minkowski Inequality: 𝒑 = 𝟐


𝑛 𝑛 𝑛

√∑ 𝑎𝑖2 + √∑ 𝑏𝑖2 ≥ √∑(𝑎𝑖 + 𝑏𝑖 )2


𝑖=1 𝑖=1 𝑖=1

4.209: Minkowski Inequality: General Case


𝑝
𝑛 𝑝
𝑛 𝑝
𝑛

√∑ 𝑎𝑖𝑝 + √∑ 𝑏𝑖𝑝 ≥ √∑(𝑎1 + 𝑏1 )2


𝑖=1 𝑖=1 𝑖=1

4.6 Recursive and Cyclic Sequences


A. Recursive Sequences

4.210: Recursive Sequences


➢ When any sequence is defined in terms of preceding terms of the sequence, the sequence is recursively
defined.
➢ A recursive sequence also needs a bootstrap (or a starting point) to be completely defined.

Example 4.211
Each sequence below is given in terms of a recursive definition. Write an explicit definition for the sequence. If
the sequence is of a common type, identify the nature and parameters and the 𝑛𝑡ℎ term of the sequence.

A. 𝑎𝑛 = 𝑛𝑎𝑛−1 , 𝑛 ≥ 2, 𝑎1 = 1
B. 𝑎𝑛 = 𝑎𝑛−1 + 𝑛, 𝑛 ≥ 2, 𝑎1 = 1

P a g e 216 | 274
Get all the files at: https://bit.ly/azizhandouts
Aziz Manva (azizmanva@gmail.com)
1
C. 𝑎𝑛 = 𝑎𝑛−1 + 5, 𝑛 ≥ 2, 𝑎1 = 2
2 81
D. 𝑎𝑛 = 3 𝑎𝑛−1 , 𝑛 ≥ 2, 𝑎1 = 64
E. 𝑎𝑛 = 𝑎𝑛−1 + 2𝑛 − 1, 𝑛 ≥ 2, 𝑎1 = 1
F. 𝑓𝑛 = 𝑓𝑛−1 + 𝑓𝑛−2 , 𝑛 ≥ 3, 𝑓1 = 1, 𝑓2 = 1

Part A and common difference 5. The 𝑛𝑡ℎ term is


𝑎1 = 1 = 𝑎 + (𝑛 − 1)𝑑 = 0.5 + (𝑛 − 1)5 = 5𝑛 − 4.5
𝑎2 = 2 × 1 = 2 Part D
𝑎3 = 3(2) = 6 The first few terms are:
𝑎4 = 4(3) = 24 81 27 9
, , ,…
𝑎5 = 5(4) = 120 64 32 16
𝑎6 = 6(120) = 720 81 34
This is a geometric sequence with first term 64 = 26 ,
The explicit definition for factorials: 2
𝑛! = 𝑛 ⏟(𝑛 − 1)(𝑛 − 2) … 1 = ⏟ 𝑛(𝑛 − 1)! and common ratio 3. It has 𝑛𝑡ℎ term
(𝒏−𝟏)! 𝑹𝒆𝒄𝒖𝒓𝒔𝒊𝒗𝒆 34 2𝑛−1 2𝑛−7
𝑫𝒆𝒇𝒊𝒏𝒕𝒊𝒐𝒏 = 𝑎𝑟 𝑛−1 = × =
26 3𝑛−1 3𝑛−5
Hence, the 𝑛𝑡ℎ term is: Part E
𝑎𝑛 = 𝑛! The first few terms:
Part B 1,4,9,16,25,36, …
𝑎1 = 1 These are the square numbers given by
𝑎2 = 𝑎1 + 2 = 1 + 2 = 3 𝑛2
𝑎3 = 𝑎2 + 3 = 3 + 3 = 6 Part F
𝑎4 = 𝑎3 + 4 = 6 + 4 = 10 The first few terms are:
Triangular Numbers with explicit formula: 1,1,2,3,5,8,13,21,34, …
1 + 2 + 3 + ⋯ + (𝑛 − 2) + (𝑛 − 1) + 𝑛 This sequence is the famous Fibonacci sequence.
𝑡ℎ
The 𝑛 term is 𝜑𝑛 − 𝜓𝑛
𝑛+1 𝐹𝑛 =
= 𝑛( ) √5
2
Part C 1 + √5 1 − √5
𝜑= ,𝜓 =
The first few terms are: 2 2
0.5,5.5,10.5, … These are the solutions to the quadratic
This is an arithmetic sequence with first term 0.5 𝑥2 − 𝑥 − 1 = 0

Example 4.212: Shifting

B. Cyclic Sequences

4.213: Cyclic Property


When a sequence repeats, it is said to be cyclic.

4.214: Self Inverse functions have cyclicity 2


Self-inverse functions result in a sequence that has cyclicity 2.

➢ A function that reverses the input of a function 𝑓 is called the inverse function.
➢ There are certain functions that are their own inverse. Such functions are called self-inverse.

Example 4.215
Give examples of simple self-inverse functions.

P a g e 217 | 274
Get all the files at: https://bit.ly/azizhandouts
Aziz Manva (azizmanva@gmail.com)

A. Give them the input 5, and show that they have cyclicity 2.
B. Write them as recursive sequences and show that they are cyclical.

Reciprocal Function
1 1 1
𝑓(𝑛) = , 𝑓(5) = , 𝑓 ( ) = 5
𝑛 5 5
1 1 1
𝑎1 = 5, 𝑎𝑛 = ⇒ 𝑎1 = 5, 𝑎2 = , 𝑎3 = 5, 𝑎4 = , …
𝑎𝑛−1 5 5
Negation Function
𝑓(𝑛) = −𝑛, 𝑓(5) = −5, 𝑓(−5) = 5
𝑎1 = 5, 𝑎𝑛 = −𝑎𝑛−1 ⇒ 𝑎1 = 5, 𝑎2 = −5, 𝑎3 = 5, 𝑎4 = −5, …

Example 4.216
3 1−𝑎
The first term of a sequence is 8. If 𝑎 is a term, the next term is 1+𝑎. The 2007𝑡ℎ term is: (NMTC Primary-
Screening, 2007/11).

We calculate the second and the third terms:


3 5
1 − 𝑡1 1 − 8 8 = 5
𝑡⏟2 = = =
𝑺𝒆𝒄𝒐𝒏𝒅
1 + 𝑡1 1 + 3 11 11
𝑻𝒆𝒓𝒎 8 8
5 6
1 − 𝑡2 1 − 11 11 = 6 = 3
𝑡⏟3 = = =
1 + 𝑡2 1 + 5 16 16 8
𝑻𝒉𝒊𝒓𝒅
𝑻𝒆𝒓𝒎 11 11

The third term is the same as the first term. Hence, the sequence is cyclical. The pattern is:
3 5
,
⏟8 ⏟
11
𝑂𝑑𝑑 𝑇𝑒𝑟𝑚 𝐸𝑣𝑒𝑛 𝑇𝑒𝑟𝑚
𝑁𝑢𝑚𝑏𝑒𝑟 𝑁𝑢𝑚𝑏𝑒𝑟
3
2007th term ⇒ Odd Term ⇒
8

Example 4.217
Find the sum to 100 terms if:
𝑡𝑛−1 + 1
𝑡1 = 1, 𝑡2 = 2, 𝑡𝑛 =
𝑡𝑛−2
(NMTC Primary-Final, 2006/2)

We use the definition to find the first few terms


𝑡2 + 1 2 + 1 3
𝑡3 = = = =3
𝑡1 1 1
𝑡3 + 1 3 + 1 4
𝑡4 = = = =2
𝑡2 2 2
𝑡4 + 1 2 + 1 3
𝑡5 = = = =1
𝑡3 3 3
𝑡5 + 1 1 + 1 2
𝑡6 = = = =1
𝑡4 2 2
𝑡6 + 1 1 + 1 2
𝑡7 = = = =2
𝑡5 1 1

P a g e 218 | 274
Get all the files at: https://bit.ly/azizhandouts
Aziz Manva (azizmanva@gmail.com)

𝑡3 = 3, 𝑡4 = 2, 𝑡5 = 1, 𝑡6 = 1, 𝑡7 = 2
This is a cyclical sequence which repeats every five terms, and the total of one cycle is:
1+2+3+2+1=9
To find the sum to 100 terms, note that the cycle repeats
100
= 20 𝑡𝑖𝑚𝑒𝑠
5
And hence, the total is:
20 × 9 = 180

Example 4.218
A. A sequence of numbers starts with 0,0,1 and every number of the sequence from the fourth term is the
sum of the previous three numbers. The tenth number in the sequence is:
B. A sequence of numbers starts with 1, 2, and 3. The fourth number of the sequence is the sum of the
previous three numbers in the sequence: 1+2+3=6. In the same way, every number after the fourth is
the sum of the previous three numbers. What is the eighth number in the sequence? (AMC 8 2009/5)
C. A sequence of numbers starts with 1, 2, 3 and every number of the sequence from the fourth term is the
sum of the previous three numbers. The tenth number in the sequence is: (NMTC Primary-
III/Screening/8)

Part A
0,0,1,1,2,4,7,13,24,44
Part B
1,2,3,6,11,20,37,68
Part C
1,2,3,6,11,20,37,68,125,230

Example 4.219
A list of 8 numbers is formed by beginning with two given numbers. Each new number in the list is the product
of the two previous numbers. Find the first number if the last three are shown:
−, −, −, −, −,16,64,1024
(AMC 8 1990/21)

𝑡1 , 𝑡2 , 𝑡3 , 𝑡4 , 𝑡5 , 16,64,1024

𝐶ℎ𝑒𝑐𝑘: 16 × 64 = 1024
𝑡1 , 𝑡2 , 𝑡3 , 𝑡4 , 4,16,64,1024
𝑡1 , 𝑡2 , 𝑡3 , 4,4,16,64,1024
𝑡1 , 𝑡2 ,1,4,4,16,64,1024
𝑡1 , 4 ,1,4,4,16,64,1024
1
, 4 ,1,4,4,16,64,1024
4
C. Fibonacci Sequence (1, 1)
Recursive sequences have a distinct flavor compared to other mathematical topics. It is important to be
understand and apply the concepts. In particular, the properties that we show for the Fibonacci sequence are
applicable to Fibonacci-type sequences as well.
The Fibonacci sequence was first made popular by the Italian mathematician Fibonacci. It has many interesting
properties, and hence many questions in competitions are based on this series:
Definition

P a g e 219 | 274
Get all the files at: https://bit.ly/azizhandouts
Aziz Manva (azizmanva@gmail.com)

4.220: Fibonacci Sequence


𝑓𝑛 = 𝑓𝑛−1 + 𝑓𝑛−2 , 𝑓1 = 1, 𝑓2 = 1, 𝑛 ≥ 3


2=⏟
1+⏟
1
𝑓3 𝑓2 𝑓1

Example 4.221
Find the first few values of the Fibonacci Sequence

First two terms, by definition, of the Fibonacci series are 1 and 1:


➢ 𝑓1 = 1, 𝑓2 = 1
For all terms after the first two, any term is the sum of the two terms before it:
➢ 𝑓3 = 𝑓2 + 𝑓1 = 1 + 1 = 2
➢ 𝑓4 = 𝑓3 + 𝑓2 = 2 + 1 = 3
➢ 𝑓5 = 𝑓4 + 𝑓3 = 3 + 2 = 5
1,1,2,3,5,8,13,21,34,55,89, …

Example 4.222
Calculate 𝑓7. Also, find 𝑓7 in terms of 𝑓1 and 𝑓2, where 𝑓𝑛 represents the 𝑛𝑡ℎ term in the Fibonacci sequence. And
verify that your answer both methods are the same via substitution.

𝑓6 = 8, 𝑓7 = 13
𝑓7 = 𝑓
⏟6 + 𝑓5 = 2𝑓5 + 𝑓4 = 3𝑓4 + 2𝑓3 = 5𝑓3 + 3𝑓2 = 8𝑓2 + 5𝑓1
𝒇𝟔 =𝒇𝟓 +𝒇𝟒

Example 4.223
The Fibonacci sequence 1, 1, 2, 3, 5, 8, 13, 21, … starts with two 1s, and each term afterwards is the sum of its
two predecessors. Which one of the ten digits is the last to appear in the units position of a number in the
Fibonacci sequence? (AMC 10 2000/6)

We can do this by finding the terms of the Fibonacci sequence. But since we only want the unit’s digit, we only
add that, which reduces our work considerably:
1,1,2,3,5,8,3,1,4,5,9,4,3,7,0,7,7,4,1,5,6, ….

Strike out each Units Digit as it appears:


0 1 2 3 4 5 6 7 8 9
9th 1st 2nd 3rd 6th 4th 10th 8th 5th 7th

D. Back Calculations in Fibonacci Sequence (1, 1)


Given terms in the Fibonacci Sequence, it can be extended backwards.

Example 4.224: Back Calculation


Suppose that
𝑓𝑛 = 3, 𝑓𝑛+1 = 5, 𝑓𝑛 = 𝑓𝑛−1 + 𝑓𝑛−2
Find the terms
𝑓𝑛−1 , 𝑓𝑛−2 , 𝑓𝑛−3 … . upto 10 terms
And hence find a pattern connecting this sequence with the Fibonacci sequence.

P a g e 220 | 274
Get all the files at: https://bit.ly/azizhandouts
Aziz Manva (azizmanva@gmail.com)

The given formula is that of the Fibonacci sequence, as are the given terms. We need to calculate backwards.
𝑓𝑛+1 = 𝑓𝑛 + 𝑓𝑛−1 ⇒ 𝑓𝑛−1 = 𝑓𝑛+1 − 𝑓𝑛 = 5 − 3 = 2

Tabulate the terms to see the pattern


𝑓𝑛 |𝑓𝑛 |
𝑓𝑛+1 − 𝑓𝑛 = 5 − 3 2
𝑓𝑛−2 𝑓𝑛 − 𝑓𝑛−1 = 3 − 2 1
𝑓𝑛−3 𝑓𝑛−1 − 𝑓𝑛−2 = 2 − 1 1
𝑓𝑛−4 𝑓𝑛−2 − 𝑓𝑛−3 = 1 − 1 0 0
𝑓𝑛−5 𝑓𝑛−3 − 𝑓𝑛−4 = 1 − 0 1 1
𝑓𝑛−6 𝑓𝑛−4 − 𝑓𝑛−5 = 0 − 1 −1 1
𝑓𝑛−7 𝑓𝑛−5 − 𝑓𝑛−6 = 1 − (−1) 2 2
𝑓𝑛−8 𝑓𝑛−6 − 𝑓𝑛−7 = −1 − 2 −3 3
𝑓𝑛−9 𝑓𝑛−7 − 𝑓𝑛−8 = 2 − (−3) 5 5
𝑓𝑛−10 𝑓𝑛−8 − 𝑓𝑛−9 = −3 − 5 −8 8

Taking the absolute of 𝑓𝑛 recovers the Fibonacci sequence, even if we go backwards in the sequence.

Consider the sequence defined as above. Let


𝑓𝑚 = 0
Find the sum of the twenty-one terms mentioned below:
𝑓𝑚−10 , 𝑓𝑚−9 , … , 𝑓𝑚−1 , 𝑓𝑚 , 𝑓𝑚+1 , … , 𝑓𝑚+9 , 𝑓𝑚+10

𝑚 𝑚+1 𝑚+2 𝑚+3 𝑚+4 𝑚+5 𝑚+6 𝑚+7 𝑚+8 𝑚+9 𝑚 + 10


0 1 1 2 3 5 8 13 21 34 55

𝑚 𝑚−1 𝑚−2 𝑚−3 𝑚−4 𝑚−5 𝑚−6 𝑚−7 𝑚−8 𝑚−9 𝑚 − 10


0 1 −1 2 −3 5 −8 13 −21 34 −55

2 4 10 26 68

2 + 4 + 10 + 26 + 68 = 110
E. Fibonacci-Type Series (n, m)
You can generalize the Fibonacci sequence. The original sequence is:
𝑓𝑛 = 𝑓𝑛−1 + 𝑓𝑛−2 , 𝑓1 = 1, 𝑓2 = 1, 𝑛 ≥ 3

If we change the initial values, we get a different sequence, but one that still satisfies the recurrence relations
that the Fibonacci sequence does.

Example 4.225
Find the first ten terms of the sequence given by:
A. 𝑓1 = 1, 𝑓2 = 3, 𝑓𝑛 = 𝑓𝑛−1 + 𝑓𝑛−2 , 𝑛 ≥ 3
B. 𝑓1 = −1, 𝑓2 = 0, 𝑓𝑛 = 𝑓𝑛−1 + 𝑓𝑛−2 , 𝑛 ≥ 3
1 1
C. 𝑓1 = , 𝑓2 = , 𝑓𝑛 = 𝑓𝑛−1 + 𝑓𝑛−2 , 𝑛 ≥ 3
2 3

Part A
1,3,4,7,11,18,29,47,76,123
Part B

P a g e 221 | 274
Get all the files at: https://bit.ly/azizhandouts
Aziz Manva (azizmanva@gmail.com)

−1,0, −1, −1, −2, −3, −5, −8, −13


Part C
1 1 5 7 12 19 31 50
, , , , , , ,
2 3 6 6 6 6 6 6

Example 4.226
The increasing sequence of positive integers 𝑎1 , 𝑎2 , 𝑎3 … has the property that
𝑎𝑛+2 = 𝑎𝑛 + 𝑎𝑛+1 for all 𝑛 ≥ 1
If 𝑎7 = 120, then 𝑎8 = (AHSME 1992/18)

Diophantine Equations
𝑎7 = 8𝑎
⏟2 + 5𝑎
⏟1 = 120 = 15
⏟×8
𝑴𝒖𝒍𝒕𝒊𝒑𝒍𝒆 𝒐𝒇 𝟖 𝑯𝒂𝒔 𝒕𝒐 𝒃𝒆 𝑴𝒖𝒍𝒕𝒊𝒑𝒍𝒆 𝒐𝒇 𝟖
𝒎𝒖𝒍𝒕𝒊𝒑𝒍𝒆 𝒐𝒇 𝟖
Try values:
𝑎1 > 0 ⇒ 𝒂𝟏 ≠ 𝟎
𝑎1 = 8 ⇒ 5𝑎1 = 40 ⇒ 8𝑎2 = 80 ⇒ 𝑎2 = 10
𝑎1 = 16 ⇒ 5𝑎1 = 80 ⇒ 8𝑎2 = 40 ⇒ 𝑎2 = 5 ⇒ 𝑎1 > 𝑎2 ⇒ 𝑵𝒐𝒕 𝑽𝒂𝒍𝒊𝒅

𝑎3 = 8 + 10 = 18
𝑎4 = 10 + 18 = 28
𝑎5 = 18 + 28 = 46
𝑎6 = 28 + 46 = 74
𝑎7 = 46 + 74 = 120
𝑎8 = 74 + 120 = 194
System of Equations in Two Variables
Trying to create a system of equations in this question is not useful, since we have two variables, and only one
equation.
𝑎6 = 3𝑥 + 5𝑦
𝑎7 = 5𝑥 + 8𝑦 = 120
𝑎8 = 120 + 3𝑥 + 5𝑦 = 8𝑥 + 13𝑦
Hence, the Diophantine equation methods used above are relevant for solving the question.

Example 4.227
The list 11, 20, 31, 51,82 is an example of an increasing list of five positive integers in which the first and second
integers add to the third, the second and third add to the fourth, and the third and fourth add to the fifth. How
many such lists of five positive integers have 124 as the fifth integer? (Gauss 7 2020/23)

Let the first two numbers in the list be 𝑎 and 𝑏. Then, we must have:
𝑎, 𝑏, 𝑎 + 𝑏, 𝑎 + 2𝑏, 2𝑎 + 3𝑏

2𝑎
⏟ + 3𝑏
⏟ = 124
⏟, 𝑏>𝑎
𝑬𝒗𝒆𝒏 𝑬𝒗𝒆𝒏 𝑬𝒗𝒆𝒏

124 − 2𝑎 123 − 2𝑎 + 1 123 2𝑎 − 1


𝑏= = = −
3 3 3 3

2𝑎 should be 1 more than a multiple of 3:


124 − 2𝑎 124 − 4 120
𝑎=2⇒𝑏= = = = 40
3 3 3

P a g e 222 | 274
Get all the files at: https://bit.ly/azizhandouts
Aziz Manva (azizmanva@gmail.com)

124 − 2𝑎 124 − 10 120


𝑎=5⇒𝑏= = = = 38
3 3 3

𝑏 ∈ {40,38,36,34,32,30,28,26} ⇒ 8 𝑉𝑎𝑙𝑢𝑒𝑠

Example 4.228
Consider the sequence of numbers:
4, 7, 1, 8 , 9, 7, 6
𝑡ℎ
For 𝑛 > 2, the 𝑛 term of the sequence is the units digit of the sum of the two previous terms. Let 𝑆𝑛 denote the
sum of the first 𝑛 terms of this sequence. The smallest value of 𝑛 for which 𝑆𝑛 > 10,000 is: (AMC 12A 2002/21)

The given sequence has the formula:


𝑎𝑛 = Units Digit(𝑎𝑛−1 + 𝑎𝑛−2 )
Calculate the first few terms of the sequence in order to try and get a cycle:
𝟒, 𝟕, 1 , 8 , 9, 7, 6, 3, 9, 2, 1, 3 , 𝟒, 𝟕

𝑪𝒚𝒄𝒍𝒆=𝟏𝟐 𝒏𝒖𝒎𝒃𝒆𝒓𝒔
The sum of the cycle is:
4 + 7 + 1 + 8 + 9 + 7 + 6 + 3 + 9 + 2 + 1 + 3 = 60
Calculate the number of times the cycle must be repeated:
10,000 40
= 166
60 60
Hence, we need 166 complete cycles, giving us
60 × 166 = 9960
And the remaining 40 will be made up by:
4 + 7 + 1 + 8 + 9 + 7 + 6 = 42 ⇒ 7 𝑁𝑢𝑚𝑏𝑒𝑟𝑠
And the total number of numbers needed is:
= 166 × 12 + 7 = 1999

Erdos was trying the question above. He attempted to calculate the cyclicity of the sequence, which in this case,
for the starting two numbers 4 and 7 is a cycle of 12. However, considering the decimal number system, what is
the worst-case scenario for a sequence defined like the one above with positive integers 𝑎 and 𝑏:
𝑎, 𝑏, …

𝑎 and 𝑏 must be a units digit in the decimal system. Hence, by the multiplication principle, the possible number
of values that two consecutive numbers in the above sequence can take is:
10 + 10 = 102 = 100
𝑀𝑎𝑥 𝑙𝑒𝑛𝑔𝑡ℎ 𝑜𝑓 𝑐𝑦𝑐𝑙𝑒 = 100
F. Fibonacci-Type Series

Example 4.229
Let the sequence 𝑆, 𝑇, 𝑈 and 𝑉 be defined as
𝑆 = 12,21,2112, 211221
𝑡ℎ
𝑆𝑛 : the 𝑛 term of 𝑠 – the digits of 𝑆𝑛−1 followed by the digits of 𝑆𝑛−2 for 𝑛 ≥ 3
𝑇𝑛 : the number of 1’s in 𝑆𝑛
𝑈𝑛 : the number of 2’s in 𝑆𝑛
𝑉𝑛 : the sum of the digits of 𝑆𝑛

Tabulate the first few values of each of 𝑇𝑛 , 𝑈𝑛 and 𝑉𝑛 . (Charm of Problem Solving, Sequences/18)

P a g e 223 | 274
Get all the files at: https://bit.ly/azizhandouts
Aziz Manva (azizmanva@gmail.com)

𝑛 𝑆𝑛 𝑇𝑛 𝑈𝑛 𝑉𝑛
1 12 1 1 3
2 21 1 1 3
3 2112 2 2 6
4 211221 3 3 9
5 5 5 15
6 8 8 24

Example 4.230
Define 𝑆 as follows
𝑆 = 1, 2, 221, ,2212212 …
𝑆𝑛 = 𝑡ℎ𝑒 𝑑𝑖𝑔𝑖𝑡𝑠 𝑜𝑓 𝑆𝑛−1 𝑟𝑒𝑝𝑒𝑎𝑡𝑒𝑑 𝑡𝑤𝑖𝑐𝑒 𝑖𝑛 𝑡ℎ𝑎𝑡 𝑜𝑟𝑑𝑒𝑟 𝑓𝑜𝑙𝑙𝑜𝑤𝑒𝑑 𝑏𝑦 𝑡ℎ𝑒 𝑑𝑖𝑔𝑖𝑡𝑠 𝑜𝑓 𝑆𝑛−2 𝑓𝑜𝑟 𝑛 ≥ 3

Write the first 10 terms. Generate three new sequences as in the previous example, and hence find the number
of 2’s and 1’s used in the 15th term of S by extending the terms of 𝑇, 𝑈, 𝑉. Also, find the digital sum in the 15th
term of S.

𝑛 𝑆𝑛 𝑇𝑛 𝑈𝑛 𝑉𝑛
1 1 1 0
2 2 0 1
3 221 1 2
4 (221)(221)(2) 2 5
5 [(221)(221)(2)][(221)(221)(2)][221] 5 12
6 {[(221)(221)(2)][(221)(221)(2)][221]} 12 29
{[(221)(221)(2)][(221)(221)(2)][221]}
{(221)(221)(2)}
7 29 70
8 70 169
9 169 408
10 408 985

𝑇𝑛 satisfies a linear recurrence relationship:


𝑇𝑛 = 2𝑇𝑛−1 + 𝑇𝑛−2
G. Fibonacci-Type Series: Difference
The Fibonacci sequence can also be changed, more fundamentally, by making it the difference of the prior two
terms:
𝑓𝑛 = 𝑓𝑛−1 − 𝑓𝑛−2
This is a ⏟
𝑐𝑦𝑐𝑙𝑖𝑐𝑎𝑙 𝑠𝑒𝑞𝑢𝑒𝑛𝑐𝑒, as we will see.
𝑠𝑒𝑞𝑢𝑒𝑛𝑐𝑒 𝑟𝑒𝑝𝑒𝑎𝑡𝑠

Example 4.231
Show that the sum of any six consecutive terms of the sequence below is zero:
𝑓𝑛 = 𝑓𝑛−1 − 𝑓𝑛−2 , 𝑓1 = 3, 𝑓2 = 5

𝑓1 = 3
𝑓2 = 5
𝑓3 = 5 − 3 = 2
𝑓4 = 2 − 5 = −3

P a g e 224 | 274
Get all the files at: https://bit.ly/azizhandouts
Aziz Manva (azizmanva@gmail.com)

𝑓5 = −3 − 2 = −5
𝑓6 = −5 − (−3) = −5 + 3 = −2
𝑓7 = −2 − (−5) = −2 + 5 = 3
𝑓8 = 3 − (−2) = 3 + 2 = 5

The sequence is only defined by the previous two terms, and these are exactly the same as the two starting
terms.
Hence, the sequence will repeat with a cyclicity of six.

Any six terms of the sequence must be, in some order:


𝟑, 𝟓, 𝟐, −𝟑, −𝟓, −𝟐
Note that terms in the same color add up to zero, and hence the entire set of six terms adds up to zero.

4.232: Cyclicity of Fibonacci-type sequence


For the sequence 𝑓𝑛 = 𝑓𝑛−1 − 𝑓𝑛−2 , 𝑛 ≥ 3
➢ The sum of any six terms is zero
➢ The sequence is cyclical with a cyclicity of six.

Assume that the first two terms of the sequence are 𝑓1 and 𝑓2.
𝑓3 = 𝑓2 − 𝑓1
𝑓4 = 𝑓2 − 𝑓1 − 𝑓2 = −𝑓1
𝑓5 = −𝑓1 − (𝑓2 − 𝑓1 ) = −𝑓2
𝑓6 = −𝑓2 + 𝑓1
𝑓7 = −𝑓2 + 𝑓1 − (−𝑓2 ) = 𝑓1
𝑓8 = 𝑓1 − (−𝑓2 + 𝑓1 ) = 𝑓2

As before, note that the seventh and the eighth term are exactly the same as the first and the second term.
Hence, the sequence repeats with cyclicity six.

Add any six terms of the sequence to get:


𝒇𝟏 + 𝒇𝟐 + (𝑓2 − 𝑓1 ) + (−𝑓1 ) + (−𝑓2 ) + (−𝑓2 + 𝑓1 )

Example 4.233
A sequence of integers 𝑎1 , 𝑎2 , 𝑎3 , . .. is chosen so that 𝑎𝑛 = 𝑎𝑛−1 − 𝑎𝑛−2 for each 𝑛 ≥ 3. What is the sum of the
first 2001 terms of this sequence if the sum of the first 1492 terms is 1985, and the sum of the first 1985 terms
is 1492? (AIME 1985/5)

Let the first term be 𝑥. Let the second term be 𝑦.


Sum of any six consecutive terms of 𝑎𝑛 = 𝑎𝑛−1 − 𝑎𝑛−2 is zero. Therefore, we can ignore multiples of six when
finding the value of the sum of the terms
1492
𝑅( ) = 4 ⇒ 𝑆𝑢𝑚 𝑜𝑓 𝑓𝑖𝑟𝑠𝑡 4 𝑡𝑒𝑟𝑚𝑠 = 𝑥 + 𝑦 + (𝑦 − 𝑥) − 𝑥 = ⏟ 2𝑦 − 𝑥 = 1985
6
𝑬𝒒𝒖𝒂𝒕𝒊𝒐𝒏 𝑰
1985
𝑅( ) = 5 ⇒ 𝑆𝑢𝑚 𝑜𝑓 𝑓𝑖𝑟𝑠𝑡 5 𝑡𝑒𝑟𝑚𝑠 = 𝑦
⏟− 𝑥 = 1492
6
𝑬𝒒𝒖𝒂𝒕𝒊𝒐𝒏 𝑰𝑰
Subtract the second equation from the first:
2𝑦 − 𝑥 − (𝑦 − 𝑥) = 1985 − 1492 ⇒ 𝑦 = 493
𝑆𝑢𝑚 𝑜𝑓 𝑓𝑖𝑟𝑠𝑡 3 𝑡𝑒𝑟𝑚𝑠 = 𝑥 + 𝑦 + 𝑦 − 𝑥 = 2𝑦 = 986

P a g e 225 | 274
Get all the files at: https://bit.ly/azizhandouts
Aziz Manva (azizmanva@gmail.com)

H. General Recursive Sequences

Example 4.234
A grocer stacks oranges in a pyramid-like stack whose rectangular base is 5 oranges by 8 oranges. Each orange
above the first level rests in a pocket formed by four oranges below. The stack is completed by a single row of
oranges. How many oranges are in the stack? (AMC 10A 2004/8)

Each orange rests in a pocket formed by four oranges below. Hence,


each successive level above the first level will reduce the length and
the width each of the stack by one. We calculate the total number of
oranges below.

Width Length
5 8 40
4 7 28
3 6 18
2 5 10
1 4 4

Total 100

Example 4.235
Oranges are stacked in a pyramid format. A single orange is stacked in the dip or gap between four oranges in
the layer below it. If the number of oranges in the topmost layer is one, then
A. find an explicit formula for the number of oranges in the 𝑛𝑡ℎ layer,
B. find a recursive formula for the number of oranges in the 𝑛𝑡ℎ layer,

Part A
𝑇𝑛 = 𝑛2
Part B
1,4,9,16,25
1, 22 , 32 , 42 , 52
2
𝑇𝑛 = (√𝑇𝑛−1 + 1)

Example 4.236
𝑛
Let 𝑥1 = 97, and for 𝑛 > 1, let 𝑥𝑛 = 𝑥 . Calculate the product 𝑥1 𝑥2 𝑥3 𝑥4 𝑥5 𝑥6 𝑥7 𝑥8. (AIME 1985/1)
𝑛−1

Using the Definition


2
Let 𝑥1 = 𝑎, 𝑥2 = 𝑎. Then
3 3𝑎 4 8 5 15𝑎 6 48 7 105𝑎 8 384
𝑥3 = = , 𝑥4 = = , 𝑥5 = = , 𝑥6 = = , 𝑥7 = = , 𝑥8 = =
2 2 3𝑎 3𝑎 8 8 15𝑎 15𝑎 48 48 105𝑎 105𝑎
𝑎 2 3𝑎 8 15𝑎 48
2 3𝑎 8 15𝑎 48 105𝑎 384
𝑥1 𝑥2 𝑥3 𝑥4 𝑥5 𝑥6 𝑥7 𝑥8 = 𝑎 × × × × × × × = 384
𝑎 2 3𝑎 8 15𝑎 48 105𝑎
Simplifying the Formula
𝑛
𝑥𝑛 =
𝑥𝑛−1
Solve the above for 𝑛:

P a g e 226 | 274
Get all the files at: https://bit.ly/azizhandouts
Aziz Manva (azizmanva@gmail.com)

𝑥𝑛 ∙ 𝑥𝑛−1 = 𝑛

Substitute 𝑛 = 2:
2 = 𝑥2 ∙ 𝑥1
Substitute 𝑛 = 4:
4 = 𝑥4 ∙ 𝑥3
Putting it all together, we get:
𝑥1 𝑥2 ∙ 𝑥3 𝑥4 ∙ 𝑥5 𝑥6 ∙ 𝑥7 𝑥8 = ⏟
2 ∙ ⏟
4 ∙ ⏟
6 ∙ ⏟
8 = 384
𝒙𝟏 𝒙𝟐 𝒙𝟑 𝒙𝟒 𝒙𝟓 𝒙𝟔 𝒙𝟕 𝒙𝟖

Example 4.237
In the sequence 2001, 2002, 2003, , each term after the third is found by subtracting the previous term from
the sum of the two terms that precede that term. For example, the fourth term is 2001 + 2002 − 2003 = 2000.
What is the 2004𝑡ℎ term in this sequence? (AMC 10B 2004/19)

We can generalize the calculation for the fourth term as:



2001 + 2002
⏟ − 2003
⏟ = 2000

𝑎𝑛 𝑎𝑛+1 𝑎𝑛+2 𝑎𝑛+3
Which we can write as:
𝑎𝑛 + 𝑎𝑛+1 − 𝑎𝑛+2 = 𝑎𝑛+3
𝑎𝑛 + 𝑎𝑛+1 = 𝑎𝑛+2 + 𝑎𝑛+3

And now note that we can form pairs:


2001,2002
⏟ , 2003,2000
⏟ , 2005,1998

𝑃𝑎𝑖𝑟 1 𝑃𝑎𝑖𝑟 2 𝑃𝑎𝑖𝑟 3

𝑎2𝑛 = 2002 − (2𝑛 − 2)

𝑎2 = 2002
𝑎4 = 2002 − 2
𝑎6 = 2002 − 4
.
.
.
𝑎2004 = 2002 − 2002 = 0

Example 4.238: Finding the General Term


Question on finding the general term15

I. Adding

Example 4.239
If the sequence 𝑎𝑛 is defined by:
𝑎1 = 2, 𝑎𝑛+1 = 𝑎𝑛 + 2𝑛
where 𝑛 ≥ 1, then 𝑎100 equals (AHSME 1984/12)

15 By blackpenredpen

P a g e 227 | 274
Get all the files at: https://bit.ly/azizhandouts
Aziz Manva (azizmanva@gmail.com)

Using Recursion
Substitute 𝑛 = 100 in 𝑎𝑛+1 = 𝑎𝑛 + 2𝑛:
𝑎100 = 𝑎99 + 2(99)
Substitute 𝑎99 = 𝑎98 + 2(98):
= 𝑎98 + 2(98) + 2(99)
Substitute 𝑎98 = 𝑎97 + 2(97):
= 𝑎97 + 2(97) + 2(98) + 2(99)
We can keep repeating this until we get:
= 𝑎1 + 2(1) + 2(2) + ⋯ + 2(98) + 2(99)
Factor 2 from all the terms except the first term:
= 𝑎1 + 2[1 + 2 + ⋯ + 99]
𝑛(𝑛+1)
Use the formula for the sum of the first 𝑛 natural numbers = 2 :
99 × 100
= 𝑎1 + 2 [ ] = 𝑎1 + 99 × 100
2
Substitute 𝑎1 = 2:
= 2 + 9900 = 9902
Using the Pattern
We can generalize the difference of successive terms in the sequence:
𝑎𝑛+1 − 𝑎𝑛 = 𝑎𝑛 + 2𝑛 − [𝑎𝑛−1 + 2(𝑛 − 1)] = 𝑎𝑛 + 2𝑛 − 𝑎𝑛−1 − 2𝑛 + 2 = (𝑎𝑛 − 𝑎𝑛−1 ) + 2
Hence, the difference of successive terms follows an arithmetic sequence:
𝑎2 − 𝑎1 = 4 − 2 = 2 = 2(1)
𝑎3 − 𝑎2 = 8 − 4 = 4 = 2(2)
𝑎4 − 𝑎3 = 14 − 8 = 6 = 3(2)
.
.
.
𝑎99 − 𝑎98 = 2(98)
𝑎100 − 𝑎99 = 2(99)
Add all the equations above:
(𝑎100 − 𝑎99 ) + (𝑎99 − 𝑎98 ) + ⋯ + (𝑎3 − 𝑎2 ) + (𝑎2 − 𝑎1 ) = 2(1) + 2(2) + ⋯ + 2(99)
99 × 100
𝑎100 − 𝑎1 = 2 [ ]
2
𝑎100 = 𝑎1 + 9900 = 2 + 9900 + 9902
Use the Pattern

𝑎1 𝑎2 𝑎3 𝑎4 . . . 𝑎100
Sequence 2 4 8 14 . . .
Successive 2 4 6 . . . 99 × 2
Differences
Formula 2+2 2+2+4 2+2+4+6

Note that successive differences from an arithmetic sequence:

99 × 100
2 + (2 + 4 + 6 + ⋯ + 198) = 2 + 2(1 + 2 + ⋯ + 99) = 2 + (2) ( ) = 2 + 9900 = 9902
2

Quadratic Sequence

𝑎1 𝑎2 𝑎3 𝑎4 . . .

P a g e 228 | 274
Get all the files at: https://bit.ly/azizhandouts
Aziz Manva (azizmanva@gmail.com)

Sequence 2 4 8 14 . . .
Successive 2 4 6 . . .
Differences

Note that successive differences form an arithmetic sequence. And hence the sequence is the sum of terms of an
arithmetic sequence, which results in a quadratic sequence.

𝑦 = 𝑎𝑥 2 + 𝑏𝑥 + 𝑐
𝑎1 = ⏟
𝑎 + 𝑏 + 𝑐 = 2, 𝑎2 = ⏟
4𝑎 + 2𝑏 + 𝑐 = 4 , 𝑎3 = ⏟
9𝑎 + 3𝑏 + 𝑐 = 8
𝑬𝒒𝒖𝒂𝒕𝒊𝒐𝒏 𝑰 𝑬𝒒𝒖𝒂𝒕𝒊𝒐𝒏 𝑰𝑰 𝑬𝒒𝒖𝒂𝒕𝒊𝒐𝒏 𝑰𝑰𝑰

Subtract Equation I from Equation II, and Equation II from Equation III:
3𝑎 + 𝑏 = 2 , ⏟
⏟ 5𝑎 + 𝑏 = 4
𝑬𝒒𝒖𝒂𝒕𝒊𝒐𝒏 𝑰𝑽 𝑬𝒒𝒖𝒂𝒕𝒊𝒐𝒏 𝑽
Subtract Equation IV from Equation V:
2𝑎 = 2 ⇒ 𝑎 = 1
Substitute 𝑎 = 1 in Equation IV
3(1) + 𝑏 = 2 ⇒ 𝑏 = −1
Substitute 𝑎 = 1, 𝑏 = −1 in Equation I
1 + (−1) + 𝑐 = 2 ⇒ 𝑐 = 2
(𝑎, 𝑏, 𝑐) = (1, −1,2) ⇒ 𝑎𝑥 2 + 𝑏𝑥 + 𝑐 = 𝑥 2 − 𝑥 + 2

𝑎100 = 1002 − 100 + 2 = 10,000 − 100 + 2 = 9902

Example 4.240
Let {𝑎𝑘 } be a sequence of integers such that 𝑎1 = 1 and 𝑎𝑚+𝑛 = 𝑎𝑚 + 𝑎𝑛 + 𝑚𝑛, for all positive integers 𝑚 and 𝑛.
Then 𝑎12 is (AMC 10B 2002/23)

Use the pattern


𝑎1 = 1
𝑎2 = 𝑎1+1 = 𝑎1 + 𝑎1 + (1)(1) = 1 + 1 + 1 = 3
𝑎3 = 𝑎1+2 = 𝑎1 + 𝑎2 + (1)(2) = 1 + 3 + 2 = 6

These are the triangular numbers.


𝑎12 = 78

Add Equations
Substitute 𝑚 = 1 in the given definition:
𝑎1+𝑛 = 𝑎1 + 𝑎𝑛 + 𝑛 = 𝑎𝑛 + 𝑛 + 1
Start with the definition:
𝑎1 = 1
𝑎2 = 𝑎1+1 = 𝑎1 + 2
𝑎3 = 𝑎1+2 = 𝑎2 + 3
.
.
.
𝑎12 = 𝑎1+11 = 𝑎11 + 12
Add up the equations:
𝑎1 + 𝑎2 + ⋯ + 𝑎12 = 𝑎1 + 𝑎2 + ⋯ + 𝑎11 + 1 + 2 + ⋯ + 12
𝑎12 = 78

P a g e 229 | 274
Get all the files at: https://bit.ly/azizhandouts
Aziz Manva (azizmanva@gmail.com)

J. Estimation

Example 4.241
2 +5𝑥 +4
𝑥𝑛 𝑛
Define a sequence recursively by 𝑥0 = 5 and 𝑥𝑛+1 = 𝑥𝑛 +6
for all nonnegative integers 𝑛. Let 𝑚 be the least
1
positive integer such that 𝑥𝑚 ≤ 4 + 220 . In which of the following intervals does 𝑚 lie?
A. [9,26]
B. [27,80]
C. [81,242]
D. [243,728]
E. [729, ∞] (AMC 10B 2019/24)

1 1
𝑥𝑚 ≤ 4 + ⇒ 𝑥𝑚 − 4 ≤
220 220
Part A: Property I: 𝒙𝒏 > 𝟒, for all 𝒏 (Proved using induction)
𝑥0 = 5 > 4 is the base case. The inductive case is that if 𝑥𝑛 > 4, then :
(𝑥𝑛 − 4) ⏟
⏟ (𝑥𝑛 + 5)
𝑥𝑛2 + 5𝑥𝑛 + 4 𝑥𝑛2 + 5𝑥𝑛 + 4 − 4(𝑥𝑛 + 6) 𝑥𝑛2 + 𝑥𝑛 − 20 +𝒗𝒆 +𝒗𝒆
𝑥𝑛+1 − 4 = −4= = = → 𝑃𝑜𝑠𝑖𝑡𝑖𝑣𝑒
𝑥𝑛 + 6 𝑥𝑛 + 6 𝑥𝑛 + 6 𝑥⏟
𝑛+6
+𝒗𝒆
⇒ 𝑥𝑛+1 − 4 𝑖𝑠 𝑝𝑜𝑠𝑖𝑡𝑖𝑣𝑒 ⇒ 𝑥𝑛+1 − 4 > 0 ⇒ 𝑥𝑛+1 > 4

Part B: Property II: 𝒙𝒏 is decreasing


𝑥𝑛2 + 5𝑥𝑛 + 4 𝑥𝑛2 + 5𝑥𝑛 + 4 − 𝑥𝑛 (𝑥𝑛 + 6) 𝑥𝑛2 + 5𝑥𝑛 + 4 − 𝑥𝑛2 − 6𝑥𝑛 4 − 𝑥𝑛
− 𝑥𝑛 = = =
⏟ 𝑥𝑛 + 6 𝑥𝑛 + 6 𝑥𝑛 + 6 𝑥𝑛 + 6
𝒙𝒏+𝟏
Given that 𝑥𝑛 > 4:
4⏟− 𝑥𝑛 ÷ 𝑥⏟
𝑛 + 6 → 𝑁𝑒𝑔𝑎𝑡𝑖𝑣𝑒
−𝒗𝒆 +𝒗𝒆
Since the difference between successive terms is negative, the sequence is decreasing.

Part C: Range of Next Term


𝑥𝑛 is a decreasing sequence always greater than 4. Compare the next term of the sequence to the previous term.
(𝑥𝑛 − 4)(𝑥𝑛 + 5) 𝑥𝑛 + 5
𝑥𝑛+1 − 4 = = (𝑥𝑛 − 4) ∙
𝑥𝑛 + 6 𝑥𝑛 + 6
From Property I, we know that 𝑥𝑛 > 4. From Property II and 𝑥0 = 5, we know that 𝑥𝑛 ≤ 5
𝑥𝑛 + 5 4+5 5+5 9 10
𝑓𝑜𝑟 4 ≤ 𝑥𝑛 ≤ 5 ∈ [ , ]=[ , ]
𝑥𝑛 + 6 4+6 5+6 10 11

Part D: Range of 𝒏𝒕𝒉 term


𝑥𝑛 + 5 𝑥𝑛 + 5
(𝑥𝑛 − 4) ∙ = 𝑥𝑛+1 − 4 = (𝑥𝑛 − 4) ∙
𝑥𝑛 + 6 𝑥𝑛 + 6
𝑥𝑛 +5 9 10
Use the range from part C for 𝑥 +6. The lower value is 10, and the upper value is 11.
𝑛
9 10
(𝑥𝑛 − 4) ∙< 𝑥𝑛+1 − 4 ≤ (𝑥𝑛 − 4) ∙
10 11
The above expression is for a jump from one term to the next. If the jump is from 𝑥0 to 𝑥𝑛 , which is 𝑛 terms, then
the bounds are multiplied by themselves 𝑛 times:
9 𝑛 10 𝑛
(𝑥0 − 4) ∙ ( ) < 𝑥𝑛 − 4 ≤ (𝑥0 − 4) ∙ ( )
10 11

P a g e 230 | 274
Get all the files at: https://bit.ly/azizhandouts
Aziz Manva (azizmanva@gmail.com)

Substitute 𝑥0 = 5:
9 𝑛 10 𝑛
( ) < 𝑥𝑛 − 4 ≤ ( )
10 11

Part E: Lower Bound


The above is a compound inequality which can be solved by breaking into separate inequalities:
9 𝑛 1 9 𝑛 1
( ) < 𝑥𝑛 − 4 ≤ 20 ⇒ ( ) ≤ 20
10 2 10 2
9 3 1
( ) ≈ 0.73, 0.94 ≈ 0.66, 0.95 ≈ 0.6, 0.96 = 0.54, 0.97 = 0.48 ⇒ 0.96.5 ≈
10 2
Exponentiate both sides of the last approximation to the power 20:
1 20 1
(0.96.5 )20 ≈ ( ) ⇒ 0.9130 ≈ 20 ⇒ 𝑛 > 130 ⇒ 130 ∈ [81,243] ⇒ 𝑂𝑝𝑡𝑖𝑜𝑛 𝐶 𝑖𝑠 𝑐𝑜𝑟𝑟𝑒𝑐𝑡
2 2

K. Geometry

𝑆1 = 1
1
𝑆2 = 1 +
2
1 1
𝑆3 = 1 + +
2 4
1 1
𝑆𝑛 = 1 + + + ⋯
⏟ 2 4
𝑛 𝑡𝑒𝑟𝑚𝑠
lim 𝑆𝑛 = 2
𝑛→∞

Example 4.242
2
A. Zeno’s frog is on the number line. It starts at the number 4, and hops 3 of a unit to the right. Its 𝑛𝑡ℎ hop
is 𝑘 times of the (𝑛 − 1)𝑠𝑡 hop, where 𝑛 ≥ 2. Zeno has calculated that if the frog hops an infinite number
of times, it will reach the number 7. Find the value of 𝑘.
B. Zeno constructs a square of area 𝑎 on a line. Adjoining the first square (and still on the line), he
constructs a second square that has area √𝑎 times the first square. If Zeno constructs an infinite number
of such squares, and the total area of the squares is 2 units, then find the value of 𝑎.

Part A 2
= 3 − 3𝑘
The recursive formula is: 3
ℎ𝑛 = 𝑘ℎ𝑛−1 , 𝑛 ≥ 2 7
3𝑘 =
The length of the hops is: 3
2 2 2 7
ℎ1 = , ℎ2 = 𝑘, ℎ3 = 𝑘 2 𝑘=
3 3 3 9
The total length of 𝑛 hops: Part B
2 2 2 The area of the 𝑛𝑡ℎ square is:
= + 𝑘 + 𝑘2 + ⋯ 𝐴𝑛 = √𝑎 𝐴𝑛−1 , 𝑛 ≥ 2
⏟3 3 3
𝒏 𝒕𝒆𝒓𝒎𝒔 2
2 𝑎 + (√𝑎)𝑎 + (√𝑎) 𝑎 + ⋯
This is a geometric series with 𝑓𝑖𝑟𝑠𝑡 𝑡𝑒𝑟𝑚 = 𝑎 = 3
,
and 𝑐𝑜𝑚𝑚𝑜𝑛 𝑟𝑎𝑡𝑖𝑜 = 𝑟 = 𝑘, and it has sum This is a geometric series with 𝑓𝑖𝑟𝑠𝑡 𝑡𝑒𝑟𝑚 = 𝑎, and
2 𝑐𝑜𝑚𝑚𝑜𝑛 𝑟𝑎𝑡𝑖𝑜 = √𝑎, and it has sum
𝑎
= 3 =7−4 𝑎
1−𝑟 1−𝑘 =2
1 − √𝑎
P a g e 231 | 274
Get all the files at: https://bit.ly/azizhandouts
Aziz Manva (azizmanva@gmail.com)

𝑎 = 2 − 2√𝑎 Use the quadratic formula:


𝑎 + 2√𝑎 − 2 = 0 𝑥 = √3 − 1
2
Use a change of variable. Let 𝑥 = √𝑎: 𝑎 = 𝑥 2 = (√3 − 1) =
𝑥 2 + 2𝑥 − 2 = 0

L. Triangle Inequality

Example 4.243: Triangle Inequality


A triangle can be formed having side lengths 4, 5 and 8. It is impossible, however, to construct a triangle with
side lengths 4, 5 and 9. Ron has eight sticks, each having an integer length. He observes that he cannot form a
triangle using any three of these sticks as side lengths. The shortest possible length of the longest of the eight
sticks is (Gauss Grade 7 2001/25)

Three Sticks (1,1,2,3)


Start with two sticks with the smallest length Note that, again because of the triangle inequality,
possible: no three of the sticks form a triangle.
(1,1)
Convert the third stick to have lengths equal to the Generalize:
sum of the first two: Note that the sequence of sticks obtained is exactly
(1,1,2) the Fibonacci sequence.
By the triangle inequality, the above three lengths
do not form a triangle. Extend the sequence to eight terms:
(1,1,2,3,5,8,13,21)
Four Sticks
Add a fourth stick, which has length equal to the Shortest possible length
longest sticks: = 21

Example 4.244
Let 𝑇1 be a triangle with sides 2011, 2012, and 2013. For 𝑛 ≥ 1, if 𝑇𝑛 =△ 𝐴𝐵𝐶 and 𝐷, 𝐸, and 𝐹 are the points of
tangency of the incircle of △ 𝐴𝐵𝐶 to the sides 𝐴𝐵, 𝐵𝐶 and 𝐴𝐶, respectively, then 𝑇𝑛+1 is a triangle with side
lengths 𝐴𝐷, 𝐵𝐸, and 𝐶𝐹, if it exists. What is the perimeter of the last triangle in the sequence (𝑇𝑛 )? (AMC 10B
2011/25)

4.7 Arithmetico-Geometric Sequences/Series


A. Arithmetico-Geometric Sequences

4.245: Arithmetico-Geometric Sequence


An arithmetico-geometric sequence has the form:
𝑎, (𝑎 + 𝑑)𝑟, (𝑎 + 2𝑑)𝑟 2 , … , [𝑎 + (𝑛 − 1)𝑑]𝑟 𝑛−1

As the name implies, arithmetico-geometric sequences combine features of both arithmetic and geometric
sequences:
➢ One component of the term increases by 𝑑 each time. This is the common difference.
➢ One component of the term gets multiplied by 𝑟 each time. This is the common ratio.

P a g e 232 | 274
Get all the files at: https://bit.ly/azizhandouts
Aziz Manva (azizmanva@gmail.com)

Example 4.246: General and Specific terms


Consider the infinite arithmetico-geometric sequence with first term 1, common difference 2 and common ratio
3.
A. Write the first few terms
B. Write the general term
C. Evaluate the tenth term. Write your answer in exponent notation.

Part A
𝑇1 = 1
𝑇2 = [1 + 2(1)]31 = 3(3) = 9
𝑇3 = [1 + 2(2)]32 = 5(9) = 45
Part B
𝑇𝑛 = [1 + (𝑛 − 1)(2)]3𝑛−1 = [1 + 2𝑛 − 2]3𝑛−1 = [2𝑛 − 1]3𝑛−1
Part C
𝑇10 = [2(10) − 1]310−1 = [19]39

Example 4.247
Find the common difference and the common difference for the following arithmetico-geometric sequences.
Show that they are correct by calculating 𝑇2 and 𝑇3 .
1 2 3
A. 10 , 102 , 103 , …
3 2 5
B. , , ,…
2 3 18
1 5 4
C. , , ,…
3 30 75

Part A
Notice that the numerator is an AP, and the denominator is a GP
1
𝑁𝑢𝑚𝑒𝑟𝑎𝑡𝑜𝑟𝑠 𝑓𝑜𝑟𝑚 𝑎𝑛 𝐴𝑃 → 1,2,3 ⇒ 𝑑 =
10
1
𝐷𝑒𝑛𝑜𝑚𝑖𝑛𝑎𝑡𝑜𝑟𝑠 𝑓𝑜𝑟𝑚 𝑎𝑛 𝐺𝑃 → 10,102 , 103 ⇒ 𝑟 =
10
1
𝑇1 = 𝑎 =
10
1 1 1 2 1 2
𝑇2 = (𝑎 + 𝑑)𝑟 = ( + ) = ( )( ) = 2
10 10 10 10 10 10
2
1 2 1 3 1 3
𝑇3 = (𝑎 + 2𝑑)𝑟 = ( + ) 2 = ( ) ( 2 ) = 3
10 10 10 10 10 10
Part B
This becomes much easier to decode if you rewrite the sequence as below:
3 4 5 3
, , , … ⇒ 𝑇1 = 𝑎 =
2 6 18 2
1
𝑁𝑢𝑚𝑒𝑟𝑎𝑡𝑜𝑟𝑠 𝑓𝑜𝑟𝑚 𝑎𝑛 𝐴𝑃 → 3,4,5 ⇒ 𝑑 =
2
1
𝐷𝑒𝑛𝑜𝑚𝑖𝑛𝑎𝑡𝑜𝑟𝑠 𝑓𝑜𝑟𝑚 𝑎𝑛 𝐺𝑃 → 2,6,18 ⇒ 𝑟 =
3
3 1 1 4 1 4 2
𝑇2 = ( + ) ( ) = ( ) ( ) = =
2 2 3 2 3 6 3
3 2 1 5 1 5
𝑇3 = ( + ) ( 2 ) = ( ) ( ) =
2 2 3 2 9 18
Part C

P a g e 233 | 274
Get all the files at: https://bit.ly/azizhandouts
Aziz Manva (azizmanva@gmail.com)

1 5 4 1 1
, , ,… ⇒ 𝑑 = ,𝑟 =
3 30 75 2 5
1
𝑇1 =
3
1 1 1 5 1 5
𝑇2 = ( + ) ( ) = ( ) ( ) =
3 2 5 6 5 30
1 1 4 1 4
𝑇3 = ( + 1) ( 2 ) = ( ) ( ) =
3 5 3 25 75

1 3 7 15 1 1
+ + + + ⋯ ⇒ 𝑎 = , 𝑑 = 1, 𝑟 =
2 4 8 16 2 2
1
𝑇1 =
2
1 1 3 1 3
𝑇2 = ( + 1) ( ) = ( ) ( ) =
2 2 2 2 4
1 1 5 1 5
𝑇3 = ( + 2) ( ) = ( ) ( ) =
2 4 2 4 8

Example 4.248
Back Calculations
Word Problems

B. Infinite Arithmetico-geometric Series

Example 4.249
Prove the formula for the sum of a geometric series.16

For −1 < 𝑟 < 1, assign a value to the sum of an infinite geometric series:
𝑆 = 𝑎 + 𝑎𝑟 + 𝑎𝑟 2 + ⋯

𝑬𝒒𝒖𝒂𝒕𝒊𝒐𝒏 𝑰
Multiply both sides by 𝑟:
𝑟𝑆 = 𝑎𝑟 + 𝑎𝑟 2 + ⋯

𝑬𝒒𝒖𝒂𝒕𝒊𝒐𝒏 𝑰𝑰

Subtract Equation II from Equation I, factor 𝑆 on the LHS, and then solve for S:
𝑎
𝑆 − 𝑟𝑆 = 𝑎 ⇒ 𝑆(1 − 𝑟) = 𝑎 ⇒ 𝑆 =
1−𝑟

4.250: Infinite Arithmetico-geometric Series


𝑎 + (𝑎 + 𝑑)𝑟 + (𝑎 + 2𝑑)𝑟 2 + ⋯ + [𝑎 + (𝑛 − 1)𝑑]𝑟 𝑛−1 + ⋯

The process to find the sum of an infinite arithmetic-geometric uses the same process as that to prove the
formula for the sum of an infinite geometric series.

Example 4.251
JEE 2024

16 We will assume convergence of the series for −1 < 𝑟 < 1. That is, we assume that it has a finite sum.

P a g e 234 | 274
Get all the files at: https://bit.ly/azizhandouts
Aziz Manva (azizmanva@gmail.com)

Example 4.252
1 2 3 𝑛
The limiting sum of the infinite series, 10 + 102 + 103 + ⋯ whose 𝑛𝑡ℎ term is 10𝑛 is: (AHSME 1962/40)

1 2 3 4 1
Multiply both sides of 𝑆
⏟= 10 + 102 + 103 + 104 + ⋯ by 𝑟 = 10:
𝑬𝒒𝒖𝒂𝒕𝒊𝒐𝒏 𝑰
𝑆 1 2 3
=0+ + 3+ 4+⋯

10 10 2 10 10
𝑬𝒒𝒖𝒂𝒕𝒊𝒐𝒏 𝑰𝑰

Subtract Equation II from Equation I:


1 1 2 1 3 2 4 3
𝑆 − 𝑆 = ( − 0) + ( 2 − 2 ) + ( 3 − 3 ) + ( 4 − 4 ) + ⋯
10 10 10 10 10 10 10 10

Simplify both sides to get:


9 1 1 1 1
𝑆= + 2+ 3+ 4+⋯
10 10 10 10 10
1 1
1 1 𝑎 10 10 1
The RHS is an infinite geometric series. Substitute 𝑎 = 10 , 𝑟 = 10 in 1−𝑟 = 1 = 9 = 9:
1−
10 10
9 1 10
𝑆 = ⇒𝑆=
10 9 81
Alternate Solution
Split up the terms:
1 1
1 1 1 1 1 1 10 10 1
+ 2+ 3+ 4+⋯⇒𝑎 = ,𝑟 = ⇒ 𝑆1 = = =
10 10 10 10 10 10 1 9 9
1 − 10 10
1
1 1 1 1 1 10 2
+ 2 + 3 + 4 +⋯ ⇒ 𝑎 = 2,𝑟 = ⇒ 𝑆2 =
10 10 10 10 10 1
1 − 10
1
1 1 1 1 10 3
+ 3+ 4+⋯⇒𝑎 = ,𝑟 = ⇒ 𝑆3 =
10 10 100 3 10 1
1 − 10
We need to find the sum:
1 1 1 1 1 1
10 10 2 10 3 10 + 102 + 103 + ⋯ 𝑆1 1 10 10
𝑆1 + 𝑆2 + 𝑆3 + ⋯ = + + +⋯= = = × =
1 1 1 9 9 9 9 81
1 − 10 1 − 10 1 − 10 10 10

Example 4.253
2 6 10 1 1 1
Find 𝛼 2𝛽 given that 𝛼 = 1 + 3 + 32 + 33 + ⋯ , 𝛽 = log 0.25 (3 + 32 + 33 + ⋯ ). (JEE Main 2021, 25 July, Shift-I,
Adapted)

Let
2 6 10
𝛼 =1+ + 2+ 3 +⋯
⏟ 3 3 3
𝑬𝒒𝒖𝒂𝒕𝒊𝒐𝒏 𝑰
Note if you ignore the first term, 𝛼 has an arithmetic series in the numerator, and a geometric series in the

P a g e 235 | 274
Get all the files at: https://bit.ly/azizhandouts
Aziz Manva (azizmanva@gmail.com)
1
denominator. Hence, multiply 𝛼 by the common ratio = 𝑟 = 3:
𝛼 1 2 6
=0+ + 2+ 3+⋯

3 3 3 3
𝑬𝒒𝒖𝒂𝒕𝒊𝒐𝒏 𝑰𝑰
4 4
Subtract Equation II from Equation I, and simplify to get a geometric series with 𝑎 = 3 , 𝑟 = 3, which we
𝑎
substitute in 1−𝑟:
4 4
2𝛼 1 4 4 4 4 4 3 4
=1+ + 2+ 3+⋯= + 2+ 3+⋯= = 3 = =2⇒𝛼=3
3 3 3 3 3 3 3 1 2 2
1−3 3
1 1
Also note that 𝛽 has a geometric series with 𝑎 = , 𝑟 = :
3 3
1 1
1 1 1 3 1 1
𝛽 = log 0.25 ( + 2 + 3 + ⋯ ) = log 1 ( ) = log 1 ( 3 ) = log 1 ( ) =
3 3 3 4 1−1 4 2 4 2 2
3 3
Finally,
1
𝛼 2𝛽 = 32×2 = 31 = 3

Example 4.254
1 1 1
Find the value of 24 ∙ 48 ∙ 816 ∙ … ∞ (JEE Main 2002)

Convert each term to have a prime factor base:


1 1 1
(21 )4 ∙ (22 )8 ∙ (23 )16 ∙ … ∞
Use the exponent law (𝑎𝑚 )𝑛 = 𝑎𝑚𝑛 :
1 2 3
24 ∙ 28 ∙ 216 ∙ … ∞
Use the exponent law 𝑎 𝑥1 ∙ 𝑎 𝑥2 ∙ 𝑎 𝑥3 ∙ … = 𝑎 𝑥1 +𝑥2 +𝑥3 +⋯
1 2 3
24+8+16+⋯
𝑁𝑢𝑚𝑒𝑟𝑎𝑡𝑜𝑟𝑠 𝑓𝑜𝑟𝑚 𝑎𝑛 𝑎𝑟𝑖𝑡ℎ𝑚𝑒𝑡𝑖𝑐 𝑠𝑒𝑟𝑖𝑒𝑠: 1,2,3, …
𝐷𝑒𝑛𝑜𝑚𝑖𝑛𝑎𝑡𝑜𝑟𝑠 𝑓𝑜𝑟𝑚 𝑎 𝑔𝑒𝑜𝑚𝑒𝑡𝑟𝑖𝑐 𝑠𝑒𝑟𝑖𝑒𝑠: 4,8,16, …
𝑇ℎ𝑒 𝑜𝑣𝑒𝑟𝑎𝑙𝑙 𝑠𝑒𝑟𝑖𝑒𝑠 𝑖𝑠 𝑎𝑟𝑖𝑡ℎ𝑚𝑒𝑡𝑖𝑐𝑜 − 𝑔𝑒𝑜𝑚𝑒𝑡𝑟𝑖𝑐.

We want to find the value of the exponent:


1 2 3
𝑆= + + +⋯
⏟ 4 8 16
𝑬𝒒𝒖𝒂𝒕𝒊𝒐𝒏 𝑰
1
Multiply both sides of the above by 2
:
1 1 2 3
𝑆 =0+ + + +⋯

2 8 16 32
𝑬𝒒𝒖𝒂𝒕𝒊𝒐𝒏 𝑰𝑰
Subtract Equation II from Equation I:
1 1 1 1
𝑆= + + +⋯
2 4 8 16
1 1
1 1 𝑎 4 4 1 1
The RHS is a geometric series with 𝑎 = 4
, 𝑟 = and 𝑠𝑢𝑚 =
2 1−𝑟
= 1 = 1 = 4 × 2 = 2:
1−
2 2
1 1 1 2 3
𝑆 = ⇒ 𝑆 = 1 ⇒ 24+8+16+⋯ = 2𝑆 = 21 = 2
2 2

P a g e 236 | 274
Get all the files at: https://bit.ly/azizhandouts
Aziz Manva (azizmanva@gmail.com)

Example 4.255: Probability


The probability distribution for discrete variable 𝑥 = 0,1,2, … is given by 𝑃(𝑋 = 𝑥) = 𝑘(𝑥 + 1)3−𝑥 . The
probability of 𝑃(𝑋 ≥ 2) is given by: (JEE Main 2023)

The sum of 𝑚𝑢𝑡𝑢𝑎𝑙𝑙𝑦 𝑒𝑥𝑐𝑙𝑢𝑠𝑖𝑣𝑒 𝑎𝑛𝑑 𝑐𝑜𝑙𝑙𝑒𝑐𝑡𝑖𝑣𝑒𝑙𝑦 𝑒𝑥ℎ𝑎𝑢𝑠𝑡𝑖𝑣𝑒 events is 1. Therefore:


1 = 𝑃(0) + 𝑃(1) + 𝑃(2) + 𝑃(3) + ⋯

𝑬𝒒𝒖𝒂𝒕𝒊𝒐𝒏 𝑰
Using 𝑃(𝑋 = 𝑥) = 𝑘(𝑥 + 1)3−𝑥 :
2 3 4
1 = 𝑘 [1 + + 2 + 3 + ⋯ ]
⏟ 3 3 3
𝑬𝒒𝒖𝒂𝒕𝒊𝒐𝒏 𝑰𝑰
1
Multiply both sides by the 𝑐𝑜𝑚𝑚𝑜𝑛 𝑟𝑎𝑡𝑖𝑜 = 3
:
1 1 2 3
= 𝑘 [0 + + 2 + 3 + ⋯ ]

3 3 3 3
𝑬𝒒𝒖𝒂𝒕𝒊𝒐𝒏 𝑰𝑰𝑰
Subtract Equation III from Equation II:
2 1 1 1
= 𝑘 [1 + + 2 + 3 + ⋯ ]
3 3 3 3
1 1 1 3
The above is a geometric series with 𝑓𝑖𝑟𝑠𝑡 𝑡𝑒𝑟𝑚 = 𝑎 = 1, 𝑐𝑜𝑚𝑚𝑜𝑛 𝑟𝑎𝑡𝑖𝑜 = 3 ⇒ 𝑆𝑢𝑚 = 1 = 2 = 2:
1−
3 3
2 3 4
= 𝑘[ ] ⇒ 𝑘 =
3 2 9
4
Rearrange Equation I to get the required probability, and use 𝑃(𝑋 = 𝑥) = (𝑥 + 1)3−𝑥 :
9
4 1 4 2 4 5 20 7
𝑃(𝑋 ≥ 2) = 1 − 𝑃(0) − 𝑃(1) = 1 − ( 0 ) − ( 1 ) = 1 − ∙ = 1 − =
9 3 9 3 9 3 27 27

Example 4.256: Statistics Application


You conduct independent trials, with only two possible outcomes: success or failure. The probability of success
is 𝑝. Show that the:
A. range of values that 𝑝 can take is 0 ≤ 𝑝 ≤ 1.
B. probability of failure is 1 − 𝑝.
C. probability that the first success will occur on the 𝑥 𝑡ℎ trial is given by 𝑝(𝑥) = (1 − 𝑝) 𝑥−1 𝑝.
1
D. the mean number of trials required is . Use the formula 𝑀𝑒𝑎𝑛 = 𝐸[𝑋] = ∑𝑥∈ℕ 𝑥𝑝(𝑥) to calculate the
𝑝
mean.

Part A
Part B
Part C
Part C
The formula for the mean is:
𝐸[𝑋] = ∑ 𝑥𝑝(𝑥)
𝑥∈ℕ

From Part C, substitute 𝑝(𝑥) = (1 − 𝑝)𝑥−1 𝑝:


𝐸[𝑋] = ∑ 𝑥(1 − 𝑝) 𝑥−1 𝑝
𝑥∈ℕ

Move the 𝑝 out of the summation sign since it is a constant:

P a g e 237 | 274
Get all the files at: https://bit.ly/azizhandouts
Aziz Manva (azizmanva@gmail.com)

𝐸[𝑋] = 𝑝 ∑ 𝑥(1 − 𝑝) 𝑥−1


𝑥∈ℕ

Expand the summation:


⏟ = 𝑝[1 + 2(1 − 𝑝) + 3(1 − 𝑝)2 + 4(1 − 𝑝)3 + ⋯ ]
𝐸[𝑋]
𝑬𝒒𝒖𝒂𝒕𝒊𝒐𝒏 𝑰

The RHS is an AGP with 𝑐𝑜𝑚𝑚𝑜𝑛 𝑟𝑎𝑡𝑖𝑜 = 𝑟 = 1 − 𝑝. Multiply both sides of the above by the common ratio. Add
an extra zero:
⏟ − 𝑝)𝐸[𝑋] = 𝑝[0 + (1 − 𝑝) + 2(1 − 𝑝)2 + 3(1 − 𝑝)3 + ⋯ ]
(1
𝑬𝒒𝒖𝒂𝒕𝒊𝒐𝒏 𝑰𝑰

Subtract Equation II from Equation I:


𝐸[𝑋] − 𝐸[𝑋] + 𝑝𝐸[𝑋] = 𝑝[1 + (1 − 𝑝) + (1 − 𝑝)2 + (1 − 𝑝)3 + ⋯ ]

Simplify and divide both side by 𝑝:


𝐸[𝑋] = 1 + (1 − 𝑝) + (1 − 𝑝)2 + (1 − 𝑝)3 + ⋯

Use the formula for the sum of an infinite geometric series with 𝑎 = 1, 𝑟 = 1 − 𝑝:
𝑎 1 1
𝐸[𝑋] = = =
1 − 𝑟 1 − (1 − 𝑝) 𝑝

4.257: Sum of Infinite Arithmetico-geometric Series

C. Finite Arithmetico-geometric Series

4.258: Finite Arithmetico-geometric Series


𝑎 + (𝑎 + 𝑑)𝑟 + (𝑎 + 2𝑑)𝑟 2 + ⋯ + [𝑎 + (𝑛 − 1)𝑑]𝑟 𝑛−1

The method for finding the sum of a finite arithmetico-geometric series is similar that if the series is infinite.
➢ Multiply both sides by the common ratio.
➢ Find the difference
➢ There will be one extra term compared to the original series. Be careful not to lose track of this.
➢ Sum the resulting finite geometric series.

Example 4.259
Find the value of:
20
1
(∑ 𝑘 )−2
2𝑘
𝑘=1
(JEE Main 2019, 8 April, Shift-I, Adapted)

1
Let 𝑆 = ∑20
𝑘=1 𝑘 , and expand the summation:
2𝑘
1 2 3 20
𝑆 = + 2 + 3 + ⋯ + 20
⏟ 2 2 2 2
𝑬𝒒𝒖𝒂𝒕𝒊𝒐𝒏 𝑰−𝟏𝟎 𝑻𝒆𝒓𝒎𝒔
This is a finite arithmetico-geometric series. Multiply the above equation by the common ratio:

P a g e 238 | 274
Get all the files at: https://bit.ly/azizhandouts
Aziz Manva (azizmanva@gmail.com)

𝑆 1 2 19 20
= 0 + 2 + 3 + ⋯ + 20 + 21

2 2 2 2 2
𝑬𝒒𝒖𝒂𝒕𝒊𝒐𝒏 𝑰𝑰−𝟏𝟏 𝑻𝒆𝒓𝒎𝒔
Subtract Equation II from Equation I. Note that because Equation II has 11 terms, and Equation I has 10 terms,
their difference has 11 terms:
𝑆 1 1 1 1 20
= + 2 + 3 + ⋯ + 20 − 21
2 2 2 2 2 2
1 1
Now the first ten terms are a finite geometric series with 𝑎 = 2 , 𝑟 = 2:
1 1
𝑆 2 (1 − 220 ) 20 1 20 1 10 11
= − 21 = 1 − 20 − 21 = 1 − 20 − 20 = 1 − 20
2 1 2 2 2 2 2 2
1−
2
Multiply by 2 both sides:
11
𝑆 = 2 − 19
2
Subtract 2 from both sides:
11
𝑆 − 2 = − 19
2

4.260: Common Ratio greater than 1


If the common ratio of an AGP is greater than 1, the method to solve it remains the same.

Example 4.261
If 109 + 2(11)1 (10)8 + 3(11)2 (10)7 +. . . +10(11)9 = 𝑘(10)9 then 𝑘 is equal to: (JEE Main 2014)

Divide both sides by 109 :


11 1 11 2 11 9
𝑘 = 1 + 2 ( ) + 3 ( ) +. . . +10 ( )
⏟ 10 10 10
𝑬𝒒𝒖𝒂𝒕𝒊𝒐𝒏 𝑰−𝟏𝟎 𝑻𝒆𝒓𝒎𝒔
11 11
This is an AGP with common ratio 𝑟 = 10
. Multiply both sides by 𝑟 = 10:
11 11 11 2 11 3 11 10
𝑘 = 0 + 1 ( ) + 2 ( ) + 3 ( ) + ⋯ + 10 ( )

10 10 10 10 10
𝑬𝒒𝒖𝒂𝒕𝒊𝒐𝒏 𝑰𝑰−𝟏𝟏 𝑻𝒆𝒓𝒎𝒔
Subtract Equation II from Equation I:
1 11 1 11 2 11 9 11 10
− 𝑘 = 1 + ( ) + ( ) + ⋯ + ( ) − 10 ( )
10 10 10 10 10
11 𝑎(𝑟 𝑛 −1)
Note that the first ten terms of the RHS form a finite geometric series with 𝑎 = 1, 𝑟 = in :
10 𝑟−1
11 10
1 [(10) − 1]
1 11 10
− 𝑘= − 10 ( )
10 11 10
10 − 1
Simplify:
1 11 10 11 10
− 𝑘 = 10 ( ) − 10 − 10 ( )
10 10 10
Note that the first term and the last term on the RHS cancel:
1
− 𝑘 = −10 ⇒ 𝑘 = 100
10

4.262: Sum of Finite Arithmetico-geometric Series

P a g e 239 | 274
Get all the files at: https://bit.ly/azizhandouts
Aziz Manva (azizmanva@gmail.com)

D. More Uses of the Method


Sometimes a series may not be an AGP, but the method used in AGP can still be useful. However, this may
require additional steps.

Example 4.263
1 3 7 15
Sum of first 𝑛 terms of the series 2 + 4 + 8 + 16 + ⋯ is equal to (JEE Adv. 1988)

Let:
1 3 7 15 2𝑛 − 1
𝑆 = 1 + 2 + 3 + 4 + ⋯+
⏟ 2 2 2 2 2𝑛
𝑬𝒒𝒖𝒂𝒕𝒊𝒐𝒏 𝑰
Note that the numerators do not form an arithmetic progression, but the denominators do form a geometric
progress. We can try our AGP method.
Divide both sides of the above by the common ratio= 𝑟 = 2:
𝑆 1 3 7 2𝑛 − 1
= 0 + 2 + 3 + 4 + ⋯ + 𝑛+1

2 2 2 2 2
𝑬𝒒𝒖𝒂𝒕𝒊𝒐𝒏 𝑰𝑰
Subtract Equation II from Equation I:
𝑆 1 2 4 2𝑛 − 1
= ( + + + ⋯ ) − 𝑛+1
2 ⏟2 4 8 2
𝒏 𝒕𝒆𝒓𝒎𝒔
1
Each of the 𝑛 terms simplifies to :
2
𝑆 1 1 1 2𝑛 − 1
= ( + + + ⋯ ) − 𝑛+1
2 ⏟2 2 2 2
𝒏 𝒕𝒆𝒓𝒎𝒔
Simplify:
𝑆 𝑛 2𝑛 − 1
= − 𝑛+1
2 2 2
Multiply by 2 both sides
2𝑛+1 − 2
𝑆=𝑛−
2𝑛+1
Split the fraction and simplify:
1
𝑆 =𝑛−1+ = 𝑛 − 1 + 2−𝑛
2𝑛
E. Quadratic-Geometric Sequence

4.264: Quadratic-Geometric Sequence


A quadratic-geometric sequence is a combination of a quadratic sequence, and a geometric sequence.

➢ The first difference of a quadratic sequence is an arithmetic sequence.

Example 4.265
7 9 13 19
If 𝑆 = 5 + 52 + 53 + 54 + ⋯ then 160𝑆 is equal to (JEE Main 2021, 31 Aug, Shift-II)

Quadratic-Geometric Sequence 7 9 13 19
𝑆 = + 2+ 3 + 4 +⋯
We know that: ⏟ 5 5 5 5
𝑬𝒒𝒖𝒂𝒕𝒊𝒐𝒏 𝑰

P a g e 240 | 274
Get all the files at: https://bit.ly/azizhandouts
Aziz Manva (azizmanva@gmail.com)

This is a quadratic-geometric sequence with 4 2 2


1 𝐾 = 2+ 3+⋯
common ratio = 𝑟 = . Multiply both sides by the 5 5 5
5 The RHS is an infinite geometric series with 𝑎 =
common ratio: 2 1
𝑆 7 9 13 25
, 𝑟 = 5:
=0+ 2+ 3+ 4 +⋯ 2

5 5 5 5 4 25 2 5 1
𝑬𝒒𝒖𝒂𝒕𝒊𝒐𝒏 𝑰𝑰 𝐾= = × =
Subtract Equation II from Equation I: 5 1 25 4 10
1−
4 7 2 4 6 5
𝑆 = + 2+ 3+ 4+⋯ Hence:
5 5 5 5 5 1 5 1
𝐾= × =
10 4 8
Arithmetico-Geometric Sequence
7
Subtract 5 from both sides, and let 𝐾 equal the new Value of S
quantity: Finally,
4 7 2 4 6 8 4 7 1
𝐾 = 𝑆− = 2+ 3+ 4+ 5+⋯ 𝐾= 𝑆− =
⏟ 5 5 5 5 5 5 5 5 8
𝑬𝒒𝒖𝒂𝒕𝒊𝒐𝒏 𝑰𝑰𝑰 7
Add 5 to both sides:
Note that 𝐾 is an arithmetico geometric series. 4 1 7 61
Hence, multiply both sides by the common ratio = 𝑆= + =
1 5 8 5 40
𝑟 = 5: 5
Multiply both sides by 4:
𝐾 2 4 6 61 5 61
=0+ 3+ 4+ 5+⋯ 𝑆= × =

5 5 5 5 40 4 32
𝑬𝒒𝒖𝒂𝒕𝒊𝒐𝒏 𝑰𝑽
Multiply both sides by 160:
Subtract Equation IV from Equation III: 61
160𝑆 = × 160 = 305
32

4.8 Telescoping
A. Sum of Odd Numbers

4.266: Sum of First 𝒏 odd Numbers: Visual Proof


The sum of the first 𝑛 odd numbers:
1 + 3 + 5 + ⋯ = 𝑛2

𝑛 𝑡𝑒𝑟𝑚𝑠

Consider the “proof without words” below which shows that every extra term added increases the size of
square figure by exactly the number needed to make it one larger.

1 1 3 1 3 5 1 3 5 7 1 3 5 7 9 1 3 5 7 9 11

1 + 3 = 4 = 22
1 + 3 + 5 = 9 = 32
1 + 3 + 5 + 7 = 16 = 42

P a g e 241 | 274
Get all the files at: https://bit.ly/azizhandouts
Aziz Manva (azizmanva@gmail.com)

1 + 3 + 5 + 7 + 9 = 25 = 52
1 + 3 + 5 + 7 + 9 + 11 = 36 = 62

4.267: Sum of First 𝒏 odd Numbers: Algebraic Proof


The sum of the first 𝑛 odd numbers:
1 + 3 + 5 + ⋯ = 𝑛2

𝑛 𝑡𝑒𝑟𝑚𝑠

We prove this using mathematical induction.


Base Case: 𝒏 = 𝟏:
𝐿𝐻𝑆 = 1 3 + ⋯ + (2𝑛 − 1) = 1
𝑅𝐻𝑆 = 𝑛2 = 1
𝐿𝐻𝑆 = 𝑅𝐻𝑆 ⇒ 𝐵𝑎𝑠𝑒 𝐶𝑎𝑠𝑒 𝑖𝑠 𝑣𝑎𝑙𝑖𝑑
Inductive Case:
Let the statement be true for 𝑘:
1 + 3 + ⋯ + (2𝑘 − 1) = 𝑘 2
Add (2(𝑘 + 1) − 1) = 2𝑘 + 1 to both sides:
LHS = 1 + 3 + ⋯ + (2𝑘 − 1) + (2𝑘 + 1)
𝑅𝐻𝑆 = 𝑘 2 + 2𝑘 + 1 = (𝑘 + 1)2
Hence, it is true for 𝑘 + 1. This completes the proof.

4.268: A useful identity


2𝑛 + 1 = (𝑛 + 1)2 − 𝑛2

2𝑛 + 1 = 𝑛2 + 2𝑛 + 1 − 𝑛2 = (𝑛 + 1)2 − 𝑛2

There is an interesting interpretating of this identity:


1 + 3 + 5 + ⋯ + (2𝑛 + 1) = (𝑛 + 1)2
1 + 3 + 5 + ⋯ + (2𝑛 − 1) = 𝑛2
Subtract Equation II from Equation I:
2𝑛 + 1 = (𝑛 + 1)2 − 𝑛2

4.269: Another useful identity


2𝑛 + 1 1 1
= 2−
𝑛2 (𝑛+ 1)2 𝑛 (𝑛 + 1)2

Going from the RHS to the LHS is simple addition.

Going from the LHS to the RHS requires a little more creativity. We use the identity from the previous point.
The LHS is:
2𝑛 + 1
𝑛 (𝑛 + 1)2
2

Split the fraction


(𝑛 + 1)2 − 𝑛2 (𝑛 + 1)2 𝑛2
= 2 = −
𝑛 (𝑛 + 1)2 𝑛2 (𝑛 + 1)2 𝑛2 (𝑛 + 1)2
Simplify:
1 1
= 2−
𝑛 (𝑛 + 1)2

P a g e 242 | 274
Get all the files at: https://bit.ly/azizhandouts
Aziz Manva (azizmanva@gmail.com)

Example 4.270
The sum of 10 terms of the series:
3 5 7
+ 2 + 2 +⋯
12 ×2 2 2 ×3 2 3 × 42
Is (JEE Main 2021, 21 Aug, Shift-I)

Write out the first ten terms of the series. Note that the first numerator is the second odd number, and hence
the 10th numerator is the 11th odd number:
3 5 7 21
+ + + ⋯+
12 × 22 22 × 32 3 2 × 42 10 × 11

Using the identity 2𝑛 + 1 = (𝑛 + 1)2 − 𝑛2:


22 − 12 32 − 22 42 − 32 112 − 102
+ + + ⋯+
12 × 22 22 × 32 32 × 42 10 × 11

Separate each fraction into two:


1 𝟏 𝟏 𝟏 𝟏 1 1 1
= ( 2 − 𝟐) + ( 𝟐 − 𝟐) + ( 𝟐 − 2) + ⋯ + ( 2 − 2)
1 𝟐 𝟐 𝟑 𝟑 4 10 11

Telescope. In each parentheses, the second term cancels with the first term of the next parentheses. We are only
left with the first term of the first parentheses, and the last term of the last parenthesis:
1 1 1 1 120
= − 2= − =
1 11 1 121 121

B. Telescoping: Splitting Fractions

𝟏
4.271: Breaking
𝒏(𝒏+𝟏)
1 1 𝑛+1 𝑛 1
− = − =
𝑛 𝑛 + 1 𝑛(𝑛
⏟ + 1) 𝑛(𝑛 + 1) 𝑛(𝑛 + 1)
𝑇𝑎𝑘𝑒 𝑡ℎ𝑒 𝐿𝐶𝑀

1 1 3 2 1 1
− = − = =
2 3 6 6 6 2×3
1 1 5 4 1 1
− = − = =
4 5 20 20 20 4 × 5

Example 4.272: Breaking


1 1 1 1
D. Evaluate + + +⋯+
30 42 56 𝑛(𝑛−1)
1 1 1 1
E. Evaluate 1677 (1560 + 1640 + 1722 + 1806)

Part A
Rewrite the denominators:
1 1 1
+ + ⋯+
5×6 6×7 𝑛(𝑛 − 1)
Apply the formula to break each term into two:
1 𝟏 𝟏 𝟏 𝟏 1
( − )+ ( − ) + ⋯+( − )
5 𝟔 𝟔 𝟕 𝒏−𝟏 𝑛
Everything cancels, except the first and the last term:

P a g e 243 | 274
Get all the files at: https://bit.ly/azizhandouts
Aziz Manva (azizmanva@gmail.com)

1 1 𝑛−5
− =
5 𝑛 5𝑛
Part B
1 1 1 1
1677 ( + + + )
39 × 40 40 × 41 41 × 42 42 × 43
Split the fractions:
1 1 1 1 1 1 1 1
= 1677 [( − ) + ( − ) + ( − ) + ( − )]
39 40 40 41 41 42 42 43
Telescope:
1 1 43 39 4
= 1677 [ − ] = 1677 [ − ] = 1677 ( )=4
39 43 39 43 1677

𝟏
4.273: Breaking
𝒏(𝒏+𝒌)
1 1 1 1
( − )=
𝑘 𝑛 𝑛+𝑘 𝑛(𝑛 + 𝑘)

1 1 𝑛+𝑘 𝑛 𝑘
− = − =
𝑛 𝑛 + 𝑘 𝑛(𝑛 + 𝑘) 𝑛(𝑛 + 𝑘) 𝑛(𝑛 + 𝑘)

1 1 5 2 3 𝑥 1 1 1 1
𝑥= − = − = ⇒ = ( )( − ) =
2 5 10 10 10 3 3 2 5 2 × (2 + 3)

Example 4.274:
1 1 1 1
A. Evaluate 8 + 24 + 48 + 80
1 1 1 1
B. Evaluate + + +
10 40 88 154
1 1 1
C. Evaluate + + ⋯ +
3 15 𝑛(𝑛+2)

Part A
Rewrite the denominators:
1 1 1 1
+ + +
2 × 4 4 × 6 6 × 8 8 × 10
Apply the formula to break each term into two:
1 1 1 1 1 1 1 1 1 1 1 1
( )( − ) + ( )( − ) + ( )( − ) + ( )( − )
2 2 4 2 4 6 2 6 8 2 8 10
1
Factor from all the terms:
2
1 1 𝟏 𝟏 𝟏 𝟏 𝟏 𝟏 1
( ) [( − ) + ( − ) + ( − ) + ( − )]
2 2 𝟒 𝟒 𝟔 𝟔 𝟖 𝟖 10
Within the brackets, cancel all except the first and the last terms:
1 1 1 1 5 1 1 4 2 1
( )[ − ] = [ − ] = [ ] = =
2 2 10 2 10 10 2 10 10 5
Part B
Rewrite the denominators:
1 1 1 1
+ + +
2 × 5 5 × 8 8 × 11 11 × 14
Apply the formula to break each term into two:
1 1 1 1 1 1 1 1 1 1 1 1
( )( − ) + ( )( − ) + ( )( − ) + ( )( − )
3 2 5 3 5 8 3 8 11 3 11 14

P a g e 244 | 274
Get all the files at: https://bit.ly/azizhandouts
Aziz Manva (azizmanva@gmail.com)
1
Factor 3 from all the terms:
1 1 𝟏 𝟏 𝟏 𝟏 𝟏 𝟏 1
( ) [( − ) + ( − ) + ( − ) + ( − )]
3 2 𝟓 𝟓 𝟖 𝟖 𝟏𝟏 𝟏𝟏 14
Telescope. Within the brackets, cancel all except the first and the last terms:
1 1 1 1 7 1 1 6 1
( )[ − ] = [ − ] = [ ] =
3 2 14 3 14 14 3 14 7
Part C
Rewrite the denominators:
1 1 1
+ +⋯+
1×3 3×5 𝑛(𝑛 + 2)
Apply the formula to break each term into two:
1 1 1 1 1 1 1 1
( ) (1 − ) + ( ) ( − ) + ⋯ ( ) ( − )
2 3 2 3 5 2 𝑛 𝑛+2
1
Factor 2 from all the terms:
1 𝟏 𝟏 𝟏 𝟏 1
( ) [(1 − ) + ( − ) + ⋯ ( − )]
2 𝟑 𝟑 𝟓 𝒏 𝑛+2
Within the brackets, cancel all except the first and the last terms:
1 1 1 𝑛+2−1 𝑛+1
( ) [1 − ]= [ ]=
2 𝑛+2 2 𝑛+2 2(𝑛 + 2)

Challenge 4.275
Evaluate:
1 1 1 1
+ 2 + 2 + ⋯+ 2 = (𝑪𝑨𝑻 𝟐𝟎𝟎𝟎)
22 −1 4 −1 6 −1 20 − 1

Strategy: Factor
We need to do a little additional work here before we see the series in the form that will let us use the formula.
First, factor the denominators using 𝑎2 − 𝑏 2 = (𝑎 + 𝑏)(𝑎 − 𝑏):
1 1 1 1
+ + +⋯+
(2 − 1)(2 + 1) (4 − 1)(4 + 1) (6 − 1)(6 + 1) (20 − 1)(20 + 1)
Then simplify the brackets:
1 1 1 1
+ + +⋯+
(1)(3) (3)(5) (5)(7) (19)(21)
Telescope
Now the series is in the form that we need:
1 1 𝟏 𝟏 𝟏 𝟏 𝟏 𝟏 1
[( − ) + ( − ) + ( − ) + ⋯ + ( − )]
2 1 𝟑 𝟑 𝟓 𝟓 𝟕 𝟏𝟗 21
Telescope and simplify:
1 1 1 20 10
[1 − ] = [ ] =
2 21 2 21 21

Example 4.276
Show that the sum of the reciprocals of the first 𝑛 triangular numbers is less than 2.

𝑛(𝑛 + 1) 2
𝑛𝑡ℎ Triangular No. = ⇒ Reciprocal =
2 𝑛(𝑛 + 1)
Sum of the reciprocals of the first 𝑛 triangular numbers:

P a g e 245 | 274
Get all the files at: https://bit.ly/azizhandouts
Aziz Manva (azizmanva@gmail.com)

2 2 2 2
= + + + ⋯+
1×2 2×3 3×4 𝑛(𝑛 + 1)
Factor 2 out of every term in the series:
1 1 1 1
2[ + + + ⋯+ ]
1×2 2×3 3×4 𝑛(𝑛 + 1)
Use fraction decomposition to re-write each term and then telescope:
1 𝟏 𝟏 𝟏 𝟏 𝟏 𝟏 1 1 𝑛
2 [( − ) + ( − ) + ( − ) + ⋯ + ( − )] = 2 (1 − ) = 2( )<2
1 𝟐 𝟐 𝟑 𝟑 𝟒 𝒏 𝑛+1 𝑛+1 𝑛+1

Example 4.277
Find the sum:
1 1 1 1 1 1
√1 + 2
+ 2 + √1 + 2 + 2 + ⋯ + √1 + 2
+ (𝐂𝐀𝐓 𝟐𝟎𝟎𝟖)
1 2 2 3 2007 20082

Identifying the Pattern


1 1 9 3 1
√1 + 2
+ 2=√ = =1
1 2 4 2 2

1 1 49 7 1
√1 + 2
+ 2=√ = =1
2 3 36 6 6

1 1 169 13 1
√1 + 2
+ 2=√ = =1
3 4 144 12 12
We have perfect squares every time inside the radical. But, we don’t know what the sum will be. Switch over to
Algebra.
Find the nth term
The nth term of the series is
1 1 𝑛2 (𝑛 + 1)2 + (𝑛 + 1)2 + 𝑛2 √𝑛4 + 2𝑛3 + 3𝑛2 + 2𝑛 + 1
√1 + + = √ =
𝑛2 (𝑛 + 1)2 𝑛2 (𝑛 + 1)2 𝑛2 + 𝑛
Since the numerator in the numbers above is always more than the denominator, try:
(𝑛2 + 𝑛 + 1)2 = 𝑛4 + 2𝑛3 + 3𝑛2 + 2𝑛 + 1
Simplify the nth term
𝑛4 + 2𝑛3 + 3𝑛2 + 2𝑛 + 1 (𝑛2 + 𝑛 + 1)2 𝑛2 + 𝑛 + 1 1
√ = √ = =1+
𝑛2 (𝑛 + 1)2 𝑛2 (𝑛 + 1)2 𝑛2 + 𝑛 𝑛(𝑛 + 1)

Find the sum of the series


Substitute the value of 𝑛. We want to find
1 1 1
1+ +1+ + ⋯+ 1 +
⏟ (2) ⏟ 2(3) ⏟ 2007(2008)
𝒏=𝟏 𝒏=𝟐 𝒏=𝟐𝟎𝟎𝟕
Separate out the 1’s:
1 1 1 1 1
1 + 1 + ⋯+1 +
⏟ + + ⋯+ = 2007 + 1 − = 2008 −
1(2) 2(3)
⏟ 2007(2008) ⏟ 2008 2008
2007 𝑡𝑖𝑚𝑒𝑠=1
𝒚 𝒚

Factor the Numerator

P a g e 246 | 274
Get all the files at: https://bit.ly/azizhandouts
Aziz Manva (azizmanva@gmail.com)

If we had wanted to factor the numerator as a perfect square without a guess as to what it was, we could still
have done it using the method below:
➢ The square root of a fourth-degree equation must be a second-degree equation.
➢ The first and last coefficient must be 1, since the first and last coefficient of 𝑛4 + 2𝑛3 + 3𝑛2 + 2𝑛 + 1 is
1.
Hence, we are looking for an expression of the form:
(𝑛2 + 𝑎𝑛 + 1)2

Expand the above, and equate it to the numerator:


𝑛4 + 2𝑎𝑛3 + (2 + 𝑎2 )𝑛2 + 2𝑎𝑛 + 1 = 𝑛4 + 2𝑛3 + 3𝑛2 + 2𝑛 + 1

By equating coefficients, we get three equations:


𝑛3 : 2𝑎 = 2 ⇒ 𝑎 = 1
𝑛 : 2 + 𝑎2 = 3 ⇒ 𝑎 = ±1
2

Hence, 𝑎 = 1 satisfies all the above equations. This tells us that:


𝑛4 + 2𝑛3 + 3𝑛2 + 2𝑛 + 1 = (𝑛2 + 𝑛 + 1)2

Example 4.278
If
40
1 1 𝑏
∑ √1 + 2
+ 2
=𝑎+
𝑘 (𝑘 + 1) 𝑐
𝑘=1
Where 𝑎, 𝑏, 𝑐 ∈ ℕ, 𝑏 < 𝑐, gcd(𝑏, 𝑐) = 1 then find the value of 𝑎 + 𝑏. (IOQM 2021/18)

This is the same question as one we did above, except that it is written in scarier notation. If we substitute 𝑘 =
1,2,3, … ,40, we get:
1 1 1 1 1 1
√1 + 2
+ 2 + √1 + 2 + 2 + ⋯ + √1 + 2 + 2
1 2 2 3 40 41
Apply the same method as above to get:
1 1 1 1 40
1 + 1 + ⋯+ 1 +
⏟ + + ⋯+ = 40 + 1 − = 40 ⇒ 𝑎 + 𝑏 = 40 + 40 = 80
1(2)
⏟ 2(3) 40(41) ⏟ 41 41
40 𝑡𝑖𝑚𝑒𝑠=1
𝒚 𝒚

C. Rationalizing the denominator

4.279: Rationalize the Denominator


If you have a series with radicals in the denominator, one of the options to telescope it is to rationalize the
denominator.

This will not work with all series. Only the ones which can be telescoped.

Example 4.280
1 1 1
+ + ⋯+ =
√1 + √2 √2 + √3 √8 + √9

Multiply each term by the 𝑐𝑜𝑛𝑗𝑢𝑔𝑎𝑡𝑒 𝑠𝑢𝑟𝑑 of the denominator:

P a g e 247 | 274
Get all the files at: https://bit.ly/azizhandouts
Aziz Manva (azizmanva@gmail.com)

1 √2 − √1 1 √3 − √2 1 √9 − √8
∙ + ∙ + ⋯+ ∙
√1 + √2 √2 − √1 √2 + √3 √3 − √2 √8 + √9 √9 − √8

Use the formula (𝑎 + 𝑏)(𝑎 − 𝑏) = 𝑎2 − 𝑏 2 in the denominator, and then telescope:


√𝟐 − √1 √3 − √𝟐 √4 − √3 √9 − √𝟖 √9 − √1 3 − 1 2
+ + + ⋯+ = = = =2
1 1 1 1 1 1 1

Example 4.281
Find the sum to 𝑛 terms of
1 1 1
+ + +⋯
√7 + √10 √10 + √13 √13 + √16

Recognize that the expression inside the first term and the second term of the denominator is each an
arithmetic progression:
7,10,13, … ⇒ 𝑎 = 4, 𝑑 = 3 ⇒ 𝑛𝑡ℎ 𝑡𝑒𝑟𝑚 = 4 + 3𝑛
10,13,16 … ⇒ 𝑎 = 7, 𝑑 = 3 ⇒ 𝑛𝑡ℎ 𝑡𝑒𝑟𝑚 = 7 + 3𝑛
Rewrite the series to include the 𝑛𝑡ℎ term:
1 1 1 1
+ + + ⋯+
√7 + √10 √10 + √13 √13 + √16 √4 + 3𝑛 + √7 + 3𝑛

Rationalize the denominators:


√𝟏𝟎 − √7 √𝟏𝟑 − √𝟏𝟎 √16 − √𝟏𝟑 √7 + 3𝑛 − √𝟒 + 𝟑𝒏
= + + + ⋯+
3 3 3 3
Telescope
√7 + 3𝑛 − √7
=
3
D. Completing the Square

Example 4.282
𝑁
2𝑘 + 1
If ∑ = 0.9999 then determine the value of N. (IOQM 2021/3)
(𝑘 2 + 𝑘)2
𝑘=1

Consider the expression inside the summation sign.


2𝑘 + 1
(𝑘 2 + 𝑘)2
The denominator is already a perfect square. Complete the square in the numerator. Add and subtract 𝑘 2:
𝑘 2 + 2𝑘 + 1 − 𝑘 2
(𝑘 2 + 𝑘)2
Factor the numerator and the denominator:
(𝑘 + 1)2 − 𝑘 2
= 2
𝑘 (𝑘 + 1)2
Split the fraction into two:
1 1
= 2−
𝑘 (𝑘 + 1)2

Hence, the LHS becomes:

P a g e 248 | 274
Get all the files at: https://bit.ly/azizhandouts
Aziz Manva (azizmanva@gmail.com)

𝑁 𝑁
2𝑘 + 1 1 1
𝐿𝐻𝑆 = ∑ 2 2
= ∑( 2− )
(𝑘 + 𝑘) 𝑘 (𝑘 + 1)2
𝑘=1 𝑘=1

Write as the summation:


1 𝟏 𝟏 𝟏 𝟏 1
( − 𝟐) + ( 𝟐 − 𝟐) + ⋯ + ( − )
1 𝟐 𝟐 𝟑 𝑵 (𝑁 + 1)2

Note that the series telescopes. We are left with the first and the last term only. Equate this to the RHS:
1
1− = 0.9999
(𝑁 + 1)2

Rearrange to get:
1 1
2
= 1 − 0.9999 = 0.0001 =
(𝑁 + 1) 10000

Take the reciprocal of the first and the last quantity:


(𝑁 + 1)2 = 10,000

Take the square root on both sides, reject the negative root, and solve for 𝑁:
𝑁 + 1 = 100 ⇒ 𝑁 = 99

Telescoping in Summation Notation17


Split the summation:
𝑁 𝑁
1 1
∑ 2−∑
𝑘 (𝑘 + 1)2
𝑘=1 𝑘=1
Split:
1 𝑁 𝑁−1 𝑁
1 1 1 1
∑ 2+∑ 2−∑ − ∑
𝑘 𝑘 (𝑘 + 1)2 (𝑘 + 1)2
𝑘=1 𝑘=2 𝑘=1 𝑘=𝑁
Simplify the first and last term by substitution:
𝑁 𝑁−1
1 1 1
1+∑ 2− ∑( ) −
𝑘 (𝑘 + 1)2 (𝑁 + 1)2
𝑘=2 𝑘=1
Reindex the third term:
𝑁 𝑁
1 1 1
1+∑ 2−∑ 2−
𝑘 𝑘 (𝑁 + 1)2
𝑘=2 𝑘=2
Simplify:
1
1−
(𝑁 + 1)2

E. Introducing a New Quantity


Completing the square requires adding and subtracting the same quantity that lets us then write part of an
expression as a perfect square. However, there can be other ways of rewriting an expression, some of which can
be very creative, or difficult to think of when first seeing a problem.

17 This method is relevant more for learning summation notation then for its use in solving the question.

P a g e 249 | 274
Get all the files at: https://bit.ly/azizhandouts
Aziz Manva (azizmanva@gmail.com)

Example 4.283: Rewriting the expression


Find the sum to 𝑛 terms of:
1(2) + 2(3) + ⋯ + 𝑛(𝑛 + 1)

Let 𝑇𝑛 be the 𝑛𝑡ℎ term. Then we can write:


𝑇𝑛 = 𝑛(𝑛 + 1)
Multiply both sides by 3:
3𝑇𝑛 = 3𝑛(𝑛 + 1)
Now make a substitution whose reason is not immediately obvious.
Substitute 3 = (𝑛 + 2) − (𝑛 − 1) in the above expression:
3𝑇𝑛 = 𝑛(𝑛 + 1) [(𝑛
⏟ + 2) − (𝑛 − 1)]
=𝟑
Expand:
3𝑇𝑛 = 𝑛(𝑛 + 1)(𝑛 + 2) − (𝒏 − 𝟏)𝒏(𝒏 + 𝟏)
3𝑇𝑛−1 = (𝒏 − 𝟏)(𝒏)(𝒏 + 𝟏) − (𝒏 − 𝟐)(𝒏 − 𝟏)(𝒏)
3𝑇𝑛−2 = (𝒏 − 𝟐)(𝒏 − 𝟏)(𝒏) − (𝑛 − 3)(𝑛 − 2)(𝑛 − 1)
.
.
.
3𝑇1 = (𝟏)(𝟐)(𝟑) − (𝟎)(𝟏)(𝟐)

Add up all the terms above, and note that they telescope:
3𝑇𝑛 + 3𝑇𝑛−1 + ⋯ + 3𝑇1 = 𝑛(𝑛 + 1)(𝑛 + 2)
3(𝑇𝑛 + 𝑇𝑛−1 + ⋯ + 𝑇1 ) = 𝑛(𝑛 + 1)(𝑛 + 2)
𝑛(𝑛 + 1)(𝑛 + 2)
𝑇𝑛 + 𝑇𝑛−1 + ⋯ + 𝑇1 =
3

Method II: With Summation Notation


Write out the given series as a summation:
𝑥=𝑛

𝑋 = ∑ 𝑥(𝑥 + 1)
𝑥=1
Multiply both sides by 3:
𝑥=𝑛

3𝑋 = ∑ 3𝑥(𝑥 + 1)
𝑥=1
Substitute 3 = (𝑥 + 2) − (𝑥 − 1):
𝑥=𝑛

3𝑋 = ∑ 𝑥(𝑥 + 1)[(𝑥 + 2) − (𝑥 − 1)]


𝑥=1
Use the distributive property:
𝑥=𝑛

3𝑋 = ∑ 𝑥(𝑥 + 1)(𝑥 + 2) − (𝑥 − 1)(𝑥)(𝑥 + 1)


𝑥=1
Split the summation:
𝑥=𝑛 𝑥=𝑛

3𝑋 = ∑ 𝑥(𝑥 + 1)(𝑥 + 2) − ∑(𝑥 − 1)(𝑥)(𝑥 + 1)


𝑥=1 𝑥=1
Reindex the second summation by substituting 𝑢 = 𝑥 − 1 ⇒ 𝑥 = 𝑢 + 1:
𝑢+1=𝑛 𝑢=𝑛−1

∑ (𝑢 + 1 − 1)(𝑢 + 1)(𝑢 + 1 + 1) = ∑ (𝑢)(𝑢 + 1)(𝑢 + 1 + 1)


𝑢+1=1 𝑢=0

P a g e 250 | 274
Get all the files at: https://bit.ly/azizhandouts
Aziz Manva (azizmanva@gmail.com)

Since the index of summation is a dummy variable, change it back to 𝑥:


𝑥=𝑛 𝑥=𝑛−1

3𝑋 = ∑ 𝑥(𝑥 + 1)(𝑥 + 2) − ∑ (𝑥)(𝑥 + 1)(𝑥 + 2)


𝑥=1 𝑥=0

Note that substituting 𝑥 = 0 in the second summation results in a zero term, and hence we can ignore it:
𝑥=𝑛 𝑥=𝑛−1

3𝑋 = ∑ 𝑥(𝑥 + 1)(𝑥 + 2) − ∑ (𝑥)(𝑥 + 1)(𝑥 + 2)


𝑥=1 𝑥=1

All the terms cancel, since both the summation terms have the same formula, and the same index of summation,
except that the first summation term has an extra 𝑛𝑡ℎ term, and we are left with:
3𝑋 = 𝑛(𝑛 + 1)(𝑛 + 2) + 0
𝑛(𝑛 + 1)(𝑛 + 2)
𝑋=
3

Writing Assignment 4.284


Show that
𝑛(𝑛 + 1)(𝑛 + 2)(𝑛 + 3)(𝑛 + 4)
1(2)(3)(4) + 2(3)(4)(5) + ⋯ + 𝑛(𝑛 + 1)(𝑛 + 2)(𝑛 + 3) =
5

Example 4.285
Find the sum to 𝑛 terms of
1(2)(3) … (1 + 𝑘) + 2(3)(4) … (2 + 𝑘) + ⋯ + 𝑛(𝑛 + 1) … (𝑛 + 𝑘)

Write the expression in summation notation:


𝑛 𝑛

𝑆𝑛 = ∑ 𝑇𝑖 = ∑ 𝑖(𝑖 + 1) × … × (𝑖 + 𝑘)
𝑖=1 𝑖=1

Multiply both sides of the above by 𝑘 + 2 = (𝒊 + 𝒌 + 𝟏) − (𝒊 − 𝟏):


𝑛

(𝑘 + 2)𝑆𝑛 = ∑ 𝑖(𝑖 + 1) × … × (𝑖 + 𝑘) [(𝒊 + 𝒌 + 𝟏) − (𝒊 − 𝟏)]


𝑖=1
Use the distributive property on the RHS:
𝑛

(𝑘 + 2)𝑆𝑛 = ∑ 𝑖(𝑖 + 1) × … × (𝑖 + 𝑘) (𝒊 + 𝒌 + 𝟏) − 𝒊(𝒊 + 𝟏) × … × (𝒊 + 𝒌)(𝒊 − 𝟏)


𝑖=1
Split up the sum on the RHS:
𝑛 𝑛

(𝑘 + 2)𝑆𝑛 = ∑ 𝑖(𝑖 + 1) × … × (𝑖 + 𝑘) (𝑖 + 𝑘 + 1) − ∑(𝑖 − 1)𝑖(𝑖 + 1) × … × (𝑖 + 𝑘)


𝑖=1 𝑖=1

To telescope, make the terms the same form by reducing the index of summation on the red term:
𝑛 𝑛−1

(𝑘 + 2)𝑆𝑛 = ∑ 𝑖(𝑖 + 1) × … × (𝑖 + 𝑘) (𝑖 + 𝑘 + 1) − ∑(𝑖)(𝑖 + 1) × … × (𝑖 + 𝑘 + 1)


𝑖=1 ⏟
𝑖=1
𝐶ℎ𝑎𝑛𝑔𝑒 𝑡ℎ𝑒 𝑖𝑛𝑑𝑒𝑥 𝑜𝑓 𝑠𝑢𝑚𝑚𝑎𝑡𝑖𝑜𝑛 𝑎𝑛𝑑 𝑑𝑟𝑜𝑝 𝑡ℎ𝑒
𝑖=0 𝑡𝑒𝑟𝑚 𝑠𝑖𝑛𝑐𝑒 𝑖𝑡 𝑣𝑎𝑛𝑖𝑠ℎ𝑒𝑠

Simplify:

P a g e 251 | 274
Get all the files at: https://bit.ly/azizhandouts
Aziz Manva (azizmanva@gmail.com)

(𝑘 + 2)𝑆𝑛 = 𝑛(𝑛 + 1) × … × (𝑛 + 𝑘)(𝑛 + 𝑘 + 1)

Solve for 𝑆𝑛 :
𝑛(𝑛 + 1) × … × (𝑛 + 𝑘)(𝑛 + 𝑘 + 1)
𝑆𝑛 =
(𝑘 + 2)

A. Find the sum to 𝑛 terms:


𝐴. 1 × 1 + 2 × 2 + 3 × 6 + 4 × 10 + ⋯.
𝐵. 3 × 8 + 6 × 11 + 9 × 14 …
𝐶. 3 + 7 + 13 + 21 + 31 + ⋯
12 + 22 12 + 22 + 32
𝐸. 1 + +
1+2 1+2+3

F. Comparing

Example 4.286
Determine without a calculator, which is larger:
1.005200 𝑜𝑟 2

Con
5 200 1 200 1 1 1
𝑃 = (1 + 0.005)200 = (1 + ) = (1 + ) = (1 + ) (1 + ) … (1 + )
1000 200 200 200 200

400 201 202 203 399 400


𝑄=2= =( )( )( )…( )( )
200 200 201 202 398 399
Which has
399 − 200 + 1 = 400 − 200 = 200 𝑡𝑒𝑟𝑚𝑠

1 1 1 1 1
= (1 + ) (1 + ) (1 + ) … (1 + ) (1 + )
200 201 202 398 399

Compare term by term:


1𝑠𝑡 𝑇𝑒𝑟𝑚: 𝐸𝑞𝑢𝑎𝑙
1 1
2𝑛𝑑 𝑇𝑒𝑟𝑚: 1 + > 1+
200 201
1 1
3𝑟𝑑 𝑇𝑒𝑟𝑚: 1 + > 1+
200 202
.
.
.
1 1
200𝑡ℎ 𝑇𝑒𝑟𝑚: 1 + > 1+
200 399

Overall, since one term in P is equal, and other terms in P are greater than Q, we must have:
𝑃>𝑄

P a g e 252 | 274
Get all the files at: https://bit.ly/azizhandouts
Aziz Manva (azizmanva@gmail.com)

4.9 Miscellaneous Sequences


A. Rearranging

Example 4.287
1 3 7 15
Sum of first 𝑛 terms of the series 2 + 4 + 8 + 16 + ⋯ is equal to (JEE Adv. 1988)

The general term is:


2𝑛 − 1 2𝑛 1 1
𝑛
= 𝑛− 𝑛 =1− 𝑛
2 2 2 2
Use this idea to rewrite the series as a sum of two series:
1 1 1 1
(1 − ) + (1 − ) + (1 − ) + (1 − ) + ⋯
2 4 8 16
Separate the two series out:
1 1 1 1 1
=⏟ 1 + 1 + ⋯+ 1 − ( + + + + ⋯ + 𝑛)
𝒏 𝒕𝒊𝒎𝒆𝒔
⏟2 4 8 16 2
𝑮𝒆𝒐𝒎𝒆𝒕𝒓𝒊𝒄 𝑺𝒆𝒓𝒊𝒆𝒔
1 1
1 1 (1− 𝑛 )
The first series is easy to sum. The second is a finite geometric series with 𝑎 = 2 , 𝑟 = 2 and 𝑆𝑢𝑚 = 2 2
1 :
1−
2
1 1 𝟏 1
2 (1 − 2𝑛 ) 𝟐 (1 − 2𝑛 ) 1
=𝑛−[ ]= 𝑛−[ ] = 𝑛 − 1 + 𝑛 = 𝑛 − 1 + 2−𝑛
1 𝟏 2
1−2 𝟐

B. Product

4.288: Product of Consecutive Numbers


𝑛(𝑛 + 1)(𝑛 + 2)
1(2) + 2(3) + ⋯ + 𝑛(𝑛 + 1) =
3
𝑛(𝑛 + 1)(𝑛 + 2)(𝑛 + 3)
1(2)(3) + 2(3)(4) + ⋯ + 𝑛(𝑛 + 1)(𝑛 + 2) =
4
𝑛(𝑛 + 1)(𝑛 + 2)(𝑛 + 3)(𝑛 + 4)
1(2)(3)(4) + 2(3)(4)(5) + ⋯ + 𝑛(𝑛 + 1)(𝑛 + 2)(𝑛 + 3) =
5

Example 4.289
Evaluate:
9

∑ 𝑘(𝑘 + 1)
𝑘=1
7

∑ 𝑘(𝑘 + 1)
𝑘=0

9
9 × 10 × 11
∑ 𝑘(𝑘 + 1) = = 330
3
𝑘=1

P a g e 253 | 274
Get all the files at: https://bit.ly/azizhandouts
Aziz Manva (azizmanva@gmail.com)
7 7
7×8×9
∑ 𝑘(𝑘 + 1) = ∑ 𝑘(𝑘 + 1) = = 168
3
𝑘=0 𝑘=1

Example 4.290
𝑘=10
𝑛2 + 3𝑛 1
𝑎1 + 𝑎2 + ⋯ + 𝑎𝑛 = = 𝑆𝑛 , 28 ∑ = 𝑃1 × 𝑃2 × … × 𝑃𝑚
(𝑛 + 1)(𝑛 + 2) 𝑎𝑘
𝑘=1
Where 𝑃𝑚 are first prime numbers, then find 𝑚. (JEE Main 2023)

Use 𝑎𝑘 = 𝑆𝑘 − 𝑆𝑘−1 :
𝑘(𝑘 + 3) (𝑘 − 1)(𝑘 + 2) (𝑘 3 + 3𝑘 2 ) − (𝑘 − 1)(𝑘 + 2)(𝑘 + 2)
= − =
(𝑘 + 1)(𝑘 + 2) 𝑘(𝑘 + 1) 𝑘(𝑘 + 1)(𝑘 + 2)

Multiply and simplify to get:


(𝑘 3 + 3𝑘 2 ) − (𝑘 3 + 3𝑘 2 − 4) 4
= =
𝑘(𝑘 + 1)(𝑘 + 2) 𝑘(𝑘 + 1)(𝑘 + 2)

1 𝑘(𝑘+1)(𝑘+2)
Substitute 𝑎 = 4
:
𝑘
𝑘=10 𝑘=10 𝑘=10
1 𝑘(𝑘 + 1)(𝑘 + 2) 28
28 ∑ = 28 ∑ = ∑ 𝑘(𝑘 + 1)(𝑘 + 2)
𝑎𝑘 4 4
𝑘=1 𝑘=1 𝑘=1

𝑛(𝑛+1)(𝑛+2)(𝑛+3)
Use the formula: 1(2)(3) + 2(3)(4) + ⋯ + 𝑛(𝑛 + 1)(𝑛 + 2) =
4
28 10 × 11 × 12 × 13
= ×
4 4
= 7 × 10 × 11 × 3 × 13
= 2 × 3 × 5 × 7 × 11 × 13

Since the above expression has the first six prime numbers:
𝑚=6

C. Triangular Numbers

4.291: Sum of First 𝒏 natural Numbers


𝑛
𝑛(𝑛 + 1)
∑𝑥 = 1 + 2 + ⋯+ 𝑛 =
2
𝑥=1

➢ These numbers are also called the triangular numbers.

Example 4.292
If the sum of the first 20 terms of the series log 1 𝑥 + log 1 𝑥 + log 1 𝑥 + ⋯ is 460, then the value of 𝑥 is: (JEE
72 73 74
Main 2020, 5 Sep, Shift-II)

By the power rule extension (or by using change of base), the LHS is
2 log 7 𝑥 + 3 log 7 𝑥 + 4 log 7 𝑥 + ⋯ + 21 log 7 𝑥 = 460
Factor log 7 𝑥:

P a g e 254 | 274
Get all the files at: https://bit.ly/azizhandouts
Aziz Manva (azizmanva@gmail.com)

log 7 𝑥 (2 + 3 + ⋯ + 21)
Use the formula for the sum of the first 𝑛 natural numbers:
21 × 22
log 7 𝑥 ( − 1) = 460
2
log 7 𝑥 (230) = 460
Divide both sides by 230:
log 7 𝑥 = 2
Convert from logarithmic from to exponential form:
𝑥 = 72 = 49

4.293: Union of Two Sets


𝑛(𝐴
⏟ ∪ 𝐵) = 𝑛(𝐴)
⏟ + 𝑛(𝐵)
⏟ − 𝑛(𝐴
⏟ ∩ 𝐵)
𝑼𝒏𝒊𝒐𝒏 𝒐𝒇 𝑨 𝒂𝒏𝒅 𝑩 𝑬𝒍𝒆𝒎𝒆𝒏𝒕𝒔 𝒊𝒏 𝑨 𝑬𝒍𝒆𝒎𝒆𝒏𝒕𝒔 𝒊𝒏 𝑩 𝑬𝒍𝒆𝒎𝒆𝒏𝒕𝒔 𝒊𝒏 𝑰𝒏𝒕𝒆𝒓𝒔𝒆𝒄𝒕𝒊𝒐𝒏
𝒐𝒇 𝑨 𝒂𝒏𝒅 𝑩

Example 4.294
The sum of all natural numbers 𝑛 such that 100 < 𝑛 < 200, and 𝐻𝐶𝐹(91, 𝑛) > 1 is: (JEE Main, 8 April, Shift-I)

𝐻𝐶𝐹(91, 𝑛) > 1 ⇒ 𝐻𝐶𝐹(13 × 7, 𝑛) > 1


Hence, 𝑛 must be a multiple of 7, or a multiple of 13, or both.

Multiples of 7
The multiples of 7 from 100 to 200 have sum:
= 105 + 112 + ⋯ + 196
= 7(15 + 16 + ⋯ + 28)
Use complementary counting:
= 7[(1 + 2 + ⋯ + 28) − (1 + 2 + ⋯ + 14)]
Use the formula for the sum of the first 𝑛 natural numbers:
28 × 29 14 × 15
= 7[ − ] = 7[14 × 29 − 7 × 15]
2 2
Simplify:
= 49[2 × 29 − 15] = 49[43] = 2107

Multiples of 13
The multiples of 13 from 100 to 200 have sum:
= 13(8 + 9 + ⋯ + 15)
Use complementary counting:
= 13[(1 + 2 + ⋯ + 15) − (1 + 2 + ⋯ + 7)]
Use the formula for the sum of the first 𝑛 natural numbers:
15 × 16 7 × 8
= 13 [ − ] = 13[15 × 8 − 7 × 4]
2 2
Simplify:
= 52[15 × 2 − 7] = 52[23] = 1192

Multiples of 91
There is only a single number that is a multiple of 91 from 100 to 200:

P a g e 255 | 274
Get all the files at: https://bit.ly/azizhandouts
Aziz Manva (azizmanva@gmail.com)

= 182

Hence, the final answer is:


2107 + 1196 − 182 = 3,121

4.295: Number of Elements in Three Sets


𝑛(𝐴 ∪ 𝐵 ∪ 𝐶) = ⏟ [𝑛(𝐴 ∩ 𝐵) + 𝑛(𝐵 ∩ 𝐶) + 𝑛(𝐴 ∩ 𝐶)] + ⏟
𝑛(𝐴) + 𝑛(𝐵) + 𝑛(𝐶) − ⏟ 𝑛(𝐴 ∩ 𝐵 ∩ 𝐶)
𝑶𝒏𝒆 𝒂𝒕 𝒂𝒕 𝒂 𝑻𝒊𝒎𝒆 𝑻𝒘𝒐 𝒂𝒕 𝒂 𝑻𝒊𝒎𝒆 𝑻𝒉𝒓𝒆𝒆 𝒂𝒕 𝒂 𝒕𝒊𝒎𝒆

Left Hand Side


= 𝑛(𝐴 ∪ 𝐵 ∪ 𝐶) = 𝑎 + 𝑏 + 𝑐 + 𝑑 + 𝑒 + 𝑓 + 𝑔

Right Hand Side


= (𝑎 + 𝑏 + 𝑒 + 𝑑) + (𝑏 + 𝑐 + 𝑒 + 𝑓) + (𝑑 + 𝑒 + 𝑓 + 𝑔)
− [(𝑏 + 𝑒) + (𝑒 + 𝑓) + (𝑑 + 𝑒) ] + 𝑒
= 𝑎 + 2𝑏 + 3𝑒 + 2𝑑 + 2𝑓 + 𝑔 − 𝑏 − 3𝑒 − 𝑑 − 𝑓 + 𝑒
= 𝑎+𝑏+𝑒+𝑑+𝑓

Example 4.296
How many positive integers not exceeding 2001 are multiples of 3 or 4 but not 5? (AMC 10 2001/25)

𝑀𝑢𝑙𝑡𝑖𝑝𝑙𝑒𝑠 𝑜𝑓 3 = { }

𝑛(𝐴 ∪ 𝐵 ∪ 𝐶) = 𝑛(𝐴) + 𝑛(𝐵) + 𝑛(𝐶) − 𝑛(𝐴 ∩ 𝐵) − 𝑛(𝐵 ∩ 𝐶) − 𝑛(𝐴 ∩ 𝐶) + 𝑛(𝐴 ∩ 𝐵 ∩ 𝐶)

(Calculator) Example 4.297


Ashley writes out the first 2017 positive integers. She then underlines any of the 2017 integers that is a multiple
of 2, and then underlines any of the 2017 integers that is a multiple of 3, and then underlines any of the 2017
integers that is a multiple of 5. Finally, Ashley finds the sum of all the integers which have not been underlined.
What is this sum? (Gauss Grade 7 2017/25)

We will use complementary counting. Find the sum of all the numbers from 1 to 2017, and then subtract the
sum of the numbers which have been underlined.

𝑛(𝑛+1)
The sum of the first 𝑛 natural numbers is given by Hence:
2
.
2017 × 2018
1 + 2 + ⋯ + 2017 = = 2,035,153
2

𝑛(𝑆2 ∪ 𝑆3 ∪ 𝑆5 ) = 𝑛(𝑆2 ) + 𝑛(𝑆3 ) + 𝑛(𝑆5 ) − 𝑛(𝑆6 ) − 𝑛(𝑆15 ) − 𝑛(𝑆10 ) + 𝑛(𝑆30 )

1008 × 1009
𝑆2 = 2 + 4 + ⋯ + 2016 = 2(1 + 2 + ⋯ + 1008) = 2 ( ) = 1,017,072
2
672 × 673
𝑆3 = 3 + 6 + ⋯ + 2016 = 3(1 + 2 + ⋯ + 672) = 3 ( ) = 678,384
2
403 × 404
𝑆5 = 5 + 10 + ⋯ + 2015 = 5(1 + 2 + ⋯ + 403) = 5 ( ) = 407,030
2

P a g e 256 | 274
Get all the files at: https://bit.ly/azizhandouts
Aziz Manva (azizmanva@gmail.com)

336 × 337
𝑆6 = 6 + 12 + ⋯ + 2016 = 6(1 + 2 + ⋯ + 336) = 6 ( ) = 339,696
2
134 × 135
𝑆15 = 15 + 30 + ⋯ + 2010 = 15(1 + 2 + ⋯ + 134) = 15 ( ) = 135,675
2
201 × 202
𝑆10 = 10 + 20 + ⋯ + 2010 = 10(1 + 2 + ⋯ + 201) = 10 ( ) = 203,010
2
67 × 68
𝑆30 = 30 + 60 + ⋯ + 2010 = 30(1 + 2 + ⋯ + 67) = 30 ( ) = 68,340
2

𝑛(𝑆2 ∪ 𝑆3 ∪ 𝑆5 ) = 1017072
⏟ + 678,384
⏟ + 407,030
⏟ − 339,696
⏟ − 135,675
⏟ − 203,010
⏟ + 68,340
⏟ = 1,492,445
𝑛(𝑆2 ) 𝑛(𝑆3 ) 𝑛(𝑆5 ) 𝑛(𝑆6 ) 𝑛(𝑆15 ) 𝑛(𝑆10 ) 𝑛(𝑆30 )

2,035,153 − 1,492,445 =
D. Sum of Squares

4.298: Sum of Squares


𝑛
𝑛(𝑛 + 1)(2𝑛 + 1)
∑ 𝑥 2 = 12 + 22 + ⋯ + 𝑛 2 =
6
𝑥=1

Example 4.299
Find 𝐴 if: (JEE 2019, 12 Jan, Shift-II)
1 + 2 + ⋯+ 𝑘 5
𝑆𝑘 = , 𝑆12 + 𝑆22 + ⋯ + 𝑆10
2
= 𝐴
𝑘 12

Use the formula for the sum of the first 𝑘 numbers:


𝑘(𝑘 + 1) 𝑘 + 1 (𝑘 + 1)2
𝑆𝑘 = = ⇒ 𝑆𝑘2 =
2𝑘 2 4

The LHS of the given equality is:


22 32 112 1 2
𝑆12 + 𝑆22 + ⋯ + 𝑆10
2
= + + ⋯+ = [2 + 32 + ⋯ + 112 ]
4 4 4 4

Use the formula for the sum of the squares of the first 𝑛 numbers:
1 11(12)(23) 1 1
= [ − 1] = [506 − 1] = [505]
4 6 4 4

5 1
𝐴 = [505]
12 4
1 12
𝐴 = [505] × = 303
4 5

Example 4.300
The sum of the series below upto 11th term is: (JEE Main 2019, 9 April, Shift-II)
1 + 2 × 3 + 3 × 5 + 4 × 7+. ..

= 1 + 2 × 3 + 3 × 5 + 4 × 7+. ..
It is critical in this question to get the general term correct:
(𝑁 𝑡ℎ 𝑁𝑢𝑚𝑏𝑒𝑟)(𝑁 𝑡ℎ 𝑂𝑑𝑑 𝑁𝑢𝑚𝑏𝑒𝑟) = 𝑟(2𝑟 − 1) = 2𝑟 2 − 𝑟

P a g e 257 | 274
Get all the files at: https://bit.ly/azizhandouts
Aziz Manva (azizmanva@gmail.com)

Hence, the summation that we are looking for


11 11 11
2 2
∑ 2𝑟 − 𝑟 = 2 ∑ 𝑟 − ∑ 𝑟
𝑟=1 𝑟=1 𝑟=1

Substitute 𝑟 = 11, and use the corresponding formulas:


11(12)(23) 11 × 12
= 2[ ]−( )
6 2
Simplify:
= 11(4)(23) − 11 × 6
Factor out 11, and simplify:
= 11[92 − 6]
= 11[86]
= 946

Example 4.301
The sum of 20 terms of the series 1 + (1 + 2) + (1 + 2 + 3) + ⋯ + (1 + 2 + 3 + ⋯ + 𝑘) is (JEE Main 2020, 8
Jan, Shift-I, Adapted)

Write as a summation:
20

∑(1 + 2 + ⋯ + 𝑘)
𝑘=1
Use the formula for the sum of the first 𝑛 natural numbers:
20
𝑘(𝑘 + 1)

2
𝑘=1
1
Move 2 out of the summation sign:
20
1
= ∑(𝑘 2 + 𝑘)
2
𝑘=1
Split the summation:
20 20
1
= [∑ 𝑘 2 + ∑ 𝑘 ]
2
𝑘=1 𝑘=1
Use the formulas:
1 20(20 + 1)(40 + 1) 20(20 + 1)
= [ + ]
2 6 2
1
= [10(7)(41) + 10(21)]
2

P a g e 258 | 274
Get all the files at: https://bit.ly/azizhandouts
Aziz Manva (azizmanva@gmail.com)

10 10 3080
= [287 + 21] = [308] = = 1540
2 2 2

Example 4.302
A. Find the sum of the first 𝑛 triangular numbers.
B. Use the formula to evaluate ∑20 𝑘=1(1 + 2 + ⋯ + 𝑘)
C. Is it a good idea to memorize this “formula”.

The 𝑛𝑡ℎ triangular number is the sum of the first 𝑛 Convert the second fraction to have a denominator
natural numbers. of 12:
𝑥=𝑛 𝑥=𝑛 𝑛(𝑛 + 1)(2𝑛 + 1) 3𝑛(𝑛 + 1)
𝑥(𝑥 + 1) 𝑥2 + 𝑥 +
∑ =∑ 12 12
2 2
𝑥=1 𝑥=1 Factor 𝑛(𝑛 + 1):
Split the terms in the summation expression: 𝑛(𝑛 + 1)[2𝑛 + 1 + 3]
𝑥=𝑛 𝑥=𝑛
𝑥2 𝑥 12
(∑ ) + (∑ ) Simplify:
2 2
𝑥=1 𝑥=1 𝑛(𝑛 + 1)[2𝑛 + 4]
Use the formula for the sum of the squares of the
12
first 𝑛 natural numbers in the first term, and the Cancel:
formula for the sum of the first 𝑛 natural numbers 𝑛(𝑛 + 1)(𝑛 + 2)
in the second term:
6
𝑛(𝑛 + 1)(2𝑛 + 1) 𝑛(𝑛 + 1)
+
12 4

4.303: Sum of Cubes of Natural Numbers


The sum of the cube of the first 𝑛 natural numbers is the square of the sum of the first 𝑛 natural numbers.
That is:
𝑛 2
3 3 3 3
𝑛(𝑛 + 1) 2
∑ 𝑥 = 1 + 2 + ⋯ + 𝑛 = (1 + 2 + ⋯ + 𝑛) = [ ]
2
𝑥=1

Example 4.304
The sum below is equal to:
7
3 3 3
∑ [ ∙ 12 + ∙ 22 + ⋯ + ∙ 𝑛2 ] (𝑱𝑬𝑬 𝑴𝒂𝒊𝒏 𝟐𝟎𝟐𝟎, 𝟖 𝑱𝒂𝒏, 𝑺𝒉𝒊𝒇𝒕 − 𝑰𝐼)
2 2 2
𝑛=1

3
Factor out 2, and use the sum of squares of the natural numbers:
7 7
3 3 𝑛(𝑛 + 1)(2𝑛 + 1)
∑ [ (12 + 22 + ⋯ + 𝑛2 )] = ∑ [ ( )]
2 2 6
𝑛=1 𝑛=1

1
Multiply, and move the 4 outside, and split the summation:
7 7 7 7
1 1
= ∑[2𝑛3 + 3𝑛2 + 𝑛] = [∑ 2𝑛3 + ∑ 3𝑛2 + ∑ 𝑛]
4 4
𝑛=1 𝑛=1 𝑛=1 𝑛=1

Use the formula for the corresponding sums:

P a g e 259 | 274
Get all the files at: https://bit.ly/azizhandouts
Aziz Manva (azizmanva@gmail.com)

2
1 7(8) 7(8)(15) 7(8)
= [2 ( ) + 3( )+ ]
4 2 6 2

Simplify, cancel the 4


1
= [2(7 × 4)2 + 28(15) + 7(4)]
4
Cancel the 4
= [72 ∙ 8 + 7(15) + 7]
Factor out 7, and simplify:
= 7[56 + 15 + 1] = 7[72] = 504

Example 4.305
If the sum of the first 15 terms of the series below is equal to 225𝑘, then 𝑘 is: (JEE Main 2019, 12 Jan, Shift-II)
3 3 1 3 1 3 3 3
( ) + (1 ) + (2 ) + 33 + (3 ) + ⋯
4 2 4 4

Rewrite the given series:


3 3 6 3 9 3 12 3 15 3
( ) +( ) +( ) +( ) +( ) +⋯
4 4 4 4 4

Factor out :
3 3 3
( ) [1 + 23 + ⋯ + 153 ]
4

Use the corresponding formula:


3 3 15 × 16 2 27
( ) [ ] = × 152 × 82 = 27 × 225
4 2 64

Compare with the equality:


27 × 225 = 222𝑘 ⇒ 𝑘 = 27

Example 4.306
The sum of the following series up to 15 terms is: (JEE 2019, 9 Jan, Shift-II)
12 + 22 + 32 12 + 22 + 32 + 42 12 + 22 + ⋯ + 52
1 + 6 + 9( ) + 12 ( ) + 15 ( )+⋯
7 9 11

We have two hanging terms at the beginning of the series. But, we see that they also follow the pattern, and we
can rewrite the given series as:
12 12 + 22 12 + 22 + 32 12 + 22 + 32 + 42 12 + 22 + ⋯ + 52
3 ( ) + 2(3) ( ) + 3(3) ( ) + 3(4) ( ) + 3(5) ( )+⋯
3 5 7 9 11

Write the general term, and use the formula for the sum of the first 𝑟 squares:
12 + 22 + ⋯ + 𝑟 2 3𝑟 𝑟(𝑟 + 1)(2𝑟 + 1) 𝑟(𝑟 + 1) 1
𝑇𝑟 = 3𝑟 ( )= ( ) = 𝑟( ) = (𝑟 3 + 𝑟 2 )
2𝑟 + 1 2𝑟 + 1 6 2 2

Hence, we need to find the sum:

P a g e 260 | 274
Get all the files at: https://bit.ly/azizhandouts
Aziz Manva (azizmanva@gmail.com)

15 15 15 15
1 1
∑ 𝑇𝑟 = ∑ (𝑟 3 + 𝑟 2 ) = (∑ 𝑟 3 + ∑ 𝑟 2 )
2 2
𝑟=1 𝑟=1 𝑟=1 𝑟=1

Use the corresponding formulas:


2
1 𝑛(𝑛 + 1) 𝑛(𝑛 + 1)(2𝑛 + 1)
= {[ ] + }
2 2 6
𝑛=15
𝑛(𝑛+1)
Factor 2 out of each term:
1 𝑛(𝑛 + 1) 𝑛(𝑛 + 1) 2𝑛 + 1
= { [ + ]}
2 2 2 3 𝑛=15

Get to a common denominator, and add:


1 𝑛(𝑛 + 1) 3𝑛2 + 3𝑛 4𝑛 + 2 1 𝑛(𝑛 + 1) 3𝑛2 + 7𝑛 + 2
= { [ + ]} = { [ ]}
2 2 6 6 𝑛=15
2 2 6 𝑛=15

Substitute 𝑛 = 15, and simplify:


1
= {15(16)[3(15)2 + 7(15) + 2]} = 10[675 + 105 + 2] = 10[782] = 7820
24

Example 4.307
111:

Example 4.308
If 1 + (1 − 22 ∙ 1) + (1 − 42 ∙ 3) + (1 − 62 ∙ 5) + ⋯ + (1 − 202 ∙ 19) = 𝛼 − 220𝛽, then an ordered pair (𝛼, 𝛽) is
equal to: (JEE Main 2020, 4 Sep, Shift-I)
A. (11,97)
B. (10,103)
C. (10,97)
D. (11,103)

Rearrange the LHS:


1 + 1 + ⋯ + 1 − (22 ∙ 1 + 42 ∙ 3 + 62 ∙ 5+. . . +202 ∙ 19)

𝟏𝟏 𝑻𝒆𝒓𝒎𝒔

Note each term in the parentheses is of the form (2𝑛)2 (2𝑛 − 1):
10

= 11 − ∑(2𝑛)2 (2𝑛 − 1)
𝑛=1
Multiply:
10

= 11 − ∑(8𝑛3 − 4𝑛2 )
𝑛=1
Split the summation:

P a g e 261 | 274
Get all the files at: https://bit.ly/azizhandouts
Aziz Manva (azizmanva@gmail.com)
10 10

= 11 − 8 ∑ 𝑛 + 4 ∑ 𝑛2
3

𝑛=1 𝑛=1
Use the formula for the sum:
10 ∙ 11 2 (10)(11)(21)
= 11 − 8 ( ) +4
2 6
Factor 10 ∙ 11 from the second and the third terms:
10 ∙ 11 (21)
= 11 − 10 ∙ 11 [8 −4 ]
4 6
Simplify:
= 11 − 110[2 ∙ 10 ∙ 11 − 14] = 11 − 110[206] = 11 − 220[103]

Hence, the correct answer is:


𝑂𝑝𝑡𝑖𝑜𝑛 𝐷

4.10 Polynomial Sequences


A. Linear Sequences

4.309: 𝒙 vs 𝒏
By convention
➢ 𝑛 is used for term numbers in a sequence where 𝑛 is a natural number.
➢ A variable that can take any real number would usually be 𝑥, 𝑦, 𝑧, …

4.310: Linear Sequence


A sequence which is also a linear function is a linear sequence.

For example:
➢ 𝑇𝑛 = 2𝑛 is a linear sequence
➢ 𝑇𝑛 = 4𝑛 + 9 is a linear sequence

In general:
𝑇𝑛 = 𝑎𝑛 + 𝑏
Is a linear sequence.

4.311: Linear Sequences are Arithmetic


𝑇𝑛 = 𝑎𝑛 + 𝑏 ⇒ 𝑇𝑛 𝑖𝑠 𝑎𝑟𝑖𝑡ℎ𝑚𝑒𝑡𝑖𝑐

𝑇𝑛 = 𝑎𝑛 + 𝑏
𝑇𝑛−1 = 𝑎(𝑛 − 1) + 𝑏 = 𝑎𝑛 − 𝑎 + 𝑏
In order to be an arithmetic sequence, the difference of successive terms should be constant. That is:

𝑇𝑛 − 𝑇𝑛−1 = (𝑎𝑛 + 𝑏) − (𝑎𝑛 − 𝑎 + 𝑏) = 𝑎 = 𝐶𝑜𝑛𝑠𝑡𝑎𝑛𝑡

4.312: Arithmetic Sequences are Linear

The 𝑛𝑡ℎ term of an arithmetic sequence with first term 𝑎 and common difference 𝑑 is:

P a g e 262 | 274
Get all the files at: https://bit.ly/azizhandouts
Aziz Manva (azizmanva@gmail.com)

𝑇𝑛 = 𝑎 + (𝑛 − 1)𝑑 = 𝑎 + 𝑛𝑑 − 𝑑 = 𝑛𝑑 + 𝑎 − 𝑑

Example 4.313
Write the sequence below as a linear function:
7,9,11, …

The general term is:


𝑎 + (𝑛 − 1)𝑑 = 7 + (𝑛 − 1)2 = 7 + 2𝑛 − 2 = 2𝑛 + 5

𝑓(𝑛) = 2𝑛 + 5, 𝑛 ∈ ℕ
B. Quadratic Sequences

4.314: Sum of an Arithmetic Series


For an arithmetic sequence with first term 𝑎 and common difference 𝑑, the sum to 𝑛 terms is:
𝑛
𝑆𝑛 = [2𝑎 + (𝑛 − 1)𝑑]
2
𝑎 = 𝑓𝑖𝑟𝑠𝑡 𝑡𝑒𝑟𝑚
𝑛 = 𝑛𝑢𝑚𝑏𝑒𝑟 𝑜𝑓 𝑡𝑒𝑟𝑚𝑠
𝑑 = 𝑐𝑜𝑚𝑚𝑜𝑛 𝑑𝑖𝑓𝑓𝑒𝑟𝑒𝑛𝑐𝑒

Example 4.315
A. Show that the sum to 𝑛 terms of the series 3 + 7 + 11 + 15 + ⋯ is a quadratic expression.
B. What are the coefficients?

Part A
𝑛
Substitute 𝑎 = 3, 𝑑 = 4 in 𝑆𝑛 = 2 [2𝑎 + (𝑛 − 1)𝑑]:
𝑛 𝑛 𝑛
𝑆𝑛 = [2(3) + (𝑛 − 1)4] = [6 + 4𝑛 − 4] = [4𝑛 + 2] = 2𝑛2 + 𝑛
2 2 2

Part B
𝑎𝑛2 + 𝑏𝑛 + 𝑐 = 2𝑛2 + 𝑛 + 0
𝑎 = 2, 𝑏 = 1, 𝑐 = 0

4.316: Sum of Arithmetic Sequence is Quadratic


The sum of an arithmetic sequence is a quadratic sequence.

Consider an arithmetic sequence with


𝐹𝑖𝑟𝑠𝑡 𝑇𝑒𝑟𝑚 = 𝑎, 𝑁𝑜. 𝑜𝑓 𝑡𝑒𝑟𝑚𝑠 = 𝑛, 𝐶𝑜𝑚𝑚𝑜𝑛 𝐷𝑖𝑓𝑓𝑒𝑟𝑒𝑛𝑐𝑒 = 𝑑

The arithmetic sequence has sum:


𝑛
𝑆𝑛 = [2𝑎 + (𝑛 − 1)𝑑]
2

Distribute the 𝑅𝐻𝑆 to get:


2𝑎𝑛 𝑛(𝑛 − 1)𝑑 2𝑎𝑛 𝑑𝑛2 − 𝑛𝑑
= + = +
2 2 2 2
Simplify:
𝑑 𝑑𝑛
= 𝑎𝑛 + 𝑛2 −
2 2

P a g e 263 | 274
Get all the files at: https://bit.ly/azizhandouts
Aziz Manva (azizmanva@gmail.com)

Rearrange:
𝑑 2 𝑑𝑛
= 𝑛 + 𝑎𝑛 −
2 2
Factor 𝑛 from the last two terms:
𝑑 2 1
= 𝑛 + (𝑎 − 𝑑) 𝑛
2 2

Note that the first term is quadratic, and the second term is linear:
𝑑 2 1
𝑛 + (𝑎 − 𝑑) 𝑛

2 ⏟ 2
𝑸𝒖𝒂𝒅𝒓𝒂𝒕𝒊𝒄 𝑳𝒊𝒏𝒆𝒂𝒓
𝑻𝒆𝒓𝒎 𝑻𝒆𝒓𝒎

4.317: Sum of Arithmetic Sequence is Quadratic


The sum to 𝑛 terms of an arithmetic sequence can be written:
𝑆𝑛 = 𝐴𝑛2 + 𝐵𝑛

From the previous:


𝑑 1
𝐴= , 𝐵=𝑎− 𝑑
2 2

Example 4.318
Consider the parabola
𝑦 = 𝐴𝑥 2 + 𝐵𝑥
A. What is the nature of the expression?
B. What is the 𝑦 intercept of the above?
C. Interpret the 𝑦 intercept that you found in Part A.

Part A
𝑄𝑢𝑎𝑑𝑟𝑎𝑡𝑖𝑐 𝐸𝑥𝑝𝑟𝑒𝑠𝑠𝑖𝑜𝑛
Part B
𝑦 𝑖𝑛𝑡𝑒𝑟𝑐𝑒𝑝𝑡 = 0
Part C
The graph passes through the origin.

Example 4.319
Consider the sum of the first 𝑛 odd numbers.
A. What is the sum?
B. Plot the parabola

P a g e 264 | 274
Get all the files at: https://bit.ly/azizhandouts
Aziz Manva (azizmanva@gmail.com)

Example 4.320
Consider the sum of the first 𝑛 natural numbers.
A. What is the sum?
B. Plot the parabola

Example 4.321
A. What is the definition of an upward, and a downward parabola?
B. What kind of arithmetic sequence generates an upward parabola? What kind generates a downward
parabola?

Part A
A parabola is given by
𝐴𝑥 2 + 𝐵𝑥 + 𝐶, 𝐴 ≠ 0
𝑈𝑝𝑤𝑎𝑟𝑑 𝑃𝑎𝑟𝑎𝑏𝑜𝑙𝑎: 𝐴 > 0
𝐷𝑜𝑤𝑛𝑤𝑎𝑟𝑑 𝑃𝑎𝑟𝑎𝑏𝑜𝑙𝑎: 𝐴 < 0
Part B
𝑑 2 1
𝑛 + (𝑎 − 𝑑) 𝑛 = 𝐴𝑥 2 + 𝐵𝑥 + 𝐶

2 ⏟ 2
𝑸𝒖𝒂𝒅𝒓𝒂𝒕𝒊𝒄 𝑳𝒊𝒏𝒆𝒂𝒓
𝑻𝒆𝒓𝒎 𝑻𝒆𝒓𝒎

Comparing, we see that:


𝑑
𝐴=
2
𝑑
𝐴>0⇒ >0⇒𝑑>0
2
If the common difference is positive, then it is an upward parabola.
If the common difference is negative, then it is a downward parabola.

Example 4.322
Find the sum to 𝑛 terms of the series 3 + 7 + 11 + 15 + ⋯ using the polynomial method

P a g e 265 | 274
Get all the files at: https://bit.ly/azizhandouts
Aziz Manva (azizmanva@gmail.com)

The sum is:


𝑆1 = 3
𝑆2 = 3 + 7 = 10
Let the sum to 𝑛 terms be given by 𝑓(𝑛) = 𝐴𝑛2 + 𝐵𝑛:
𝑓(1) = 𝐴 + 𝐵 = 3 ⇒ ⏟2𝐴 + 2𝐵 = 6
𝑬𝒒𝒖𝒂𝒕𝒊𝒐𝒏 𝑰
𝑓(2) = ⏟
4𝐴 + 2𝐵 = 10
𝑬𝒒𝒖𝒂𝒕𝒊𝒐𝒏 𝑰𝑰
Subtract Equation I from Equation II:
2𝐴 = 4 ⇒ 𝐴 = 2 ⇒ 𝐵 = 1

𝐴𝑛2 + 𝐵𝑛 = 2𝑛2 + 𝑛

4.323: Difference Method


Given a sequence 𝑇1 , 𝑇2 , 𝑇3 , …
𝐹𝑖𝑟𝑠𝑡 𝑑𝑖𝑓𝑓𝑒𝑟𝑒𝑛𝑐𝑒 = 𝑑𝑛 = 𝑇𝑛+1 − 𝑇𝑛
𝑆𝑒𝑐𝑜𝑛𝑑 𝑑𝑖𝑓𝑓𝑒𝑟𝑒𝑛𝑐𝑒 = 𝐷𝑛 = 𝑑𝑛+1 − 𝑑𝑛

Example 4.324
Find the first and the second difference of the sequence below:
𝑇 = {2,6,13,23,36,52}

C. Quadratic Expression for Sums

4.325: Connecting Terms with Sums


For any sequence, 𝑇𝑛 = 𝑆𝑛 − 𝑆𝑛−1
𝑆𝑛 − 𝑆𝑛−1 = (𝑇1 + 𝑇2 + ⋯ + 𝑇𝑛 ) − (𝑇1 + 𝑇2 + ⋯ + 𝑇𝑛−1 ) = 𝑇𝑛

Example 4.326
A. If the sum to 12 terms of a sequence is 134, and the sum to 11 terms is −112, then find the twelfth term.
B. For every 𝑛 the sum of 𝑛 terms of an arithmetic progression is 2𝑛 + 3𝑛2. The rth term is: (AHSME
1965/20)

Part A
𝑇12 = 𝑆12 − 𝑆11 = 246
Part B

P a g e 266 | 274
Get all the files at: https://bit.ly/azizhandouts
Aziz Manva (azizmanva@gmail.com)

The sum to r terms is:


𝑆𝑟 = 2𝑟 + 3𝑟 2
The sum to 𝑟 − 1 terms is:
𝑆𝑟−1 = 2(𝑟 − 1) + 3(𝑟 − 1)2
= 2𝑟 − 2 + 3(𝑟 2 − 2𝑟 + 1)
= 2𝑟 − 2 + 3𝑟 2 − 6𝑟 + 3
= 3𝑟 2 − 4𝑟 + 1

𝑇𝑟 = 𝑆𝑟 − 𝑆𝑟−1 = 2𝑟 + 3𝑟 2 − (3𝑟 2 − 4𝑟 + 1) = 6𝑟 − 1

Example 4.327
A. Show that the sequence with sum 𝑆𝑛 = 3𝑛2 − 11𝑛 is an arithmetic sequence.

𝑆𝑛−1 = 3(𝑛 − 1)2 − 11(𝑛 − 1) = 3𝑛2 − 6𝑛 + 3 − 11𝑛 + 11 = 3𝑛2 − 17𝑛 + 14


𝑡𝑛 = 𝑆𝑛 − 𝑆𝑛−1 = 3𝑛2 − 11𝑛 − 3𝑛2 + 17𝑛 − 14 = 6𝑛 − 14

𝑡𝑛−1 = 6(𝑛 − 1) − 14 = 6𝑛 − 20
𝑡𝑛 − 𝑡𝑛−1 = 6𝑛 − 14 − 6𝑛 + 20 = 6

4.328: Finding common difference from Sum

The sum to 𝑛 terms of an arithmetic progression can be written as:


𝑛 2𝑎𝑛 + 𝑛2 𝑑 − 𝑛𝑑 𝑛2 𝑑 + 𝑛(2𝑎 − 𝑑) 𝑑 2𝑎 − 𝑑
𝑆 = [2𝑎 + (𝑛 − 1)𝑑] = = = ( ) 𝑛2 + ( )𝑛
2 2 2 ⏟2 2
𝑸𝒖𝒂𝒅𝒓𝒂𝒕𝒊𝒄 𝒘𝒊𝒕𝒉 𝒄𝒐𝒏𝒔𝒕𝒂𝒏𝒕 𝒕𝒆𝒓𝒎 𝒛𝒆𝒓𝒐
𝑮𝒐𝒆𝒔 𝒕𝒉𝒓𝒐𝒖𝒈𝒉 𝒕𝒉𝒆 𝒐𝒓𝒊𝒈𝒊𝒏
The common difference is twice the coefficient of the square term in a quadratic expression that represents the
sum of 𝑛 terms of an arithmetic progression.

Example 4.329
If the sum of 𝑛 terms of an arithmetic progression is 𝑝𝑛 + 𝑞𝑛2 , where 𝑝 and 𝑞 are constants, find the common
difference. (NCERT)

The common difference is twice the coefficient of the square term in a quadratic expression that represents the
sum of 𝑛 terms of an arithmetic progression.
𝑑 = 2𝑞
We can also do this using the method of undetermined coefficients:
𝑑 2𝑎 − 𝑑 𝑑
𝑞𝑛2 + 𝑝𝑛 ≡ ( ) 𝑛2 + ( ) 𝑛 ⇒ 𝑞 = ⇒ 𝑑 = 2𝑞
2 2 2
D. Graphing Quadratics

4.330: Definition

Example 4.331
Show that if, for an arithmetic progression, the sum to 𝑚 terms is the same as the sum to 𝑛 terms, then the sum
to 𝑚 + 𝑛 terms is zero.

𝑆𝑛 of an 𝐴𝑃 is a parabola passing through the origin.

P a g e 267 | 274
Get all the files at: https://bit.ly/azizhandouts
Aziz Manva (azizmanva@gmail.com)

𝑆𝑚 = 𝑆𝑛 ⇒ By symmetry, 𝑆𝑛+𝑚 is a root of the quadratic ⇒ 𝑆𝑛+𝑚 = 0

E. General Quadratic Sequences

Example 4.332
Let a sequence 𝑢𝑛 be defined by 𝑢1 = 5 and the relationship 𝑢𝑛+1 − 𝑢𝑛 = 3 + 4(𝑛 − 1), 𝑛 = 1,2,3 … .
A. If 𝑢𝑛 is expressed as a polynomial in 𝑛, the algebraic sum of its coefficients is: (AHSME 1969/32)
B. Find 𝑢𝑛 as a polynomial in 𝑛.
C. The question has given a recursive definition for 𝑢𝑛+1 in terms of 𝑢𝑛 . Find 𝑢𝑛+1 in terms of 𝑢𝑛−1 .

Part A
𝑢1 = 5
𝑢2 = 𝑢1 + 3 + 4(𝑛 − 1) = 5 + 3 + 0 = 8
𝑢3 = 𝑢2 + 3 + 4(𝑛 − 1) = 8 + 3 + 4 = 15
𝑢4 = 𝑢3 + 3 + 4(𝑛 − 1) = 15 + 3 + 8 = 26

𝑓(𝑛) = 𝐴𝑛2 + 𝐵𝑛 + 𝐶
𝑓(1) = 𝐴 + 𝐵 + 𝐶 = 5

𝑬𝒒𝒖𝒂𝒕𝒊𝒐𝒏 𝑰

The sum of coefficients is also


𝐴+𝐵+𝐶 =5
Part B
𝑓(2)
⏟ = 4𝐴 + 2𝐵 + 𝐶 = 8
𝑬𝒒𝒖𝒂𝒕𝒊𝒐𝒏 𝑰𝑰
𝑓(3) = 9𝐴 + 3𝐵 + 𝐶 = 15

𝑬𝒒𝒖𝒂𝒕𝒊𝒐𝒏 𝑰𝑰𝑰

Subtract Eq. I from Eq II, and Eq. II from Eq. III


3𝐴 + 𝐵 = 3 ,
⏟ 5𝐴 + 𝐵 = 7

𝑰𝑰−𝑰=𝑰𝑽 𝑰𝑰𝑰−𝑰𝑰=𝑽

Subtract Eq. IV from Eq. V:


2𝐴 = 4 ⇒ 𝐴 = 2
𝐵 = −3
𝐶=6
𝐴𝑛 + 𝐵𝑛 + 𝐶 = 2𝑛2 − 𝑛 + 6
2

Part C
Rearrange 𝑢𝑛+1 − 𝑢𝑛 = 3 + 4(𝑛 − 1) to get:
𝑢𝑛+1 = 𝑢𝑛 + 3 + 4(𝑛 − 1)
𝑢𝑛+1 = 𝑢𝑛−1 + 3 + 4(𝑛 − 2) + 3 + 4(𝑛 − 1)
𝑢𝑛+1 = 𝑢𝑛−1 + 6 + 4(2𝑛 − 3)

Example 4.333

P a g e 268 | 274
Get all the files at: https://bit.ly/azizhandouts
Aziz Manva (azizmanva@gmail.com)

(Calc) Example 4.334


The rectangular spiral shown in the diagram is constructed as follows.
Starting at (0, 0), line segments of lengths 1, 1, 2, 2, 3, 3, 4, 4, . .. are
drawn in a clockwise manner, as shown. The integers from 1 to 1000
are placed, in increasing order, wherever the spiral passes through a
point with integer coordinates (that is, 1 at (0, 0), 2 at (1, 0), 3 at (1,
−1), and so on). What is the sum of all of the positive integers from 1
to 1000 which are written at points on the line 𝑦 = −𝑥? (CSMC
2017/A/5)

Note that from the diagram each time the length of the line segments
increases by 2, and hence the pattern will also continue.

The given pattern is:


1,3,7,13,21, …
Find the common difference
3−1=2
7−3=4
13 − 7 = 6
21 − 13 = 8

The sequence of common differences is


2,4,6,8, …

𝑇0 = 1 = 1
𝑇1 = 3 = 1 + {2}
𝑇2 = 7 = 1 + {2 + 4}
𝑇3 = 13 = 1 + {2 + 4 + 6}

𝑇𝑛 = 1 + (2 + 4 + 6 + ⋯ + 2𝑛)
𝑇𝑛 = 1 + 2(1 + 2 + 3 + ⋯ + 𝑛)
𝑛(𝑛 + 1)
𝑇𝑛 = 1 + 2 ( )
2
𝑇𝑛 = 1 + 𝑛(𝑛 + 1)
𝑇𝑛 = 𝑛2 + 𝑛 + 1

P a g e 269 | 274
Get all the files at: https://bit.ly/azizhandouts
Aziz Manva (azizmanva@gmail.com)

𝑛2 + 𝑛 + 1 < 1000
2
We know that 32 = 1024. Check 𝑛 = 31:
312 + 31 + 1 = 993

𝑇0 + 𝑇1 + 𝑇2 + ⋯ + 𝑇31
𝑛=31 𝑛=31 𝑛=31 𝑛=31
2 2
∑ 𝑛 +𝑛+1= ∑ 𝑛 + ∑ 𝑛+ ∑ 1
𝑛=0 𝑛=0 𝑛=0 𝑛=0

𝑛=31

∑ 1 = 32
𝑛=0
𝑛=31
𝑛(𝑛 + 1) 31 ∙ 32
∑ 𝑛= = = 496
2 2
𝑛=0
𝑛=31
𝑛(𝑛 + 1)(2𝑛 + 1) 31(32)(63)
∑ 𝑛2 = = = 10416
6 6
𝑛=0

32 + 496 + 10416 = 10944

F. Polynomial Sequences and Cubic Sequences

Example 4.335

G. Exponential Sequences

Example 4.336

4.11 Series Expansions


A. 𝒆

4.337: Factorials

Example 4.338

4.339: Expansion
𝑛=∞
𝑥
𝑥2 𝑥𝑛 𝑥𝑛
𝑒 = 1 + 𝑥 + + ⋯+ = ∑
2! 𝑛! 𝑛!
𝑛=0

Example 4.340

P a g e 270 | 274
Get all the files at: https://bit.ly/azizhandouts
Aziz Manva (azizmanva@gmail.com)

4.12 Further Topics


Example 4.341
Let 𝑆𝑛 be the sum of the first 𝑛 terms of an arithmetic progression. If 𝑆3𝑛 = 3𝑆2𝑛 , then the value of 𝑆𝑛 (as a
number) is:

Method I: Quadratic Sequence


Substitute 𝑆𝑛 = 𝐴𝑛2 + 𝐵𝑛 = 𝑛(𝐴𝑛 + 𝐵) in 𝑆3𝑛 = 3𝑆2𝑛 :
3𝑛[𝐴(3𝑛) + 𝐵] = 3 ∙ 2𝑛[𝐴(2𝑛) + 𝐵]
Divide by 3𝑛 both sides:
𝐴(3𝑛) + 𝐵 = 2[𝐴(2𝑛) + 𝐵]
Simplify and distribute:
3𝐴𝑛 + 𝐵 = 4𝐴𝑛 + 2𝐵
Collate all terms on one side:
𝐴𝑛 + 𝐵 = 0

𝑬𝒒𝒖𝒂𝒕𝒊𝒐𝒏 𝑰
Substitute Equation I in 𝑆𝑛 :
𝑆𝑛 = 𝑛(𝐴𝑛 + 𝐵) = 𝑛(0) = 0
Hence, the sum to 𝑛 terms is zero.

Method II: Sum to 𝒏 terms


𝑆3𝑛 = 3𝑆2𝑛
𝑛
Use the formula 𝑆𝑛 = [2𝑎 + (𝑛 − 1)𝑑]:
2
3𝑛 2𝑛
[2𝑎 + (3𝑛 − 1)𝑑] = 3 ∙ [2𝑎 + (2𝑛 − 1)𝑑]
2 2
3𝑛
Cancel 2
on both sides:
2𝑎 + (3𝑛 − 1)𝑑 = 2[2𝑎 + (2𝑛 − 1)𝑑]
2𝑎 + (3𝑛 − 1)𝑑 = 4𝑎 + (4𝑛 − 2)𝑑
(1 − 𝑛)𝑑 = 2𝑎

𝑬𝒒𝒖𝒂𝒕𝒊𝒐𝒏 𝑰𝑰
𝑛
Substitute Equation II in 𝑆𝑛 = [2𝑎
+ (𝑛 − 1)𝑑]:
2
𝑛 𝑛 𝑛
𝑆𝑛 = [(1 − 𝑛)𝑑 + (𝑛 − 1)𝑑] = [𝑑(1 − 𝑛 + 𝑛 − 1)] = [𝑑(0)] = 0
2 2 2

Example 4.342
𝑆
6: Let 𝑆𝑛 be the sum of the first 𝑛 terms of an arithmetic progression. If 𝑆3𝑛 = 3𝑆2𝑛 , then the value of 𝑆4𝑛 (as a
2𝑛
number) is: (JEE Main 2021, 25 July, Shift-I)

𝑆4𝑛 4𝑛[𝐴(4𝑛) + 𝐵] 2[4𝐴𝑛 + 𝐵] 2[3𝐴𝑛 + 𝐴𝑛 + 𝐵] 2[3𝐴𝑛 + 0] 2[3𝐴𝑛]


= = = = = =6
𝑆2𝑛 2𝑛[𝐴(2𝑛) + 𝐵] [2𝐴𝑛 + 𝐵] [𝐴𝑛 + 𝐴𝑛 + 𝐵] [𝐴𝑛 + 0] [𝐴𝑛]

4𝑛
𝑆4𝑛 [(1 − 𝑛)𝑑 + (4𝑛 − 1)𝑑] 2[3𝑛𝑑]
= 2 = =6
𝑆2𝑛 2𝑛 [(1 − 𝑛)𝑑 + (2𝑛 − 1)𝑑] [𝑛𝑑]
2

Example 4.343
1+2+3+⋯
Is 𝑆2 = 2𝑆1

P a g e 271 | 274
Get all the files at: https://bit.ly/azizhandouts
Aziz Manva (azizmanva@gmail.com)

𝑆1 = 1, 𝑆2 = 3

Example 4.344
−1,0,1
𝑆3 = 0
𝑆6 = 4

−2, −1,0,1,2

Example 4.345
7: Let 𝑆𝑛 denote the sum of first 𝑛 terms of an arithmetic progression. If 𝑆10 = 530, 𝑆5 = 140, then 𝑆20 − 𝑆6 =
(JEE Main 2021, 22 July, Shift-II)

Using 𝑆𝑛 = 𝐴𝑛2 + 𝐵𝑛
𝑆10 = ⏟
100𝐴 + 10𝐵 = 530
𝑬𝒒𝒖𝒂𝒕𝒊𝒐𝒏 𝑰
𝑆5 = 25𝐴 + 5𝐵 = 140 ⇒ ⏟
50𝐴 + 10𝐵 = 280
𝑬𝒒𝒖𝒂𝒕𝒊𝒐𝒏 𝑰𝑰

𝐴 = 5, 𝐵 = 3 ⇒ 𝑆𝑛 = 5𝑛2 + 3𝑛

𝑆20 − 𝑆6 = 5(202 ) − 3(20) − [5(62 ) − 3(6)] = 1862

Example 4.346
1
19: Five numbers are in 𝐴𝑃, whose sum is 25, and product is 2520. If one of these five numbers in − , then the
2
greatest number among them is: (JEE Main 2020, 7 Jan, Shift-I)

Let the five terms be:


𝑎 − 2𝑑, 𝑎 − 𝑑, 𝑎, 𝑎 + 𝑑, 𝑎 + 2𝑑

𝑆𝑢𝑚 = 5𝑎 = 25 ⇒ 𝑎 = 5

Substitute 𝑎 = 5 in 𝑃𝑟𝑜𝑑𝑢𝑐𝑡 = 𝑎(𝑎2 − 𝑑2 )(𝑎2 − 4𝑑2 ) = 2520:


5(25 − 𝑑2 )(25 − 4𝑑2 ) = 2520

Divide both sides by 5 and expand:


4𝑑4 − 125𝑑2 + 625 = 504

Collate all terms on one side:


4𝑑4 − 125𝑑 2 + 121 = 0

This is a disguised quadratic. Substitute 𝑦 = 𝑑2 :


4𝑦 2 − 125𝑦 + 121 = 0
4𝑦 2 − 4𝑦 − 121𝑦 + 121 = 0
(𝑦 − 1)(4𝑦 − 121) = 0
121
𝑦 = 1 𝑂𝑅 𝑦 =
4

P a g e 272 | 274
Get all the files at: https://bit.ly/azizhandouts
Aziz Manva (azizmanva@gmail.com)

11
𝑑 ∈ {±1, ± }
2
If 𝑑 = ±1:
𝑆𝑒𝑞𝑢𝑒𝑛𝑐𝑒 = 3,4,5,6,7 ⇒ 𝑅𝑒𝑗𝑒𝑐𝑡

11
If 𝑑 = + 2
:
−6, −0.5,5,10.5,16 ⇒ 𝐿𝑎𝑟𝑔𝑒𝑠𝑡 𝑇𝑒𝑟𝑚 = 16
11
If 𝑑 = − 2:
16,10.5,5, −0.5, −6 ⇒ 𝐿𝑎𝑟𝑔𝑒𝑠𝑡 𝑇𝑒𝑟𝑚 = 16

Example 4.347
20: If the sum of the first 40 terms of the series 3 + 4 + 8 + 9 + 13 + 14 + 18 + 19 + ⋯ is 102(𝑚), find the
value of 𝑚:

7,17,27, … ⇒ 𝑎 = 7, 𝑑 = 10
20
𝑆20 = [14 + 19 × 10] = 10[14 + 190] = 2040 = 20(102) ⇒ 𝑚 = 20
2

Example 4.348
Find the number of terms common to the sequences below:
𝑆𝑒𝑞𝑢𝑒𝑛𝑐𝑒: 3,7,11, … ,407
𝑆𝑒𝑞𝑢𝑒𝑛𝑐𝑒: 2,9,16, … ,709

3,7,11,15,19,23, … ,407 ⇒ 𝑎1 = 3, 𝑑1 = 4
2,9,16,23, … ,709 ⇒ 𝑎2 = 2, 𝑑2 = 7

The sequence of terms common to both sequences will an arithmetic sequence with
𝑐𝑜𝑚𝑚𝑜𝑛 𝑑𝑖𝑓𝑓𝑒𝑟𝑒𝑛𝑐𝑒 = 𝑑 = 𝐿𝐶𝑀(4,7) = 28
𝑓𝑖𝑟𝑠𝑡 𝑡𝑒𝑟𝑚 = 23

The sequence is:


23,23 + 28,23 + 2(28), … ⇒ 𝑛𝑡ℎ 𝑡𝑒𝑟𝑚 = 23 + (𝑛 − 1)28

23 + (𝑛 − 1)28 ≤ 407
23 + 28𝑛 − 28 ≤ 407
28𝑛 ≤ 412
412
𝑛≤
28
𝑛 ∈ {1,2,3, … ,14}
14 𝑇𝑒𝑟𝑚𝑠

Example 4.349
24: Some identical balls are arranged in rows to form an equilateral triangle. The first row consists of one ball,
the second row consists of two balls and so on. If 99 more identical balls are added to the total number of balls
used in forming the equilateral triangle, then all these balls can be arranged in a square whose each side
contains exactly 2 balls less than number of balls each side of the triangle contains. Then, the number of balls
used to form the equilateral triangle is (JEE Main 2019, 9 April, Shift-II)

The number of balls in the equilateral triangle is:

P a g e 273 | 274
Get all the files at: https://bit.ly/azizhandouts
Aziz Manva (azizmanva@gmail.com)

𝑛(𝑛 + 1)
1 + 2 + ⋯+𝑛 =
2
The number of balls in the square is:
(𝑛 − 2)2

Since from
𝑛(𝑛 + 1)
+ 99 = (𝑛 − 2)2
2
𝑛2 + 𝑛 + 198 = 2(𝑛2 − 4𝑛 + 4)
𝑛2 − 9𝑛 − 190 = 0
𝑛 ∈ {19, −10}
𝑛 = 19
𝑛(𝑛 + 1) 19 × 20
= = 190
2 2

Example 4.350

8:The sum of all the elements in the set {n\in(1,2,...,100)}: HCF of n and 2040 is 1} is equal to:

351 Examples

P a g e 274 | 274

You might also like